Quick viewing(Text Mode)

Guide for Answering Theory Questions in MS Surgery

Guide for Answering Theory Questions in MS Surgery

WBUHS (2011-2015) MS- PAPER – I -IV

Guide for Answering theory questions in MS

Dr. Arkaprovo Roy ASSOCIATE

PROFESSOR DEPARTMENT OF GENERAL SURGERY

Dr. Arkaprovo Roy ASSOCIATE PROFESSOR DEPARTMENT OF GENERAL SURGERY MEDICAL COLLEGE AND HOSPITAL, KOLKATA

THE WEST BENGAL UNIVERSITY OF HEALTH SCIENCES

MS (General Surgery) Examination, 2015 PAPER I

Time Allowed: 3 Hours Full Marks: 100 Attempt all questions

1. How will you assess the nutritional status of a surgical patient? Define and classify artificial nutritional support (ANS). Give an account of enteral nutrition and its advantages and drawbacks. 4+4+8+4 2. Describe the lymph node status in relation to spread of . Discuss in detail the different types of gastric carcinoma and prognosis in respect to lymph node harvest. 5+10+5 3. Write short notes of the following: 5x6

a) Pharmacological therapy in patients awaiting surgery for pheochromocytoma. b) Retroperitoneal fibrosis. c) Ethics and law in surgical practice. d) Pathophysiology of short bowel syndrome. e) Metabolic response to trauma.

4. Answer briefly on the following. 4x71/2

a) Laparoscopic versus conventional surgery in pregnancy. b) Component separation and role of blood components in surgery. c) Graft rejection in transplants. d) Immunohistochemistry.

THE WEST BENGAL UNIVERSITY OF HEALTH SCIENCES

MS (General Surgery) Examination, 2015

April 2015

PAPER I

Time Allowed: 3 Hours Full Marks: 100 Attempt all questions

1. How will you assess the nutritional status of a surgical patient? Define and classify artificial nutritional support (ANS). Give an account of enteral nutrition and its advantages and drawbacks. 4+4+8+4

Answer. The first task when considering perioperative nutritional recommendations is to assess whether or not the patient has malnutrition. The basic principle of dietary and nutritional assessment in the general population is discussed elsewhere. Important aspects of nutritional assessment that pertain to surgical patients are reviewed below.

Based upon expert consensus, a diagnosis of malnutrition requires that the patient exhibit two or more of the following:

●Insufficient energy intake

●Weight loss

●Loss of muscle mass

●Loss of subcutaneous fat

●Localized or generalized fluid accumulation that may sometimes mask weight loss

●Diminished functional status as measured by handgrip strength

History and physical Several aspects of the past medical history are of particular importance, including chronic disease (particularly diabetes), , recent hospitalization, and prior surgery (particularly gastrointestinal— surgery).

On review of systems, a history of weight loss or gain is important. Any recent losses or gains (and whether they were purposeful or not) prior to the hospital stay should be assessed. The details of the current hospitalization also play a central role. Newly admitted trauma patients who are otherwise well have drastically different needs from patients who have had surgery several weeks in the past but have remained hospitalized due to complications. In addition to collecting information on current medications, nonprescription medicines and other supplements should be noted. The use of dietary supplements, such as protein shakes, should also be determined. Lastly, any allergies or food intolerances should also be noted. A diet history should be collected from the patient, family, or care facility. Although there are several methods of dietary assessment, the most useful and straightforward may be to assess the usual intake on an average day before hospitalization or before the onset of the current illness.

In addition to vital signs and a general physical examination, the following should be noted:

●Height and weight (calculate body mass index [BMI] using weight in kg divided by height in meters squared, or using a nomogram)

●General: Loss of subcutaneous fat, any generalized fluid accumulation

●Head and neck exam: Hair loss, bitemporal wasting, conjunctival pallor, xerosis, glossitis, bleeding or sores on the gums and oral mucosa, angular cheilosis or stomatitis, dentition

●Cardiovascular: Evidence of heart failure or high-output state

●Neck: Thyromegaly

●Extremities: Edema, loss of muscle mass

●Neurologic: Evidence of peripheral neuropathy, reflexes, tetany, mental status, handgrip strength

●Skin: Ecchymoses, petechiae, pallor, pressure ulcers, assessment of surgical wound healing and signs of surgical site infection (if postoperative).

Signs of specific nutritional deficiencies should also be sought. Appropriate micronutrient levels should be investigated depending on clinical exam findings.

Several clinical tools are available to quickly assess and score nutrition status. The Subjective Global Assessment of Nutritional Status is a brief tool that includes history and physical examination findings, and allows standardized assessment. The Nutritional Risk Screening tool (NRS 2002) can be applied rapidly and used to screen for poor baseline nutritional status. A study that assessed the ability of the NRS 2002 score to predict the incidence and severity of postoperative complications found the overall incidence of nutritional risk was 14 percent among 608 patients undergoing gastrointestinal surgery. A significantly higher overall complication rate was found in patients at nutritional risk compared with those with a normal NRS 2002 risk score (40 versus 15 percent). Severe complications were also significantly higher in patients at nutritional risk (54 versus 15 percent).

Assessing protein status Assessing protein status is particularly important in the surgical patient because of the close relationship between protein status and wound healing, and because protein- calorie malnutrition can be— treated with supplementation as discussed below. Protein status is affected by previous intake, muscle mass, duration of current illness, blood loss, wound healing, , and gastrointestinal absorption. Three serum measures of protein status have differing half-lives. These serum components do not directly indicate nutritional status, but rather reflect the severity of illness and must be used in conjunction with other clinical data such as the duration of the current surgical illness to be useful in determining therapy. Although decreased levels for these protein markers correlate with adverse outcomes, improvements in these markers with nutritional supplementation are not reliably associated with a clinical benefit.

●Serum albumin has the longest half-life at 18 to 20 days and is the most extensively used parameter. Low serum albumin (<2.2 g/dL) is a marker of a negative catabolic state, and a predictor of poor outcome.Surgical stress, other acute stresses, hepatic disease, and renal disease decrease serum albumin levels.

●Serum transferrin has an intermediate half-life of eight to nine days, reflecting protein status over the past two to four weeks. Transferrin also reflects iron status, and low transferrin should be considered an indicator of protein status only in the setting of normal serum iron.

●Serum prealbumin (transthyretin) has the shortest half-life at two to three days. Although prealbumin responds quickly to the onset of malnutrition and rises rapidly with adequate protein intake, the level can be altered in the acute phase response due to acute or chronic inflammation. In general, inflammatory cytokines reduce the level of prealbumin synthesis by the , and it can also be reduced with renal and hepatic disease. Therefore, serum prealbumin is the least helpful of the three for assessing overall nutritional status.

Other laboratory studies In addition to assessing protein status, a few other laboratory studies may be helpful. Electrolytes, glucose, and BUN/creatinine help assess overall clinical and fluid volume status and need to— be obtained if parenteral (intravenous) nutrition will be instituted. Iron levels should be measured in the setting of unexplained anemia, as should specific vitamin levels if clinically indicated (eg, B12/folate in macrocytic anemias, others based upon specific physical signs). Serum calcium, magnesium, and phosphorous should also be assessed periodically, particularly in the setting of poor oral intake or diarrhea.

 Artificial nutrition support: refers to the administration of nutrient solutions by the enteral or intravenous (parenteral) routes. o Enteral feeding includes the use of oral supplements and tube feeding.  The method of tube feeding may be by nasogastric, nasojejunal, percutaneous endoscopic (PEG), percutaneous endoscopic (PEJ), or fine needle jejunostomy. o Parenteral nutrition may be administered by peripheral or central veins.

Enteral nutrition - See the answer of question 3.d of Paper – I of 2010.

2. Describe the lymph node status in relation to spread of carcinoma stomach. Discuss in detail the different types of gastric carcinoma and prognosis in respect to lymph node harvest. 5+10+5

Answer. See the answer of question 2 of Paper – II of 2006. See the answer of question 2 of Paper – III of 2007. See the answer of question 1 of Paper – III of 2010.

3. Write short notes of the following: 5x6

a) Pharmacological therapy in patients awaiting surgery for pheochromocytoma. b) Retroperitoneal fibrosis. c) Ethics and law in surgical practice. d) Pathophysiology of short bowel syndrome. e) Metabolic response to trauma. Answer. a) Pharmacological therapy in patients awaiting surgery for pheochromocytoma.

Answer. Medical therapy is used for preoperative preparation prior to surgical resection, for acute hypertensive crises, and as primary therapy for patients with metastatic pheochromocytomas. Preoperative preparation requires combined alpha and beta blockade to control blood pressure and to prevent an intraoperative hypertensive crisis. Alpha-adrenergic blockade, in particular, is required to control blood pressure and prevent a hypertensive crisis. High circulating catecholamine levels stimulate alpha receptors on blood vessels and cause vasoconstriction.

Beta blockers are used if significant tachycardia occurs after alpha blockade. Beta blockers are not administered until adequate alpha blockade has been established, however, because unopposed alpha-adrenergic receptor stimulation can precipitate a hypertensive crisis. Noncardioselective beta blockers, such as propranolol or nadolol, are the usual choice; however, cardioselective agents, such as atenolol and metoprolol, also may be used.

Labetalol is a noncardioselective beta-adrenergic blocker and selective alpha-adrenergic blocker that has been shown to be effective in controlling hypertension associated with pheochromocytoma. However, it has also been associated with paradoxic episodes of hypertension thought to be secondary to incomplete alpha blockade. Thus, its use in the preoperative treatment of patients with pheochromocytoma is controversial.

During surgery, intravenous phentolamine, a rapid-acting alpha-adrenergic antagonist, is used to control blood pressure. Rapid-acting intravenous beta blockers, such as esmolol, are also used to normalize blood pressure.

Selective alpha1 blocking agents, such as prazosin, terazosin, and doxazosin, have more favorable adverse effect profiles and are used when long-term therapy is required (metastatic pheochromocytoma). These medications are not used to prepare patients for surgery, because of their incomplete alpha blockade.

The patient with pheochromocytoma is invariably volume depleted. In other words, the chronically elevated adrenergic state characteristic of an untreated pheochromocytoma leads to near-total inhibition of renin-angiotensin activity, resulting in excessive fluid loss in the urine and thus reduced blood volume. Hence, once the pheochromocytoma has been resected, thereby removing the major source of circulating catecholamines, a situation arises where there is both very low sympathetic activity and volume depletion. This can result in profound hypotension. Therefore, it is usually advised to "salt load" pheochromocytoma patients before their surgery. This may consist of simple interventions such as consumption of high salt food pre-operatively, direct salt replacement or through the administration of intravenous saline solution.

b) Retroperitoneal fibrosis.

Answer. Retroperitoneal fibrosis (RPF) (Ormond disease) is a condition that has previously been described as chronic periaortitis. It is an uncommon fibrotic reaction in the retroperitoneum that typically presents with ureteral obstruction.

The disease is part of a spectrum of entities that have a common pathogenetic process consisting of an inflammatory response to advanced atherosclerosis of the abdominal aorta, combined with auto- immunologic factors:

 Idiopathic retroperitoneal fibrosis (IRF)  Perianeurysmal retroperitoneal fibrosis  Isolated periaortitis: corresponds to a non-aneurysmal form of chronic periaortitis  Inflammatory abdominal aortic aneurysm (IAAA)

Epidemiology: Retroperitoneal fibrosis is an uncommon condition with a reported estimated incidence of 1.38 cases per 100,000 people. The mean age is approximately 64 years with a male-to- female ratio of 3:1.

Aetiology:

 Idiopathic: Ormond disease (70% benign)  Radiation  Medication o Methyldopa o Ergotamyl o Methysergide (discontinued migraine medication)  Inflammation: , pyelonephritis  Malignant: desmoplastic reaction, lymphoma  Prolonged exposure to asbestos  Retroperitoneal bleeding, e.g. after trauma or medical procedure

Radiographic features: Contrast enhancing fibrosis encompassing the retroperitoneal structures causing ureteric and vascular obstruction and displacement.

Fluoroscopy: IVU: The ureters are classically medially deviated, giving a distinct path different from their usual course.

CT: Retroperitoneal fibrosis is visible as a mass and has a density similar to soft tissue. In most cases it is located around the aorta and/or iliac artery. The lesion neither enhances in the arterial nor in the excretory phase and does not invade the ureters or arteries. In almost all cases there is no growth posterior to the aorta and the aorta is not displaced. When displacement is present, lymphoma is more likely.

MRI: This is usually dark on T1W MRI and T2W unless there is an active inflammation whereby the T2W images can be hyperintense. Pelvic extension and medial deviation of the ureters are common features.

FDG-PET: Uptake will be seen in active inflammation and absent in metabolically inactive disease. Treatment and prognosis: Retroperitoneal fibrosis may lead to ureteric obstruction and consequent renal failure. In some cases a blow-out of the ureter is described as a result of the obstruction.

Differential diagnosis

Imaging differential considerations include

 Retroperitoneal lymphoma

c) Ethics and law in surgical practice. Answer. The fundamental contract in surgery is an undertaking by one individual to cure another by operation, in the expectation of reward. Autonomy:  Respect for Informed consent for surgery  Truth-telling  Consent for involvement of trainees in surgical procedures  Confidentiality  Respecting patient s requests (for procedures/ particular surgeons)  Good communication skills ’ Beneficence:  Surgical competence  Ability to exercise sound judgment  Continuous professional development  Research and innovation in surgery  Responsible conduct  Functioning equipment and optimal operating conditions  Minimizing harm (including pain control)  Good communication skills Nonmaleficence:  Surgical competence  Continuous professional development  Ability to exercise sound judgment  Recognizing the limits of one s professional competence  Research and auditing  Disclosure and discussion of surgical’ complications including medical errors  Good communication skills Justice:  Allocation of scarce resources  Legal issues  Whistle blowing

o Ethics is an essential discipline in the practice of surgery o Represents your best understanding of moral responsibility o Requires an ability to distinguish degrees of value or lack of o Evolves as reasoned reflection on clinical experience o Ethical study investigates what our character and conduct should be o Morality is subject to re-examination and improvement o Ideas of justice and fairness require critical assessment and improvement o Ethical argument should maintain relevance and integrity Role of the Surgeon is to act as the patient’s fiduciary Expectations:  Patient will get better  Patient s interests are paramount  Resolution of the presenting problem Surgical obligations:’  Knowledge, technical ability and ethical integrity Informed Consent and Disclosure Physician Responsibilities: • Explanation of the patient s disease • Explanation of untreated natural history • Recommendation of most ’appropriate treatment • Discussion of risks and benefits • Anticipated outcome • Treatment alternatives

o Only adult patients are competent to decide which treatments to accept or reject. o Information provided to the patient should be accurate and reasonably complete. o Reasonable person standard identify clinically salient information about the patient s condition and it s management . o Patient s do not have the right“ to dictate treatment. ’ ’ ” d) Pathophysiology’ of short bowel syndrome.

Answer. Physiologic derangements in short-bowel syndrome are the result of the loss of large amounts of intestinal absorptive surface area. The sequelae of this loss include malabsorption of water, electrolytes, macronutrients (ie, proteins, carbohydrates, fats), and micronutrients (ie, vitamins, minerals, trace elements). o The GI tract is a vital locus for water and electrolyte absorption and transport. In addition to managing exogenously obtained sources of these nutrients, such as daily water intake and the electrolytes found in liquid and solid foods, the GI tract must deal with its own considerable daily secretions. o GI tract processes 8000-9000 mL of fluid per day, with the vast majority of this derived from endogenous secretions. Fluid reabsorption by the healthy GI tract is efficient (98%), and only 100-200 mL are lost in fecal matter each day. The great majority (80%) of this reabsorption occurs in the . o Disturbances in the major determinants of intestinal fluid absorption negatively impact the ability to reabsorb this large fluid load. The major determinants include intestinal mucosal surface area, the health or integrity of the mucosa, the status of small bowel motility, and the osmolarity of solutes in the intestinal lumen. o Clinically, these disturbances can manifest as major components of short-bowel syndrome, namely diarrhea, dehydration, and electrolyte imbalance. Thus, short-bowel syndrome can be produced by clinical entities that result in critical loss of mucosal surface area (eg, massive small-) or degrade mucosal integrity (eg, radiation enteritis). o Macronutrients and micronutrients are absorbed along the length of the small intestine. However, the jejunum has taller villi, deeper crypts, and greater enzyme activity than the ileum.Therefore, under normal conditions, about 90% of digestion and absorption of significant macronutrients and micronutrients are accomplished in the proximal 100-150 cm of the jejunum.This includes absorption of proteins; carbohydrates; fats; vitamins B, C, and folic acid; and the fat-soluble vitamins A, D, E, and K. o However, if a significant portion or all of the jejunum is resected, the absorption of proteins, carbohydrates, and most vitamins and minerals can be unaffected because of adaptation in the ileum. Unfortunately, enzymatic digestion suffers because of the irreplaceable loss of enteric hormones produced by the jejunum. Biliary and pancreatic secretions decrease. Gastrin levels rise, causing gastric hypersecretion. The resultant high acid output from the stomach may injure the small bowel mucosa. o Additionally, the low intraluminal pH creates unfavorable conditions for optimal activity of the pancreatic enzymes that are present. Diarrhea may then result if a large osmotically active solute load of unabsorbed nutrients is delivered to the ileum and colon. o Ileal resection severely decreases the capacity to absorb water and electrolytes. In addition, the terminal ileum is the site of absorption of bile salts and vitamin B12. Loss of significant lengths of ileum almost invariably results in diarrhea. Continued loss of bile salts following resection of the terminal ileum leads to fat malabsorption, steatorrhea, and loss of fat-soluble vitamins. o Retention of the ileocecal valve plays a pivotal role in massive small bowel resection. If the ileocecal valve can be preserved, intestinal transit is slowed, allowing more time for absorption. If the ileocecal valve is lost, transit time is faster, and loss of fluid and nutrients is greater. Furthermore, colonic bacteria can colonize the small bowel, worsening diarrhea and nutrient loss. o Preservation of the colon has positive and negative attributes. Philips and Giller demonstrated that colonic water absorption could be increased to as much as five times its normal capacity following small bowel resection. o Also, by virtue of its resident bacteria, the colon has the inherent capacity to metabolize undigested carbohydrates into short-chain fatty acids, such as butyrate, propionate, and acetate. These are a preferred fuel source for the colon. Interestingly, work by Pomare and colleagues and Halverstad demonstrated that the colon can absorb up to 500 kcal daily of these metabolites, which then can be transported via the portal vein to be used as a somatic fuel source. o In contrast, maintenance of the colon increases the incidence of urinary calcium oxalate stone formation. Oxalate is normally bound by calcium in the small bowel and thus is insoluble when it reaches the colon. After massive enterectomy, much of this calcium is bound by free intraluminal fats. Free oxalate is delivered to the colon, where it is absorbed. This can eventually lead to saturation of the urine with calcium oxalate crystals and result in stone formation. Retention of the colon in the absence of a competent ileocecal valve can lead to small intestinal bacterial overgrowth.

The physiologic changes and adaptation of patients with short-bowel syndrome can be viewed in three phases.

 The acute phase occurs immediately after massive bowel resection and may last up to 3-4 months. it is associated with malnutrition and fluid and electrolyte loss through the GI tract. Fluid and electrolyte loss through the GI tract may be as high as 6-8 L/day. Patients will have abnormal liver function test results and transient hyperbilirubinemia.  Enteral feedings may also be initiated, but it should be relatively slow. Patients with less than 100 cm of small intestine will require TPN. The presence of ileocecal valve or colon may play a significant role in the outcome of these patients.  The adaptation phase generally begins 2-4 days after bowel resection and may last up to 12-18 months. During this second phase, up to 90% of the bowel adaptation may occur. Villous hyperplasia, increased crypt depth, and intestinal dilatation occur. Early continuous feedings with a high viscosity elemental diet may reduce the duration of TPN.

In the maintenance phase, the absorptive capacity of the GI tract is at its maximum. Some patients may still require TPN. In other patients, nutritional and metabolic homeostasis can be achieved by small meals and supplemental nutritional support for life. These patients will also require vitamins and mineral supplements, including vitamins A, B12, and D, magnesium, and zinc.

To summarize, the acute phase has the following characteristics:

 Starts immediately after bowel resection and lasts 1-3 months  Ostomy output of greater than 5 L/day  Life-threatening dehydration and electrolyte imbalances  Extremely poor absorption of all nutrients  Development of hypergastrinemia and hyperbilirubinemia

The adaptation phase has the following characteristics:

 Begins within 48 hours of resection and lasts up to 1-2 years  Approximately 90% of the bowel adaptation takes place during this phase  Enterocyte hyperplasia, villous hyperplasia, and increased crypt depth occur, resulting in increased surface area; intestinal dilatation and lengthening also occur  Luminal nutrition is essential for adaptation and should be initiated as early as possible; parenteral nutrition is also essential throughout this period

The maintenance phase has the following characteristics:

 The absorptive capacity of the intestine is at its maximum  Nutritional and metabolic homeostasis can be achieved by oral feeding, or patients are committed to receiving supplemental or complete nutritional support for life e) Metabolic response to trauma. Answer. The response to injury is graded: the more severe the injury, the greater the response. This concept not only applies to physiological/metabolic changes but also to immunological changes/sequelae. Thus, following elective surgery of intermediate severity, there may be a transient and modest rise in temperature, heart rate, respiratory rate, energy expenditure and peripheral white cell count. Following major trauma/sepsis, these changes are accentuated, resulting in a systemic inflammatory response syndrome (SIRS), hypermetabolism, marked catabolism, shock and even multiple organ dysfunction (MODS).

Mediators of the metabolic response to injury: The classical neuroendocrine pathways of the stress response consist of afferent nociceptive neurones, the spinal cord, thalamus, hypothalamus and pituitary. Corticotrophin-releasing factor (CRF) released from the hypothalamus increases adrenocorticotrophic hormone (ACTH) release from the anterior pituitary. ACTH then acts on the adrenal to increase the secretion of cortisol. Hypothalamic activation of the sympathetic nervous system causes release of adrenalin and also stimulates release of glucagon. Intravenous infusion of a cocktail of these counter-regulatory hormones (glucagon, glucocorticoids and catecholamines) reproduces many aspects of the metabolic response to injury. Alterations in insulin release and sensitivity, Hypersecretion‘ of prolactin’ and growth hormone (GH). Of note, GH has direct lipolytic, insulin-antagonising and pro-inflammatory properties. Inactivation of peripheral thyroid hormones and gonadal function. The innate immune system (principally macrophages) interacts in a complex manner with the adaptive immune system (T cells, B cells) in co-generating the metabolic response to injury. Pro-inflammatory cytokines including interleukin-1 (IL-1), tumour necrosis factor alpha (TNF alpha), IL-6 and IL-8 are produced within the first 24 hours and act directly on the hypothalamus to cause pyrexia. Such cytokines also augment the hypothalamic stress response and act directly on to induce proteolysis while inducing acute phase protein production in the liver. Pro-inflammatory cytokines also play a complex role in the development of peripheral insulin resistance. Other important pro-inflammatory mediators include nitric oxide [(NO) via inducible nitric oxide synthetase.

The metabolic response to trauma: o Within hours of the upregulation of pro-inflammatory cytokines, endogenous cytokine antagonists enter the circulation [e.g. interleukin-1 receptor antagonist (IL-1Ra) and TNFsoluble receptors (TNF-sR-55 and 75)] and act to control the pro-inflammatory response. o A complex further series of adaptive changes includes the development of counterinflammatory response [regulated by IL-4, -5, -9 and -13 and transforming growth factor beta (TGF beta)] which, if accentuated and prolonged in critical illness, is characterised as the CARS and results in immunosuppression and an increased susceptibility to opportunistic (nosocomial) infection. o There are many complex interactions between the neuroendocrine, cytokine and metabolic axes. For example, although cortisol is immunosuppressive at high levels, it acts synergistically with IL-6 to promote the hepatic acute phase response. o ACTH release is enhanced by pro-inflammatory cytokines and the noradrenergic system. The resulting rise in cortisol levels may form a weak feedback loop attempting to limit the pro- inflammatory stress response. o Finally, hyperglycaemia may aggravate the inflammatory response via substrate overflow in the mitochondria, causing the formation of excess free oxygen radicals and also altering gene expression to enhance cytokine production.

Systemic inflammatory response syndrome (SIRS) following major injury ■ Is driven initially by pro-inflammatory cytokines (e.g. IL-1, IL-6 and TNF) ■ Is followed rapidly by increased plasma levels of cytokine antagonists and soluble receptors (e.g. IL-1Ra, TNF-sR) ■ If prolonged or excessive may evolve into a counterinflammatory response syndrome (CARS)

The metabolic stress response to surgery and trauma: the ‘Ebb and flow’model In the natural world, if an animal is injured, it displays a characteristic response, which includes immobility, anorexia and catabolism. In 1930, Sir David Cuthbertson divided the metabolic response to injury in humans into ebb and flow phases. Ebb phase The ebb phase begins at the time of injury and lasts for approximately 24 48 hours. It may be attenuated by proper resuscitation, but not completely abolished.‘ ’ ‘The ebb’ phase is characterised by: – 1) Hypovolaemia, 2) Decreased basal metabolic rate, 3) Reduced cardiac output, 4) Hypothermia and lactic acidosis.

The predominant hormones regulating the ebb phase are catecholamines, cortisol and aldosterone (following activation of the renin angiotensin system). The magnitude of this neuroendocrine response depends on the degree of blood loss and the stimulation of somatic afferent nerves at the site of injury. The main physiological– role of the ebb phase is to conserve both circulating volume and energy stores for recovery and repair. Flow phase Following resuscitation, the ebb phase evolves into a hypermetabolic flow phase, which corresponds to the SIRS. This phase involves the mobilisation of body energy stores for recovery and repair, and the subsequent replacement of lost or damaged tissue. It is characterised by: 1) Tissue oedema (from vasodilatation and increased capillary leakage), 2) Increased basal metabolic rate (hypermetabolism), 3) Increased cardiac output, 4) Raised body temperature, 5) Leukocytosis, 6) Increased oxygen consumption and 7) Increased gluconeogenesis. The flow phase may be subdivided into an initial catabolic phase, lasting approximately 3 10 days, followed by an anabolic phase, which may last for weeks if extensive recovery and repair are required following serious injury. During the catabolic phase, the increased– production of counter- regulatory hormones (including catecholamines, cortisol, insulin and glucagon) and inflammatory cytokines (e.g. IL-1, IL-6 and TNFalpha) results in significant fat and protein mobilisation, leading to significant weight loss and increased urinary nitrogen excretion. The increased production of insulin at this time is associated with significant insulin resistance and, therefore, injured patients often exhibit poor glycaemic control. The combination of pronounced or prolonged catabolism in association with insulin resistance places patients within this phase at increased risk of complications, particularly infectious and cardiovascular. Obviously, the development of complications will further aggravate the neuroendocrine and inflammatory stress responses, thus creating a vicious catabolic cycle.

Physiological response to injury:The natural response to injury includes:

■ Immobility/rest ■ Anorexia ■ Catabolism: The changes are designed to aid survival of moderate injury in the absence of medical intervention.

Purpose of neuroendocrine changes following injury: The constellation of neuroendocrine changes following injury acts to: ■ Provide essential substrates for survival ■ Postpone anabolism ■ Optimise host defence.

Key catabolic elements of the flow phase of the metabolic stress response:

Hypermetabolism:

The causes of increase patient energy expenditure are: 1. Central thermodysregulation (caused by the pro-inflammatory cytokine cascade), 2. Increased sympathetic activity, 3. Abnormalities in wound circulation [ischaemic areas produce lactate, which must be metabolised by the adenosine triphosphate (ATP)-consuming hepatic Cori cycle; hyperaemic areas cause an increase in cardiac output], 4. Increased protein turnover and nutritional support. Counteracts of the hypermetabolism: 1) Standard intensive care (including bed rest, paralysis, ventilation and external temperature regulation). 2) The skeletal muscle wasting experienced by patients with prolonged catabolism actually limits the volume of metabolically active tissue.

Alterations in skeletal muscle protein metabolism: Skeletal muscle wasting ■ Provides amino acids for protein synthesis in central organs/tissues ■ Can result in immobility and contribute to hypostatic pneumonia and death if prolonged and excessive Alterations in hepatic protein metabolism: the acute phase protein response (APPR)

Hepatic acute phase response: The hepatic acute phase response represents a reprioritization of body protein metabolism towards the liver and is characterised by: ■ Positive reactants (e.g. CRP): plasma concentration ■ Negative reactants (e.g. albumin): plasma concentration

Insulin resistance o Changes in body composition following major surgery/critical illness ■ Catabolism leads to a decrease in fat mass and skeletal muscle mass ■ Body weight may paradoxically increase because of expansion of extracellular fluid space.

Avoidable factors that compound the response to injury: ■ Continuing haemorrhage ■ Hypothermia ■ Tissue oedema ■ Tissue underperfusion ■ Starvation ■ Immobility

4. Answer briefly on the following. 4x71/2

a) Laparoscopic versus conventional surgery in pregnancy. b) Component separation and role of blood components in surgery. c) Graft rejection in transplants. d) Immunohistochemistry. Answer. a) Laparoscopic versus conventional surgery in pregnancy.

The advantages of laparoscopic surgery are similar for pregnant and nonpregnant women; nevertheless, this procedure had been avoided during pregnancy because of concerns that it may be harmful to the fetus. Potential concerns include: o The rise in intraabdominal pressure during pneumoperitoneum could decrease utero-placental blood flow and result in fetal hypoxia. o Fetal acidosis could develop from absorption of carbon dioxide (CO2. o The fetus could be injured directly or indirectly if the uterus is perforated by a trocar or Veress needle. o Uterine perforation may result in preterm premature rupture of the membranes and preterm delivery.

Advantages of in pregnancy:

1. Short hospital stay 2. Early return to normal activities 3. Small incision, so rapid post operative recovery and less incision complications such as , post operative wound infection and pain. 4. Less uterine manipulation and hence decrease uterine irritability and foetal loss.

Risk of laparoscopy in pregnancy:

1. More chance of uterine injury during port enters as uterus becomes an abdominal organ after first trimester. 2. Problems associated with pneumoperitonization as discussed already. 3. CO2 absorption causes increase CO2 pressure and decrease arterial PH. 4. Risk of exposure to intra abdominal smoke including carbon monoxide generated by electro surgery and laser.

Strategies for safe laparoscopic surgery in pregnancy:

1. Surgery should be done in second trimester. 2. If pt presents in late third trimester, surgery should be postponed if possible until after delivery. 3. Nasogastric incubation is a must in all case as there is a high risk o aspiration into the lungs. 4. Patient can be placed in dorsal lithotomy position in the first half of pregnancy, but in second half to prevent inferior venacaval compression patient is ideally placed in lateral recumbent position. 5. Hypotension should be avoided; proper fluid replacement should be done. 6. Ideal method for commencing pneumoperitonium is open Hasson trocar method. Placement of trocar depends on the size of gravid uterus. 7. Tocolysis is indicated if signs of uterine irritability are present. 8. Decrease operation time by using adequate number of ports, and using most experienced surgeons. 9. Maternal hyperventilation to maintain end-tidal CO2 Pressure at 32mmHg. 10. Lower CO2 insufflations pressure of < 12 mm Hg should be used to avoid foetal acidosis. 11. Electrocautery should be used with care; the smokes containing carbon monoxide should be evacuated promptly to avoid toxic effect to foetus. 12. Entry of all instruments must be under direct vision; care should be taken to avoid injury to the gravid uterus. 13. All specimens should be removed with endobag to avoid spillage. 14. Manipulators should never be fixed to vagina or cervix.

Society of American Gastrointestinal Endoscopic Surgery (SAGES) Recommendations:

1. Obstetrical consultation should be obtained preoperatively. 2. When possible, operative intervention should be deferred until the second trimester, when foetal risk is lowest. 3. Pneumoperitonium enhances lower extremity venous stasis already present in the gravid patient and pregnancy induces a hypercoagulable state. Therefore pneumatic compression devices should be utilized whenever possible. 4. Fatal and uterine status, as maternal end tidal CO2 and/or arterial blood gases, should be monitored. 5. The uterus should be protected with a lead shield if intraoperative is a possibility. 6. Fluoroscopy should be utilized selectively. 7. Given the enlarged gravid uterus, abdominal access should be attained using an open technique. 8. Dependent positioning should be utilized to shift the uterus away from the inferior vena cava. 9. Pneumoperitoneum pressures should be kept at 10 mm Hg. 10. Future studies into methods that increase the safety of laparoscopy in pregnant patient should be done. b) Component separation and role of blood components in surgery.

Answer. Anticoagulants Preservative Solutions:

 Anticoagulants prevent blood clotting  Preservatives provide nutrients for cells  Heparin – Rarely if ever used anymore – Anticoagulant ONLY – Transfuse within 48 hours, preferably 8

Anticoagulants

CPD or CP2-D CPD-A1 Storage time 21 days 35 days Temperature 1-6 C 1-6 C Slows glycolytic activity Adenine None Substrate for ATP synthesis Volume 450 +/- 10% Dextrose Supports ATP generation by glycolytic pathway Citrate Prevents coagulation by binding calcium

Additive Solution (AS)

 Primary bag with satellite bags attached.  One bag has additive solution (AS)  Unit drawn into CPD anticoagulant  Remove platelet rich plasma within 72 hours  Add additive solution to RBCs, ADSOL, which consists of:  Saline  Adenine  Glucose  Mannitol  Extends storage to 42 days  Final hematocrit approximately 66% Changes Occur During Storage:

 Shelf life = expiration date – At end of expiration must have 75% recovery – At least 75% of transfused cells remain in circulation 24 hours after transfusion

Storage Lesion:

 Biochemical changes which occur at 1-6C  Affects oxygen dissociation curve, increased affinity of hemoglobin for oxygen. – Low 2, 3-DPG, increased O2 affinity, less O2 released. – pH drops causes 2,3-DPG levels to fall – Once transfused RBCs regenerate ATP and 2,3-DPG  Few functional platelets present  Viable (living) RBCs decrease  Significant for infants and massive transfusion.  Summary of biochemical changes – pH decreases – 2,3 DPG decreases – ATP decreases – Potassium increases – Sodium decreases – Plasma hemoglobin increases

Preparation of Components:

 Collect unit within 15 minutes to prevent activation of coagulation system  Draw into closed system primary bag with satellite bags with hermetic seal between.  If hermetic seal broken transfuse within 24 hours if stored at 1-4C, 4 hours if stored at 20-24C  Centrifuge light spin, platelets– suspended  Remove platelet rich plasma (PRP)  Centrifuge PRP– heavy spin  Remove platelet poor plasma  Freeze plasma solid within 8 hours  Thaw plasma at 1-4C precipitate forms  Centrifuge, express plasma leaving cryoprecipitate. Store both at -18C  RBCs CPD 21 days,– ADSOL 42 days 1-6C

One unit– of –whole blood can –produce:–

. Packed RBCs . Fresh frozen plasma (FFP) . Cryoprecipitate (CRYO) . Single donor plasma (SDP) cyro removed . Platelets  Sterile docking device joins tubing– . Used to add satellite bags to maintain original expiration of component . May be used to pool components Quality Control:

o Requires certain number of blood products to be tested at regular intervals. o Usually 4 per month. o Must ensure product preparation results in a component that meets the regulatory agencies guidelines for number or quantity.

 Blood separated into components to specifically treat patients with product needed.  Advantages of component separation:

 Allow optimum survival of each component  Transfuse only component needed

Transfusion practice:

o Transfusion requires doctor s prescription o All components MUST be administered through a filter o Infuse quickly, within 4 hours’ o D (Rh) neg require D neg cellular products o ABO identical preferred, ABO compatible OK o Universal donor RBCs group O, plasma AB

Blood“ Components:” –

 Fresh Whole Blood – Blood not usually available until 12-24 hours – Candidates . Newborns needing exchange transfusion . Patients requiring leukoreduced products in US products leukoreduced immediately after collection.  Summary of storage temperatures: – – Liquid rbcs 1-6C – Platelets, Cryo (thawed) and granulocytes 20-24C (room temperature) – -18C – ANY liquid plasma product except Cryo 1-6C ANY frozen plasma product ≤ Blood Components:

o Cellular – Red blood cell products – Platelets – Granulocytes o Plasma – FFP – Cryoprecipitate

o Whole Blood:

o Clinical indications for use of whole blood are extremely limited. o Used for massive transfusion to correct acute hypovolemia such as in trauma and shock, exchange transfusion. o Rarely used today, platelets non-functional, labile coagulation factors gone. o Must be ABO identical

o Red Blood Cells (RBC):

o Used to treat symptomatic anemia and routine blood loss during surgery o Hematocrit is approximately 80% for non-additive (CPD), 60% for additive (ADSOL). o Allow WB to sediment or centrifuge WB, remove supernatant plasma.

o RBCs Leukocyte Reduced:

o Leukocytes can induce adverse affects during transfusion, primarily febrile, non-hemolytic reactions. o Reactions to cytokines produced by leukocytes in transfused units. o Other explanations to reactions include: immunization of recipient to transfused HLA or granulocyte antigens, micro aggregates and fragmentation of granulocytes. o Historically, indicated only for patients who had 2 or more febrile transfusion reactions, now a commonly ordered, popular component. o CMV safe blood, since CMV lives in WBCs. o Most blood centers now leukoreduce blood immediately after collection. o “Bed side” filters are available to leukoreduce products during transfusion.

 Washed Red Blood Cells (W-RBCs):

o Washing removes plasma proteins, platelets, WBCs and micro aggregates which may cause febrile or urticarial reactions. o Patient requiring this product is the IgA deficient patient with anti-IgA antibodies. o Prepared by using a machine which washes the cells 3 times with saline to remove and WBCs. o Two types of labels: . Washed RBCs - do not need to QC for WBCs. . Leukocyte Poor WRBCs, QC must be done to guarantee removal of 85% of WBCs. No longer considered effective method for leukoreduction. o Expires 24 hours after unit is entered.

 Frozen RBCs; Deglycerolized RBCs:

o Blood is frozen to preserve: rare types, for autologous transfusion, stock piling blood for military mobilization and/or civilian natural disasters. o Blood is drawn into an anticoagulant preservative. – Plasma is removed and glycerol is added. – After equilibration unit is centrifuged to remove excess glycerol and frozen. o Expiration – If frozen, 10 years. – After deglycerolization, 24 hours. o Storage temperature – high glycerol -65 C. – low glycerol -120 C, liquid nitrogen.

 Frozen RBCs; Deglycerolized RBCs

o Thaw unit at 37C, thawed RBCs will have high concentration of glycerol. o A solution of glycerol of lesser concentration of the original glycerol is added. o This causes glycerol to come out of the red blood cells slowly to prevent hemolysis of the RBCs. o After a period of equilibration the unit is spun, the solution is removed and a solution with a lower glycerol concentration is added. o This procedure is repeated until all glycerol is removed, more steps are required for the high glycerol stored units. o The unit is then washed.

Blood fractions. Fractions of blood are simply smaller components of blood used in treating patients for various conditions.

The resulting components are:

 A clear solution of blood plasma in the upper phase (which can be separated into its own fractions, see Blood plasma fractionation),  The buffy coat, which is a thin layer of leukocytes (white blood cells) mixed with platelets in the middle, and  Erythrocytes (red blood cells) at the bottom of the centrifuge tube. Suggested Transfusion Guidelines for Red Blood Cells Hemoglobin <8 g/dL or acute blood loss in an otherwise healthy patient with signs and symptoms of decreased oxygen delivery and two or more of the following:

Estimated or anticipated acute blood loss of >15% of total blood volume (750 mL in a 70- ▪ kg male) ▪ Diastolic blood pressure <60 mm Hg ▪ Systolic blood pressure drop >30 mm Hg from baseline ▪ Tachycardia (>100 beats/min) ▪ Oliguria/anuria ▪ Mental status changes

Hemoglobin <10 g/dL in patients with a known increased risk for coronary artery disease or pulmonary insufficiency who have sustained or are expected to sustain significant blood loss

Symptomatic anemia with any of the following:

▪ Tachycardia (>100 beats/min) ▪ Mental status changes ▪ Evidence of myocardial ischemia, including angina ▪ Shortness of breath or dizziness with mild exertion ▪ Orthostatic hypotension

Unfounded/questionable indications:

▪ To increase wound healing ▪ To improve the patient's sense of well-being Hemoglobin between 7 and 10 g/dL (or hematocrit from 21%-30%) in an otherwise ▪ stable, asymptomatic patient ▪ Mere availability of predonated autologous blood without medical indication

Indications for leukocyte-reduced blood components:

 To decrease the incidence of subsequent refractoriness to platelet transfusion caused by HLA

alloimmunization in patients requiring long-term platelet support

 To provide blood components with reduced risk for transmission of cytomegalovirus (CMV)

 To prevent subsequent febrile nonhemolytic transfusion reactions in patients who have had one

documented episode  To decrease the incidence of HLA alloimmunization in nonhepatic solid-organ transplant candidates.

Granulocyte transfusions have been used for profound granulocytopenia (<500/mm3) with evidence of infection (e.g., positive blood culture, persistent temperature higher than 38.5°C) unresponsive to antibiotic therapy. Daily transfusions are given until the infection is under control or the granulocyte count is greater than 1000/mm3.

Suggested Transfusion Guidelines for Platelets

▪ Recent (within 24 hours) platelet count <10,000/mm3 (for prophylaxis) ▪ Recent (within 24 hours) platelet count <50,000/mm3 with demonstrated microvascular

bleeding ( oozing ) or a planned surgical/invasive procedure ▪ Demonstrated microvascular bleeding and a precipitous fall in the platelet count “ ” ▪ Adult patients in the operating room who have had complicated procedures or have required more than 10 units of blood and have microvascular bleeding. Giving platelets assumes that adequate surgical hemostasis has been achieved ▪ Documented platelet dysfunction (e.g., prolonged bleeding time >15 minutes; abnormal platelet function tests) with petechiae, purpura, microvascular bleeding ( oozing ), or a surgical/invasive procedure “ ” Unwarranted indications:

▪ Empirical use with massive transfusion when the patient is not exhibiting clinically

evident microvascular bleeding ( oozing ) Prophylaxis in patients with thrombotic thrombocytopenic purpura/hemolytic-uremic ▪ ” syndrome or idiopathic thrombocytopenic“ purpura ▪ Extrinsic platelet dysfunction (e.g., renal failure, von Willebrand's disease)

Suggested Transfusion Guidelines for Plasma Treatment of multiple or specific coagulation factor deficiency with an abnormal prothrombin time and/or activated partial thromboplastin time

Abnormal specific factor deficiency in the presence of one of the following:

Congenital deficiency of antithrombin III; prothrombin; factors V, VII, IX, X, and XI; ▪ protein C or S; plasminogen or antiplasmin Acquired deficiency related to warfarin therapy, vitamin K deficiency, liver disease, ▪ massive transfusion, or disseminated intravascular coagulation ▪ Also indicated as prophylaxis for the above if a surgical/invasive procedure is planned

Unwarranted indications:

Empirical use during massive transfusion if the patient does not exhibit clinical ▪ coagulopathy ▪ Volume replacement ▪ Nutritional supplement ▪ Hypoalbuminemia

Cryoprecipitate is useful in treating factor deficiency (hemophilia A), von Willebrand's disease, and hypofibrinogenemia and may help treat uremic bleeding. Each 5- to 15-mL unit contains more than 80 units of factor VIII and about 200 mg of fibrinogen. Because the proteins mentioned previously are in relatively high concentration, a smaller volume may be given than would be required if plasma were used. Cryoprecipitate is usually administered as a transfusion of 10 single units.

c) Graft rejection in transplants. Answer. Transplant rejection

Transplant rejection occurs when transplanted tissue is rejected by the recipient's immune system, which destroys the transplanted tissue. Transplant rejection can be lessened by determining the molecular similitude between donor and recipient and by use of immunosuppressant drugs after transplant.

ABO-incompatible transplants: Because very young children (generally under 12 months, but often as old as 24 months) do not have a well-developed immune system, it is possible for them to receive organs from otherwise incompatible donors. This is known as ABO-incompatible (ABOi) transplantation. Graft survival and patient mortality is approximately the same between ABOi and ABO-compatible (ABOc) recipients. While focus has been on infant heart transplants, the principles generally apply to other forms of solid organ transplantation.

The most important factors are that the recipient not have produced isohemagglutinins, and that they have low levels of T cell-independent antigens. Immunologic mechanisms of rejection

Rejection is an adaptive immune response via cellular immunity (mediated by killer T cells inducing apoptosis of target cells) as well as humoral immunity (mediated by activated B cells secreting antibody molecules), though the action is joined by components of innate immune response (phagocytes and soluble immune proteins).

Tissue Mechanism Blood Antibodies (isohaemagglutinins) Kidney Antibodies, cell-mediated immunity (CMI) Heart Antibodies, CMI Skin CMI Bonemarrow CMI Cornea Usually accepted unless vascularised: CMI

Medical categories of rejection:

Hyperacute rejection:

Initiated by preexisting humoral immunity, hyperacute rejection manifests within minutes after transplant, and if tissue is left implanted brings systemic inflammatory response syndrome. Of high risk in kidney transplants is rapid clumping, namely agglutination, of red blood cells (RBCs or erythrocytes), as an antibody molecule binds multiple target cells at once.

Acute rejection

Developing with formation of cellular immunity, acute rejection occurs to some degree in all transplants, except between identical twins, unless immunosuppression is achieved (usually through drugs). Acute rejection begins as early as one week after transplant, the risk being highest in the first three months, though it can occur months to years later. Highly vascular tissues such as kidney or liver often host the earliest signs particularly at endothelial cells lining blood vessels though it eventually occurs in roughly 10 to 30% of liver transplants, and 10 to 20% of kidney transplants. A single episode of acute rejection— can be recognized and promptly treated, usually — preventing organ failure, but recurrent episodes lead to chronic rejection. It is believed that the process of acute rejection is mediated by the cell mediated pathway, specifically by mononuclear macrophages and T-lymphocytes.

Chronic rejection

Micrograph showing a glomerulus with changes characteristic of a transplant glomerulopathy. Transplant glomerulopathy is considered a form of chronic antibody-mediated rejection. PAS stain.

The term chronic rejection initially described long-term loss of function in transplanted organs via fibrosis of the transplanted tissue's blood vessels. This is now chronic allograft vasculopathy, however, leaving chronic rejection referring to rejection due to more patent aspects of immunity.

Chronic rejection explains long-term morbidity in most lung-transplant recipients, the median survival roughly 4.7 years, about half the span versus other major organ transplants. In histopathology the condition is bronchiolitis obliterans, which clinically presents as progressive airflow obstruction, often involving dyspnea and coughing, and the patient eventually succumbs to pulmonary insufficiency or secondary acute infection.

Rejection due to non-adherence

One principal reason for transplant rejection is non-adherence to prescribed immunosuppressant regimens. This is particularly the case with adolescent recipients, with non-adherence rates near 50% in some instances.

Rejection detection

Diagnosis of acute rejection relies on clinical data patient signs and symptoms but also calls on laboratory data such as tissue biopsy. The laboratory pathologist generally seeks three main histological signs: (1) infiltrating T cells, perhaps —accompanied by infiltrating eosinophils— , plasma cells, and neutrophils, particularly in telltale ratios, (2) structural compromise of tissue anatomy, varying by tissue type transplanted, and (3) injury to blood vessels. Tissue biopsy is restricted, however, by sampling limitations and risks/complications of the invasive procedure. Cellular magnetic resonance imaging (MRI) of immune cells radiolabeled in vivo might offer noninvasive testing.

Rejection treatment

Hyperacute rejection manifests severely and within minutes, and so treatment is immediate: removal of the tissue. Chronic rejection is generally considered irreversible and poorly amenable to treatment only retransplant generally indicated if feasible though inhaled cyclosporine is being investigated to delay or prevent chronic rejection of lung transplants. Acute rejection is treated with —one or multiple of a few strategies. —

Immunosuppressive therapy

A short course of high-dose corticosteroids can be applied, and repeated. Triple therapy adds a calcineurin inhibitor and an anti-proliferative agent. Where calcineurin inhibitors or steroids are contraindicated, mTOR inhibitors are used.

Immunosuppressive drugs:

 Corticosteroids o Prednisolone o Hydrocortisone  Calcineurin inhibitors o Cyclosporin o Tacrolimus  Anti-proliferatives o Azathioprine o Mycophenolic acid  mTOR inhibitors o Sirolimus o Everolimus Antibody-based treatments

Antibody specific to select immune components can be added to immunosuppressive therapy. The monoclonal anti-T cell antibody OKT3, once used to prevent rejection, and still occasionally used to treat severe acute rejection, has fallen into disfavor, as it commonly brings severe cytokine release syndrome and late post-transplant lymphoproliferative disorder. (OKT3 is available in the United Kingdom for named-patient use only.)

Antibody drugs:

 Monoclonal anti-IL- o Basiliximab o Daclizumab 2Rα receptor antibodies  Polyclonal anti-T-cell antibodies o Anti-thymocyte globulin (ATG) o Anti-lymphocyte globulin (ALG)  Monoclonal anti-CD20 antibodies o Rituximab

Blood transfer

Cases refractory to immunosuppressive or antibody therapy are sometimes given blood transfusions removing antibody molecules specific to the transplanted tissue.

Marrow transplant—

Bone marrow transplant can replace the transplant recipient's immune system with the donor's, and the recipient accepts the new organ without rejection. The marrow's hematopoietic stem cells the reservoir of stem cells replenishing exhausted blood cells including white blood cells forming the immune system must be of the individual who donated the organ or of an identical twin— or a clone. There is a risk of graft-versus-host disease (GVHD), however, whereby mature lymphocytes entering with marrow— recognize the new host tissues as foreign and destroy them. d) Immunohistochemistry.

Answer. Immunohistochemistry (IHC) refers to the process of detecting antigens (e.g. proteins) in cells of a tissue section by exploiting the principle of antibodies binding specifically to antigens in biological tissues. Immunohistochemical staining is widely used in the diagnosis of abnormal cells such as those found in cancerous tumors. Specific molecular markers are characteristic of particular cellular events such as proliferation or cell death (apoptosis). Immunohistochemistry is also widely used in basic research to understand the distribution and localization of biomarkers and differentially expressed proteins in different parts of a biological tissue.

Sample preparation: o Preparation of the sample is critical to maintain cell morphology, tissue architecture and the antigenicity of target epitopes. This requires proper tissue collection, fixation and sectioning. A solution of paraformaldehyde is often used to fix tissue, but other methods may be used. o Depending on the method of fixation and tissue preservation, the sample may require additional steps to make the epitopes available for antibody binding, including deparaffinization and antigen retrieval. o Dependent on the tissue type and the method of antigen detection, endogenous biotin or enzymes may need to be blocked or quenched, respectively, prior to antibody staining. Although antibodies show preferential avidity for specific epitopes, they may partially or weakly bind to sites on nonspecific proteins (also called reactive sites) that are similar to the cognate binding sites on the target antigen. o A great amount of non-specific binding causes high background staining which will mask the detection of the target antigen.

Antibody types

 The antibodies used for specific detection can be polyclonal or monoclonal. Polyclonal antibodies are made by injecting animals with the protein of interest, or a peptide fragment and, after a secondary immune response is stimulated, isolating antibodies from whole serum. Thus, polyclonal antibodies are a heterogeneous mix of antibodies that recognize several epitopes. Monoclonal antibodies show specificity for a single epitope.  For immunohistochemical detection strategies, antibodies are classified as primary or secondary reagents. Primary antibodies are raised against an antigen of interest and are typically unconjugated (unlabelled), while secondary antibodies are raised against immunoglobulins of the primary antibody species. The secondary antibody is usually conjugated to a linker molecule, such as biotin, that then recruits reporter molecules, or the secondary antibody itself is directly bound to the reporter molecule.

IHC reporters:

Reporter molecules vary based on the nature of the detection method, the most popular being chromogenic and fluorescence detection mediated by an enzyme or a fluorophore, respectively. With chromogenic reporters, an enzyme label is reacted with a substrate to yield an intensely colored product that can be analyzed with an ordinary light microscope. While the list of enzyme substrates is extensive, alkaline phosphatase (AP) and horseradish peroxidase (HRP) are the two enzymes used most extensively as labels for protein detection. An array of chromogenic, fluorogenic and chemiluminescent substrates is available for use with either enzyme, including DAB or BCIP/NBT, which produce a brown or purple staining, respectively, wherever the enzymes are bound.

Target antigen detection methods:

The direct method is a one-step staining method and involves a labeled antibody (e.g. FITC- conjugated antiserum) reacting directly with the antigen in tissue sections. While this technique utilizes only one antibody and therefore is simple and rapid, the sensitivity is lower due to little signal amplification, in contrast to indirect approaches. However, this strategy is used less frequently than its multi-phase counterpart.

The indirect method involves an unlabeled primary antibody (first layer) that binds to the target antigen in the tissue and a labeled secondary antibody (second layer) that reacts with the primary antibody. As mentioned above, the secondary antibody must be raised against the IgG of the animal species in which the primary antibody has been raised. This method is more sensitive than direct detection strategies because of signal amplification due to the binding of several secondary antibodies to each primary antibody if the secondary antibody is conjugated to the fluorescent or enzyme reporter.

Further amplification can be achieved if the secondary antibody is conjugated to several biotin molecules, which can recruit complexes of avidin-, streptavidin- or NeutrAvidin protein-bound enzyme. The difference between these three biotin-binding proteins is their individual binding affinity to endogenous tissue targets leading to nonspecific binding and high background; the ranking of these proteins based on their nonspecific binding affinities, from highest to lowest, is: 1) avidin, 2) streptavidin and 3) NeutrAvidin protein.

The indirect method, aside from its greater sensitivity, also has the advantage that only a relatively small number of standard conjugated (labeled) secondary antibodies needs to be generated. For example, a labeled secondary antibody raised against rabbit IgG, which can be purchased "off the shelf," is useful with any primary antibody raised in rabbit. With the direct method, it would be necessary to label each primary antibody for every antigen of interest.

Counterstains

After immunohistochemical staining of the target antigen, a second stain is often applied to provide contrast that helps the primary stain stand out. Many of these stains show specificity for specific classes of biomolecules, while others will stain the whole cell. Both chromogenic and fluorescent dyes are available for IHC to provide a vast array of reagents to fit every experimental design, and include: hematoxylin, Hoechst stain and DAPI are commonly used.

IHC troubleshooting

In immunohistochemical techniques, there are several steps prior to the final staining of the tissue antigen, and many potential problems affect the outcome of the procedure. The major problem areas in IHC staining include strong background staining, weak target antigen staining and autofluorescence. Endogenous biotin or reporter enzymes or primary/secondary antibody cross- reactivity are common causes of strong background staining, while weak staining may be caused by poor enzyme activity or primary antibody potency. Furthermore, autofluorescence may be due to the nature of the tissue or the fixation method. These aspects of IHC tissue prep and antibody staining must be systematically addressed to identify and overcome staining issues.

Diagnostic IHC markers

Immunohistochemical staining of normal kidney with CD10.

IHC is an excellent detection technique and has the tremendous advantage of being able to show exactly where a given protein is located within the tissue examined. It is also an effective way to examine the tissues. This has made it a widely used technique in the neurosciences, enabling researchers to examine protein expression within specific brain structures. Its major disadvantage is that, unlike immunoblotting techniques where staining is checked against a molecular weight ladder, it is impossible to show in IHC that the staining corresponds with the protein of interest. For this reason, primary antibodies must be well-validated in a Western Blot or similar procedure. The technique is even more widely used in diagnostic surgical for immunophenotyping tumors (e.g. immunostaining for e-cadherin to differentiate between DCIS ( in situ: stains positive) and LCIS ( in situ: does not stain positive) . More recently, Immunohistochemical techniques have been useful in differential diagnoses of multiple forms of salivary , head, and neck .

The diversity of IHC markers used in diagnostic surgical pathology is substantial. Many clinical laboratories in tertiary hospitals will have menus of over 200 antibodies used as diagnostic, prognostic and predictive biomarkers. Examples of some commonly used markers include:

 Cytokeratins: used for identification of carcinomas but may also be expressed in some .  CD15 and CD30 : used for Hodgkin's disease  Alpha fetoprotein: for yolk sac tumors and  CD117 (KIT): for gastrointestinal stromal tumors (GIST) and mast cell tumors  CD10 (CALLA): for and acute lymphoblastic leukemia  Prostate specific antigen (PSA): for prostate  estrogens and progesterone receptor (ER & PR) staining are used both diagnostically (breast and gyn tumors) as well as prognostic in and predictive of response to therapy (estrogen receptor)  Identification of B-cell lymphomas using CD20  Identification of T-cell lymphomas using CD3

Directing therapy

A variety of molecular pathways are altered in cancer and some of the alterations can be targeted in cancer therapy. Immunohistochemistry can be used to assess which tumors are likely to respond to therapy, by detecting the presence or elevated levels of the molecular target.

Chemical inhibitors

Tumor biology allows for a number of potential intracellular targets. Many tumors are hormone dependent. The presence of hormone receptors can be used to determine if a tumor is potentially responsive to antihormonal therapy. One of the first therapies was the antiestrogen, tamoxifen, used to treat breast cancer. Such hormone receptors can be detected by immunohistochemistry.[8] Imatinib, an intracellualar tyrosine kinase inhibitor, was developed to treat chronic myelogenous leukemia, a disease characterized by the formation of a specific abnormal tyrosine kinase. Imitanib has proven effective in tumors that express other tyrosine kinases, most notably KIT. Most gastrointestinal stromal tumors express KIT, which can be detected by immunohistochemistry.

Monoclonal antibodies

HER-2 status characterization of xenografts by immunohistochemistry of HER-2.

Many proteins shown to be highly upregulated in pathological states by immunohistochemistry are potential targets for therapies utilising monoclonal antibodies. Monoclonal antibodies, due to their size, are utilized against cell surface targets. Among the overexpressed targets are members of the epidermal growth factor receptor (EGFR) family, transmembrane proteins with an extracellular receptor domain regulating an intracellular tyrosine kinase.. Of these, HER2/neu (also known as Erb-B2) was the first to be developed. The molecule is highly expressed in a variety of cancer cell types, most notably breast cancer. As such, antibodies against HER2/neu have been FDA approved for clinical treatment of cancer under the drug name Herceptin.

Similarly, EGFR (HER-1) is overexpressed in a variety of including head and neck and colon. Immunohistochemistry is used to determine patients who may benefit from therapeutic antibodies such as Erbitux (cetuximab).

THE WEST BENGAL UNIVERSITY OF HEALTH SCIENCES MS (General Surgery) Examination, 2014 PAPER I

Time Allowed: 3 Hours Full Marks: 100 Attempt all questions.

1) Describe the segmental anatomy of liver. Discuss the clinical features, investigations and management of Primary liver carcinoma. 5+3+7+5 2) Describe in brief the molecular biology of breast cancers, particularly in reference to identify a woman at high risk. Describe the presently available medical and surgical options in the risk reduction . 10+5+5 3) Write short notes of the following: 5x6

a) Gastrointestinal hormones. b) Parathyroid localisation. c) Distributive shock. d) Borderline resectable pancreatic tumours. e) Pharmacotherapy of Peripheral Arterial Disease. 4) Answer briefly on the following. 4x71/2

a) Pathogenesis of different organ dysfunction in abdominal compartment syndrome. b) Premalignant conditions of skin. c) Pathogenesis of sepsis induced hypotension d) Immune-nutrition in surgical patients.

THE WEST BENGAL UNIVERSITY OF HEALTH SCIENCES

MS (General Surgery) Examination, 2014

April 2014

PAPER I

Time Allowed: 3 Hours Full Marks: 100

Attempt all questions.

1) Describe the segmental anatomy of liver. Discuss the clinical features, investigations and management of Primary liver carcinoma. 5+3+7+5

Answer. Segmental anatomy of liver see the answer of question 3.a of Paper – I of 2012. Clinical presentation: o Most commonly, patients presenting– with HCC are men 50 to 60 years of age who complain of right upper quadrant abdominal pain and weight loss, and have a palpable mass. o In countries endemic for HBV, presentation at a younger age is common and probably related to childhood infection. o Unfortunately, in unscreened populations, HCC tends to present at a later stage because of the lack of symptoms in early stages. o Presentation at an advanced stage is often with vague right upper quadrant abdominal pain that sometimes radiates to the right shoulder. o Nonspecific symptoms of advanced such as anorexia, nausea, lethargy, and weight loss are also common. o Another common presentation of HCC is hepatic decompensation in a patient with known mild cirrhosis or even in patients with unrecognized cirrhosis. o HCC can rarely present as a rupture, with the sudden onset of abdominal pain followed by hypovolemic shock secondary to intraperitoneal bleeding. Other rare presentations include: o Hepatic vein occlusion (Budd-Chiari syndrome), o Obstructive jaundice, o Hemobilia, and o Fever of unknown origin. o Less than 1% of cases of HCC present with a paraneoplastic syndrome, usually o Hypercalcemia, hypoglycemia, and erythrocytosis. o Small incidentally noted tumors have become a more common presentation because of the knowledge of . Specific risk factors, . Screening programs for diagnosed HBV or HCV infection, . Increasing use of high-quality abdominal imaging. For patients suspected of suffering from HCC, the aims of diagnostic investigations are: . Verification of diagnosis, . Determine extent of disease, . Determine functional liver reserve, and . Assess biologic determinants that are predictors of long-term prognosis. Diagnosis:  Diagnosis is often confirmed with a biopsy.  Diagnosis can sometimes be confirmed with blood or imaging tests.  Physical examination.  Blood test for alpha-fetoprotein (AFP); 50%-70% of people with primary liver cancer have elevated levels.  Ultrasound of the abdomen.  Computed tomography (CT or CAT) scan.  Magnetic resonance imaging (MRI).  Angiogram.  Laparoscopy.  Radiologic investigation is a critical part of the diagnosis of HCC.  Ultrasound plays a significant role in screening and early detection of HCC, but definitive diagnosis and treatment planning rely on CT and/or MRI.  Contrast-enhanced CT and MRI protocols aimed at diagnosing HCC take advantage of the hypervascularity of these tumors, and arterial phase images are critical to assess the extent of disease adequately.  CT and MRI also evaluate the extent of disease in terms of peritoneal metastases, nodal metastases, and extent of vascular and biliary involvement.  Detection of bland or tumor thrombus in the portal or hepatic venous system is also important and can be diagnosed with any of these modalities.  AFP measurements can be helpful in the diagnosis of HCC. . An AFP level higher than 20 ng/mL is noted in approximately 75% of documented cases of HCC. . False-positive elevations of serum AFP levels can be seen in inflammatory disorders of the liver, such as chronic active viral hepatitis. . The specificity and positive predictive values of AFP improve with higher cutofflevels (e.g., 400 ng/mL), but at the cost of sensitivity. . With improvements in imaging technology and the ability to detect smaller tumors, AFP is largely used as an adjunctive test in patients with liver masses. . AFP levels are particularly useful in monitoring treated patients for recurrence after normalization of levels.

 If atypical features appear on imaging, a biopsy should be obtained for histologic diagnosis.  For hepatic nodules larger than 2 cm, a triple-phase CT or MRI scan is required if typical features of HCC are identified in combination with an AFP level higher than 200 ng/mL.  If typical features appear on imaging, the diagnosis of HCC is confirmed.  If atypical features are seen, then biopsy is required to confirm the histologic diagnosis.

 At present, the diagnosis of HCC can be made according to the guidelines of the Barcelona-2000 European Association for the Study of the Liver (EASL) Conference and to successive modifications of guidelines from the American Association for the Study of Liver Disease (AASLD).  Pathologic diagnoses of HCC are made according to the International Working Party criteria. For hepatic nodules from 1 to 2 cm in size, a triplephase CT and MRI scan must show typical features of HCC arterially enhancing mass with washout of contrast in delayed phases to confirm the diagnosis. Treatment: — —  There are a large number of treatment options for patients with HCC, reflecting the heterogeneity of this disease and the lack of a proven superior treatment, except complete resection.  Selection of the appropriate patient for resection is critical and must take into account the condition of the liver and extent of disease.  Patients with Child-Pugh class B or C cirrhosis or do not tolerate resection. Treatment Options for Hepatocellular Carcinoma: Surgical: Resection Orthotopic Ablative: Ethanol (EtOH) injection Acetic acid injection Thermal ablation (cryotherapy, radiofrequency ablation, microwave) Transarterial Embolization Chemoembolization Radiotherapy Combination Transarterial and Ablative: External Beam Radiation Systemic Chemotherapy Hormonal Immunotherapy

Pugh's Modification of Child's Grading of Cirrhosis: Measurements 1 point 2 points 3 points

Bilirubin (mg/dL) 1 1.9 2 2.9 >2.9

– – Prothrombin time prolongation 1 3 4 6 >6 (secs) – –

Albumin (g/dL) >3.5 2.8 3.4 <2.8

– Ascites none mild moderate to severe

Cancer Treatment: Surgery:  Most successful in patients with small tumors (smaller than 5 cm) and with good liver function  : portion of the liver is removed when the cancer is limited to one part of the liver  Liver transplantation: used to treat cancer confined to the liver if a suitable donor is found. Must fulfill strict criteria  A variety of prognostic factors predictive of survival after resection have been identified, but none are universally agreed on.  The most commonly cited negative prognostic factors are tumor size, cirrhosis, infiltrative growth pattern, vascular invasion, intrahepatic metastases, multifocal tumors, lymph node metastases, margin less than 1 cm, and lack of a capsule.  The best outcomes are found in patients with single small tumors, but size alone should not contraindicate resection.  Multifocal tumors and major vascular invasion are generally associated with a poor outcome but some groups advocate resection in highly select patients. Ablation of liver tumour: Percutaneous Laparoscopy Open method Percutaneous technique is best choice for early stage liver tumor who are not surgical candidates.  Selection criteria:  Single tumor < 5cm  Multiple tumors< 3 nodule and < 3cm  No evidence of vascular invasion  No extrahepatic spread  Performance status test 0  Cirrhosis with Child Pugh class A or B Percutaneous methods: – Chemical ablation Thermal ablation

Ethanol injection RFA Acetic acid injection Microwave Laser ablation Cryoablation

Cancer Treatment: Radiation Therapy:  The use of high-energy x-rays to destroy cancer cells  The use of high energy x-rays or other particles to destroy cancer cells  Internal beam: use of implants inside the body. Radioactive beads may be inserted into the artery that supplies the tumor with blood  External beam: outside the body rarely used for HCC  Side effects can include fatigue, mild skin reactions, upset stomach and loose bowel movements – Cancer Treatment: Chemoherapy:  Use of drugs to kill cancer cells  One or a combination of drugs may be used  Side effects may include nausea and vomiting, loss of appetite, diarrhea, fatigue, and risk of infection  Side effects often go away after treatment is finished

Cancer Treatment: Targeted Therapy:  Targets faulty genes or proteins that contribute to cancer growth and development. Sorafenib (Nexavar), an anti-angiogenic and anti-proliferative drug (starves the tumor by disrupting its blood supply), may be used to treat tumors that cannot be removed with surgery.

2) Describe in brief the molecular biology of breast cancers, particularly in reference to identify a woman at high risk. Describe the presently available medical and surgical options in the risk reduction surgeries. 10+5+5 Answer.

Breast Cancer Risk Factors Established risks: Age: As with many other diseases, your risk of breast cancer goes up as you get older. About two out of three invasive breast cancers are found in women 55 or older. Family History: Women with close relatives who've been diagnosed with breast cancer have a higher risk of developing the disease. If you've had one first-degree female relative (sister, mother, daughter) diagnosed with breast cancer, your risk is doubled. Genetics: About 5% to 10% of breast cancers are thought to be hereditary, caused by abnormal genes passed from parent to child. Personal history of breast cancer: If you've been diagnosed with breast cancer, you're 3 to 4 times more likely to develop a new cancer in the other breast or a different part of the same breast. This risk is different from the risk of the original cancer coming back (called risk of recurrence). Radiation to chest or face before age 30: If you had radiation to the chest to treat another cancer (not breast cancer), such as Hodgkin's disease or non-Hodgkin's lymphoma, you have a higher-than-average longer done), you are at higher risk of developing breast cancer later in life. Certainrisk of breast Breast cancer. Changes: If you If you've had radiation been diagnosed to the face with at certainan adolescen benignt to (not treat cancer) acne (somethingbreast conditions, that’s no you may have a higher risk of breast cancer. There are several types of benign breast conditions that affect breast cancer risk Race/Ethnicity: White women are slightly more likely to develop breast cancer than African American, Hispanic, and Asian women. But African American women are more likely to develop more aggressive, more advanced-stage breast cancer that is diagnosed at a young age. Being Overweight: Overweight and obese women have a higher risk of being diagnosed with breast cancer compared to women who maintain a healthy weight, especially after menopause. Being overweight also can increase the risk of the breast cancer coming back (recurrence) in women who have had the disease. Pregnancy History -term pregnancy or have their first child after age 30 have a higher risk of breast cancer compared to women who gave birth before age 30. Breastfeeding History:: Women Breastfeeding who haven’t can had lower a full breast cancer risk, especially if a woman breastfeeds for longer than 1 year. Menstrual History: Women who started menstruating (having periods) younger than age 12 have a higher risk of breast cancer later in life. The same is true for women who go through menopause when they're older than 55. Using HRT (Hormone Replacement Therapy): Current or recent past users of HRT have a higher risk of being diagnosed with breast cancer. Since 2002 when research linked HRT and risk, the number of women taking HRT has dropped dramatically.

Drinking Alcohol: Research consistently shows that drinking alcoholic beverages -- beer, wine, and liquor -- increases a woman's risk of hormone-receptor-positive breast cancer. Having dense breasts:Research has shown that dense breasts can be 6 times more likely to develop cancer and can make it harder for mammograms to detect breast cancer. Lack of Exercise: Research shows a link between exercising regularly at a moderate or intense level for 4 to 7 hours per week and a lower risk of breast cancer. Smoking: Smoking causes a number of diseases and is linked to a higher risk of breast cancer in younger, premenopausal women. Research also has shown that there may be link between very heavy second-hand smoke exposure and breast cancer risk in postmenopausal women. Emerging risks: Low of Vitamin D Levels: Research suggests that women with low levels of vitamin D have a higher risk of breast cancer. Vitamin D may play a role in controlling normal breast cell growth and may be able to stop breast cancer cells from growing. Light Exposure at Night: The results of several studies suggest that women who work at night -- factory workers, doctors, nurses, and police officers, for example -- have a higher risk of breast cancer compared to women who work during the day. Other research suggests that women who live in areas with high levels of external light at night (street lights, for example) have a higher risk of breast cancer. DES (Diethylstilbestrol) Exposure: Some pregnant women were given DES from the 1940s through the 1960s to prevent miscarriage. Women who took DES themselves have a slightly higher risk of breast cancer. Women who were exposed to DES while their mothers were pregnant with them also may have slightly higher risk of breast cancer later in life. Eating Unhealthy Food: Diet is thought to be at least partly responsible for about 30% to 40% of all cancers. No food or diet can prevent you from getting breast cancer. But some foods can make your body the healthiest it can be, boost your immune system, and help keep your risk for breast cancer as low as possible. Exposure to Chemicals in Cosmetics: Research strongly suggests that at certain exposure levels, some of the chemicals in cosmetics may contribute to the development of cancer in people. Exposure to Chemicals in Food: There's a real concern that pesticides, antibiotics, and hormones used on crops and livestock may cause health problems in people, including an increase in breast cancer risk. There are also concerns about mercury in seafood and industrial chemicals in food and food packaging. Exposure to Chemicals for Lawns and Gardens: Research strongly suggests that at certain exposure levels, some of the chemicals in lawn and garden products may cause cancer in people. But because the products are diverse combinations of chemicals, it's difficult to show a definite cause and effect for any specific chemical. Exposure to Chemicals in Plastic: Research strongly suggests that at certain exposure levels, some of the chemicals in plastic products, such as bisphenol A (BPA), may cause cancer in people. Exposure to Chemicals in Sunscreen: While chemicals can protect us from the sun's harmful ultraviolet rays, research strongly suggests that at certain exposure levels, some of the chemicals in some sunscreen products may cause cancer in people.

Exposure to Chemicals in Water: Research has shown that the water you drink -- home faucet or bottled water from a store -- may not always be as safe as it could be. Everyone has a role in protecting the water supply. There are steps you can take to ensure your waterwhether is as it’s safe from as ityour can be. Exposure to Chemicals When Food Is Grilled/Prepared: Research has shown that women who ate a lot of grilled, barbecued, and smoked meats and very few fruits and vegetables had a higher risk of breast cancer compared to women who didn't eat a lot of grilled meats.

Risk-reducing surgery:  For women without a personal history of cancer who have a BRCA mutation, prophylactic (or preventative) surgery reduces the risk of developing cancer. However, risk-reducing surgery does not completely eliminate the risk of developing cancer as residual risks remain after mastectomy and oophorectomy. While prophylactic surgery is effective in cancer risk reduction, women should be counseled preoperatively about the potential morbidity of such procedures, and the possibility that surgery may affect libido, sexual functioning, and body image.  Oophorectomy in premenopausal women can be associated with increased risks for bone and heart disease, and raises concerns about how to optimally manage surgical menopause and hormone therapy. These procedures are discussed further below.  Mastectomy National Comprehensive Cancer Network and recommend that BRCA carriers be offered prophylactic bilateral mastectomy. However, the decision about whether or not to undergo such surgery— is based on personal preference, given that effective screening is available.  Patients who opt to proceed with mastectomy should undergo a bilateral total mastectomy rather than a subcutaneous mastectomy because the latter leaves behind more glandular tissue that remains at risk for future cancers. However, skin-sparing mastectomy with or without preservation of the nipple-areolar complex followed by immediate breast reconstruction is increasingly being performed as it provides superior cosmetic results.  There has been considerable debate about whether mutation carriers should undergo hysterectomy at the time of BSO. Few high-quality data exist, but the limited information suggests there is no increased risk of endometrial cancers. For example, one study that included over 3000 women with a BRCA mutation found only 17 uterine cancers, and most were attributable to tamoxifen use in whom the ten-year absolute risk was 2.8 percent. While this and other studies have also reported a possible small excess risk of uterine cancers in mutation carriers,the absolute risk is low and it is not clear that the benefits associated with hysterectomy are sufficiently large enough to warrant the risks associated with surgery. Hormone replacement therapy — Pre- and perimenopausal women who undergo BSO will likely experience side effects of surgically induced menopause. Given their relatively young age, there is concern regarding the effect of hormone replacement therapy (HRT) on breast cancer risk. Although data are limited, it has become commonplace to offer carriers HRT from the time of BSO until about age 50, particularly for those undergoing risk-reducing mastectomy. Of note, women should be counseled about the availability of different hormonal medications, doses, and methods of delivery (eg, patch, pill, vaginal cream). It is critical to have at least an initial discussion about these options preoperatively and to be aware that not all women require or desire hormonal therapy to manage menopausal symptoms. Alternatives to BSO? The only proven risk-reducing procedure for ovarian cancer in BRCA mutation carriers is BSO. However, there is controversy about whether it is appropriate to perform a salpingectomy alone —for BRCA mutation carriers who wish to defer oophorectomy, based upon a possible fallopian tube origin for some ovarian cancers. The Society of Gynecology (SGO) Clinical Practice Statement opens with the statement: "Salpingectomy may be appropriate and feasible as a strategy for ovarian risk reduction"/However, the statement and a lengthier explication make clear that this procedure does not eliminate the risk of ovarian cancer, and it does not reduce the risk of breast cancer. Guidelines from the National Comprehensive Cancer Network thus indicate that "salpingectomy alone is not the standard of care and is discouraged outside a clinical trial". Cancer surveillance For patients with a BRCA mutation who do not wish to pursue (or would rather delay) surgical risk reduction, breast cancer surveillance should be offered, and ovarian cancer screening may be performed.— While breast and ovarian (including fallopian tube and peritoneal) cancers present the greatest risk, individuals with BRCA mutations have elevated risks for other cancers, including prostate cancer and . Breast examinations o Beginning at age 18, self breast exams performed periodically may facilitate awareness of changes, and clinical— breast examination should be performed every 6 to 12 months beginning at age 25. o Annual mammography Mammography should begin at age 30 or be individualized if the earliest age of onset in the family is under age 25. However, the sensitivity of mammography for detecting breast cancer —in mutation carriers appears to be lower than in other high-risk women, which may be due to:  Higher breast density.  Differences in morphologic features (eg, less spiculation due to lack of tumor- surrounding fibrosis).  The frequent development of interval . While the risk of radiation-associated breast cancer from breast imaging for the average-risk patient is believed to be small or nonexistent, women at high genetic risk may be more susceptible to radiation- induced carcinogenesis because of the role of BRCA proteins in DNA repair. Studies almost exclusively performed in women with a BRCA mutation differ on the effect of diagnostic radiation, but growing evidence suggests the effect depends on the age when exposure occurs and the total dose. A history of having a mammogram before age 30 in BRCA mutation carriers was associated with a statistically insignificant increased risk of breast cancer. •Increased risk of breast cancer among women in this study occurred at radiation dose levels considerably lower than that among women without BRCA. •There was no association between breast cancer risk and radiation exposure from ages 30 to 39. Although these data are provocative, further research is needed, and current recommendations by most groups• support screening these women with a combination of mammography and MRI scanning rather than breast MRI exclusively. In contrast, in a report based upon a questionnaire sent to known BRCA mutation carriers, exposure to mammographic screening (mean age at first screening mammogram 35 years of age) was not associated with an increased risk of breast cancer when adjusted for parity, oral contraceptive use, family history, and ethnicity. Breast magnetic resonance imaging MRI for is recommended annually beginning at age 25 and can be scheduled six months after annual mammogram. — The addition of breast MRI to the breast cancer surveillance strategy in high-risk women increases breast cancer detection rates, increases the number of patients diagnosed at an earlier stage of disease, is cost-effective , and is supported in multiple guidelines. However, the mortality impact of including breast MRI in the surveillance strategy is not clear. ●A systematic review of 11 studies compared test performance of screening MRI with mammography in high-risk women and reported that MRI: Was significantly more sensitive than mammogram Was significantly less specific. •Was both more sensitive and specific when used alongside mammography •Despite these results, the impact of MRI screening on breast cancer mortality has not been established. •Breast ultrasound Data from large studies evaluating the role of breast ultrasound in addition to other imaging modalities have not demonstrated any additional benefit of this procedure. Chemoprevention — Chemopreventive strategies to reduce the risk of breast cancer have focused exclusively on prevention in high-risk women and involve the use of selective estrogen receptor modulators (SERMs)— and aromatase inhibitors for breast cancer prevention. There is also a significant amount of data on the role of oral contraceptives in reducing the risk of hereditary ovarian cancer. ● Use of tamoxifen for risk reduction for women who opt against mastectomies, especially if they are BRCA2 mutation carriers. However, the option for prophylactic mastectomy should be rediscussed periodically with patients, as medical chemoprevention is less effective than prophylactic mastectomy. ●The use of oral contraceptives in women with a known BRCA mutation is not contraindicated. However, while it may reduce the risk of ovarian cancer, it may increase the risk of breast cancer. Tamoxifen Only limited data are available regarding the preventive benefit of tamoxifen in BRCA mutation carriers. Evidence for the benefit of tamoxifen in women who have never had a diagnosis of breast cancer— comes from a subset analysis of the National Surgical Adjuvant Breast and Bowel Project (NSABP) Breast Cancer Prevention trial (P-1 trial). Tamoxifen reduced breast cancer risk by 62 percent in BRCA2 carriers (relative risk [RR] 0.38, 95% CI 0.06-1.56), but not in BRCA1 carriers (RR 1.67, 95% CI 0.32-10.07). However, this analysis is limited by the small number of mutation carriers (of the 288 women in the study who developed breast cancer, only eight had BRCA1 mutations and 11 had BRCA2 mutations). There are no data addressing the preventive benefit of raloxifene or an aromatase inhibitor (AI) in patients with BRCA mutations. However, in large chemoprevention studies of postmenopausal women at increased risk for breast cancer, both raloxifene and AIs have been demonstrated to significantly reduce the risk of breast cancer.

A differential effect of tamoxifen in BRCA2 as compared with BRCA1 mutation carriers may be attributed to estrogen receptor (ER) status of BRCA1 and BRCA2-associated tumors. Tamoxifen might be expected to have an impact only against ER-positive tumors, and BRCA2-associated tumors have a greater likelihood than BRCA1-associated tumors of being ER-positive. Indirect evidence of a beneficial effect of tamoxifen for both BRCA1 and BRCA2-associated tumors, irrespective of ER-status, comes from data regarding its impact on risk of contralateral breast cancer. The main benefit of tamoxifen is in reducing hormone receptor-positive breast cancer. Whether tamoxifen is effective in reducing recurrence in BRCA mutation carriers with hormone receptor- negative breast cancers is unclear.  For BRCA mutation carriers with breast cancer who do not opt for contralateral mastectomy, surveillance with clinical breast examination, mammography, and magnetic resonance imaging (MRI) are recommended, as outlined above.  Selection of agents The available data regarding the impact of BRCA status on breast cancer- related prognosis are inconclusive, but the majority of studies suggest that mutation status is not an independent prognostic— factor. Thus, mutation status generally does not factor into the decision- making process regarding systemic therapy. However, there is accumulating evidence of altered sensitivity of systemic agents in BRCA-related breast cancer, including to platinums and poly ADP- ribose polymerase (PARP) inhibitors. Platinum agents There are emerging data that BRCA-associated breast cancers are more sensitive to platinum agents (eg, cisplatin and carboplatin) than other breast cancer subtypes. However, larger trials are needed— before they can be adopted as preferred agents for these patients. ●A phase III trial randomly assigned 376 patients with metastatic breast cancer with either germline BRCA mutations or triple-negative breast in the first-line setting to treatment with either carboplatin or docetaxel. PARP inhibitors PARP inhibitors block the repair of DNA single-strand breaks, and for tumors associated with BRCA mutations, they result in cell death due to inefficiencies in cell repair mechanisms. Olaparib— is the first of this class to be approved for use in the United States. While specifically approved for the treatment of ovarian cancer in BRCA carriers, it holds great promise in the treatment of breast cancer associated with BRCA mutations. Cancer surveillance In general, recommendations are as follow: ●Beginning at age 18, self breast exams performed periodically ●Annual screening mammogram— beginning at age 30 ●Annual breast magnetic resonance imaging (MRI) beginning at age 30 ●Clinical breast exam every 6 to 12 months beginning at age 30. Cancer Screening Programme and modify the above recommendations as follows: ●Women should undergo annual MRI breast screening beginning at age 25 years. ●Mammography should be performed every 18 months between ages 40 and 49 years, and then every three years starting at age 50. Risk reduction Decisions about chemoprevention and risk-reducing mastectomy and/or salpingo- oophorectomy should be highly individualized based upon the woman's mutation status as well as personal and family— history. For example: ●Women with a 20 percent or higher lifetime risk of developing breast cancer may consider risk- reducing bilateral mastectomy. ●Women with CHEK2 mutations are more likely to develop estrogen receptor-positive breast cancers and thus may be good candidates for chemoprevention with tamoxifen or related agents, although no data are available regarding efficacy specifically in this group of mutation carriers. ● Women without a personal history of cancer The options for women who test negative or have an uninformative result on genetic testing depend on their family history and their risk of developing breast and other possible cancers. — Women at high risk Women deemed to be at high lifetime risk of breast cancer (defined as a risk of at least 20 percent) should undergo annual screening mammogram, annual breast magnetic resonance imaging (MRI), and clinical— breast exam every 6 to 12 months beginning at age 30. However, these screening guidelines should be considered in the context of each woman's personal and family history, and may be modified accordingly. While chemoprevention has not been well studied in women with uninformative negative genetic testing results, we approach these discussions based on the data obtained from trials of these agents in the general population. Women with a personal history of breast cancer Like women with BRCA mutations, women with uninformative results who are clinically felt to be at high risk of second breast cancers may still be candidates for breast conservation; however, such— treatment does not address the risk of contralateral breast cancer (CBC). These women should be individually counseled regarding options for both the primary treatment of breast cancer and the potential role of contralateral prophylactic mastectomy. For some breast cancer survivors with a family history of breast cancer, their lifetime risk of developing a CBC may make them good candidates for heightened surveillance with breast MRI in addition to mammography. Although a panel convened by the American Cancer Society concluded that there was insufficient evidence to recommend for or against MRI in breast cancer survivors, when considering an competing mortality risk, MRI may be beneficial and is now a reasonable option for screening. Recommendations:individual woman’s lifetime CBC risk in addition to other factors related to her treatment and ●Women with inherited mutations in breast cancer type 1 and 2 susceptibility genes (BRCA1 and BRCA2) have markedly elevated risks of breast cancer and ovarian cancer. Men with BRCA mutations have increased risk for breast and prostate cancer. ●For women with a known BRCA mutation, risk-reducing mastectomy is a highly effective strategy for breast cancer risk reduction. For BRCA carriers diagnosed with advanced ovarian cancer, we prefer not proceeding with risk-reducing mastectomies until at least five years after the ovarian cancer diagnosis, as survival is dominated by the mortality rate of ovarian cancer. ●For women with a BRCA mutation, risk-reducing salpingo-oophorectomy (BSO) is indicated by age 35 to 40 and when childbearing is completed. BSO is also indicated for carriers who are diagnosed with early-stage breast cancer. ●Women with a BRCA mutation who do not opt for bilateral mastectomy should undergo breast cancer surveillance with annual mammography and magnetic resonance imaging (MRI). ●For women with a BRCA mutation who do not opt for risk-reducing mastectomy, chemoprevention with tamoxifen, raloxifene, or an aromatase inhibitor is appropriate. ●For women who carry mutations in genes other than BRCA that confer moderate risk for breast cancer (eg, PALB2, CHEK2, and ATM gene mutations), the options for breast cancer prevention are similar. ●Oral contraceptive use in BRCA mutation carriers decreases the risk of ovarian cancer and does not appear to increase risk of breast cancer. ●For women with negative or uninformative test results, quantitative models can help identify women with a high lifetime risk of breast cancer of at least 20 percent. These women are candidates for breast cancer screening with MRI in addition to mammography. ●Women with uninformative negative BRCA results without a family history of ovarian cancer do not appear to be at increased risk of developing ovarian cancer. Thus, BSO is not indicated for ovarian cancer risk reduction.

3) Write short notes of the following: 5x6

a) Gastrointestinal hormones. b) Parathyroid localisation. c) Distributive shock. d) Borderline resectable pancreatic tumours. e) Pharmacotherapy of Peripheral Arterial Disease.

Answer. a) Gastrointestinal hormones. Cholecystokinin is produced by duodenal and jejunal I cells and enteric nerves in response to intraluminal amino acids and fats. CCK induces contraction, pancreatic enzyme secretion, and relaxation of the of Oddi. Enteroglucagon from ileal and colonic L cells is produced in response to intraluminal fat and bile acids. Of note, inflammatory processes, such as Crohn disease and celiac sprue, can dramatically increase enteroglucagon secretion. Gastric inhibitory peptide (GIP), secreted by duodenal and jejunal K cells in response to active transport of monosaccharides, long-chain fatty acids, and amino acids, inhibits gastric acid and pepsinogen secretion and gastric emptying but stimulates insulin release. Duodenal G cells secrete gastrin in response to vagal stimulation and intraluminal peptides. Gastrin stimulates acid secretion by the gastric fundus and body and increases gastric mucosal blood flow. Motilin is produced by duodenal and jejunal M cells in response to duodenal acid, vagal stimulation, and gastrin-releasing peptide. Motilin initiates phase III of the MMC during the fasting state. Erythromycin is useful as a promotility agent due to its action as a motilin agonist. Duodenal and jejunal S cells release secretin in response to acid, bile salts, and fatty acids in the duodenum. Secretin increases bicarbonate and water secretion from pancreatic ducts. It inhibits gastric acid secretion and gastric motility. Somatostatin broadly inhibits gut exocrine and endocrine function. Somatostatin and its analog, octreotide, are often used to decrease the volume of intestinal secretions in patients with enterocutaneous fistulas. Intestinal D cells and enteric neurons secrete somatostatin in response to intraluminal fat, protein, and acid. Vasoactive intestinal peptide (VIP) is secreted throughout the small intestine in response to vagal stimulation. VIP increases mesenteric blood flow, intestinal motility, and pancreatic and intestinal secretions.

Parathyroid localisation. Answer.  No consensus on whether preoperative localization necessary  Preoperative localization can allow for unilateral focused parathyroidectomy. o The combination often used is:  Sesatmibi for localization  Ultrasound for information on size and relationship of the abnormal to surrounding tissue  Sestamibi scanning limited in identifying multiple and 4 gland hyperplasia  Preoperative localization essential in reoperation cases.

Preoperative Evaluation: o Neck ultrasound o MRI o Thallium-technetium dual isotope scintigraphy. o Technetium-99m sestamibi scan. o SPECT sestamibi scan: allows for 3-D localization but is expensive o Angiography w/ or w/o selective venous sampling (Angioablation)

Intra operative localization: o Intraoperative ultrasound o Intraoperative rapid PTH (50%, 80%) o Hand-held gamma probe b) Distributive shock. Answer. Distributive Shock as a result of severely diminished systemic vascular resistance. Distributive shock is a medical condition in which abnormal distribution of blood flow in the smallest blood vessels results in inadequate supply of blood to the body's tissues and organs. Septic: secondary to an overwhelming infection Anaphylactic: secondary to an overwhelming infection Neurogenic: secondary to a sudden loss of the autonomic nervous system function. Pathophysiology: The cause of inadequate tissue perfusion (blood delivery to tissues) in distributive shock is a lack of normal responsiveness of blood vessels to vasoconstrictive agents and direct vasodilation. There are four types of distributive shock. The most common, septic shock, is caused by an infection, most frequently by bacteria, but viruses, fungi and parasites have been implicated. Infection sites most likely to lead to septic shock are chest, abdomen and genitourinary tract. In septic shock the blood flow in the microvasculature is abnormal with some capillaries under perfused and others with normal to high blood flow. The endothelial cells lining the blood vessels become less responsive to vasocontrictive agents, lose their glycocalyx (normal coating) and negative ionic charge, become leaky and cause extensive over- expression of nitric oxide. The coagulation cascade is also disrupted. Tissue factor that initiates the clotting cascade is produced by activated monocytes and the endothelial cells lining the blood vessels while antithrombin and fibrinolysis are impaired. Disseminated intravascular coagulation (DIC) can result from the thrombin produced in the inflammatory response. The ability of red blood cells to change shape decreases and their tendency to clump together increases, inhibiting their flow through the microvasculature. In anaphylactic shock low blood pressure is related to decreased systemic vascular resistance (SVR) triggered primarily by a massive release of histamine by mast cells activated by antigen-bound immunoglobulin E and also by increased production and release of prostaglandins. Neurogenic shock is caused by the loss of vascular tone normally supported by the sympathetic nervous system due to injury to the central nervous system especially spinal cord injury. Rupture of a hollow organ, with subsequent evacuation of contents in the peritoneal cavity could also determine neurogenic shock, a subtype of distributive shock. This happens due to the widespread peritoneal irritation by the ruptured viscus contents, as in peptic ulcer perforation, with consequent strong vagal activation, and generalized, extensive peripheral vasodilatation and bradicardia. Distrib utive shock associated with adrenal crisis results from inadequate steroid hormones. Elbers and Ince have identified five classes of abnormal microcirculatory flow in distributive shock using side stream dark field microscopy. Class I: All capillaries are stagnant when there is normal or sluggish venular flow. Class II: There are empty capillaries next to capillaries that have flowing red blood cells. Class III: There stagnant capillaries next to capillaries with normal blood flow. Class IV: Hyperdynamic flow in capillaries adjacent to capillaries that are stagnant. Class V: Widespread hyperdynamic flow in the microcirculatory system. Treatment: The main goals of treatment in distributive shock are to reverse the underlying cause and achieve hemodynamic stabilization. Immediate treatment involves fluid resuscitation and the use of vasoactive drugs, both vasopressors and inotropes. Hydrocortisone is used for patients whose hypotension does not respond to fluid resuscitation and vasopressors.Opening and keeping open the microcirculation is a consideration in the treatment of distributive shock, as a result limiting the use of vasopressors has been suggested. Control of inflammation, vascular function and coagulation to correct pathological differences in blood flow and microvascular shunting has been pointed to as a potentially important adjunct goal in the treatment of distributive shock. Patients with septic shock are treated with antimicrobial drugs to treat the causative infection. Some sources of infection require surgical intervention including necrotizing fasciitis, cholangitis, abscess, intestinal ischemia, or infected medical devices. Anaphylactic shock is treated with epinephrine. c) Borderline resectable pancreatic tumours. See the answer of Question no. 1 of Paper –IV of 2015 d) Pharmacotherapy of Peripheral Arterial Disease. Answer. While lifestyle changes may be enough treatment for some people with peripheral artery disease, others may require medication. Examples of medications used to treat peripheral artery disease include antiplatelet or anticlotting agents, cholesterol-lowering drugs such as statins, medications that increase blood supply to the extremities such as cilostazol and pentoxifylline (Trental), and medications that control high blood pressure. Antiplatelet medications (such as aspirin and clopidogrel) make the blood platelets less likely to stick to one another to form blood clots. Low dose aspirin (81 to 325 mg/day) is usually prescribed indefinitely because it is also helpful in preventing strokes and heart attacks in patients with peripheral artery disease. Clopidogrel (Plavix) is an alternative to aspirin for those who are allergic or cannot tolerate aspirin. Antiplatelet medications also help prevent occlusion of blood vessels after angioplasty or bypass surgery. Anticoagulant medications act to prevent blood clotting. Both heparin and warfarin (Coumadin) are anticoagulant medications. Anticoagulants are sometimes prescribed for people with peripheral artery disease if they are at increased risk for formation of blood clots; these agents are used much less frequently than anti platelet agents in patients with peripheral artery disease. Cholesterol-lowering drugs of the statin family have been shown in numerous large clinical trials to help prevent heart attacks and strokes and prolong survival among patients with atherosclerosis. Statins have also been shown to slow the progression of peripheral artery disease, decrease arthrosclerosis in the arteries, and improve claudication symptoms. Cilostazol (Pletal) is a medication that can help increase physical activity (enabling one to walk a greater distance without the pain of claudication). Cilostazol works by causing dilation of the arteries and an increased supply of oxygenated blood to be delivered to the arms and legs. Cilostazol is recommended for some patients with claudication when lifestyle modifications and exercise are ineffective. Cilostazol should be taken on an empty stomach either a half an hour before or two hours after meals. High fat meals, grapefruit juice, and certain medications such as omeprazole (Prilosec) and diltiazem (Cardizem) can increase the absorption, and hence, the blood levels of cilostazol. Side effects are generally mild that includeheadache, diarrhea, and dizziness. Cilostazol should not be used in patients with heart failure because of concern over increased mortality in heart failure patients using medications similar to cilostazol. Pentoxifylline (Pentoxyl, Trental) improves blood flow to the extremities by decreasing the viscosity ("stickiness") of blood, enabling more efficient blood flow. Side effects are fewer than with cilostazol, but its benefits are weaker and have not been conclusively proven by all studies. Drugs to control hypertension may also be prescribed. Current recommendations are to treat hypertension in patients with peripheral artery disease to prevent strokes and heart attacks.

4) Answer briefly on the following. 4x71/2

a) Pathogenesis of different organ dysfunction in abdominal compartment syndrome. b) Premalignant conditions of skin. c) Pathogenesis of sepsis induced hypotension d) Immune-nutrition in surgical patients.

Answer. a) Pathogenesis of different organ dysfunction in abdominal compartment syndrome.

Effect on different organ: Cardiovascular A rise in the intra-abdominal pressure leads to a fall in cardiac output, due mainly to compression of the inferior vena cava and reduction in venous return to the heart. Cardiac output is reduced despite apparent rises in central venous pressure, pulmonary artery occlusion pressure and systemic vascular resistance. Respiratory Raised intra-abdominal pressure will effectively splint the diaphragm and lead to a rise in peak airway pressure and intra-thoracic pressure and subsequently a reduced venous return to the heart. The increase in airway pressures may also exacerbate barotrauma and contribute to the development of acute respiratory distress syndrome. Renal An acute increase in intra-abdominal pressure leads to oliguria and anuria probably due to compression of the renal vein and renal parenchyma. Renal blood flow, glomerular filtration are decreased with a corresponding increase in renal vascular resistance. Cerebral The rise in intra-abdominal pressure, intrathoracic pressure leads to a rise in central venous pressure which prevents adequate venous drainage from the brain, leading to a rise in intracranial pressure and worsening of intracerebral oedema. b) Premalignant conditions of skin. Answer. Actinic Keratosis Arsenical Keratosis Chronic Radiation Keratosis Disease Erythroplasia Of Queyrat Bowen’s Erythroplakia Leukoplakia Actinic keratosis: o Age >60 - 80% chnces of development o M>F o Phenotype of fair skin which burn & freckles eaisly and rerely tan o Blue or light coloured eyes & blond hair o Immunosuppression o Genetic syndromes - Xeroderma pigmentosum & albinism Pathogenesis: o Sunlight exposure o UV-induced mutation in o Tumor-suppressor gene p53 Clinical feature: o In elderly patient 80% of lesions found on chronically sun exposed sites head,neck,forarms & dorsum of hand o Erythematous, flat,scaly,yellow coloured papules – o Hypertrophic - cutaneous horn o Actinic cheilites Bowen’s Disease: o Squamous cell carcinoma in situ o Affects both skin & mucous membranes -having potential to progress into invasive carcinoma o Age >60 rarely before 30 years of age o Can occur at any body parts sun or non sun exposed areas of body o Sun exposure, o Arsenic exposure – o Ionizing radiation, o Immunosuppression o Infection with HPV-

Clinical features: 16 specially anogenital bowen’s disease. o Discrete o Slowly enlarging o Pink to erythematous o Thin plaque with well demarcated,irregular borders o Over lining scales or crust o Hyperkeratotic verrucous lesions o 5% of BD progress to invasive SCC.

Treatment:

o Surgical excision o Cryosurgery o Curettage o 5 fu topical chemotherapy o Imiquimod 5% cream o Laser o Photo dynamic therapy

Erythroplasia of Queyrat: o EQ- is carcinoma in situ affecting the mucosal surfaces of penis in uncircumcised males o Age 20 to 80 years o Uncircumcised o Poor hygiene o Smegma o HSV infection o HPV-16 & 18 infection. Clinical features: o Glistening o Red o Velvety plaqu on glans penis,prepuce or urethra o Usually solitary plaque o Localised pain or pruritus o Difficulty in retracting fore o Bleeding or crusting may be there at the lesion o Enlarge slowly & persist for several years o 33% of cases progress to invasive SCC.

Leukoplakia: o It is fixed prediminantly white lesion of mucosa o Oral & anogenital mucosal surfaces o Alcohol & tobacco use o Age >50 to 70 years o 5 to 25% risk of becoming invasive o Clinically o Asymptomatic o Asymmetric o White plaque o At floor of mouth o Lateral & venetral tounge o Sofe plate o Diagnosis by biopsy o Surgical excision of the lesion c) Pathogenesis of sepsis induced hypotension. Answer. Hypotension is a serious sign in sepsis. In general, hypotension can be caused by a decrease in blood volume, a decrease in vascular tone, or a decrease in cardiac output; the hypotension of sepsis can be caused by reductions in all three parameters. Initially, sepsis usually reduces blood volume by increasing capillary leakage, so the administration of fluids is an early priority during treatment. In sepsis, the blood volume is not only reduced, it is redistributed ineffectively. Fluid resuscitation will usually refill the under-perfused arteries. In septic shock, poor vascular tone has been added to the patient's other systemic problems. In shock, the administration of large amounts of fluids will not succeed in restoring normal blood pressure. d) Immune-nutrition in surgical patients. Answer.

Introduction: Oral or enteral dietary supplementation with arginine, omega 3 fatty acids and nucleotides (known as immunonutrition) significantly improve outcomes in patients undergoing elective surgery. The objective of the study was to determine the impact on hospital costs of immunonutrition formulas used in patients undergoing elective surgery for gastrointestinal cancer.

Major ingredients: glutamine, L-arginine, nucleotides, and omega-3 fatty acids. Few human studies have investigated individual enteral components and outcome.

Glutamine: Glutamine is a conditionally essential amino acid and the preferred fuel for rapidly replicating cells such as gastrointestinal mucosal cells (enterocytes and colonocytes) and immune cells (lymphocytes and macrophages). It is a precursor for nucleotide synthesis, a substrate for hepatic gluconeogenesis, a precursor of the antioxidant glutathione, and essential to the renal handling of ammonia. Patients under catabolic stress are at risk for glutamine deficiency as a result of their increased metabolic demand for this amino acid, inadequate nutritional intake, and the relative instability of this amino acid in standard protein solutions. An association between high-dose glutamine and decreased mortality has been identified. Further prospective, randomized trials are necessary to recommend widespread utilization of glutamine in nutritional support. Arginine: L-arginine, a dibasic amino acid, has numerous important roles in the transport, storage, and excretion of nitrogen; formation of nitric oxide; mediation of macrophage function after injury; and regulation of wound healing. Meta-analyses of arginine-containing enteral nutrition formulas have demonstrated a decreased incidence of infectious complications, a shorter hospital length of stay, and fewer ventilator days. Such differences were most marked in the surgical patient subpopulation and less so in the critically ill. Further prospective, randomized trials are necessary to recommend widespread utilization of arginine in nutritional support. Nucleotides: Nucleotides serve as the building blocks of deoxyribonucleic acid (DNA) and ribonucleic acid (RNA). Nucleotides are required for catalysis, energy transfer, and coordination of hormonal signals. In the absence of an adequate nucleotide supply, replication of rapidly growing cells (such as gastrointestinal mucosa and lymphocytes or macrophages) is down regulated. Prospective, randomized trials evaluating the impact of isolated nucleotide preparations on patient outcome do not currently exist. Omega-3 fatty acids: Of the long chain fatty acids, the omega-3 polyunsaturated fatty acids appear to have the greatest potential for clinical benefit in modulating immune response and improving patient outcome. These fatty acids are precursors of the prostacyclins, thromboxanes, prostaglandins, lipoxins, and leukotrienes. Unlike glutamine, L-arginine, and nucleotides, the omega-3 fatty acids do not directly stimulate the immune system, but compete with arachidonic acid (an omega-6 fatty acid) for cyclo- oxygenase metabolism.The ratio of omega-3 to omega-6 fatty acids appears to be important to optimizing immune function. The omega-3 family of fatty acids has been shown to reduce the development of atherosclerosis and incidence of myocardial infarction, reduce hypertension, and improve outcome from various proinflammatory states.

THE WEST BENGAL UNIVERSITY OF HEALTH SCIENCES

MS (General Surgery) Examination, 2013

PAPER I

Time Allowed: 3 Hours Full Marks: 100 Attempt all questions

1. Give the detailed anatomy of pararectal endopelvic fascia with the related autonomic nerves in reference to proper execution of TME. Describe the different local excision procedures for early rectal cancers. 10+10 2. Describe the pathophysiological aspect of pneumoperitoneum during laparoscopic surgery. Give a brief account of the energy sources of Endoscopic and Endoluminal surgery. 10+10

3. Write short notes of the following: 5x6

a) Development and descent of testis. b) Systemic inflammatory response syndrome. c) Complication of parenteral nutrition. d) Metabolic changes in pyloric obstruction. e) Diagnosis and management of DVT.

4. Answer briefly on the following. 4x71/2

a) Post splenectomy problems. b) Anal incontinence. c) Tumours of Endocrine . d) Silent Gall Stones.

THE WEST BENGAL UNIVERSITY OF HEALTH SCIENCES

MS (General Surgery) Examination, 2013

April 2013

PAPER I

Time Allowed: 3 Hours Full Marks: 100 Attempt all questions

1. Give the detailed anatomy of pararectal endopelvic fascia with the related autonomic nerves in reference to proper execution of TME. Describe the different local excision procedures for early rectal cancers. 10+10

Answer.

Endopelvic fascia - meshwork of , ligaments, blood vessels and lying between the parietal and visceral fascia, sometimes condensing to form fibrous fascial septa which separate and suspend the organs.

 Anatomists use the name subserous fascia whereas surgeons refer to this layer of retroperitoneal fascia as endopelvic fascia.

 Hypogastric sheath - separates retropubic space from presacral space; conduit for vessels and nerves  Transverse cerical (cardinal) ligaments - part of hypogastric sheath; runs from lateral pelvic wall to uterine cervix and vagina; transmits uterine artery and provides passive support for the uterus  Vesicovaginal septum  Rectovesical septum  Rectovaginal septum

Endopelvic Fascia - Ligaments

 Female: Pubovesical ligaments - attach bladder to pubic symphysis  Male: Puboprostatic ligaments - attach bladder to pubic symphysis  Sacrogenital - these are thickenings of the fascia, not specific ligaments  Uterosacral ligaments - attach upper vagina, upper portion of cervix and uterus to the 3rd sacral vertebra  Cardinal ligaments (aka Mackenrodt's) - attach upper vagina, cervix and uterus to the side walls of the pelvis  Round ligaments - attach uterus to mons pubis  Broad ligaments - attach uterus to medial aspect of the ilium  Uracus ligament - attaches bladder to umbilicus (formed from the remnants of the umbilican vein)

The bladder and genital tract are supplied by autonomic nerves that are in close relationship with the and for this reason are at risk during rectal mobilization. The sympathetic nerves (thoracic splanchnic nerves) arises from both sides of the thoracic spinal cord (T5-T12), pierce the diaphragmatic crura, flow down in front of the aorta and the root of the mesentery and enter the pelvis in front of the promontory of the sacrum as the hypogastric nerves. These pre-aortic nerves turn just beyond the aortic bifurcation into the superior hypogastric pelvic plexus of nerves. The sympathetic nerves are strengthened by the lumbar and sacral sympathetic nerves as they descend down the abdomen into the pelvis. They divide into two trunks, which pass to form the pelvic plexus on the side wall of the pelvis (the inferior hypogastric plexus) to be distributed to the pelvic organs by branches passing with the blood vessels or with the parasympathetic nervi erigentes (pelvic splanchnic nerves) arising from the sacral segments (S2, S3, S4 ) of the spinal cord. The parasympathetic supply are more difficult to see as it is situated deep in the pelvis and distributed to thepelvic organs in the endopelvic fascia on the surface of the pelvic floor.The autonomic fibres penetrate levator ani and the perineal membrane to supply the deep and superficial perineal spaces, and the parasympathetic fibres have the independent function of stimulating erection of the penis/clitoris. Although TME has been modified over time, the basic principle of excising tumor and the mesorectum en bloc remains its foundation. This principle is based on the original observations of Moynihan19 in 1908 regarding potential pathways for lymphatic spread and also on the hypothesis of Heald that the mesorectum represents embryological advantages conferring pr tection against tumor dissemination until the terminal stages. Lymphoscintigraphy further demonstrated this in an anatomical study of the lymphatics that drain the rectum. . Extent and technique of TME: The sigmoidal vessels are isolated. There is a clear space on either side of the vessels. Care is taken to preserve the hypogastric nerves. The sigmoid is divided. The point of vessel ligation is at the level of the left colic artery. The colon is divided and packed cephalad, out of the field. The left colon is mobilized by incising the white line of Toldt and freeing the splenic flexure attachments. . TME begins by pulling the rectum up and incising the on both sides down to meet anteriorly at the deepest point in the cul de sac. By retracting the rectum anteriorly, a loose areolar plane of avascular tissue is visible. Dissection in this plane prevents entry into presacral veins. The nerves are visible and kept posterior to the plane of dissection. Both ureters should be visualized and retracted laterally. The tumor should be removed en bloc to include organs directly involved with tumor extension, such as the ovaries, uterus, bladder wall, and vagina. The presacral fascia is incised down to the rectosacral (Waldeyer's) fascia, a thickened band attaching the rectum to the endopelvic fascia at the S4 level. Divide this fascia and continue dissection to the coccyx. . Continue the dissection laterally down to the lateral ligaments or "stalks," which may contain the middle rectal vessels. In fact, only 25% of patients have distinct branches of the middle rectal vessels in these ligaments. The lateral dissection ends at the levator muscles, the caudad boundary of the pelvic cavity. . The anterior dissection is the most difficult because the planes are less distinct and the fat of the mesorectum is thin. This dissection is continued parallel to the plane of Denonvilliers' fascia between the rectum and the posterior wall of the vagina or the seminal vesicles. These structures are elevated anteriorly. Small vessels that traverse this area have a propensity for nuisance bleeding. The entire mesorectum should be contained within the fascia propria of the extraperitoneal rectum.

 The point of transection is chosen distal to the tumor. Multiple studies comparing distal cancer spread have shown that a 2-cm margin is necessary for low rectal excisions to perform an adequate oncologic resection.  Less than 2% to 4% of tumors will have intraluminal implantation beyond 2 cm distally. Rigid may be used to identify a point at least 2 cm distal to the carcinoma.  Depending on the depth of the pelvis and therefore the diameter of the pelvis, a linear stapler of 30, 45, or 60 mm length is used for the first staple line in the double-stapling technique.  The bowel is clamped just proximal to this point, the stapler is fired, and the rectum sharply transected and handed off the field. The operating field is secondarily isolated with towels while the open colon is manipulated.  The proximal end of the colon is prepared by resecting off residual fat where the anastomosis is to be constructed. The staple line is sharply excised.  A purse-string suture of 2-0 or 3-0 polypropylene is placed using full-thickness bites at 2-mm intervals.  The anvil of the selected circular stapling device is inserted and the purse-string suture is tied around the shaft.  The stapler is inserted transanally up to the previous staple line where the rectum was cut across.  The trocar is introduced through or adjacent to the staple line. The anvil and the stapler are joined.  The colon is inspected to verify that no adjacent tissue is entrapped and that the bowel is not twisted. The stapler is fired, opened slightly, and then removed. This is the second staple line in the double-stapling technique.  The tissue removed in the stapler is checked to see that the tissue from the proximal and distal bowel is intact in two rings, the so-called "doughnuts."  Check the anastomosis by clamping the bowel proximally, filling the pelvis with saline, and insufflating air under direct vision with a rigid proctoscope to see if bubbles can be detected coming through the anastomosis from the abdominal view.  The rigid proctoscope permits assessment of the anastomosis for bleeding or saline entering the lumen from the abdomen. o For the low coloanal anastomosis and for the preoperatively radiated pelvis, we routinely perform a diverting loop at a premarked site. o The site is kept away from bony prominences, the umbilicus, scars, and creases and is determined with the patient sitting, standing, and lying supine. o This can usually be closed within 12 weeks of surgery. o A 2 cm in diameter skin opening is made; dissection is carried down to the anterior rectus sheath. o The fascial sheath is opened in a cruciate fashion, the muscle split bluntly, and the posterior peritoneum opened. o The ileum is brought through the rectus muscle, identifying which end is proximal. The ostomy is matured following skin closure and dressing placement. o The ileum is incised transversely 1 cm above the skin on the distal limb of the loop. o As these cut edges of the ileum are sewn to the skin, the proximal ileum is everted to create an elevated tip.

 Macroscopic examination of the mesorectal surface helps us evaluate the quality of the surgical specimen. Mesorectal defects are classified into three categories: (a) Complete: mesorectum is intact, smooth with only minor irregularities without defect > 5 mm; (b) Moderate: moderate bulk to mesorectum but irregularity of the mesorectal surface; muscularis propria is not visible with the exception of the area of insertion of levator muscles, and (c) Incomplete: little bulk to mesorectum with defects down into muscularis propria.

Circumferential Resection Margin:  After the introduction of TME, the CRM on the nonperitonealized surface of the resected rectal specimen isone of the most important predictors of local recurrence. Quirke et al.were the first to show that the radial or CRM is an important factor for local recurrence. The CRM can be involved by the tumor in various ways, such as direct spread, by tumor emboli in veins, lymphatics or lymph nodes or by tumor deposits discontinuous from the main growth.  The rectum is covered anteriorly by peritoneum and only in the area below the peritoneal reflection is there a risk of the CRM being involved. The posterior area, which continues up to the sigmoid mesocolon, is at risk of being involved by the tumor not only by direct spread but also with metastatic deposits in lymph nodes. The mesorectum is thinner anteriorly than posteriorly and the risk of a positive resection margin is higher for anteriorly located tumors than in posterior ones.  Microscopic tumor nodules can be observed in the mesorectum. Studies have shown that the presence of widespread microscopic nodules in the mesorectum was present in up to 38.7%.  Other studies proved that lymph nodes <5 mm in diameter are more often involved by the tumor than bigger ones. Such findings pinpoint the importance of TME.  The pathologist must differentiate the peritonealized from nonIf a tumor is detected on the non- peritonealized surface, adjuvant radiotherapy may be appropriate without the dimensions of the tumor to be considered. o The macroscopic CRM is measured with a ruler; the microscopic CRM measurement is done better by using a sheet of graph paper that is photocopied onto a sheet of acetate and cut to size than to using the Vernier scale . o The CRM is divided into two categories: an involved (positive) CRM when the tumor extends to within 1 mm of the circumferential margin and an uninvolved (negative) CRM when the distance between the tumor and CRM is >1 mm. o All the studies in the literature consider a positive CRM to be a distance between tumor and margin of <1 mm. o Several studies also indicated that the frequency of a positive CRM is higher in abdominoperineal resection than in low anterior resection.

Sphincter preservation, disease control, and long-term survival are the main goals in the treatment of rectal cancer. Local excision is a procedure which can eliminate defecation, sexual and urinary dysfunction and the risk of a permanent stoma, with a shorter hospital stay and minimal mortality and morbidity because it avoids radical surgery. T1 and T2 rectal cancers according to the tumor node metastasis classification (TMN) have been reported as early or early stage cancer in many Western publications. On the other hand, Tis and T1 cancers of the rectum have been considered as early cancer in accordance with the Japanese classification.“ ” “ ” In the ESMO clinical practice guidelines, rectal cancer“ is divided” into four groups: very early (some cT1), early (cT1 2, some cT3), more advanced (cT3, some cT4) and locally advanced (cT4).

The practice parameters– for the management of rectal cancer (revised) prepared by the standards practice task force of the American Society of Colon and Rectal Surgeons (published in 2013) recommended local excision as an appropriate treatment modality for carefully selected patients with T1 rectal cancers without high-risk features.  The criteria for local treatment include well to moderately differentiated T1 cancer, the absence of lymphovascular or perineural invasion and tumors less than 3 cm in diameter occupying less than one-third of the circumference of the bowel lumen  The NCCN clinical practice guidelines in oncology showed that transanal excision can be indicated for the following rectal cancers: <30 % circumference of the bowel, <3 cm in size, clear margins (>3 mm), mobile, nonfixed, within 8 cm of the anal verge, T1 and T2 (using caution in T2 due to the high local recurrence rate), endoscopically removed polyps with cancer or indeterminate pathology, no lymphovascular or perineural invasion, well to moderately differentiated cancer and no evidence of lymphadenopathy on pretreatment imaging.  In addition, these guidelines comment that when the tumor can be adequately identified in the rectum, transanal microsurgery may be indicated.  The ESMO clinical practice guidelines show that in the earliest, most favorable cases, chiefly the malignant polyps, a local procedure, e.g., using the transanal endoscopic microsurgery (TEM) technique, is appropriate, and that the resection should be radical (R0), and that no sign of vessel invasion or poor differentiation should be present.  An expert panel designated by the American College of Radiology showed that the optimal candidates for a local excision alone include small (<4 cm), low-lying T1 tumors without adverse pathological features.  On the other hand, the guidelines proposed by the Japanese Society for Cancer of the Colon and Rectum (JSCCR) describe very short inclusion criteria for local excision: preoperatively diagnosed Tis and slightly invasive T1 tumors located below the peritoneal reflection, adding that TEM can excise more proximal tumors than conventional local excision procedures. Instead, a more detailed description is provided for the inclusion criteria for endoscopic resection: preoperative diagnosed Tis and slightly invasive T1 cancer, tumors less than 2 cm and tumors with an undetermined morphology. In addition, these guidelines comment that the tumor histology must be evaluated. Expanding features, erosion, ulceration, fold convergence, deformity and hardness of the tumors are listed as endoscopic parameters, thus indicating the presence of massive invasive T1 tumors in the guidelines. The guidelines also recommend referencing the findings of a barium enema study, endoscopic observation with the use of dye, high-magnification endoscopic observation and ERUS for the further diagnosis of the depth of invasion.

Local excision procedures are: Local options for surgical management include transanal local excision, transsacral excision (Kraske approach), transsphincteric resection (Bevan or York-Mason approach), transanal endoscopic microsurgery, and electrocoagulation.

Transanal local excision should be performed as a full-thickness excision where the perirectal fat serves as the deep plane of dissection.

The Kraske excision is used for lesions too large or proximal for transanal local excision, and allows for removal of some perirectal lymph nodes. A perineal incision is made just above the anus, the coccyx is removed, and the fascia is divided. The rectum is mobilized through this incision, and a wide local excision or sleeve resection can be performed.

The transsphincteric procedure is identical to the Kraske procedure, except that the anal sphincter is divided posteriorly in the midline. The anus is reconstructed at completion of the operation, with little risk of functional impairment.

Transanal Endoscopic Microsurgery (TEM):

Indications:

 Lesion smaller than 3 cm  Mobile  Polypoid in shape  Anatomically accessible localized to bowel wall (T1N0)  Confined to extraperitoneal region of the rectum  Occupies less than 40% of the circumference of bowel lumen  Well or moderately differentiated  Absence of lymphovascular invasion

In transanal endoscopic microsurgery, an operating rectoscope is used to perform a full-thickness disc excision of the rectum, with primary closure. Fulguration is completed in multiple steps with the patient under general or regional anesthesia, and involves charring of the tumor, then scraping with a curette. There is a 10% to 20% risk of delayed hemorrhage due to sloughing of the scar at 7 to 10 days.

Full-thickness local excision is preferred over destructive approaches because the complete specimen is available for assessment of depth of invasion, margins, and pathologic features known to be prognostic for lymph node involvement.

2. Describe the pathophysiological aspect of pneumoperitoneum during laparoscopic surgery. Give a brief account of the energy sources of Endoscopic and Endoluminal surgery. 10+10

Answer. Pathophysiologic changes: o Respiratory changes: . The physiology of respiratory system is affected by pneumoperitoneum. . With insufflation, causing an increase in intra- abdominal pressure (IAP), the diaphragm is pushed upwards causing stiffness of the chest wall, causing the total volume of the lungs to be reduced. . Hence the pulmonary compliance is decreased to 35 40% and also a non-negligible increase in the maximum respiratory system resistance. Hypoxemia may occur from a ventilation- perfusion mismatch and intrapulmonary– shunting but is rare in healthy patients. . Carbon dioxide is usually administered at a rate of 1 2 ml/min. Being a highly soluble gas, it is readily absorbed into the circulation through the peritoneum, causing hypercapnia and acidosis. – . Carbon dioxide is mainly excreted by the lungs, depending on alveolar and mixed venous CO exchange rates, which are themselves controlled by the cardiac output, alveolar ventilation and respiratory quotient . . Normal excretion of CO 2 is 100 200 mL/min and is increased by 14 48 mL/min when CO2 is administered intraperitoneally. – – . After a long laparoscopic operation, achieving a normal CO2 value can take several hours after desufflation, since high use of peripheral storage capacity will lengthen the duration of increased PaCO2. o Cardiovascular changes: . Cardiovascular system effects during CO2 pneumoperitoneum are caused mainly by hypercarbia followed by acidosis and increased intra-abdominal pressure. . A euvolemic status is of great importance prior to surgery to reduce any cardiac depression via reducedpreload caused by the pneumoperitoneum. . Hypercarbia has direct and indirect sympathoadrenal stimulating effects on cardiovascular functions. These effects are not pronounced with mild hypercarbia (PaCO2:45 50 mmHg), whereas moderate to severe hypercarbia affects cardiac function since it is then a myocardial depressant and has direct vasodilatary effect. – . . With endoluminal endoscopic surgery, radiofrequency alternating current in the form of a circuit is essentially monopolar procedures such as snare polypectomy, sphincterotomy, removal of the lower esophageal sphincter, and "hot" biopsy. A grounding electrode is required for this form of energy. Bipolar electrocautery is used primarily for thermal hemostasis. Electrosurgical generator is activated by a pedal so that the endoscopist can keep both hands free during the endoscopic procedure. Methods of production of shock waves or heat with ultrasonic energy are also of interest. Extracorporeal by shock waves created with focused shock waves that increase the focal point of discharge is approached. When the focal point is inside the body, large amounts of energy are able to fragment the stone. Slightly different configurations of this energy can be used to provide increased internal heating of tissues. Potential applications of this technology include the ability to produce enough noninvasively internal heating to destroy tissue without an incision. A third way to use ultrasonic energy is to create instruments that are rapidly oscillating capable of heating the tissue with the friction, this technology represents a major advance in energy technology. An example of its application is laparoscopic coagulation shears device which is capable of coagulating and dividing the blood vessels in the first closing and then providing sufficient heat to weld the blood vessel walls together and to divide the tank. This non-electric coagulation and dividing the tissue with a minimum amount of collateral damage has facilitated the implementation of procedures endosurgical many. It is particularly useful in controlling bleeding from medium-sized vessels that are too big to manage with electrocoagulation monopolar and bipolar desiccation require followed by cutting.

MIS uses conventional energy sources, but the requirement for the bloodless surgery to maintain optimal visualization has led to new ways of applying energy. The energy source is the most common radio frequency electrosurgery using alternating current with a frequency of 500,000 cycles per second. Tissue heating progresses through the phases of known coagulation, vaporization and desiccation, and carbonization. The two most common methods of delivering RF electrosurgical electrodes are with monopolar and bipolar. With monopolar electrosurgery to a ground plate distance on the patient's leg or rear receives the flow of electrons which come to a point source, the surgical electrode. A fine tip electrode causes a high current density at the site of application and rapid heating of the tissues. Monopolar electrosurgery is inexpensive and easy to adjust for effects of different tissues. A short-term, high-voltage discharge current provides the extremely fast heating of tissue. Lowervoltage, more powerful current is best for drying and vaporization of tissue. With bipolar electrosurgery the electrons flow between two adjacent electrodes. The fabric between the two electrodes is heated and dehydrated. It is unlikely to cut tissue when bipolar current is used, but the ability to coapt the electrodes in a vessel provides the best method of coagulation of small vessels without thermal damage to adjacent tissues. To avoid thermal injury to adjacent structures, the laparoscopic field of view must include all metal parts of the electrosurgical electrode. In addition, the integrity of the insulation must be maintained and assured. Capacitive coupling occurs when a plastic trocar isolated from the abdominal wall current, turn the current is discharged from a metal sleeve or laparoscope in the viscera. This can lead to thermal necrosis and delayed faecal fistula. Another potential mechanism for unrecognized visceral lesions may occur with the direct coupling of current laparoscopy and adjacent bowel.

3. Write short notes of the following: 5x6

a) Development and descent of testis. b) Systemic inflammatory response syndrome. c) Complication of parenteral nutrition. d) Metabolic changes in pyloric obstruction. e) Diagnosis and management of DVT Answer. a) Development and descent of testis. See the answer of question 3.c of Paper –I of 2007. b) Systemic inflammatory response syndrome. See the answer of question 3(a) of Paper – I of 2010. c) Complications of parenteral nutrition. Answer.

TPN fully by-passes the GI tract and normal methods of nutrient absorption.Possible complications, which may be significant, are as below.

 Infection: TPN requires a chronic IV access for the solution to run through, and the most common complication is infection of this catheter. Infection is a common cause of death in these patients, with a mortality rate of approximately 15% per infection, and death usually results from septic shock.

 Blood clots: Chronic IV access leaves a foreign body in the vascular system, and blood clots on this IV line are common. Death can result from pulmonary embolism wherein a clot that starts on the IV line but breaks off goes into the lungs.

 Fatty liver and liver failure: Fatty liver is usually a more long term complication of TPN. The pathogenesis is due to using linoleic acid (an omega-6 fatty acid component of soybean oil) as a major source of calories. TPN-associated liver disease strikes up to 50% of patients within 5 7 years, correlated with a mortality rate of 2-50%. Onset of this liver disease is the major complication that leads TPN patients to requiring an intestinal transplant. –

 Hunger: Because patients are being fed intravenously, the subject does not physically eat, resulting in intense hunger pangs. The brain uses signals from the mouth (taste and smell), the stomach/G.I. Tract (fullness) and blood (nutrient levels) to determine conscious feelings of hunger.

 Cholecystitis: Total parenteral nutrition increases the risk of acute cholecystitis due to complete disuse of , which may result in bile stasis in the gallbladder. Other potential hepatobiliary dysfunctions include steatosis, steatohepatitis, cholestasis, and cholelithiasis. Six percent of patients on TPN longer than 3 weeks and 100% of patients on TPN longer than 13 weeks develop biliary sludge

In newborn infants with short bowel syndrome with less than 10% of expected intestinal length, thereby being dependent upon total parenteral nutrition, 5 year survival is approximately 20%.

 Gut atrophy: Infants who are sustained on TPN without food by mouth for prolonged periods are at risk for developing gut atrophy.

 Other complications: Other complications are either related to catheter insertion, or metabolic, including refeeding syndrome. Catheter complications include pneumothorax, accidental arterial puncture, and catheter-related sepsis. The complication rate at the time of insertion should be less than 5%. Metabolic complications include the refeeding syndrome characterised by hypokalemia, hypophosphatemia and hypomagnesemia. Hyperglycemia is common at the start of therapy, but can be treated with insulin added to the TPN solution. Hypoglycaemia is likely to occur with abrupt cessation of TPN. Liver dysfunction can be limited to a reversible cholestatic jaundice and to fatty infiltration (demonstrated by elevated transaminases). Severe hepatic dysfunction is a rare complication.[36] Overall, patients receiving TPN have a higher rate of infectious complications. This can be related to hyperglycemia.

 Special Complications in Pregnancy: Pregnancy can cause major complications when trying to properly dose the nutrient mixture. Because all of the baby s nourishment comes from the mother s blood stream, the doctor must properly calculate the dosage of nutrients to meet both recipient s needs and have them in usable forms. Incorrect’ dosage can lead to many adverse, hard-’ to-guess effects, such as death, and varying degrees of deformation or other developmental problems’ .

d) Metabolic changes in pyloric obstruction. See the answer of question 1 of Paper –I of 2008. e) Diagnosis and management of DVT. See the answer of question 3(a) of Paper – I of 2009.

4. Answer briefly on the following. 4x71/2

a) Post splenectomy problems. b) Anal incontinence. c) Tumours of Endocrine pancreas. d) Silent Gall Stones. Answer. a) Post splenectomy problems. See the answer of question 4.e of Paper – I of 2009.

b) Anal incontinence. See the answer of question 1 of Paper –I of 2011. c) Tumours of Endocrine pancreas. See the answer of question 4.d of Paper –I of 2010. d) Silent Gall Stones. Answer. Introduction:  Cholelithiasis is common (10-25%) in the general population, with the majority (70- 80%) being asymptomatic at the time of diagnosis, and only few (10-20%) produce symptoms or complications in life time. , either open or laparoscopic, is straight forward for symptomatic gallstones, unlike the controversies in asymptomatic or silent gallstones.  Asymptomatic gallstones may be defined as those having caused no symptoms or the ones detected incidentally during investigations in the absence of gallstone-related symptoms (biliary colic) or complications (cholecystitis, cholangitis and pancreatitis).  Vague or non-specific abdominal symptoms, for example dyspepsia, epigastric discomfort, flatulence or nausea are not considered gallstone-related[192225] as they are common in the general population with or without other gastrointestinal problems. Asymptomatic gallstone and the risk of gallbladder carcinoma . Since 80% of gallbladder carcinoma patients show presence of gallstone, gallstone is definitely a precancerous condition. . The risk of carcinoma is higher in patients with gallstone greater than 3 cm in diameter and in women than in men. Asymptomatic gallstone: symptoms and/or complications . The risk of biliary pain is 10% at 5. . The vast majority of gallstones (80%) do not cause any symptoms over few decades. . More than 90% of patients will have symptoms before any complications (acute cholecystitis, obstructive jaundice, ) occur. . The risk of complication is 3% at 10 years.  Treatment of asymptomatic gallstones:

Surgical treatment of asymptomatic gallstones without medically complicating diseases is discouraged. The risk of complications arising from interventions is higher than the risk of symptomatic disease. Approximately 25% of patients with asymptomatic gallstones develop symptoms within 10 years.

Persons with diabetes and women who are pregnant should have close follow-up to determine if they become symptomatic or develop complications. However, cholecystectomy for asymptomatic gallstones may be indicated in the following patients:

 Patients with large gallstones greater than 2 cm in diameter  Patients with nonfunctional or calcified (porcelain) gallbladder observed on imaging studies and who are at high risk of gallbladder carcinoma  Patients with spinal cord injuries or sensory neuropathies affecting the abdomen  Patients with sickle cell anemia in whom the distinction between painful crisis and cholecystitis may be difficult

Patients with risk factors for complications of gallstones may be offered elective cholecystectomy, even if they have asymptomatic gallstones. These groups include persons with the following conditions and demographics:

 Cirrhosis  Portal hypertension  Children  Transplant candidates  Diabetes with minor symptoms

Patients with a calcified or porcelain gallbladder should consider elective cholecystectomy due to the possibly increased risk of carcinoma (25%). Refer to a surgeon for removal as an outpatient procedure.

Medical dissolution of gallstones:

Ursodeoxycholic acid (ursodiol) is a gallstone dissolution agent. In humans, long-term administration of ursodeoxycholic acid reduces cholesterol saturation of bile, both by reducing liver cholesterol secretion and by reducing the detergent effect of bile salts in the gallbladder (thereby preserving vesicles that have a high cholesterol carrying capacity). Desaturation of bile prevents crystals from forming and, in fact, may allow gradual extraction of cholesterol from existing stones.

In patients with established cholesterol gallstones, treatment with ursodeoxycholic acid at a dose of 8-10 mg/kg/d PO divided bid/tid may result in gradual gallstone dissolution. This intervention typically requires 6-18 months and is successful only with small, purely cholesterol stones. Patients remain at risk for gallstone complications until dissolution is completed. The recurrence rate is 50% within 5 years. Moreover, after discontinuation of treatment, most patients form new gallstones over the subsequent 5-10 years.

THE WEST BENGAL UNIVERSITY OF HEALTH SCIENCES

MS (General Surgery) Examination, 2012

PAPER I

Time Allowed: 3 Hours Full Marks: 100 Attempt all questions

1. Define and discuss Subphrenic spaces and their surgical importances. 10 + 10. 2. Describe the neuroendocrine response to trauma. Write the local and systemic changes following 50% burn.10+10

3. Write short notes of the following: 5 x 6 a) Hepatic segments. b) Imaging of breast disease. c) Pharmacotherapy of Benign Prostatic Hypertrophy. d) Compartment syndrome. e) Umbilical fistulae.

1. Answer briefly on the following: 4 x 71/2 a) Tumour markers. b) Antibiotic resistance. c) Gastrointestinal hormones and their uses. d) Necrotizing soft tissue infection.

THE WEST BENGAL UNIVERSITY OF HEALTH SCIENCES

MS (General Surgery) Examination, 2012

May 2012

PAPER I

Time Allowed: 3 Hours Full Marks: 100 Attempt all questions

1. Define and discuss Subphrenic spaces and their surgical importances. 10 + 10. Answer. See the answer of question no.1 of Paper – I of 2007.

2. Describe the neuroendocrine response to trauma. Write the local and systemic changes following 50% burn.10+10 Answer. The endocrine response to trauma: Changes occur to circulating concentrations of many hormones following injury. These take place as a result of direct stimulation of the various glands that produce the hormones, and also because normal negative feedback mechanisms are altered as part of the response to injury. Factors mediating the metabolic response to injury: The acute Endothelial cell Nervous system Endocrine Bacterial inflammatory activation response infection response Inflammatory cells Adhesion of Afferent nerve Increased (macrophages, inflammatory cells. stimulation secretion of monocytes, neutrophils) stress hormones

Pro-inflammatory Vasodilatation Decreased cytokines and other secretion of inflammatory mediators Increased anabolic permeability hormones

Hormonal changes in response to trauma: Hormonal Pituitary Adrenal Pancreas Others change Increased GH Adrenaline Glucagon Rennin secretion ACTH Cortisol Angiotensin Prolactin Aldosterone ADH/AVP Unchanged TSH secretion LH FSH Decreased Insulin Testosterone secretion Oestrogen Thyroid hormones

Causes of fluid loss following trauma:

Urinary changes during the metabolic response to injury:  Reduced urine volume in response to hypovolaemia and ADH release  Low urinary sodium and increased urinary potassium excretion due to aldosterone release  Increased urinary nitrogen excretion due to the catabolic response to injury.

Neuroendocrine response altogether:

Burn injuries result in both local and systemic responses.

Local response: The three zones of a burn were described by Jackson in 1947.

 Zone of coagulation This occurs at the point of maximum damage. In this zone there is irreversible tissue loss due to coagulation of the constituent proteins.  Zone of stasis The— surrounding zone of stasis is characterised by decreased tissue perfusion. The tissue in this zone is potentially salvageable. The main aim of burns resuscitation is— to increase tissue perfusion here and prevent any damage becoming irreversible. Additional insults such as prolonged hypotension, infection, or oedema can convert this zone into an area of complete tissue loss.  Zone of hyperaemia In this outermost— zone tissue perfusion is increased. The tissue— here will invariably recover unless there is severe sepsis or prolonged hypoperfusion. — These three zones of a burn are three dimensional, and loss of tissue in the zone of stasis will lead to the wound deepening as well as widening.

Systemic response:

 The release of cytokines and other inflammatory mediators at the site of injury has a systemic effect once the burn reaches 30% of total body surface area.  Cardiovascular changes Capillary permeability is increased, leading to loss of intravascular proteins and fluids into the interstitial compartment. Peripheral and splanchnic vasoconstriction— occurs. Myocardial contractility is decreased, possibly due to

wound, result in systemic hypotension and end organ hypoperfusion.  Respiratoryrelease of tumour changes necrosisInflammatory factor α. These mediators changes, cause coupled bronchoconstriction, with fluid loss from and inthe severe burn burns adult respiratory distress syndrome can occur.  Metabolic changes —The basal metabolic rate increases up to three times its original rate. This, coupled with splanchnic hypoperfusion, necessitates early and aggressive enteral feeding to decrease— catabolism and maintain gut integrity.  Immunological changes Non-specific down regulation of the immune response occurs, affecting both cell mediated and humoral pathways. —

3. Write short notes of the following: 5 x 6 a) Hepatic segments. b) Imaging of breast disease. c) Pharmacotherapy of Benign Prostatic Hypertrophy. d) Compartment syndrome. e) Umbilical fistulae. a) Hepatic segments. Answer. o Historically, the liver was divided into left and right lobes by the obvious external landmark of the falciform ligament. Not only was this description oversimplified, but it was also anatomically incorrect in relationship to the blood supply to the liver. Later, more accurate descriptions of the lobar anatomy of the liver were developed. The liver was divided into right and left lobes determined by portal and hepatic vein branches. Briefly, a plane without any surface markings running from the gall-bladder to the left side of the IVC (known as the portal fissure, or Cantlie's line) divided the liver into right and left lobes. The right lobe was further divided into anterior and posterior segments. The left lobe was divided into a medial segment (also known as the quadrate lobe) that lies to the right of the falciform ligament and umbilical fissure and a lateral segment lying to the left. This system, although anatomically more correct, is only sufficient for mobilization of the liver and simple hepatic procedures. It does not describe the more intricate and functional segmental anatomy that is essential to understand before pursuing complex hepatobiliary surgery. o The functional anatomy of the liver is composed of eight segments, each of which is supplied by a single portal triad (also called a pedicle) composed of a portal vein, hepatic artery, and . These segments are further organized into four sectors that are separated by scissurae containing the three main hepatic veins. The four sectors are even further organized into the right and left liver (the phrase right and left liver is preferable to right and left lobe because there is no external mark that allows the identification of the right and left liver). This system was originally described in 1957 by Woodsmith and Goldburne as well as Couinaud and defines hepatic anatomy as it is most relevant to surgery of the liver. The functional anatomy is more often seen as cross-sectional imaging.

o A schematic demonstrating the segmental anatomy of the liver. Each segment receives its own portal pedicle (triad of portal vein, hepatic artery, and bile duct). The eight segments are illustrated, and the four sectors, divided by the three main hepatic veins running in scissurae, are shown. The umbilical fissure (not a scissura) is shown to contain the left portal pedicle.

Segmental anatomy of the liver as seen at in the anatomic position (A) and in the ex vivo position (B). o The main scissura contains the middle hepatic vein, which runs in an anteroposterior direction from the gallbladder fossa to the left side of the vena cava and divides the liver into right and left hemi-. The line of the main scissura is also known as Cantlie's line. The right liver is divided into an anterior (segments V and VIII) and posterior (segments VI and VII) sector by the right scissura, which contains the right hepatic vein. The right portal pedicle, composed of the right hepatic artery, portal vein, and bile duct, splits into right anterior and posterior pedicles that supply the segments of the anterior and posterior sectors. o The left liver has a visible fissure along its inferior surface called the umbilical fissure. The ligamentum teres (containing the remnant of the umbilical vein) runs into this fissure. The falciform ligament is contiguous with the umbilical fissure and ligamentum teres. The umbilical fissure is not a scissura, does not contain a hepatic vein, and in fact, contains the left portal pedicle (triad containing the left portal vein, hepatic artery, and bile duct), which runs in this fissure, branching to feed the left liver. The left scissura runs posterior to the ligamentum teres and contains the left hepatic vein. The left liver is split into an anterior (segments III and IV) and posterior (segment II the only sector composed of a single segment) sector by the left scissura. o At the hilum of the liver, the right portal triad has a short— extrahepatic course of about 1 to 1.5 cm before entering the substance of the liver and branching into anterior and posterior sectoral branches. The left portal triad, however, has a long extrahepatic course of up to 3 or 4 cm and runs transversely along the base of segment IV in a peritoneal sheath that is the upper end of the lesser omentum. The left portal triad, as it runs along the base of segment IV, is separated from the liver substance by connective tissue known as the hilar plate. The continuation of the left portal triad runs anteriorly and caudally in the umbilical fissure and gives branches to segments II and III and recurrent branches to segment IV.

The plate system is illustrated. A, The cystic plate between the gallbladder and the liver. B, The hilar plate at the biliary confluence at the base of segment IV. C, The umbilical plate above the umbilical portion of the portal vein. The arrows show the plane of dissection of the cystic plate for cholecystectomy and the hilar plate for exposure of the hepatic duct confluence and the main left hepatic duct. o The caudate lobe (segment I) is the dorsal portion of the liver and embraces the IVC on its posterior surface and lies posterior to the left portal triad inferiorly and the left and middle hepatic veins superiorly. The main bulk of the caudate lobe is to the left of the IVC, but inferiorly, it traverses between the IVC and left portal triad, where it fuses to the right liver (segments VI and VII). This part of the caudate lobe is known as the right portion or the caudate process. The left portion of the caudate lobe lies in the lesser omental bursa and is covered anteriorly by the gastrohepatic ligament (lesser omentum) that separates it from segments II and III anteriorly. The gastrohepatic ligament attaches to the ligamentum venosum (sinus venosus remnant) along the left side of the left portal triad. o The vascular inflow and biliary drainage to the caudate lobe comes from both the right and left systems. The right side of the caudate (the caudate process) largely derives its portal venous supply from the right portal vein or the bifurcation of the main portal vein, whereas the left portion of the caudate derives its portal venous inflow from the left main portal vein. The arterial supply and the biliary drainage of the right portion are generally through the right posterior sectoral system and the left portion through the left main vessels. The hepatic venous drainage of the caudate is unique in that multiple small veins drain posteriorly directly into the IVC. o The posterior edge of the left side of the caudate terminates into a fibrous component that attaches to the crura of the diaphragm and also runs posteriorly, wrapping behind the IVC and attaching to segment VII of the right liver. Up to 50% of the time, this fibrous component is composed either partially or completely of liver parenchyma, and thus liver tissue may completely encircle the IVC. This important structure is known as the IVC ligament and is important when mobilizing the right liver or the caudate lobe off of the vena cava. o Anomalous development of the liver is uncommonly encountered. Complete absence of the left liver has been reported. A tongue of tissue extending inferiorly off of the right liver has been described (Riedel's lobe). Rare cases of supradiaphragmatic liver in the absence of a hernia sac have been noted.

b) Imaging of breast disease. Answer. Imaging modalities of breast diseases:  Mammography: o Mammography has been the basic imaging method in breast diagnostics, and the only tool suitable for screening breast cancer. o In screening, its sensitivity and specificity are 90 93% and 93 97%, respectively. o Mammography has some recognized limitations and disadvantages. The sensitivity and specificity are highly dependent on the composition– of the breast– parenchyma, which for its part is influenced by age, hormonal status and possible previous interventions. In young women, the usefulness of mammography is restricted by high prevalence of dense fibroglandular tissue, which impairs both the detection and the differentiation of the lesion. o With increasing age, the breast parenchyma usually shows fatty replacement, which makes abnormalities more easily detectable. Hormone replacement therapy may decrease the sensitivity of mammography by increasing the breast density and enlarging benign masses, such as and . o After breast surgery, mass-like scars and areas of distortion may mimic a tumor or hide subtle signs of malignancy. Radiation after surgical treatment of breast carcinoma leads to skin thickening and increased focal or diffuse density of the breast due to edematous changes. The accuracy of mammography is also impaired in patients with silicon implants. o Cysts and solid tumors cannot always be definitely differentiated at mammography. Some carcinomas may have a benign appearance, and some fibroadenomas may be irregular and difficult to differentiate from a malignant tumor. o A palpable mass may be partially or completely obscured by adjacent fibroglandular tissue. Even the spiculations within the fibrous tissue surrounding a cancer may be inconspicuous at mammography, because both the spiculations and the adjacent fibrous tissue are of the same density. o In addition to standard (craniocaudal, mediolateral oblique, lateral) views, supplemental mammographic views are often needed to better or more completely visualize the area of special concern. o The commonly used modified views are spot compression and magnification views. With the spot compression, separation of overlapping structures can be achieved. Coning down with spot compression also reduces scatter radiation and sharpens image details. Magnification provides the ability to define characteristics of microcalcifications and details of masses. Also tangential views, extra lateral craniocaudal views, change-of-angle views, cleavage views and modified compression views (e.g. for patients with breast implants) can be taken. o Digital mammography has the potential to overcome some of the limitations of conventional mammography. Because of the increased contrast and decreased noise of digital systems it is possible to improve image quality, although the spatial resolution is still limited when compared to screen-film mammography. o BI-RADS is an acronym for Breast Imaging-Reporting and Data System, a quality assurance tool originally designed for use with mammography. The system is a collaborative effort of many health groups but is published and trademarked by the American College of Radiology (ACR).

BI-RADS Assessment Categories are:

 0: Incomplete  1: Negative  2: Benign  3: Probably benign  4: Suspicious  5: Highly suggestive of malignancy  6: Known biopsy proven malignancy

–  Other mammographic techniques: o In case of spontaneous nipple discharge, galactography has been the method of choice. A mammogram taken after duct cannulation and contrast injection reveals possible intraductal tumors as filling defects, and with the same method, the lesion can also be preoperatively marked with methylene blue dye. o Galactographic finding helps to localize the origin of nipple discharge but is nonspecific. Sometimes cannulating a secreting duct may be impossible. The latest high resolution US has allowed visualization of the dilated ducts and the intraductal tumors. o Pneumocystography after a fine-needle aspiration of a may reveal intracystic tumor and help to prevent cyst recurrence. o Specimen radiography is a sufficient method to confirm removal of a nonpalpable lesion after surgical excision.

 Ultrasonography:

B-mode ultrasonography: o US has been used in breast diagnostics since the 1950s. Until recently, the main indications of breast US have been differentiation between cystic and solid lesions, evaluation of a palpable lesion in a mammographically dense breast (for example young, pregnant or lactating patient), evaluation of a lesion detected at mammography or mammographic asymmetry, detection of an abscess in an infectious breast, evaluation after breast cancer treatment and breast augmentation, evaluation of axillary lymph nodes and guidance for interventional procedures. o Ultrasound can detect mammographically occult cancers, but it is generally accepted that US is not suitable for screening. Microcalcifications with no associate mass are not usually reliably detectable at US. The analysis of microcalcifications is, however, only possible with mammographic spot magnification. o In the last few years, differentiation between benign and malignant solid breast lesions by means of US has gained increased interest. The individual characteristics for malignant lesions include spiculation, angular margins, marked hypoechogenicity, shadowing, calcification, duct extension, branch pattern, vertical ( taller than wide ) orientation and microlobulation. If a single malignant feature is found, the lesion cannot be considered benign. Intense hyperechogenicity, ellipsoid shape, gentle” bi- or trilobulations,” thin, echogenic pseudocapsule and lack of malignant findings are considered benign features o Currently, most solid breast lesions undergo a diagnostic or preoperative needle biopsy.

 Doppler techniques: Doppler enables more accurate detection of the small tumor vessels than conventional color.

A considerable problem in tumor diagnostics is that the spatial resolution of Doppler imaging is limited, and only major feeding vessels of the tumors are detectable, not the abnormal complex microvascularity. Color Doppler, or even the more sensitive power Doppler, is not capable of detecting the flow information in small vessels in all directions. The technique is also equipment and operator dependent. Neither examination techniques or interpretation of the Doppler images are standardized and the results of the studies vary considerably.

 Contrast-enhanced ultrasonography: o The finding that Doppler signals may be difficult to detect either because of small vessel size or inadequate equipment has led to the development of ultrasound contrast agents. o Because US contrast agents do not extravasate from the vessel to the surrounding tissue, any echo received indicates the presence of a vessel. Contrast enhancement improves detection of small vessels with slow and low-volume blood flow.

Other ultrasonographic techniques:

 Specific imaging techniques for ultrasound contrast agents have been developed to overcome this limitation. In contrast harmonic imaging, the transducer emits at one frequency and receives at the second (or higher) harmonic frequency.  Inverted phase imaging, i.e. pulse inversion, is a new variant of second harmonic imaging. The transducer transmits not a single pulse but two pulses that are 180 degrees out of phase. The nonlinear harmonic component from the microbubbles produces a strong signal, while linear signals from the background causing most artifacts are cancelled out. It produces new possibilities for flow detection in both color Doppler and B-mode. Contrast harmonic imaging together with pulse inversion may overcome the limitations in assessing the microperfusion of tissues.  In spatial compounding technique, several real-time ultrasound images are acquired using different angles of insonation. These images are combined to provide a single composite image, which has a better signal-to-noise ratio and sharper borders with reduced speckle and clutter compared to a conventional B-mode image.  Three-dimensional (3D) ultrasound is helpful in determining the exact size and position as well as the surface characteristics of a lesion. With 3D imaging volume measurement is possible, which allows for more accurate follow-up examinations.  Pathological changes often cause changes in tissue stiffness. In breast diagnostics, breast cancers are firm and less mobile than the surrounding parenchyma. In elasticity imaging, ultrasound is used to monitor the internal tissue displacement responses to externally applied force. This method has many alternative schemes.

 Magnetic resonance imaging: MR imaging has proved to be the most sensitive method for the detection of invasive breast cancer. The detection is based on lesion enhancement after contrast agent administration. In various series, the sensitivity for invasive breast cancer has ranged from 88 to 100%, and the specificity from 37 to 97%. The value of MR imaging in patients with mammographically detected suspicious microcalcifications and DCIS has been poor. In cases of nipple discharge, the results have been controversial.  Diagnostic criteria:

. The diagnostic criteria consist of both lesion morphology and enhancement kinetics.The morphologic criteria are comparable to those used at mammography. Well-defined margins indicate benignity, while ill-defined or spiculated lesions are suggestive of malignancy. Internal septations, if seen, are specific for fibroadenomas. . Enhancement in benign lesions is homogeneous and proceeds centrifugally. Benign lesions also usually enhance less and do so more slowly than malignant lesions. In malignant lesions enhancement is often inhomogeneous or rim-like and tends to proceed centripetally. Diffuse enhancement is a nonspecific finding. . Relative enhancement > 80 90% is a widely used criterion for malignancy, but the quantity of enhancement is highly dependent on the imaging technique (field strength, imaging sequences, dosage –of contrast agent etc.) and thus, it is not generally accepted. Enhancement is not specific for breast pathology, because also normal breast tissue may enhance. . Due to hormonal changes, transient enhancement has been found in 80% of healthy premenopausal women. . There is also overlapping between the morphologic and dynamic features of benign and malignant lesions. The most important benign lesions causing false positive findings include fibroadenomas, inflammatory changes, , proliferative fibrocystic changes and radial scar. . Post-treatment changes may also be liable to misinterpretations: postoperative scars enhance for six months and post radiation therapy may cause enhancement for 18 months. Hematoma due to core biopsy causes signal increase in some patients, but fine-needle biopsy has not been shown to have influence on MR imaging.

 Indications and contraindications:

. Most breast lesions can be diagnosed by using conventional modalities, especially when combined with needle biopsies. It is, however, desirable to be able to reduce the number of biopsies performed for benign causes. In dealing with lesions that remain equivocal after mammographic and sonographic evaluation, MR imaging could be the problem-solving method. A negative MR imaging finding virtually excludes invasive carcinoma. . The aim of breast imaging is the detection of breast cancer as early as possible, as the prognosis of breast cancer depends on the stage of the disease at the time of diagnosis. MR imaging is not considered suitable for breast screening in a general population, but it might be feasible in imaging the extremely dense breasts of especially young high-risk women (family history, cancer susceptibility genes, history of contralateral breast cancer). . It is the best method for detecting an otherwise occult primary breast carcinoma in patients with axillary node metastases. After a cancer diagnosis, MR imaging is the most sensitive tool for preoperative staging and treatment planning. . MR imaging can also be an adjunctive method in post-treatment surveillance in conservatively treated breasts with suspected recurrence and evaluation of tumor response to chemotherapy. . MR imaging is also the best method for imaging of breasts with silicon prostheses. In addition to general contraindications for MR imaging (e.g. cardiac pacemaker, ferromagnetic incorporated substances) there are some specific limitations concerning breast imaging.

. It must not be used instead of x-ray mammography, because according to present knowledge, it is not suitable for detecting and evaluating microcalcifications. Due to its variable specificity, MR imaging should not be used for imaging symptomless women without an increased risk of breast cancer. . Before breast MR imaging can be a clinically used tool for breast imaging, a proper MR- guided biopsy and localization system must be available for biopsy of suspicious lesions not detected by other modalities. . The verification of lesion removal is also a major problem to solve, because MR imaging of the excised specimen is not feasible.

Other magnetic resonance imaging techniques:

 In proton MR spectroscopy malignant lesions have been shown as having high levels of choline-containing compounds when compared to benign lesions and normal breast tissue.  MR elastography is a new imaging modality that produces images with a contrast proportional to the stiffness of the tissue, as in US elasticity imaging. It has been predicted to become a potential adjunctive tool for both lesion detection and characterization.

Other imaging modalities:

. Computed tomography has not been recommended for breast imaging, mainly because of high radiation dose. It has been successfully used in regional staging of small breast cancer before breast conserving surgery. . Electrical impedance scanning is a new technique, which is based upon the principle that malignant cells exhibit altered local dielectric properties and show measurably higher conductivity values. The method has been presented as a useful tool for further evaluation of equivocal mammographic findings, but its real value remains to be seen. . The most important task of nuclear medicine with regard to breast cancer is nowadays sentinel node staging using lymphoscintigraphy. In differential diagnostics, nuclear medicine is under investigation as an adjunct to mammography. . Presented clinical indications for the most commonly used agent Tc 99m sestamibi scintimammography include examining premenopausal dense breasts, palpable lesions with low-suspicion mammographic finding and evaluation of response to neoadjuvant chemotherapy for locally advanced breast cancer. . Positron emission tomography scanning might have a role in differential diagnosis and in staging of breast tumors. c) Pharmacotherapy of Benign Prostatic Hypertrophy. Answer. See the answer of question 4(a) of Paper – I of 2008. d) Compartment syndrome. Answer. See the answer of question 3(a) of Paper – I of 2010. e) Umbilical fistulae. Answer. It is an abnormal passage from the umbilicus to the intestine or, more frequently, to the remnant of the canal in the median umbilical ligament that connects the fetal bladder with the allantois. Vitelline Umbilical fistula: . May present as Umbilical polyp. . May drain enteric contents. . Fistulogram shows communication with bowel.

D/D of umbilical fistula:

Embroyologic precursor of the urachus and ompalomesenteric duct. The urachus forms from the involution of the allantoic duct.

Imaging studies: o Barium enema o Fistulogram o USG o CT Scan.

Treatment: Surgical excision of the fistulous tract.

4. Answer briefly on the following: 4 x 71/2 a) Tumour markers. b) Antibiotic resistance. c) Gastrointestinal hormones and their uses. d) Necrotizing soft tissue infection.

Answer. a) Tumour markers. See the answer of question 3(e) of Paper-I of 2010. b) Antibiotic resistance. Answer.

 Antibiotic resistance is a type of drug resistance where a microorganism is able to survive exposure to an antibiotic. While a spontaneous or induced genetic mutation in bacteria may confer resistance to antimicrobial drugs, genes that confer resistance can be transferred between bacteria in a horizontal fashion by conjugation, transduction, or transformation. Thus, a gene for antibiotic resistance that evolves via natural selection may be shared. Evolutionary stress such as exposure to antibiotics then selects for the antibiotic resistant trait. Many antibiotic resistance genes reside on plasmids, facilitating their transfer. If a bacterium carries several resistance genes, it is called multidrug resistant (MDR) or, informally, a superbug or super bacterium.  Genes for resistance to antibiotics, like the antibiotics themselves, are ancient. However, the increasing prevalence of antibiotic-resistant bacterial infections seen in clinical practice stems from antibiotic use both within human medicine and veterinary medicine. Any use of antibiotics can increase selective pressure in a population of bacteria to allow the resistant bacteria to thrive and the susceptible bacteria to die off. As resistance towards antibiotics becomes more common, a greater need for alternative treatments arises. However, despite a push for new antibiotic therapies there has been a continued decline in the number of newly approved drugs. Antibiotic resistance therefore poses a significant problem.

Causes: o Although there were low levels of preexisting antibiotic-resistant bacteria before the widespread use of antibiotics, evolutionary pressure from their use has played a role in the development of muiltidrug resistance varieties and the spread of resistance between bacterial species. In some countries, antibiotics are sold over the counter without a prescription, which also leads to the creation of resistant strains. In medicine, the major problem of the emergence of resistant bacteria is due to misuse and overuse of antibiotics. Other practices contributing towards resistance include the addition of antibiotics to livestock feed. Household use of antibacterials in soaps and other products, although not clearly contributing to resistance, is also discouraged (as not being effective at infection control). Also unsound practices in the pharmaceutical manufacturing industry can contribute towards the likelihood of creating antibiotic-resistant strains. The procedures and clinical practice during the period of drug treatment are frequently flawed - usually no steps are taken to isolate the patient to prevent re-infection or infection by a new pathogen, negating the goal of complete destruction by the end of the course. o Certain antibiotic classes are highly associated with colonisation with "superbugs" (highly antibiotic resistant bacteria) compared to other antibiotic classes. The risk for colonisation increases if there is a lack of sensitivity (resistance) of the superbugs to the antibiotic used and high tissue penetration, as well as broad-spectrum activity against "good bacteria". In the case of MRSA, increased rates of MRSA infections are seen with glycopeptides, cephalosporins and especially quinolones. In the case of colonisation with Clostridium difficile the high risk antibiotics include cephalosporins and in particular quinolones and clindamycin.

Natural occurrence: o There is evidence that naturally occurring antibiotic resistance is common. The genes that confer this resistance are known as the environmental resistome. These genes may be transferred from non-disease-causing bacteria to those that do cause disease, leading to clinically significant antibiotic resistance. o Inappropriate prescribing of antibiotics has been attributed to a number of causes, including people who insist on antibiotics, physicians who simply prescribe them as they feel they do not have time to explain why they are not necessary, and physicians who do not know when to prescribe antibiotics or else are overly cautious for medical legal reasons. o Antibiotic resistance has been shown to increase with duration of treatment; therefore, as long as a clinically effective lower limit is observed (that depends upon the organism and antibiotic in question), the use by the medical community of shorter courses of antibiotics is likely to decrease rates of resistance. o Compliance with once-daily antibiotics is better than with twice-daily antibiotics. Patients taking less than the required dosage or failing to take their doses within the prescribed timing results in decreased concentration of antibiotics in the bloodstream and tissues, and in turn, exposure of bacteria to suboptimal antibiotic concentrations increases the frequency of antibiotic resistant organisms. o Poor hand hygiene by hospital staff has been associated with the spread of resistant organisms, and an increase in hand washing compliance results in decreased rates of these organisms.

Role of other animals: o Drugs are used in animals that are used as human food, such as cattle, pigs, chickens, fish, etc. There has been extensive use of antibiotics in animal husbandry. Historically this use has been little regulated, because very little restrictive legislation or regulatory limitations exist. These drugs can affect the meat, milk, and eggs produced from those animals and can be the source of superbugs. For example, farm animals, particularly pigs, are believed (though not proven) to be able to infect people with MRSA. o The resistant bacteria in animals due to antibiotic exposure can be transmitted to humans via three pathways, those being through the consumption of meat, from close or direct contact with animals, or through the environment. However, complete cooking of meat inactivates bacteria, whether or not they are antibiotic-resistant. The World Health Organization concluded antibiotics as growth promoters in animal feeds should be prohibited, in the absence of risk assessments.

Mechanisms:

The four main mechanisms by which microorganisms exhibit resistance to antimicrobials are:

1. Drug inactivation or modification: for example, enzymatic deactivation of penicillin G in some penicillin-resistant bacteria through the production of -lactamases 2. Alteration of target site: for example, alteration of PBP the binding target site of penicillins in MRSA and other penicillin-resistant bacteria β 3. Alteration of metabolic pathway: for example, some sulfonamide-resistant— bacteria do not require para-aminobenzoic— acid (PABA), an important precursor for the synthesis of folic acid and nucleic acids in bacteria inhibited by sulfonamides, instead, like mammalian cells, they turn to using preformed folic acid. 4. Reduced drug accumulation: by decreasing drug permeability and/or increasing active efflux (pumping out) of the drugs across the cell surface.

 There are three known mechanisms of fluoroquinolone resistance. Some types of efflux pumps can act to decrease intracellular quinolone concentration. In Gram-negative bacteria, plasmid-mediated resistance genes produce proteins that can bind to DNA gyrase, protecting it from the action of quinolones. Finally, mutations at key sites in DNA gyrase or topoisomerase IV can decrease their binding affinity to quinolones, decreasing the drug's effectiveness.  Antibiotic resistance can also be introduced artificially into a microorganism through laboratory protocols, sometimes used as a selectable marker to examine the mechanisms of gene transfer or to identify individuals that absorbed a piece of DNA that included the resistance gene and another gene of interest. A recent study demonstrated that the extent of horizontal gene transfer among Staphylococcus is much greater than previously expected and encompasses genes with functions beyond antibiotic resistance and virulence, and beyond genes residing within the mobile genetic elements. — Prevention: o Rational use of antibiotics may reduce the chances of development of opportunistic infection by antibiotic-resistant bacteria due to dysbacteriosis. o Vaccines do not have the problem of resistance because a vaccine enhances the body's natural defenses, while an antibiotic operates separately from the body's normal defenses. Nevertheless, new strains may evolve that escape immunity induced by vaccines; for example an updated influenza vaccine is needed each year. o While theoretically promising, antistaphylococcal vaccines have shown limited efficacy, because of immunological variation between Staphylococcus species, and the limited duration of effectiveness of the antibodies produced. Development and testing of more effective vaccines is under way.

Phage therapy: o Phage therapy, an approach that has been extensively researched and used as a therapeutic agent. Bacteriophages or "phages" are viruses that invade bacterial cells and, in the case of lytic phages, disrupt bacterial metabolism and cause the bacterium to lyse. Phage therapy is the therapeutic use of lytic bacteriophages to treat pathogenic bacterial infections. o Bacteriophage therapy is an important alternative to antibiotics in the current era of multidrug resistant pathogens. o Discovery of the structure of the viral protein PlyC is allowing researchers to understand the way it kills a significant range of pathogenic bacteria.

Archaeocins is the name given to a new class of potentially useful antibiotics that are derived from the Archaea group of organisms.

. In research published on October 17, 2008 in Cell, a team of scientists pinpointed the place on bacteria where the antibiotic myxopyronin launches its attack, and why that attack is successful. The myxopyronin binds to and inhibits the crucial bacterial enzyme, RNA polymerase. The myxopyronin changes the structure of the switch-2 segment of the enzyme, inhibiting its function of reading and transmitting DNA code. This prevents RNA polymerase from delivering genetic information to the ribosomes, causing the bacteria to die. . One of the major causes of antibiotic resistance is the decrease of effective drug concentrations at their target place, due to the increased action of ABC transporters. Since ABC transporter blockers can be used in combination with current drugs to increase their effective intracellular concentration, the possible impact of ABC transporter inhibitors is of great clinical interest. ABC transporter blockers that may be useful to increase the efficacy of current drugs have entered clinical trials and are available for therapeutic regimens.

Applications: Antibiotic resistance is an important tool for genetic engineering. By constructing a plasmid that contains an antibiotic resistance gene as well as the gene being engineered or expressed, a researcher can ensure that when bacteria replicate, only the copies that carry the plasmid survive. This ensures that the gene being manipulated passes along when the bacteria replicates. o The most commonly used antibiotics in genetic engineering are generally "older" antibiotics that have largely fallen out of use in clinical practice. These include:

 Ampicillin  Kanamycin  Tetracycline  Chloramphenicol c) Gastrointestinal hormones and their uses. Answer. The gastrointestinal hormones (or gut hormones) constitute a group of hormones secreted by enteroendocrine cells in the stomach, pancreas, and small intestine that control various functions of the digestive organs.

Enteroendocrine cells do not form endocrine glands but are spread throughout the digestive tract. They exert their autocrine and paracrine actions that integrate all of gastrointestinal function.

 Cholecystokinin is produced by duodenal and jejunal I cells and enteric nerves in response to intraluminal amino acids and fats. CCK induces gallbladder contraction, pancreatic enzyme secretion, and relaxation of the sphincter of Oddi.  Enteroglucagon from ileal and colonic L cells is produced in response to intraluminal fat and bile acids. Of note, inflammatory processes, such as Crohn disease and celiac sprue, can dramatically increase enteroglucagon secretion.  Gastric inhibitory peptide (GIP), secreted by duodenal and jejunal K cells in response to active transport of monosaccharides, long-chain fatty acids, and amino acids, inhibits gastric acid and pepsinogen secretion and gastric emptying but stimulates insulin release.  Duodenal G cells secrete gastrin in response to vagal stimulation and intraluminal peptides. Gastrin stimulates acid secretion by the gastric fundus and body and increases gastric mucosal blood flow.  Motilin is produced by duodenal and jejunal M cells in response to duodenal acid, vagal stimulation, and gastrin-releasing peptide. Motilin initiates phase III of the MMC during the fasting state. Erythromycin is useful as a promotility agent due to its action as a motilin agonist.  Duodenal and jejunal S cells release secretin in response to acid, bile salts, and fatty acids in the duodenum. Secretin increases bicarbonate and water secretion from pancreatic ducts. It inhibits gastric acid secretion and gastric motility.  Somatostatin broadly inhibits gut exocrine and endocrine function. Somatostatin and its analog, octreotide, are often used to decrease the volume of intestinal secretions in patients with enterocutaneous fistulas. Intestinal D cells and enteric neurons secrete somatostatin in response to intraluminal fat, protein, and acid.  Vasoactive intestinal peptide (VIP) is secreted throughout the small intestine in response to vagal stimulation. VIP increases mesenteric blood flow, intestinal motility, and pancreatic and intestinal secretions.

d) Necrotizing soft tissue infection. Answer. Definition: Necrotizing soft tissue infection is a rare but very severe type of bacterial infection that can destroy the muscles, skin, and underlying tissue. Necrotizing refers to something that causes tissue death. Causes, incidence, and risk factors: Many different types of bacteria can cause this type of infection. A very severe and usually deadly form of necrotizing soft tissue infection is due to Streptococcus pyogenes, which is sometimes called "flesh-eating bacteria." Necrotizing soft tissue infection develops when the bacteria enters the body, usually through a minor cut or scrape. The bacteria begins to grow and release harmful substances (toxins) that:

 Directly kill tissue  Interfere with the blood flow to the tissue  Break down materials in the tissue, which rapidly spreads the bacteria, leading to widespread effects such as shock.

Signs and Symptoms:

o The first sign of infection may be a small, reddish, painful spot or bump on the skin. This quickly changes to a very painful bronze- or purple-colored patch that grows rapidly. The center may become black and die off. The skin may break open and ooze fluid. The wound may quickly grow in less than an hour. o Symptoms may include general ill feeling, fever, sweating, chills, nausea, dizziness, profound weakness, and finally shock. Without treatment, death can occur rapidly.

Tests:

o Often a patient will be diagnosed in the operating room by a surgeon. Imaging tests, such as CT scans, are sometimes helpful. o Tests performed on blood, fluids, or tissue from the area may determine the bacteria that is causing the infection. Treatment:

o Powerful, broad-spectrum antibiotics must be given immediately through a vein (IV). Surgery is required to open and drain infected areas and remove dead tissue. Sometimes donor immunoglobulins (antibodies) are given by vein to help fight the infection. o Skin grafts may be needed after the infection goes away. If an arm or leg infection cannot be controlled, of the limb may be considered. o If the bacteria is determined to be an oxygen-avoiding organism (anaerobe), the patient may receive hyperbaric oxygen therapy. This involves placing the patient in a chamber that delivers 100% oxygen at high pressure.

Complications:

 Local spread of infection, progressive tissue damage  Systemic spread of infection, sepsis, shock  Scarring and disfigurement  Functional loss of an arm or leg  Death.

Prevention: Always clean the skin thoroughly after a cut, scrape, or other skin injury.

THE WEST BENGAL UNIVERSITY OF HEALTH SCIENCES MS (General Surgery) Examination, 2011 PAPER I

Time Allowed: 3 Hours Full Marks: 100 Attempt all questions

1. Describe the surgical anatomy of Rectum and . Write about aetiology and management of anal incontinence. 10 + 10. 2. Describe the aetiopathology of Acute Pancreatitis. What are the complications of pseudocyst of Pancreas? 10+10

3. Write short notes of the following: 5 x 6 a) Blood substitutes. b) Post operative pain management. c) Complications of parenteral nutrition. d) H.I.V infection in health care workers. e) Drains in gastro intestinal suegery.

4) Answer briefly on the following: 4 x 71/2 a) Anastomosis in surgery. b) Vascular access for dialysis. c) Endoluminal ultrasound. d) Incidentalomas.

THE WEST BENGAL UNIVERSITY OF HEALTH SCIENCES

MS (General Surgery) Examination, 2011

April 2011

PAPER I

Time Allowed: 3 Hours Full Marks: 100 Attempt all questions

1. Describe the surgical anatomy of Rectum and Anal Canal. Write about aetiology and management of anal incontinence. 10 + 10.

Answer.

 The rectum normally lies attached to its mesorectum within the curve of the sacrum with limited mobility. The junction of the rectosigmoid is most consistently found at the sacral promontory and descends only 2 or 3 cm during a Valsalva maneuver.  The rectum exits the pelvis anteriorly surrounded by a sling of muscle from the pubis through a slit in the pelvic floor. The sling is created by the puborectalis muscle that circles around behind the rectum and reinserts on the pubis anteriorly.  The anal canal itself measures 3 4 cm and is a funnel-shaped extension of the pelvic floor musculature. The pressure generated by this voluntary muscle prevents egress of rectal contents. The internal sphincter– muscle is a continuation of the thickened circular muscle of the rectum. As such, it is an autonomic muscle and has no voluntary control.

 The anorectum receives both sympathetic and parasympathetic nerves. The sympathetic nerves originate from thoracolumbar segments, and unite below the inferior mesenteric artery to form the inferior mesenteric plexus. These fibers then descend to the superior hypogastric plexus located just inferior to the aortic bifurcation. These purely sympathetic fibers bifurcate and descend as the hypogastric nerves. Parasympathetic fibers from S2, S3, and S4 (the nervi erigentes) join the hypogastric nerves anterolateral to the rectum to form the inferior hypogastric plexuses. Mixed fibers from the plexuses innervate the prostate, rectum, bladder, penis, and internal anal sphincter. The sympathetic innervation of the internal sphincter is motor, while the parasympathetic innervation is inhibitory.  The innervation of the voluntary muscles of the pelvic floor is via direct fibers from S2, S3, and S4 in the pelvis from the sacrum. The nerves of the external sphincter are derived from S2, S3, and S4 nerve roots from the sacral plexus and arrive at the external sphincter via the pudendal nerve around the ischial spine at Alcock's canal.

 It is useful to consider the anus and surrounding structures as a single unit, the anorectum . The anorectum includes the perianal skin, the anal canal, the anal , and the distal rectum. The three main anatomic points of reference are the anal verge, the dentate line, and the anorectal ring. The distal external boundary of the anal canal is the anal verge, which is also the junction between the anal and perianal skin. Anal (anoderm) is devoid of the hair follicles, sebaceous glands, and apocrine glands that are present in the perianal skin, a fact worth remembering when attempting to distinguish between hidradenitis (inflammation of the apocrine glands in the perianal skin) and cryptoglandular anal disease.  The cephalad border of the anal canal is a true mucocutaneous junction, the dentate line. This union of the embryonic ectoderm with the endodermal gut resides approximately 1.0 1.5 cm above the anal verge. In a transitional zone of 6 12 mm in length, the columnar epithelium of the rectum changes to cuboidal epithelium which joins the squamous epithelium at the– dentate line. –  The upper border of the anal sphincteric complex is the anorectal ring. It may be palpated by digital examination about 1.0 1.5 cm above the dentate line.  Anatomists consider the anal canal to begin at the dentate line and end at the anal verge. However, most surgeons consider– the anal canal to start at the anorectal ring and terminate at the anal verge.  Just above the dentate line, the rectal mucosa forms from 8 14 longitudinal folds known as the rectal columns. Between each two columns at the dentate line is a small pocket termed an anal crypt. The glands may extend through the internal sphincter– as far as the intersphincteric plane, but they do not extend into the external sphincter.  Below the dentate line, cutaneous sensations of heat, cold, touch, and pain are conveyed by afferent fibers in the inferior rectal nerves. Cephalad to the dentate line, poorly defined dull sensations, elicited when the mucosa is pinched or internal hemorrhoids are ligated, are probably carried by parasympathetic fibers.

The superior rectal artery, the terminal branch of the inferior mesenteric artery, descends to the upper rectum where it divides into lateral branches. Subsequent smaller divisions penetrate the rectal wall. The middle rectal arteries arise from the internal iliac arteries and supply the distal rectum and upper anal canal. The inferior rectal arteries, branches from the internal pudendal arteries, cross the ischiorectal fossae to supply the anal sphincters.

 The superior rectal artery, the terminal branch of the inferior mesenteric artery, descends to the upper rectum where it divides into lateral branches. Subsequent smaller divisions penetrate the rectal wall. The middle rectal arteries arise from the internal iliac arteries and supply the distal rectum and upper anal canal. The inferior rectal arteries, branches from the internal pudendal arteries, cross the ischiorectal fossae to supply the anal sphincters.  There are two paths for venous blood return from the anorectum. Above the dentate line, venous blood flows into the portal system through the superior rectal vein and inferior mesenteric vein. Below the dentate line, the external hemorrhoidal plexus drains into the internal iliac vein via the middle rectal vein or via the pudendal vein, which receives blood from the inferior rectal vein.

Anal Incontinence:

Anal Incontinence Etiology

Mechanical Neurogenic Idiopathic Obstetric injury Pudendal nerve stretch No clear etiology Fistula disease Strain Medical illness Trauma Prolonged labor Irradiation Iatrogenic Trauma Irritable bowel syndrome Systemic disease Multiple sclerosis, diabetes mellitus, scleroderma Diarrheal states

The normal continence mechanism has several components:

 Rectal capacitance and compliance are essential. The rectum normally holds between 200 and 250 ml. It distends readily with filling and has limited muscular activity intrinsically.  The sampling reflex is a function of rectal distension allowing anal sphincter relaxation via an intramural reflex to the internal sphincter.  The rectal contents can then be sensed in the sensory nerve rich transitional zone and anoderm to discriminate the true nature of the rectal contents. This sampling reflex occurs frequently throughout the day to provide continence and also– serves to initiate the defecation process.  The voluntary external sphincter muscle contraction in response to this sampling reflex provides the final active component of anal continence. The subconscious voluntary contraction of the external sphincter, puborectalis and pelvic floor muscles provide complete control of rectal contents.  The pelvic floor muscles maintain continual activity, even during sleep, to provide anal continence. This also seems to be a learned response since infants and children require 1 2 years to achieve control.  It is important to realize that the degree of incontinence affects the lifestyle of the patient.– The frequency of incontinence may vary and the loss of control may involve solid stool, liquid stool, or gas only.  Frequent episodes of incontinence of gas only may be as incapacitating as infrequent episodes of solid stool. It is essential to document the exact type of incontinence before planning treatment. It is especially important to clearly define the incontinence before attempting to report a series of patients who undergo a specific treatment. There is no universally accepted grading scale to assess severity and impact of .  The American Society of Colon and Rectal Surgeons validated a fecal incontinence severity index and a fecal incontinence quality of life index to help standardize the assessment of anal incontinence. Diagnosis and Evaluation: Signs of anal incontinence in the office include a thin perineal body with scarring between the vagina and the anal canal and a poor squeeze on command. There is controversy regarding the adequacy of digital assessment of the anal sphincter mechanism. It is possible that an experienced examiner may be able to determine the adequacy of resting tone, but it is difficult to quantify and accurately evaluate the voluntary squeeze pressure generated by an anal sphincter mechanism.It is also essential to rule out the presence of a rectovaginal fistula in the setting of an anterior sphincter injury.

Anal manometry: It is useful to document reduced resting and squeeze pressures as well as sphincter length in individual sphincter quadrants. Normal resting pressure is at least 40 mm Hg, normal squeeze pressure is 80 mm Hg, which is usually double the resting pressure, and sphincter length is greater than 3 cm. Normal sensation should allow detection of a balloon inflated with 10 20 mL of air. Maximal tolerable volume is usually over 100 mL of air-filled balloon distention. – : Single fiber density determination is not routinely performed. Pudendal nerve terminal motor latency determination measures the conduction velocity of the nerve action potential through the terminal 4 cm of the pudendal nerve between Alcock's canal and the external sphincter. A delay in conduction reflects injury to the fast-conducting fibers of the nerve. This injury usually is the result of stretch, direct trauma, or systemic disease. The normal terminal motor latency is 2.2 ± 0.2 ms. Any delay in conduction velocity greater than this indicates nerve injury.

Transrectal Ultrasound: The most sensitive method for documenting sphincter injury may be the anal ultrasound using a 360° rotating 10-MHz transducer. The focal length of the anal probe is approximately 1 2 cm and allows evaluation of the anal sphincter muscles in three dimensions as the probe is withdrawn from the rectum. Scarring at the site of an injury is usually easily seen by .– A rectovaginal fistula can also be detected.

MRI: High-resolution magnetic resonance imaging (MRI) with an endoanal coil is a newer diagnostic modality that can detect sphincter defects similarly to endoanal ultrasound. Endoanal coil MRI may also show sphincter atrophy or thinning not detectable by endoanal ultrasound that may have an impact on the success of surgical repair. An algorithm for the evaluation and management of anal incontinence can be produced using these diagnostic techniques

Treatment:

Anal Sphincter Reconstruction:

 Anal sphincter repair can be performed successfully in most patients who have an isolated mechanical sphincter defect.  The patient requires a complete bowel preparation in order to avoid a at the time of the procedure.  The ends of the injured sphincter are identified in the anterior perineum in patients with obstetric injury, and they are overlapped and secured or reefed in the midline to reconstruct the circular muscle.  Control of solid and liquid stool will be adequate in 90% of patients after this type of repair. However, complete continence is usually only achieved in 75 90% of patients and the long- term results may even be less satisfactory.  Leakage of liquid, mucus, and gas may continue to affect patien– ts after repair. Improvement in squeeze pressures has been shown to correlate best with functional outcome.  The presence of at least one normal pudendal nerve is necessary for an improvement after sphincter reconstruction.  Complications of wound infection, fistula formation, and breakdown of the sphincter repair are reduced by leaving a drain in the perineal body after the repair.  A repeat procedure is usually successful in those cases in whom the sphincter repair is disrupted.

Anal sphincter overlapping muscle repair. A. Anterior incision and perineal view of muscles. B. Rectal flap is created and sphincter muscles are isolated. C. Muscle flaps are fully mobilized. D. Muscle flaps are overlapped around a 15-mm rubber dilator or fingertip. E. Muscle flaps are sutured in place and the perineal body repaired. F. A drain is placed behind the vaginal wall and the wall closed.

Muscle Sensory Retraining or Biofeedback: Operant conditioning using manometric and balloon sensation techniques is possible in patients with a mechanical sphincter defect. Electromyographic anal plugs also provide biofeedback for conditioning. Pre- and postoperative biofeedback may aid in functional improvement. Patients with extremely poor pudendal nerve function or complete disruption of the anal sphincter will not benefit at all.

Other Treatments and New Modalities: New techniques have been developed, including implantation of an artificial sphincter of silicone or a neurostimulator that provides constant activity in a muscle transferred to the anal canal (i.e., dynamic graciloplasty). "Pacemakers" implanted in the pelvis for sacral nerve stimulation of the anus have also been used, similar to a urologist's technique to cure urinary dysfunction. These techniques are indicated for severe, idiopathic fecal incontinence when all else has failed. They are expensive procedures, and some have a high risk of complications, but patients with severe neuromuscular damage and even those who have had the sphincter mechanism removed may benefit from reconstruction of the anal sphincter. The Parks posterior sphincter repair has been shown to yield poor results in patients with severe anal incontinence.

2. Describe the aetiopathology of Acute Pancreatitis. What are the complications of pseudocyst of Pancreas? 10+10

Answer. See the answer of question 4(a) of Paper II of 2009 for aetiopathology of acute pancreatitis.

The risk of pseudocyst complications is less than previously believed when pseudocysts were diagnosed on the basis of symptoms. With pervasive imaging, a higher proportion of asymptomatic pseudocsyts are diagnosed.

Complications of pseudocysts include infection, leakage, and bleeding. By mass effect, a pseudocyst also can produce early satiety (stomach), stenosis (duodenum), cholestasis (bile duct), and thrombosis (portal, superior mesenteric and splenic veins) leading to portal or segmental hypertension and varices.

 Infection of the pancreas can take several forms. Sterile, organized pancreatic necrosis may appear as a mature pseudocyst on CT scan but be predominantly solid. These differences relate to the origin (isolated duct damage versus pancreatic necrosis) and to the amount of solid material contained within the collection.  The rupture of a pseudocyst can occur by erosion into the adjacent gastrointestinal tract, which may resolve the pseudocyst or leave a cystoenteric fistula.  Rupture into the peritoneum leads to pancreatic ascites and can be a dramatic presentation with acute abdominal pain and rigidity from chemical peritonitis.  Bleeding associated with a pancreatic pseudocyst is a life-threatening complication. There are several causes of bleeding. o Bleeding may occur secondary to erosion of the gut mucosa with the impending development of a cystoenteric fistula. o This may produce hematemesis and malena. More ominous is the direct erosion of a significant visceral vessel, including the splenic, gastroduodenal, and middle colic vessels. o The action of pancreatic enzymes (especially elastase) on the vessel wall can lead to thinning of the vessel wall and aneurysm and pseudoaneursym formation. This situation carries a high mortality. o The risk of bleeding is increased in the presence of local infection.

3. Write short notes of the following: 5 x 6 a) Blood substitutes. b) Post operative pain management. c) Complications of parenteral nutrition. d) H.I.V infection in health care workers. e) Drains in gastro intestinal suegery. Answer. a) Blood substitutes. Answer. A blood substitute (also called artificial blood or blood surrogates) is a substance used to mimic and fulfill some functions of biological blood, usually in the oxygen-carrying sense. It aims to provide an alternative to blood transfusion, which is transferring blood or blood-based products from one person into another. 1st generation blood substitute:  The main categories of such oxygen-carrying blood substitutes are hemoglobin-based oxygen carriers (HBOC) and perfluorocarbon-based oxygen carriers (PFBOC).  An oxygen-carrying blood substitute, sometimes called artificial haemoglobin, is an artificially made red blood cell substitute whose main function is to carry oxygen, as does natural haemoglobin. The use of oxygen-carrying blood substitutes is often called oxygen therapeutics to differentiate from true blood substitutes. The initial goal of oxygen carrying blood substitutes is merely to mimic blood's oxygen transport capacity. There is additional longer range research on true artificial red and white blood cells which could theoretically compose a blood substitute with higher fidelity to human blood.  There are two basic approaches to constructing an oxygen therapeutic. The first is perfluorocarbons (PFC), chemical compounds which can carry and release oxygen. The specific PFC usually used is perfluorodecalin . The second is haemoglobin derived from humans, animals, or artificially via recombinant technology. Stroma-free hemoglobin:  Stroma-free hemoglobin has been investigated as an oxygen carrier since the 1940s, when researchers realized that native hemoglobin is not antigeneic. A solution containing stroma-free hemoglobin has many advantages over red blood cells, including the ability to withstand sterilization and a shelf life of approximately 2 years at room temperature for some products.  Solutions of acellular hemoglobin are not as effective at oxygenation as packed red blood cells because of their high affinity for oxygen. Red blood cells have adapted to release oxygen at an oxygen half saturation pressure of hemoglobin (p-50) of approximately 26.5 mm Hg. This is due to the allosteric effect of 2,3-bisphosphoglycerate (2,3-BPG), which shifts the oxyhemoglobin dissociation curve to the right. In the absence of 2,3-BPG, stroma-free hemoglobin has a p-50 of 12-14 mm Hg.  Unmodified free hemoglobin, when infused rapidly, splits into dimers and is cleared by glomerular filtration and uptake by the reticuloendothelial system. When free hemoglobin was used initially, it caused a substantial increase in oncotic pressure because of its hyperosmolarity. Unfortunately, the initial attempts at transfusing stroma-free hemoglobin produced renal dysfunction, coagulopathy, and hypertension. Several approaches have been tried to decrease the avidity with which hemoglobin binds to oxygen. These adaptations include the addition of organic phosphate to serve the function of 2,3-BPG and adenosine triphosphate, cross-linking dimers of hemoglobin tetramers and polymerizing the tetramers to decrease oncotic pressure and prevent glomerular filtration. Hypertension has remained a significant adverse effect of stroma-free hemoglobin.  Diaspirin cross-linked hemoglobin (DCLHb) is the prototype molecule of this category of blood substitutes. It consists of cross-linking between the two alpha chains that lend stability to the molecule. The cross-linking agent is bis (dibromosalicyl) fumarate (DBBF). DCLHb made from outdated human blood has a shelf life of approximately 9 months when frozen and 24 hours when refrigerated. The intravascular half-life is 2-12 hours and is dose dependant.

Next-Generation Blood Substitutes:

 Polyethylene glycol (PEG) hemoglobin (Enzon Pharmaceuticals, Bridgewater, NJ) was a conjugated bovine hemoglobin that was evaluated for use in cancer therapy to increase tumor oxygenation and enhance the efficacy of radiation and chemotherapy. However, at present, production of this product has been discontinued.  Hemospan (Sangart Inc., San Diego, Calif), also known as MP4OX, is a PEG-conjugated human hemoglobin currently undergoing clinical trials in the US and Europe. At relatively low concentrations, Hemospan (MP4OX) is capable of transporting large amounts of oxygen.

(The trial, conducted in Sweden, involved 90 patients undergoing hip arthroplasty. Patients were randomized to receive either Hemospan (MP4OX) or Ringer acetate (control) before induction of spinal anesthesia. Hemospan (MP4OX) was found to be well tolerated in the study group with no serious adverse effects attributed to the product during the trial period. The percentage of hypotensive episodes in the Hemospan (MP4OX) group was about 45% compared to 87% among controls. Incidence of intra-operative vasopressor use was about 15% in the Hemospan (MP4OX) group compared to 32% among controls. See Sangart Inc: Clinical Trials for more information on recent studies.)

 Pyridoxylated hemoglobin polyoxyethylene conjugate (PHP) is a conjugated hemoglobin developed by Apex Bioscience that completed a phase III trial in August 2009 in patients with shock associated with systemic inflammatory response syndrome.

For hemoglobin-based oxygen carriers cross-linked with enzymes, there has been an effort to synthesize compounds that not only perform the function of carrying oxygen, as do the molecules mentioned previously, but also harbor some of the enzyme activity that normal red blood cells possess. Polymerized hemoglobin has been cross-linked with catalase and superoxide dismutase to form a compound that, in animal models can not only carry oxygen but also remove oxygen radicals that are responsible for ischemia reperfusion injuries.  Polyhemoglobin has also been cross-linked with tyrosinase to form a soluble complex that can carry oxygen and decrease the systemic levels of tyrosine. This agent can help increase the efficacy of chemotherapy and radiation therapy in tumor tissue and in a melanoma model has been shown to delay tumor growth without having significant adverse effects.

The ultimate RBC substitute would contain not only hemoglobin but also other contents of the red blood cell also encapsulated in an artificial membrane. However, production of such a product would be extremely challenging. Efforts have been made to use polyactide, a biodegradable polymer that is converted to lactic acid in the body, to create artificial red blood cells. These cells contain hemoglobin along with the RBC enzyme complement including superoxide dismutase, methemoglobin reductase, and catalase.

Potential Uses:

If such a blood substitute were indeed developed, it would presumably play a major role in the setting of trauma care and for elective surgeries. It would also benefit patients with medical conditions who are in need of long-term blood transfusions, such as patients with myelodysplastic syndrome and aplastic anemia. These products could also be used as organ preservatives to prevent or decrease reperfusion injury to donor organs. Religious and ethnic groups with concerns regarding the use of human derived blood products may accept these substances, which would provide a significant patient care advance. Adverse Effects Related to Blood Substitutes

Adverse effects: Adverse effects associated with hemoglobin-based oxygen carriers include hypertension, abdominal pain, skin rash, diarrhea, jaundice, hemoglobinuria, oliguria, fever, stroke, and laboratory anomalies such as an elevation in lipase levels. Although most of these side effects were transient and clinically asymptomatic, many clinical trials involving these agents have been discontinued or held due to the associated adverse effects. b) Post operative pain management.

Answer. See the answer of question 4(c) of Paper-I of 2009. c) Complications of parenteral nutrition.

Answer. Complications of parenteral nutrition.

 Catheter Issues in Parenteral Nutrition: Many different types of venous catheters are available for central infusion, and which is most appropriate for a given patient depends on many factors. o Catheter Sepsis: Catheter sepsis is potentially the most lethal complication in patients receiving TPN.This problem is directly related to catheter care and the incidence of hyperglycemia attributable to TPN and can be reduced to an acceptable minimum of less than 1% per year by attention to detail, avoidance of multiuse catheters, and careful metabolic management. Organisms causing line infections are generally 80% Staphylococcus (50 : 50 aureus versus epidermidis), 15% yeast, and 5% gram-negative bac-teria. Staphylococcal infections are generally related to catheter care, whereas yeast probably reaches the catheter through the gut. Additional factors influencing the incidence of line sepsis include the presence of a percutaneous stoma (e.g., a colostomy or tracheostomy), preexistent malnutrition with an increased susceptibility to infection, corticosteroid administration, recent broad-spectrum antibiotic therapy, concurrent chemotherapy, or severe neutropenia (e.g., in acute leukemia). Fungemia is the most serious type of line infection, with the entry site of Candida, the most common fungal pathogen, most probably being the GI tract. It is important to treat colonization with yeast (defined as two positive site cultures, e.g., urine and skin) in a critically ill patient aggressively with either fluconazole or amphotericin to avoid further complications such as line sepsis.

In the instance of a permanent catheter (Hickman, portacath), these devices may sometimes be salvaged in the setting of confirmed bacteremia (usually with gram-negative organisms) by prolonged antibiotic therapy, usually of 2 weeks' duration. For patients receiving long-term TPN who may have limited access options remaining because of multiple previous lines, line salvage becomes even more attractive. For S. epidermidis or gram-negatives, antibiotic therapy is effective in 60% to 70% of patients. At times a fibrin sheath at the catheter tip may be a nidus, and dissolution with tissue plasminogen activator or urokinase may be useful. Line tract infections can be more difficult to eradicate. In general, if Staphylococcus aureus or yeast is documented on blood culture, the line should simply be removed and IV antimicrobial therapy subsequently administered because these organisms are too virulent and dangerous to treat in lesser fashion. Though beyond the scope of this chapter, additional approaches to the avoidance of line infection are the use of impregnated catheters (e.g., with silver or bonded antibiotics) or the so-called antibiotic lock. The latter consists of antibiotics in high concentration, such as rifampicin, instilled into the catheter at the time that it is disconnected from the infusion, either as prophylaxis or, in some centers, as definitive therapy for line colonization.

 Catheter Thrombosis and Other Complications: o Catheter failure as a result of clogging and lack of function because of an intraluminal thrombus or a fibrin tip sheath is quite common. This problem can often be corrected by instillation of tissue plasminogen activator or urokinase and can be avoided by administering long-term prophylactic low-dose heparin (usually 6000 U/bag) or by the use of low-dose warfarin (1-2 mg/day). o Thrombosis of the great veins (subclavian, superior vena cava) occurs much less frequently, although some series report an incidence as high as 5% to 10%. o Other catheter complications include pneumothorax, vascular injuries (arterial or venous lacerations, delayed arteriovenous fistulas), brachial plexus injury, chronic pain, thoracic duct injury after left-sided cannulation, air embolism, and catheter embolism. Erosion of the catheter into the bronchus, right atrium, or other structures may occur. o Septic venous thrombosis is a life-threatening complication, and if antibiotics and anticoagulation are not successful, excision of the involved vein or Fogarty catheter embolectomy may occasionally be successful. o Hydrothorax results from catheter malposition and administration of fluid into the thoracic cavity.

Home Parenteral Nutrition: A major contribution of parenteral nutrition has been the ability to maintain patients in a functional state for decades with minimal oral intake. Unlike the continuous- infusion approach appropriate for a hospitalized patient, home parenteral alimentation is generally cycled and performed overnight over an 8- to 14-hour period.

 Metabolic Complications of Long-Term Administration of Total Parenteral Nutrition

A variety of problems can arise when TPN is administered over prolonged periods, particularly in patients who have little if any oral intake and are therefore completely dependent on the solution for the provision of essential vitamins, minerals, and fatty acids. The rare deficiency states involving these nutrients and mechanical and infectious disease considerations relevant to the TPN catheter and common in the outpatient population have been discussed earlier. o Liver Disease:

Hepatic dysfunction is commonly observed in patients receiving TPN, and these disorders occupy a spectrum ranging from simple elevations in liver function test results to cirrhosis. Most often, if hyperbilirubinemia occurs acutely in a patient receiving TPN, the cause is generally sepsis. Factors responsible for liver disease attributable primarily to TPN administration, as opposed to other causes, remain unclear and are a source of controversy. Hepatic steatosis, cholestasis (presumably from lack of enteral stimulation and reduced release of cholecystokinin), and the presence of chronic inflammation have all been implicated as relevant mechanisms. Predisposing factors include short-gut syndrome (ileal disease or resection), a history of bacterial overgrowth, and recurrent sepsis or a chronic inflammatory state. Short-bowel syndrome alone seemed an insufficient risk factor unless combined with a chronic inflammatory state such as Crohn's disease. TPN-specific factors include excessive glucose or insulin administration (with increased hepatic lipogenesis), excessive lipid administration (sequestration in hepatocytes), and alterations in fatty acid metabolism leading to the release of arachidonate-derived inflammatory leukotrienes. Deficiencies in particular nutrients, such as carnitine, choline, taurine, cysteine, and S-adenosyl methionine, have also been implicated in TPN-related liver disease.

In infants dependent on TPN, hepatic dysfunction is a more serious and potentially lethal disease, and it may have a pathophysiology different from that seen in adults. It is frequently associated with cholestasis. Even when enteral feeding is begun and TPN is discontinued, hepatic dysfunction may persist and progress to cirrhosis and death. o Metabolic Bone Disease:  In various studies, 40% to nearly 100% of patients administered TPN over prolonged periods have decreased bone mineral density (BMD) or histologic evidence of bone disease.  Some individuals can be shown to have increased urinary calcium or phosphate excretion, decreased PTH levels, or vitamin D deficiency as possible mechanisms, but even in these patients there is poor correlation with BMD.  Patients at greatest risk are postmenopausal women, patients with long-standing malnutrition or malabsorption (e.g., Crohn's disease), those with preexisting liver disease, or patients receiving steroids.  Some TPN-specific mechanisms postulated to contribute to bone loss are TPN-induced hypercalciuria, in which fixed acids generated by metabolism are buffered by bone calcium carbonate, and calcium diuresis induced by hyperglycemia or excessive sodium. TPN-associated deficiency states, such as calcium or magnesium (magnesium deficiency may decrease PTH release and vitamin D formation), copper (a cofactor for lysyl oxidase and collagen synthesis), boron, or silicon, have also been suggested to play a role.  Pamidronate, a second-generation agent shown to be effective in randomized trials in inhibiting postmenopausal osteoporosis, can be administered IV to TPN patients in the outpatient setting (30 mg/200 mL 5% dextrose in water over a 2-hour period) every 3 months, with minimal toxicity.

d) H.I.V infection in health care workers. Answer. Introduction: The incidence of HIV is steadily increasing in India. Screening of these patients by routine testing of all patients is not possible; so Universal Work Precautions take an important place to prevent transmission of HIV in the health care settings. Fundamentally this prevention can be achieved by the following means:

1. Barrier precautions:

 Use of gloves A surgeon needs to wear gloves while doing any of the following procedures:

–  Dressing a wound.  Starting an IV drip.  Taking blood samples.  Doing any operative procedures.  Doing a PR, PV or examination of the oral cavity.

Literature shows that two pairs of gloves afford better protection than one pair of gloves.

i. Use of mask and cap. ii. Use of glasses /special masks for protection of eyes. iii. Use of plastic apron. iv. Use of correct footwear.

2. A methodical approach to all procedures. 3. Proper care and disposal of the sharps: like use of puncture proof containers in OT. 4. Doing all the procedures in the operation theatre in a controlled and deliberate manner.

Post Exposure Prophylaxis

1. EC1+HIV SC1 : PEP is probably not required. 2. EC1+HIV SC2 : Basic regimen should suffice. 3. EC2+HIV SC1 : Basic regimen is recommended. 4. EC2+HIV SC2 : Expanded regimen is recommended. 5. EC3+HIV SC1 or SC2 : Expanded regimen is recommended. 6. EC2 or EC3+ HIV SCunknown : Basic regimen is recommended.

PEP should be started as early as possible after the exposure. The ideal time to start PEP is less than 6 to 12 hours. It is also known that PEP started after 72 hours of exposure is not beneficial and therefore is not recommended.

Basic regimen:

 Zidovudine (AZT) 600 mg in divided doses (300 mg twice a day or 200 mg thrice a day) for 4 weeks.  Lamivudine ( 3TC)– 150 mg twice a day for 4 weeks.

Expanded regimen: –

 Zidovudine (AZT) 600 mg in divided doses (300 mg twice a day or 200 mg thrice a day) for 4 weeks.  Lamivudine ( 3TC)– 150 mg twice a day for 4 weeks.  Indinavir 800 mg thrice daily for 4 weeks. Alternatively , in place of Indinavir we can use Nelfinavir in the dosage– of 750 mg thrice daily. – e) Drains in gastro intestinal suegery. Answer. See the answer of question no. 3(a) of Paper IV of 2010.

4) Answer briefly on the following: 4 x 71/2 a. Anastomosis in surgery. b. Vascular access for dialysis. c. Endoluminal ultrasound. d. Incidentalomas. a) Anastomosis in surgery. Answer. In surgical jargon, to create an anastomosis is to join together two hollow organs (viscus), usually to restore continuity after resection, or to bypass an unresectable disease process. Historically such procedures were performed with suture material, but increasingly mechanical staplers and biological glues are employed. While an anastomosis may be end-to-end, equally it could be performed side-to-side or end-to-side depending on the circumstances of the required reconstruction or bypass. The term reanastomosis is also used to describe a surgical reconnection usually reversing a prior surgery to disconnect an anatomical anastomosis, e.g. tubal reversal after tubal ligation. Anastomosis are typically performed on:

. Blood vessels: Arteries and veins. Most vascular procedures, including all arterial bypass operations (e.g. coronary artery bypass), aneurysmectomy of any type, and all solid organ transplants require vascular anastomoses. An anastomosis connecting an artery to a vein is also used to create an arteriovenous fistula as an access for hemodialysis.

. Gastrointestinal (GI) tract: , stomach, small bowel, large bowel, bile ducts, and pancreas. Virtually all elective resections of gastrointestinal organs are followed by anastomoses to restore continuity; is considered a massive operation, in part, because it requires three separate anastomoses (stomach, biliary tract and pancreas to small bowel). Bypass operations on the GI tract, once rarely performed, are the cornerstone of . The widespread use of mechanical suturing devices (linear and circular staplers) changed the face of gastrointestinal surgery.

. Urinary tract: Ureters, urinary bladder, urethra. Radical prostatectomy and radical cystectomy both require anastomosis of the bladder to the urethra in order to restore continuity.

. Microsurgery: The advent of microsurgical technique allowed anastomoses previously thought impossible, such as so-called "nerve anastomoses" (not strictly an anastomosis according to the above definition), and operations to restore fertility after tubal ligation or vasectomy. Fashioning an anastomosis is typically a complex and time-consuming step in a surgical operation, but almost always crucial to the outcome of the procedure. b) Vascular access for dialysis.

Answer. Preparing a vascular access is an important step before starting regular hemodialys sessions, which is the site on your body where blood is removed and returned during dialysis. To maximize the amount of blood cleansed during hemodialysis treatments, the vascular access should allow continuous high volumes of blood flow. o A vascular access should be prepared weeks or months before you start dialysis. The early preparation of the vascular access will allow easier and more efficient removal and replacement of your blood with fewer complications. o The three basic kinds of vascular access for hemodialysis are an arteriovenous (AV) fistula, an AV graft, and a venous catheter. o A fistula is an opening or connection between any two parts of the body that are usually separate. While most kinds of fistula are a problem, an AV fistula is useful because it causes the vein to grow larger and stronger for easy access to the blood system. The AV fistula is considered the best long-term vascular access for hemodialysis because it provides adequate blood flow, lasts a long time, and has a lower complication rate than other types of access. If an AV fistula cannot be created, an AV graft or venous catheter may be needed. o An AV fistula requires advance planning because a fistula takes a while after surgery to develop in rare cases, as long as 24 months. But a properly formed fistula is less likely than other kinds of vascular access to form clots or become infected. Also, properly formed fistulas tend to last— many years longer than any other kind of vascular access. o A surgeon creates an AV fistula by connecting an artery directly to a vein, frequently in the forearm. Connecting the— artery to the vein causes more blood to flow into the vein. As a result, the vein grows larger and stronger, making repeated needle insertions for hemodialysis treatments easier. For the surgery, local anesthetic will be given. In most cases, the procedure can be performed on an outpatient basis.

Forearm arteriovenous fistula ii. Arteriovenous graft: o If one that connects an artery to a vein using a synthetic tube, or graft, implanted under the skin in your arm.have The small graft veins becomes that won’t an artificial develop vein properly that can into be a usedfistula, repe youatedly can get for aneedle vascular placement access

be used sooner after placement, often within 2 or 3 weeks. o Comparedand blood accesswith properly during hemodialysis. formed fistulas, A graftgrafts doesn’t tend to need have to more develop proble as msa fistula with does,clotting so andit can infection and need replacement sooner. However, a well-cared-for graft can last several years.

One kind of AV graft iii. Venous catheter for temporary access: o If kidney disease has progressed quickly, one may not have time to get a permanent vascular access before you start hemodialysis treatments. You may need to use a venous catheter as a temporary access. o A catheter is a tube inserted into a vein in the neck, chest, or leg near the groin. It has two chambers to allow a two-way flow of blood. Once a catheter is placed, needle insertion is not necessary. o Catheters are not ideal for permanent access. They can clog, become infected, and cause narrowing of the veins in which they are placed. But if one needs to start hemodialysis immediately, a catheter will work for several weeks or months while permanent access develops.

Venous catheter for temporary hemodialysis access

For some people, fistula or graft surgery is unsuccessful, and they need to use a long-term catheter access. Catheters that will be needed for more than about 3 weeks are designed to be tunneled under the skin to increase comfort and reduce complications. Even tunneled catheters, however, are prone to infection.

What one can expect during hemodialysis:

 Every hemodialysis session using an AV fistula or AV graft requires needle insertion. Most dialysis centers use two needles one to carry blood to the dialyzer and one to return the cleansed blood to your body. Some specialized needles are designed with two openings for two- way flow of blood, but these needles— are less efficient. For some people, using this needle may mean longer treatments.  Some people prefer to insert their own needles, which requires training to learn how to prevent infection and protect the va

scular access. You can also learn a “ladder” strategy for needle  placement in which you “climb” up the entire length of the fistula, session by session, so you butwon’t insert weaken the needlean area precisely with a grouping into the ofsame needle hole sticks. made by the previous needle stick. An alternative approach is the “buttonhole” strategy in which you use a limited number of sites Complications vascular access: o All three types of vascular access AV fistula, AV graft, and venous catheter can have complications that require further treatment or surgery. The most common complications are access infection and low blood flow— due to blood clotting in the access. — o Venous catheters are most likely to develop infection and clotting problems that may require medication and catheter removal or replacement. o AV grafts can also develop low blood flow, an indication of clotting or narrowing of the access. In this situation, the AV graft may require angioplasty, a procedure to widen the small segment that is narrowed. Another option is to perform surgery on the AV graft and replace the narrow segment. o Infection and low blood flow are much less common in properly formed AV fistulas than in AV grafts and venous catheters. Still, having an AV fistula is not a guarantee against complications.

Care of vascular access:

 Make sure that nurse or technician checks your access before each treatment.  Keep the access clean at all times.  Use the access site only for dialysis.  Be careful not to bump or cut the access.  t anyone put a blood pressure cuff on the access arm.  welry or tight clothes over the access site.  Don’tt lesleep with the access arm under the head or body.  Don’t wear je objects or put pressure on the access arm.  CheckDon’ the pulse in the access every day. Don’t lift heavy c) Endoluminal ultrasound.

Answer.  Diagnostic procedure of choice in evaluation of anorectal disorders.  Best imaging modality for staging of rectal cancer.  Invaluable in workup for fecal incontinence and suppurative anorectal conditions. o Introduced in 1952 o Used in Clinical Practice since 1983. o 3D ultrasound recently developed. o Currently being evaluated in efficacy compared to 2D ultrasound and MRI.

 Equipment and Technique: . Endocavitary probe with rotating transducer. . 7or 10 MHz Transducer. . Latex Balloon covering the probe which is filled with water to distend the rectum. . Phosphossoda enema for bowel prepLeft lateral decubitus position. . DRE and . . Wide bore proctoscope to facilitate positioning of probe above lesion. . Balloon is filled with 30-60 cc of water and real time image obtained.

 Image Interpretation: . 5 Layer Model.  Beynon and Colleagues:  Three hyperechoic (white) layers separated by two hypoechoic (black) layers.  1st White layer: Interface between balloon and mucosa.  2nd black layer: mucosa and muscularis.  3rd white layer: submucosa.  4th black layer: Muscularis propria.  5th white layer: interface between muscularis propria & fat.

 Accuracy of ERUS: Accurate in determining depth of invasion. Significant learning curve. Moderately accurate in assessment of Lymph Node involvement. Highly operator dependent.

 Factors for Misinterpretation: Close proximity to anal Artifact from air or Post-biopsy or surgical Hemorrhage. verge. stool. changes. Improper balloon Refraction artifacts. Pedunculated inflation. tumors.

 Post-op Follow up: o ERUS may improve ability to diagnose recurrence in 30% o Baseline Ultrasound at 3months o Optimal interval and length of time for serial follow up has not been determined. o Suggested every 3-4 months for first 2 years then every 6 months for 5 years d) Incidentalomas.

Answer. The unexpected discovery of adrenal tumors on diagnostic imaging studies for nonadrenal clinical disorders is known as adrenal incidentalomas. These clinically inapparent adrenal tumors are becoming a more common clinical finding because of improvements in imaging technique and technology and the increasing use of imaging studies in clinical practice. This increasing incidence is best reflected in the medical literature; there has been a twentyfold increase in studies published concerning adrenal incidentalomas over the past three decades.

An adrenal incidentaloma is a mass lesion greater than 1 cm in diameter, serendipitously discovered by radiologic examination. This entity is the result of technological advances in imaging such as computed tomography (CT) and magnetic resonance imaging (MRI). Discovery of an adrenal mass raises two questions that determine the degree of evaluation and the need for therapy.

 Is it malignant? And Is it functioning?

Differential Diagnosis of Adrenal Incidentalomas  Primary Adrenal  Metastasis Non funcioning Functioning o Lung o Cortical /carcinoma o Cortical adenoma/carcinoma: o Kidney o Cyst hypercortisolism, o Breast o hyperaldosteronism o Gastrointestinal o Ganglioneuroma o Pheochromocytoma o Melanoma o Hemorrhage/hematoma

THE WEST BENGAL UNIVERSITY OF HEALTH SCIENCES MS (General Surgery) Examination, 2015 PAPER II

Time Allowed: 3 Hours Full Marks: 100 Attempt all questions

1. Define early breast carcinoma and discuss its management. 8+12 2. What are the causes of enterocutaneous fistula? Classify them and give an outline of their management. 5+5+10 3. Write short notes of the following: 5 x 6 (a) Venous ulcer. (b) tumour. (c) Hypercalcemic crisis. (d) thyroid. (e) Raynaud’s phenomenon.

1 4. Answer briefly on the following: 4 x 7 /2 (a) Pancreatic necrosectomy. (b) Hiatus hernia. (c) Solitary Rectal ulcer syndrome. (d) Management of GIST. THE WEST BENGAL UNIVERSITY OF HEALTH SCIENCES

MS (General Surgery) Examination, 2015

April 2015

PAPER II

Time Allowed: 3 Hours Full Marks: 100 Attempt all questions

1. Define early breast carcinoma and discuss its management. 8+12

Answer. TNM Staging System for Breast Cancer

Primary tumor (T) Definitions for classifying the primary tumor (T) are the same for clinical and for pathologic classification. If the measurement is made by physical examination, the examiner will use the major headings (T1, T2, or T3); if other measurements, such as mammographic or pathologic measurements, are used, the subsets of T1 can be used. Tumors should be measured to the nearest 0.1-cm increment TX Primary tumor cannot be assessed T0 No evidence of primary tumor Tis Carcinoma in situ Tis(DCIS) Tis(LCIS) Lobular carcinoma in situ Tis(Paget's) Paget's disease of the nipple with no tumor (Note: Paget's disease associated with a tumor is classified according to the size of the tumor) T1 Tumor 2 cm or less in greatest dimension T1mic Microinvasion 0.1 cm or less in greatest dimension T1a Tumor more than 0.1 cm but not more than 0.5 cm in greatest dimension T1b Tumor more than 0.5 cm but not more than 1 cm in greatest dimension T1c Tumor more than 1 cm but not more than 2 cm in greatest dimension T2 Tumor more than 2 cm but not more than 5 cm in greatest dimension T3 Tumor more than 5 cm in greatest dimension T4 Tumor of any size with direct extension to (a) chest wall or (b) skin, only as described below T4a Extension to chest wall, not including pectoralis muscle T4b Edema (including peau d'orange), or ulceration of the skin of the breast, or satellite skin nodules confined to the same breast T4c Both T4a and T4b T4d Inflammatory carcinoma Regional lymph nodes—Clinical (N) NX Regional lymph nodes cannot be assessed (e.g., previously removed) N0 No regional lymph node metastasis N1 Metastasis to movable ipsilateral axillary lymph node(s) N2 Metastases in ipsilateral axillary lymph nodes fixed or matted, or in clinically apparenta ipsilateral internal mammary nodes in the absence of clinically evident axillary lymph node metastasis

N2a Metastasis in ipsilateral axillary lymph nodes fixed to one another (matted) or to other structures N2b Metastasis only in clinically apparenta ipsilateral internal mammary nodes and in the absence of clinically evident axillary lymph node metastasis

N3 Metastasis in ipsilateral infraclavicular lymph node(s) with or without axillary lymph node involvement, or in clinically apparent a ipsilateral internal mammary lymph node(s) and in the presence of clinically evident axillary lymph node metastasis; or metastasis in ipsilateral supraclavicular lymph node(s) with or without axillary or internal mammary lymph node involvement

N3a Metastasis in ipsilateral infraclavicular lymph node(s) N3b Metastasis in ipsilateral internal mammary lymph nodes(s) and axillary lymph node(s) N3c Metastasis in ipsilateral supraclavicular lymph node(s) Distant metastasis (M) MX Distant metastasis cannot be assessed M0 No distant metastasis M1 Distant metastasis TNM Stage Groupings

Stage 0 Tis N0 M0 Stage I T1a N0 M0

Stage IIA T0 N1 M0 T1a N1 M0

T2 N0 M0 Stage IIB T2 N1 M0 T3 N0 M0 Stage IIIA T0 N2 M0 T1a N2 M0

T2 N2 M0 T3 N1 M0 T3 N2 M0 Stage IIIB T4 N0 M0 T4 N1 M0 T4 N2 M0 Stage IIIC Any T N3 M0 Stage IV Any T Any N M1 a T1 includes T1 mic.

Early Invasive Breast Cancer (Stage I, IIa, or IIb)

Management: Confirmation of diagnosis by: Triple assessment – history and clinical examination, digital mammography and core needle biopsy.

Investigations:

 Complete blood cell count, complete metabolic panel, and chest x-ray.  A bone scan if the alkaline phosphatase or calcium level is elevated.  CT scan of the liver if liver function panel is abnormal.  CT scan of the thorax to exclude presence of lung secondaries.

Surgical options:

o Mastectomy with or without reconstruction. . Modified radical mastectomy (MRM) involves total (simple) mastectomy and axillary lymph node dissection. It is indicated for patients with clinically positive lymph nodes or a positive axillary node based on previous SLNB or FNAB. . Total (simple) mastectomy with SLNB is for patients with a clinically negative axilla. A skin- sparing mastectomy (preserves skin envelope and inframammary ridge) may be performed with immediate reconstruction, resulting in improved cosmesis: The nipple-areolar complex, a rim of periareolar breast skin, and any previous excisional biopsy or partial mastectomy scars are excised. . Immediate reconstruction at the time of mastectomy should be offered to eligible patients. Options include latissimus dorsi myocutaneous flaps, transverse rectus abdominis myocutaneous flaps, and inflatable tissue expanders followed by exchange for saline or silicone implants. Immediate reconstruction has been shown not to affect patient outcome adversely. The detection of recurrence is not delayed, and the onset of chemotherapy is not changed. . Follow-up after mastectomy: physical examination every 3 to 6 months for 3 years, then every 6 to 12 months for the next 2 years, and then annually. Mammography of the contralateral breast should continue yearly. Regular gynecologic follow-up is recommended for all women (tamoxifen increases risk of endometrial cancer). o Breast conservation therapy (BCT): partial mastectomy and SLNB (or axillary lymph node dissection) followed by breast irradiation.

Contraindications to BCT:

Pregnancy Previous RT Large breast size Multicentric disease Large tumour/ breast ratio Central tumour Diffuse indeterminate Collagen vascular disease micro-calcification

For patients with large tumors who desire BCT, neoadjuvant chemotherapy or neoadjuvant hormonal therapy may be offered to attempt to reduce the size of the tumor to make BCT attempt possible. o Management of the axilla: Approximately 30% of patients with clinically negative exams will have positive lymph nodes in an axillary lymph node dissection (ALND) specimen. The presence and number of lymph nodes involved affect staging and thus prognosis. . ALND. Patients with clinically positive lymph nodes or with positive SLN should undergo ALND for local control.

ALND involves the following:

. Removal of level I and level II nodes and, if grossly involved, possibly level III nodes. Motor and sensory nerves are preserved unless there is direct tumor involvement. . An ALND should remove 10 or more nodes. The number of nodes identified is often pathologist dependent. . Patients with 4 or more positive lymph nodes should undergo adjuvant radiation to the axilla. Selective patients with 1 to 3 positive nodes may also benefit from radiation therapy to the axilla. . Intraoperative complications: potential injury to the axillary vessels and neuropathy secondary to injury to the motor nerves of the axilla (the long thoracic, thoracodorsal, and medial pectoral nerves).

Adjuvant chemotherapy is given in appropriate patients after completion of surgery. o All node-positive patients should receive adjuvant chemotherapy. . Regimens are guided by the tumor biomarkers. Typical regimens comprise four to eight cycles of a combination of cyclophosphamide and an anthracycline, followed by a taxane administered every 2 to 3 weeks. . Patients with ER-positive tumors receive adjuvant hormonal therapy for 5 years. Tamoxifen is given to premenopausal women, and aromatase inhibitors are given to postmenopausal women (aromatase inhibitors are not used in premenopausal women). . (In postmenopausal women older than 70 years, chemotherapy is performed less frequently. In postmenopausal women with tumors with ER or PR positivity, tamoxifen or an aromatase inhibitor is frequently the sole adjuvant medical therapy.) . In patients with Her2/neu-positive tumors, polychemotherapy is combined with biological therapy targeting the Her2/neu protein: Trastuzumab is a recombinant monoclonal antibody that binds to Her2/neu receptor to prevent cell proliferation. Adding trastuzumab to a chemotherapy regiment of doxorubicin, cyclophosphamide, and paclitaxel was associated with an increase in the disease-free survival by 12% and a 33% reduction in the risk of death at 3 year.

Node-negative patients may have increased disease-free survival from adjuvant chemotherapy and/or hormonal therapy.

. Up to 30% of node-negative women die of breast cancer within 10 years if treated with surgery alone. . Node-negative patients who are at high risk and benefit the most from adjuvant chemotherapy include those with tumors greater than 1 cm, higher tumor grade, Her2/neu expression, aneuploidy, Ki-67 expression, increased percentage in S phase, lymphovascular invasion, and ER/PR-negative tumors. . Polychemotherapy in combination with tamoxifen was superior to tamoxifen alone in increasing disease-free and overall survival, especially in ER-negative patients, regardless of tumor size. o Adjuvant whole-breast radiation after BCT decreases the breast cancer recurrence rate from 30% to less than 7% at 5 years.

2. What are the causes of enterocutaneous fistula? Classify them and give an outline of their management. 5+5+10 Answer. Enterocutaneous fistula is one in which there is communication between the intestinal tract and the skin.  Enterocutaneous fistulas result from several processes:  Diseased bowel extending to surrounding structures;  Extraintestinal disease involving otherwise normal bowel;  Trauma to normal bowel including inadvertent or missed enterotomies; or  Anastomotic disruption following surgery for a variety of conditions.

CAUSES RISK FACTORS Inflammatory processes, such as infection or inflammatory History bowel disease (most often Crohn's disease) of radiation Radiation to the abdomen (most often given as part of cancer Poor treatment) nutrition, Old age. Trauma, especially penetrating wounds such as those due to a Malignancy. stabbing or gunshot

Fistulas between the alimentary tract and skin may be classified as postoperative or spontaneous.

I. Anatomic Classification:

(a) External fistulas: Fistulas which communicate with the skin.

(b) Internal fistulas: Fistulas which communicate with intraperitoneal or intrathoracic organs. Internal fistulas that bypass only short segments of bowel may not be symptomatic; however, internal fistulas of bowel that bypass significant length of bowel or that communicate with either the bladder or vagina typically cause symptoms and become clinically evident.

Internal fistulas may be:

 Oral, Pharyngeal, and Esophageal Fistulas

 Gastric Fistulas

 Small Bowel Fistulas

 Colonic Fistulas

II. Physiologic Classification: Fistulas may be divided into

 high-output (>500 mL per day),

 moderate-output (200–500 mL/day), and

 low-output (<200 mL/day) groups.

Predictive Factors for Spontaneous Closure and/or Mortality

Factor Favorable Unfavorable Organ of origin Oropharyngeal Gastric Esophageal Lateral duodenal Duodenal stump Ligament of Treitz Pancreaticobiliary Ileal Jejunal Colonic Etiology Postoperative Malignancy Appendicitis Inflammatory bowel disease Diverticulitis Output Low (<200–500 mL/day) High (>500 mL/day) Nutritional status Well-nourished Malnourished Transferrin >200 mg/dL Transferrin <200 mg/dL Sepsis Absent Present State of bowel Healthy adjacent tissue Diseased adjacent bowel Intestinal continuity Distal obstruction Absence of obstruction Large abscess Bowel discontinuty Previous irradiation Fistula Tract >2 cm Tract <1 cm characteristics Bowel wall defect <1 cm2 Defect >1 cm2

Epithelialization Foreign body Miscellaneous Original operation performed at same Referred from outside institution institution

Prevention:

Proper preoperative patient preparation and meticulous surgical technique will lessen the risk of postoperative fistula formation. In the elective setting, operation may be delayed to allow for normalization of nutritional parameters, thus optimizing wound healing and immune function. Several nutritional characteristics have been suggested to increase the risk of anastomotic breakdown:

1. Weight loss of 10–15% of total body weight over 3–4 months; 2. Serum albumin less than 3 mg/dL; 3. Serum transferrin less than 220 mg/dL; 4. Anergy to recall antigens; or 5. Inability to perform activities of daily living due to weakness or fatigue.

Management Phases of Fistula

Phase Goals Time Course Recognition/stabilization Resuscitation with crystalloid, colloid, or 24–48 hours blood Control of sepsis with percutaneous or open drainage and antibiotics Electrolyte repletion Provision of nutrition Control of fistula drainage Commencement of local skin care and protection Investigation Fistulogram to define anatomy and 7–10 days characteristics of fistula CECT is also helpful. Decision Evaluate the likelihood of spontaneous 10 days to 6 weeks closure Decide duration of trial of nonoperative management Definitive management Plan operative approach When closure unlikely or Refunctionalization of entire bowel after 4–6 weeks Resection of fistula with end-to-end anastomosis Secure abdominal closure Gastrostomy and jejunostomy Healing Continue nutritional support until full oral 5–10 days after closure nutrition achieved until full oral nutrition Zinc supplementation Psychological and emotional support

3. Write short notes of the following: 5 x 6 (a) Venous ulcer. (b) Carcinoid tumour. (c) Hypercalcemic crisis. (d) Medullary carcinoma thyroid. (e) Raynaud’s phenomenon.

Answer.

(a) Venous ulcer.

Introduction:  Chronic venous disease, including chronic venous insufficiency and chronic venous ulceration, is a common and important medical problem that causes significant morbidity. Venous ulcers are expensive to treat and adversely impact patient’s quality of life.  Venous ulcers occur more commonly in the elderly, the peak prevalence occurring between ages 60 and 80 years.  A venous leg ulcer can develop after a minor injury if there is a problem with the circulation of blood in your leg veins. If this happens, the pressure inside the veins increases.

Risk factors:  Obesity – this increases the risk of high pressure in the leg veins  Not being able to move for a long period of time – this can weaken the calf muscles, which can affect circulation in the leg veins  Having previously had deep vein thrombosis (DVT)– blood clots that develop in the leg, which can damage valves  Varicose veins – swollen and enlarged veins caused by malfunctioning valves  Previous injury to the leg, such as a broken or fractured bone, which may cause DVT  Previous surgery to the leg, such as a hip replacement or knee replacement, which can prevent you from moving about  Increasing age – as people generally find it harder to move about as they get older

Pathophysiology:  Venous hypertension: Deep vein thrombosis, perforator insufficiency, superficial and deep vein insufficiencies, arteriovenous fistulas and calf muscle pump insufficiencies lead to increased pressure in the distal veins of the leg and finally venous hypertension.  Fibrin cuff theory: Fibrin gets excessively deposited around capillary beds leading to elevated intravascular pressure. This causes enlargement of endothelial pores resulting in further increased fibrinogen deposition in the interstitium. The "fibrin cuff" which surrounds the capillaries in the dermis decreases oxygen permeability 20-fold. This permeability barrier inhibits diffusion of oxygen and other nutrients, leading to tissue hypoxia causing impaired wound healing.  Inflammatory trap theory: Various growth factors and inflammatory cells, which get trapped in the fibrin cuff promote severe uncontrolled inflammation in surrounding tissue preventing proper regeneration of wounds. Leukocytes get trapped in capillaries, releasing proteolytic enzymes and reactive oxygen metabolites, which cause endothelial damage. These injured capillaries become increasingly permeable to various macromolecules, accentuating fibrin deposition. Occlusion by leukocytes also causes local ischemia thereby increasing tissue hypoxia and reperfusion damage.  Dysregulation of various cytokines.  Dysregulation of various pro-inflammatory cytokines and growth factors like tumor necrosis factor-α (TNF-α), TGF-β and matrix metalloproteinases lead to chronicity of the ulcers.  Miscellaneous: Thrombophilic conditions like factor V Leiden mutation, prothrombin mutations, deficiency of antithrombin, presence of antiphospholipid antibodies, protein C and S deficiencies and hyperhomocysteinemia are also implicated.

Clinical features: Venous leg ulcers are open, often painful, sores in the skin that take more than four to six weeks to heal. They most often develop on the inside of the leg, just above the ankle.

Symptoms:  Swollen ankles (oedema)  Discolouration and darkening of the skin around the ulcer  Hardened skin around the ulcer, which may make your leg feel hard and resemble the shape of an upside-down champagne bottle  A heavy feeling in your legs  Aching or swelling in your legs  Red, flaky, scaly and itchy skin on your legs (varicose eczema)  Swollen and enlarged veins on your legs (varicose veins)  An unpleasant and foul-smelling discharge from the ulcer

Signs of an infection: A venous leg ulcer can be susceptible to bacterial infection. Symptoms of an infected leg ulcer can include:  Worsening pain  A green or unpleasant discharge coming from the ulcer  Redness and swelling of the skin around the ulcer  A high temperature (fever) Nonsurgical Treatment A. Infected ulcers

 Necessitate treatment of the infection first.  Staphylococcus aureus, Streptococcus pyogenes, and Pseudomonas species are responsible for most infections.  Usually treated with local wound care, wet-to-dry dressings, and oral antibiotics.  Topical antiseptics should be avoided.  Severe infections require intravenous antibiotics.

B. Leg elevation Leg elevation can temporarily decrease edema and should be instituted when swelling occurs. This should be done before a patient is fitted for stockings or boots. C. Compression therapy Compression therapy is the primary treatment for CVI.

 Elastic compression stockings o Fitted to provide a compression gradient from 30 to 40 mm Hg, with the greatest compression at the ankle. o Donned on arising from bed and removed at bedtime. o Effective in healing ulcers but can take months to obtain good results. o Stockings do not correct the abnormal venous hemodynamics and must be worn after the ulcer has healed to prevent recurrence. o Principal drawback is patient compliance.  Unna boots o Paste gauze compression dressings that contain zinc oxide, calamine, and glycerin. o Used to help prevent further skin breakdown. o Provide nonelastic compression therapy. o Changed once or twice a week. o Healing time for ulcers is less than that of elastic compression alone.  Pneumatic compression devices o Provide dynamic sequential compression. o Used primarily in the prevention of deep vein thrombi in hospitalized patients. o Also used successfully to treat venous insufficiency.

D. Topical medications

 Largely ineffective as a stand-alone therapy for venous stasis ulcers.  Topical therapy is directed at absorbing wound drainage and avoiding desiccation of the wound.  Antiseptics can be counterproductive. Hydrogen peroxide, povidone-iodine, acetic acid, and sodium hypochlorite are toxic to cultured fibroblasts and should be used for the shortest duration necessary to control ulcer infection.

Surgical Therapy: Skin grafting.

(b) Carcinoid tumour. Introduction:

 Carcinoid tumours, because of their neuroendocrine origin, have neurosecretory capabilities and hence the ability to secrete a variety of peptides and bioactive amines.  Chief among these are 5-hydroxytryptamine( 5-HT) or serotonin, histamine, norepinephrine, ACTH, etc.  They arise from cells of diffuse neuroendocrine system. These cells are found throughout the GI tract and bronchopulmonary system.  Carcinoid tumours occur most frequently in the GI tract, in bronchopulmonary system, and other locations like ovary, gallbladder, thymus, testis, cervix, breast etc. Pathology:  Grossly carcinoid tumours appear as solid, yellow-tan lesions. They are usually solitary, except in stomach and ileum where they may be multicentric.  They are often submucosal and they may cause ulceration. There is intense desmoplasia within and surrounding the tumour.  Histologically they appear as uniform, small, round cells with rare mitotic figures. Two types of silver staining are commonly used to identify neuroendocrine cells—argyrophil reactions and argentafin staining. Foregut and hindgut carcinoid are typically argyrophil and midgut (serotonin secreting) are argentafin.  Immunohistochemical stains like chromogranins A, B, and C are reliable serum markers for detection of neuroendocrine tumours. About 80% of carcinoids will have elevated levels of chromogranin A. A high level of chromogranin A is a poor prognostic indicator in individuals with metastatic disease.

Clinical Presentation:  Carcinoid tumours are often discovered incidentally during surgery, endoscopic procedures or imaging studies.  They may also present either by causing symptoms due to mass effect or due to their secretory products.  Mass effect may occasionally cause pain but more commonly causes luminal obstruction (due to tumour growth or scarring).  Symptoms are often non specific consisting of malaise,vague abdominal pain or weight loss.  Carcinoid syndrome is an unusual presentation of GI carcinoids.  The classic product serotonin may produce diarrhea. ACTH producing tumours will cause Cushing’s syndrome, while gastrin producing tumours may cause ZE syndrome.

Classification:  Carcinoid tumours have been classified according to their embryologic region of origin: foregut, midgut, hind gut.  Foregut carcinoids include those arising in esophagus, stomach, pancreas, and duodenum.  Midgut carcinoids comprise of those arising in the distribution of superior mesenteric artery distribution, including appendix and ileum.  Hind gut carcinoids include those arising in the distribution of inferior mesenteric artery distribution including the rectum.

Esophagus  Esophageal carcinoids are extremely rare. is the most common presentation.  Mostly treated by esophagogastrectomy. Stomach  Stomach is the most common foregut location for carcinoid tumours.  Pernicious anaemia and chronic atrophic gastritis are risk factors for the development of gastric ECL cell tumour. This may be secondary to the hypergastrinemia caused by the absence of significant acid output in such patients.  They are much more common in those patients with MEN- type 1.  The average age of presentation of patients with gastric carcinoid tumours is 62 years.  Patients are often asymptomatic, carcinoids often found incidentally during upper GI .  When symptoms occur, they are usually dyspeptic.  EUS can be helpful in defining the depth of invasion and providing deep submucosal  biopsy specimen. CT and MRI may be helpful in finding hepatic metastasis. All patients with gastric carcinoids have elevations of chromogranin A levels.  Management of gastric carcinoid tumours is controversial.  Carcinoids secondary to pernicious anemia/ atrophic gastritis and MEN- 1 tend to have a more benign course, with occasional spontaneous regression. Pancreas  The differentiation between pancreatic carcinoids and other neuroendocrine tumours (NETs) of pancreas is primarily a matter of definition.  Pancreatic carcinoids have been defined as a tumour with the histologic features of a neuroendocrine tumour and with obviously increased serotonin metabolism.  Pancreatic carcinoids tend to present late. Abdominal pain, diarrhea, and weight loss are the most common presenting symptoms.  Most patients presents with metastasis. Treatment is by surgical resection.  Pancreaticoduodenectomy is often required.  It has better survival rates than those of other pancreatic malignancies but worse than those of carcinoids in other sites. Duodenum  Carcinoids of duodenum represent approximately 2% of carcinoids.  Duodenal carcinoids most commonly discovered during endoscopy.  They rarely produce serotonin and therefore virtually no association with carcinoid syndrome. Half of the cases produce gastrin.  Ampullary carcinoids frequently present with jaundice. Approximately 25% of patients  with periampullary carcinoids have von Recklinghausen’s disease.  The risk of metastasis increases with increased size of the primary tumour.  The treatment of duodenal and ampullary carcinoid tumour is by resection. Small Intestine  Ileum is the most common site for carcinoid tumours, accounting for almost 30 % of all carcinoids.  Carcinoids form approximately 28% to 38% of all small bowel cancers.  Average age of presentation is 62 to 65 years. Small bowel carcinoids are multifocal in 25% of cases.  They present as vague abdominal pain, intermittent intestinal obstruction, infarction of small bowel due to superior mesenteric artery occlusion.  Overt carcinoid syndrome is uncommon as an initial presentation.  Carcinoid syndrome usually indicates hepatic or retroperitoneal metastasis and signifies an unfavourable outcome.  If found early, primary tumour and the associated lymphatics should be resected surgically. Appendix  It is a less common site of GI carcinoids.  The proportion of appendiceal tumours that are carcinoids is high.  Majority are less than 1 cm in size.  Frequency of distant metastasis increases with the size of the tumour.  They cause symptoms early due to appendiceal luminal obstruction with consequent appendicitis. Have a favourable prognosis.  Simple appendicectomy is indicated for tumours less than 1 cm size and a right hemicolectomy for tumours more than 2 cm. size. Colon  Colonic carcinoids account for less than 10% of carcinoid tumours.  They occur more commonly on right side, about two thirds involving ascending or proximal transverse colon.  They tend to present as larger lesions than most oher carcinoids, averaging around 5 cm.  Symptoms are usually due to bulky advanced lesions.  Advanced lesions may cause pain or colonic obstruction.  Surgical resection and lymph node clearance should be performed. No adjuvant treatment is effective. Five year survival is worst Rectum  Constitutes 19 % of all carcinoids.  More than 80 % are still localized at the time of diagnosis.  Appear as local nodules. Tumours may produce symptoms as bleeding, pain, or decreased stool caliber, weight loss, changing bowel habits, may be detected incidentally.  Carcinoid syndrome is distinctly uncommon due to rectal lesions.  Rectal carcinoids have been treated endoscopically, by local excision, (lesions less than 2 cm) and by radical excision with either a low anterior resection or APR (lesions larger than 2 cm).

Carcinoid Syndrome

Typical Carcinoid Syndrome: Most common clinical pattern, caused by metastatic midgut carcinoids. Serotonin is released into circulation. It is then converted into 5- hydroxyindoleacetic acid (5-HIAA),which is excreted in urine. Patients with typical carcinoid syndrome have elevated plasma and platelet serotonin and increased 5-HIAA in urine. Clinically it is characterized by flushing, diarrhea, non specific abdominal pain, bronchospasm, pellagra like skin lesions and progressive right heart failure.

Atypical Carcinoid Syndrome: Most often associated with foregut carcinoid tumours. It presents with a different pattern of flushing, it occurs again in head and neck region but it appears patchier. It also shows central clearing and increased pruritus. Asthma and/ or signs of peptic ulceration are occasional presentations due to increased systemic levels of histamine. Patients have high plasma levels of 5-HTP and normal levels of serotonin. Urinary levels of 5-HIAA are typically normal to slightly elevated.

Diagnosis: Based on typical clinical history, 24 hour urinary 5-HIAA levels.

Carcinoid Crisis:  Acute exacerbation of carcinoid syndrome may occur in a number of situations but most commonly in the setting of surgical or anaesthetic stresses.  Patients with known carcinoid tumours should receive somatostatin analogs before surgery to suppress mediator release.  Symptoms like hyperglycemia, hypertension, bronchospasm are treated by routine medical means.

Imaging Studies  Visualization of primary carcinoid tumour is more difficult than identification of metastatic disease.  CT scan and MRI are non specific and often non –localizing. They are useful when lesions are more than 1 cm in size. May be used to identify metastatic disease.  Somatostatin Receptor Scanning using radiolabelled somatostatin analogs (Indium¹¹¹ labeled pentetreotide, I123 labeled tyrosine) is helpful in localizing primary occult tumours and in staging of patients with known metastatic disease.

Treatment of Metastatic Disease / Carcinoid Syndrome: Majority of such patients are not the candidates for curative treatment. The focus of therapy is palliation of symptoms.

1. Surgery: For hepatic metastasis: Resection , OLT. Cytoreductive therapy. Hepatic artery embolization and chemoembolization. 2. Immunotherapy: ? Interferon ( controversial). 3. Chemotherapy : Marginal benefit. 4. Pharmacologic Therapy: Mainstay of palliative therapy. . Octreotide, lantreotide(long acting)---- Somatostatin Analogs. . Methysergide, cyproheptadine, ketanserin, ondansetron--- serotonin receptor blockers. . Parachlorophenylalanine--- inhibitor of serotonin synthesis. 5. Radiotherapy: For palliation of metastasis to bone and central nervous system.

(c) Hypercalcemic crisis.

Introduction: Hypercalcemic crisis or severe hypercalcemia represents a life-threatening emergency. The most common cause is hypercalcemia of malignancy, although granulomatous diseases, previously undetected primary hyperparathyroidism, medication-induced hypercalcemia, and a few rarer causes may result in this endocrine emergency as well.

 The clinical presentation and prognosis depend on the acuity of the development of hypercalcemia, the degree of hypercalcemia, and the underlying cause.

Clinical presentation: The clinical presentation of patients with hypercalcemic crisis varies depending once again on the underlying cause and degree and rapidity of the hypercalcemia. Most patients experience some constitutional symptoms, neurologic symptoms, gastrointestinal symptoms, and renal manifestations of hypercalcemia.

Treatment:

Medical treatment  Hypercalcaemic crisis needs aggressive rehydration.  Establish large calibre IV access. Give 1L in first hour, further 4-6L in first 24h.  Monitor urine output and central venous pressure until normalized.  Frusemide can be added to increase urinary excretion of calcium once rehydration is adequate.  Bisphosphonates (e.g. IV pamidronate) should be avoided in PHPT when parathyroidectomy is anticipated, since they impair the ability to maintain normocalcaemia after the excision of an overactive parathyroid adenoma.

(d) Medullary carcinoma thyroid.

Introduction:

 Medullary Thyroid Carcinoma (MTC) is an uncommon tumor accounting for 3% to 5 % of all thyroid tumors.  These tumors arise from the para-follicular or C cells with ability to produce different bioactive substances like calcitonin (TC) and CEA3. TC is as an ideal tumor marker in early diagnosis, in patients‘ follow up and evaluation of their tumor.  Majority of MTC ( 80%)are sporadic variety where as 20% of these tumors have hereditary background.  The hereditary form are autosomal dominant traits associated with germ line mutation of RET proto-oncogene.  MTC arises in upper 2/3rd of the thyroid gland where C cells reside. These cells are of neuro- ectodermal in origin. Clinical Presentation :  There are three principle components to the primary diagnosis of MTC : detection, staging and evaluation for hereditary disease.  80% of MTC occur sporadically, presenting with painless, progressive nodule in the thyroid gland.  Affecting predominantly the patients in 5th or 6th decade of life  Female to male preponderance as 1.5:1.  At the time of presentation as palpable nodule, nearly 50% of patients have lymph node metastasis and approximately 10% have lung, liver or bony metastasis.  Local effects viz. dysphagia, recurrent laryngeal nerve palsy, tracheal obstruction causing stridor is seen only in very few cases.  Paraneoplastic manifestations such as diarrhea and flushing is seen usually in advanced cases.  As MTC does not have the ability to concentrate iodine, thyroid scan shows it as cold nodule which on ultra sound is found to be solid.  Plain x-ray of neck may sometimes reveal characteristic dense, coarse calcification which is distinct from the fine calcification of papillary carcinoma. CT scan may reveal similar calcification at the metastatic sites in lungs, liver or mediastinum.

Diagnosis:  Fine Needle Aspiration Cytology (FNAC) shows highly cellular smears constituted by heterogeneous oval, spindle shaped or plasmacytoid cells lacking papillary or follicular pattern and devoid of colloid. Calcitonin immuno-histochemical staining on the specimen confirms the cytological diagnosis. Elevated basal serum calcitonin levels further corroborates the diagnosis of MTC.  Several imaging methods have been found to be useful in assessing the extent of disease. CT Scan, MRI and ultra-sonography will help in detecting local invasion or regional lymphadenopathy. CT or MRI are recommended for detecting mediastinal and lung metastasis.  Nuclear medicine studies including 131In-octeotride 13, and to a lesser extent pentavalent dimercaptosuccinic acid 14 and 131 I-metaiodobenzylguanidine 15 appear to have a more limited role in the primary work up, but may be useful in identifying residual disease.

According to TNM classification, the staging of the MTC is as follows: The“T” Stage : T1- tumor 1 cm. or less within the thyroid gland T2- tumor sized 1-4 cms. within the thyroid gland T3- tumor sized greater than 4 cms. within the thyroid gland T4-tumor of any size extending outside of thyroid gland itself The “N” Stage: N0- No spread to lymph nodes N1a-Ipsilateral neck nodes involvement N1b-Bilateral or Contra lateral lymph node involvement The “M” Stage : M0- No tumor spread to other organs M1- Tumor spread to other organs · Stage I: T1N0M0 · Stage II : T2N0M0 · Stage III : any N1 M0 · Stage IV : any M1 The last but not the least task in diagnostic work-up for MTC is the evaluation for hereditable disease. Screening test for pheochromocytoma either with urinary catecholamines or metanephrines is the least one should do not only to rule out the hereditary possibility but also to avoid the harmful effects of high levels of circulating catecholamines during thyroidectomy.

Hereditary MTC has three variants- MEN 2A: MTC, Pheochromocytoma, Hyperparathyriodism MEN 2B : MTC, Pheocromocytoma, Mucosal neuromas FMTC : Isolated Familial Medullary Thyroid Carcinoma o MEN 2A is the most common type of hereditary MTC with C Cell hyperplasia and MTC as its cardinal lesion in95% of individuals. Pheochromocytoma and hyperparathyroidism are less penetrant with life time incidence of 40% to 60% and 10% to 25% on an average respectively. o The thyroid tumors in MEN 2B are usually more aggressive, with widespread metastasis at an early age as compared to MEN 2A. Pheochromocytoma appears at an earlier age and are almost bilateral (versus a 50% bilateral incidence in MEN 2A ).

Treatment:  Surgery is the primary modality of treatment of MTC whether sporadic or inherited. Pheochromocytoma must be carefully excluded and if present treated surgically before embarking on the surgical management of MTC to avoid intra-operative catecholamine crisis.  All patients with palpable disease localized to thyroid must undergo total thyroidectomy with central node dissection from the hyoid bone to the innominate veins and ipsilateral mid jugular node sampling and if found positive ,then functional ipsilateral lymph node dissection preserving jugular vein, sternocleidomastoid muscle and accessory nerve is recommended.  A prophylactic contra-lateral neck dissection should be done when the primary tumor is bilateral and when there is extensive lymph adenopathy on the side of the primary tumor.  Parathyroids needs to be identified and preserved in absence of clinical hyperparathyroidism.or frank parathyroid enlargement.In patient with hereditary MTC presenting with palpable disease, total thyroidectomy must be combined with bilateral functional neck dissection.  Clinically detectable cervical node metastasis calls for radical neck dissection of the affected site.  Patients with biochemical evidence of primary hyperparathyroidism must undergo bilateral neck exploration and total parathyroidectomy and autotransplantation of 30-60 mg. of most normal parathyroid tissue to the non dominant forearm if asymmetric parathyroid hyperplasia is present.  Patients with FMTC should have a continued follow up for pheochromocytoma and primary hyperparathyroidism. Prophylactic Thyroidedctomy : Prophylactic thyroidectomy has been recommended in hereditary MTC. MEN 2A. In most centers , thyroidectomy is performed in patients by the age of 5 years or when a mutation is identified.. FMTC : Some centers recommend management similar to that of MEN 2A MEN 2B : In most centers , surgery is performed with in first 6 months of life, preferably within first month , because of early age of MTC onset and particularly aggressive behavior. Treatment of Residual Disease :  By definition the surgical cure of MTC means normal post operative serum calcitonin levels. Elevation of post operative calcitonin levels or rising levels indicate persistent or recurrent disease.  The other sites of persistent or recurrent disease are mediastinal lymph nodes, lungs and liver. Liver metastasis are difficult to pick up with available imaging and nuclear medicine techniques.  Reoperation for minimal residual disease with curative intent is controversial.  Palliative debulking surgery in patients with advanced MTC does have a role in controlling compression on vital structures in the mediastinum and in large liver metastasis for poorly controlled diarrhea and flushing.  Cytoreductive procedures such as radiofrequency ablation or cryoablation for liver metastases should be considered in symptomatic patients to reduce tumor burden  External beam radiotherapy may give some relief.  Combination chemotherapy using decarbazine, cyclophosphamide and vincristine has been shown to exhibit some response in pheocromcytoma.  Chemotherapy or radioactive immunotherapy ( iodine131 labelled carcinoembryonic antigen monoclonal antibody) protocols have been advocated in patients with non-operable widely metastatic progressing medullary thyroid carcinoma.  Biological response modifiers such as somatostatin analogues, -interferon have been shown to be of some benefit in reliving the symptoms of paraneoplastic syndrome.  Monoclonal antibodies to CEA have been found to be unsuccessful.

Prognostic Factors:  Screening for MTC and early treatment ( total thyroidectomy with central node clearance ) gives nearly 100% cure rate.  A high proportion of patients continue to receive less than optimal initial surgical treatment.  Patients with postoperative hypercalcitonemia with out clinical or radiological evidence of residual tumor after apparently curative surgery enjoy a long term survival but have occult metastasis30.  Age at presentation and clinical staging have been found to independent predictors of survival.  Patients without capsular invasion have been shown to have 92 % survival at 10 years compared to only 67% in those with capsular invasion.  Calcitonin immunostaining is another important prognostic indicator. Calcitonin immunostaining if positive in more than 50% of cells is attended with 87% survival as compared to only less than 60% in those with less than 10% staining. Calcitonin levels per se are not of much predictive value.

(e) Raynaud’s phenomenon.

Introduction: • In 1862 Maurice Raynaud was the first to document that some individuals developed discoloration oftheir digits upon cold exposure, which was presumed to be an abnormal response of the central nervous system. • It is now recognized that this phenomenon is secondary to vasoconstriction of the digital arteries,precapillary arterioles, and cutaneous arteriovenous shunts. • Although the exact mechanism of this vasospastic abnormality remains elusive and in most patients the etiology ill defined, recent investigations suggest that elevated levels of homocysteine or recent viral infection may play a role in at least a subset of patients. • In addition, Raynaud's syndrome may occur in response to interferon treatment, as a paraneoplastic syndrome from non-small-cell lung cancer, or as a generalized vasospastic disorder associated with Prinzmetal's angina, migraines, or scleroderma. • In its classic form, Raynaud's syndrome presents with digital ischemia in response to a cold exposure. It typically commences in one or several fingers and then spreads to both hands. The duration of the attack is variable and culminates in a erythematous recovery known as reactive hyperemia. Before recovery, the digits can turn white, blue, and red. The lack of a clinical history of tricolor changes excludes the diagnosis of Raynaud's syndrome. • Women are more commonly affected than men, and although this gender discrepancy occurs in both pediatric and adult populations, the syndrome most commonly arises in the second and third decades of life. • The syndrome is more common in geographic regions with cooler daily temperatures and in winter months. Primary versus secondary Raynod’s syndrome: Raynaud's syndrome is classically divided into primary and secondary and each is defined by certain clinical criteria. Primary Raynaud's syndrome occurs in patients with symmetric attacks; absence of tissue necrosis, ulceration, or gangrene; the absence of a secondary cause on thebasis of history and physical examination; normal nail-fold capillaries; a negative test for antinuclear antibody; and a normal erythrocyte sedimentation rate. Secondary Raynaud's syndrome is suggested by an age at onset of older than 30 years; episodes that are intense, painful, asymmetric, or associated with ischemic skin lesions; clinical features suggestive of an underlying connective tissue disorder; specific autoantibodies; and evidence of microvascular disease on microscopy of nailfold capillaries. Recent work has suggested that duplex determination of carotid elasticity and stiffness may also assist in differentiating between the two forms. A clear distinction between primary and secondary Raynaud's syndrome may be difficult.

Management:  The initial therapy for Raynaud's syndrome is avoidance of a cold stimulus and use of appropriate attire that minimizes cold exposure such as gloves and stockings. Because Raynaud's syndrome is a sympathetic-mediated response, anxiety can provoke vasospasm. Thus reduction of anxiety or avoidance of stressful situations can assist in reducing symptom occurrence.  Although smoking has never been implicated as a causative factor in the development of Raynaud's syndrome, it adversely effects digital blood flow and should be avoided. In addition to these recommendations, a variety of pharmacologic and surgical approaches are available for managing this disorder.

Medication:  Nifedipine is a calcium channel blocker – a type of medication that encourages the blood vessels to dialate.  Angiotensin-converting enzyme (ACE) inhibitors  Fluoxetine – which was widely used in the treatment of depression  Sildenafil – which is used to treat erectile dysfunction (inability to get or maintain an erection) – 5 phosphodiesterase inhibitor.

Surgery: Surgery for Raynaud's is rare. It's usually only recommended if the symptoms are so severe that there's a risk the affected body part, such as your fingers, could lose their blood supply and begin to die.  Sympathectomy is sometimes used. It involves cutting the nerves causing the affected blood vessels to spasm.  The results of a sympathectomy are often only temporary and further treatment and possibly more surgery may be required after a few years. 1 4. Answer briefly on the following: 4 x 7 /2 (a) Pancreatic necrosectomy. (b) Hiatus hernia. (c) Solitary Rectal ulcer syndrome. (d) Management of GIST.

Answer. (a) Pancreatic necrosectomy.

Introduction: Pancreatic necrosis is a serious infection usually associated with acutepancreatitis. During recurring attacks of pancreatitis, tissue within thepancreas may die (necrotize) and later become infected. This condition is called acute necrotizing pancreatitis.

Rationale of Necrosectomy: Infected and necrotic pancreatic tissue serves as nidus for proinflammatory factors to be released in large amounts both locally and systemically. This elicits a severe inflammatory reaction thereby exacerbating or amplifying features of SIRS. Persistence of these effects eventually leads to septicemia and multiple organ dysfunction syndrome (MODS), eventually terminating to death. Therefore removal of this infected necrotic debris undoubtedly reduces the quantum of proinflammatory factors released into the system thereby reducing the systemic inflammatory response. A single sitting of necrosectomy may not always offer a complete eradication of the source. In most of the cases multiple sittings may be required to ensure complete eradication of the infected source.

Therapeutic options: 1) Open Surgical Necrosectomy:Through a midline incision the entire abdominal cavity is assessed. A diverting ileostomy is performed in those cases wherein the retrocolic area is also involved. Once the focus of necrosis is exposed, blunt necrosectomy is performed, which ensures atraumatic removal of necrotic tissue without damaging the residual pancreatic tissue. This is followed by one of the four techniques. a) Open Packing – The cavity is irrigated vigorously with saline and is packed with a non-adherent dressing.The patient needs to be brought to the operating room every 48 hours for further debridement and repacking until the process of necrosis is halted and no necrotic tissue debris are seen. The abdomen is closed over drains. b) Planned staged re- with repeated lavage –This method involves planned reoperations after primary necrosectomy on every alternate day until all devitalized tissue has been removed, growth of granulation tissue has commenced and there is substantial evidence that the necrotic process has been brought under control. Repeated opening of the peritoneal cavity may be cumbersome and cause irreversible damage to the abdominal wall leading to significant morbidity. This can be eased by using a zipper being attached to the incised edges. Peritoneal drains also help in drainage of infected material. c) Continuous lavage of the lesser sac and retroperitoneum is a very effective method of cleaning all necrotic residual debris. After primary necrosectomy multiple tube drains, usually 4, are kept in the retroperitoneal area in juxtaposition to the pancreatic remnant. Continuous irrigation with dialysis fluid or even normal saline measuring approximately 35 liters/day may help reduce the septic process significantly. The end point of lavage is a clear effluent from the drains. d) Closed Packing – After primary necrosectomy the cavity is irrigated and packed with large gauze mops. Drains are also kept. Drains are removed after 7 days along with removal of the packs. 2) Minimally Invasive Approach: The introduction of laparoscopy has added an additional modality to the therapeutic armamentarium. Minimally invasive techniques confirm significant advantage with respect to surgical outcome. However, the technique has its limitations and is best utilized if the pancreatic necrosis is <30% with massive fluid collections in the left retroperitoneal space. The advantages of laparoscopy are reduced inflammatory response to intervention, considerably reduced incidence of bacteremia, reduced rate of development of multiorgan failure, reduced rate of wound complications, shorter stay in ICU and fast recovery. The limitations of this method are availability of expertise and lack of evidence to substantiate a statistical advantage of this methodology over the open technique.

3) Step-up Approach: This consists of percutaneous drainage followed by minimal invasive retroperitoneal necrosectomy. The first step is percutaneous or endoscopic drainage of collection of infected fluid to reduce sepsis. This may postpone the need for surgical necrosectomy during the acute phase of systemic inflammatory response (SIRS). If the drainage is still inadequate, the next step is minimally invasive retroperitoneal necrosectomy. The main factor which limits the use of this methodology is blockage of drainage tube before complete drainage has been achieved, thereby necessitating early necrosectomy by either minimal access approach or open method.

4) Endoscopic Methods: The transgastric endoscopic approach which has evolved recently is an alternative approach to drain an infected pancreatic necrosis. Endoscopic necrosectomy has similarity to natural orifice transluminal endoscopic surgery (NOTES). The main disadvantage of this technique is significant bleeding which can in quite few cases prove to be fatal as it is difficult to control.

(a) Hiatus hernia.

Introduction: Hiatal are classified according to the position of the esophagogastric junction and the existence of a true hernia sac.

Types:  Type I (sliding) – Leading edge of the hernia is the esophagogastric junction, which is displaced into an intrathoracic position. – The longitudinal axis of the stomach is aligned with the esophagus. There is often no true hernia sac nor is there any paraesophageal component.  Type II (rolling) – The esophagogastric junction is in its normal intraabdominal location – The hernia sac (containing portions of the gastric fundus and body) develops alongside the esophagus  Type III (combined sliding-rolling or mixed) – The esophagogastric junction is displaced into the thorax and like a Type II, the hernia sac contains portions of the gastric fundus or body.  Type IV hiatal hernia: is declared in some taxonomies, when an additional organ, usually the colon, herniates as well. The end stage of type I and type II hernias occurs when the whole stomach migrates up into the chest by rotating 180° around its longitudinal axis, with the cardia and pylorus as fixed points. In this situation the abnormality is usually referred to as an intrathoracic stomach.

• Increasingly common with advancing age • More often among women than men • Symptoms are often associated with GERD

Risk factors: Increased pressure within the abdomen caused by:

 Heavy lifting or bending over  Frequent or hard coughing  Hard sneezing  Violent vomiting  Straining  Stress

Diagnosis: • Typical symptoms • Suspicious CXR • Chest C.T. • Upper GI Series • In urgent situations: – Placement of NG tube & subsequent coiling Management: • Evaluation – Endoscopy – Esophageal Motility Studies – Manometry & pH Monitoring • 1/3 of pts will have atypical of the esophageal body • ½ of symptomatic pts will have abnormal pH results

 Indications for Operation – Type I – Type II & III • Associated with a high-risk of complications • “catastrophic” in 20 – 30% of patients • Symptoms do not predict risk  Findings that may prompt surgery (even in those pts that are “not optimal”) – Symptoms of obstruction – Reflux – Anemia  Trying to avoid: – Further aspiration – Hemorrhage – Transfusion requirements

Surgical techniques:

• Principles similar to other hernia operations • Need to anchor the stomach • Fundoplication is controversial • Transthoracic vs. Transabdominal The basic principles of operative management in hiatal hernia are • Complete reduction of the hernia and removal of the hernia sac • Close the hiatus • Restore the GEJ to an intra-abdominal location • Addition of an antireflux procedure

(b) Solitary Rectal ulcer syndrome. Definition: Solitary rectal ulcer syndrome is a condition that occurs when a sore (ulcer) develops in the rectum. The rectum is a muscular tube that's connected to the end of your colon. Stool passes through the rectum on its way out of the body. Solitary rectal ulcer syndrome can cause rectal bleeding with straining when you pass bowel movements. Solitary rectal ulcer syndrome most commonly involves one ulcer, but sometimes more than one rectal ulcer can occur or polyp-like masses may form. Solitary rectal ulcer syndrome is a rare and poorly understood disorder that occurs in people with chronic constipation.

Treatments for solitary rectal ulcer syndrome range from changing your diet and fluid intake to surgery.

Signs and symptoms of solitary rectal ulcer syndrome include:

 Constipation  A feeling of incomplete passing of stool  Pain or a feeling of fullness in your pelvis  Passing mucus from your rectum  Rectal pain or anal sphincter spasms  Rectal bleeding  Straining during bowel movements However, some people with solitary rectal ulcer syndrome may experience no symptoms.

Causes: It's not always clear what causes solitary rectal ulcer syndrome. Doctors believe stress or injury to the rectum may cause rectal ulcers to form.

Examples of situations that could injure the rectum include:

 Attempts to manually remove impacted stool  Constipation or impacted stool  Intussusception, which occurs when part of the intestine slides inside another part  Radiation therapy used to treat cancer in the abdomen or pelvis  Rectal prolapse, which occurs when the rectum protrudes from the anus  Straining during bowel movements  Use of ergotamine suppositories, an anti-migraine treatment  Uncoordinated tightening of the pelvic floor muscles that slows blood flow to the rectum

Tests and procedures used to diagnose rectal ulcers include:

 Using a scope to examine the rectum. During sigmoidoscopy, a flexible tube equipped with a lens is inserted into the rectum to examine the rectum and part of colon. If a lesion is found, doctor may take a tissue sample for laboratory testing.  Using sound waves to create images of the rectum. Ultrasound is an imaging technique that uses sound waves to create pictures. Ultrasound can help to differentiate solitary rectal ulcer syndrome from other conditions.  An X-ray of the rectum. During a procedure called defecation proctography, a soft paste made of barium is inserted into the rectum. Patient then passes the barium paste as one would stool. The barium shows up on X-rays and may reveal a prolapse or problems with muscle function and muscle coordination. Treatments and drugs: Treatment for solitary rectal ulcer syndrome depends on the severity of condition. People with mild signs and symptoms may find relief by making lifestyle changes to control chronic constipation. People with more severe signs and symptoms may require treatments such as behavior therapy and surgery.

 Behavior therapy to stop straining during bowel movements. Some people strain during bowel movements out of habit. Behavior therapy can help you learn to relax the pelvic muscles and avoid straining during bowel movements. In one technique called biofeedback, a specialist teaches the patient to control certain involuntary body responses, such as tightening of anus or pelvic floor muscles during defecation. Biofeedback may make the patient more aware of straining and help to control it.

Surgery: Surgical procedures used to treat rectal ulcer include:

 Surgery to remove the rectum. An operation to remove the rectum may be an option for people with severe rectal ulcer signs and symptoms. The surgeon may connect the colon to an opening in the abdomen for waste to leave the body (colostomy). If one have a colostomy, a pouch or bag is then attached to abdomen to collect waste.  Rectal prolapse surgery. If one have a rectal prolapse that's causing rectal ulcer, surgeon may recommend a rectopexy procedure. Rectopexy corrects rectal prolapse by using stitches to secure the rectum in its anatomically correct position.

(c) Management of GIST. Management The management of GISTs should be undertaken by a multidisciplinary team (MDT) with experience in this disease. Diagnosis  Pathological review of all cases should be made by a pathologist experienced in this tumour type  For resectable tumours, a definitive diagnosis is often made after surgery  For patients with unresectable and/or metastatic tumours, a biopsy should be taken and a definitive diagnosis made before treatment

Classification of Primary Gastrointestinal Stromal Tumors by Risk of Metastasis

Risk Category Size Mitotic Count Very Low <2 cm <5 per 50 HPFs* Low 2–5 cm <5 per 50 HPFs Intermediate <5 cm 6–10 per 50 HPFs 5–10 cm <5 per 50 HPFs High >5 cm >5 per 50 HPFs >10 cm Any mitotic rate Any size >10 per 50 HPFs

Imaging studies:  Endoscopic ultrasonography (EUS), especially of the oesophagus, stomach, duodenum, and the anorectum, can confirm the diagnosis of small incidental GISTs <2 cm  For large tumours, computed tomography (CT) of chest, abdomen and pelvis is recommended to assess primary tumour extension and to stage for metastases  For ano-rectal tumours, magnetic resonance imaging (MRI) is useful in locoregional staging  PET-CT may be considered to aid assessment when radical surgery is required, particularly of the duodenum, rectum and oesophagus.  Contrast-enhanced CT scanning is the standard imaging modality for assessing response to tyrosine kinase therapies.  PET-CT may be useful for monitoring response to therapy, emerging resistance and where there is diagnostic uncertainty on CT scans, but is not a substitute for contrast-enhanced CT scanning.  PET-CT scans should be reviewed by an experienced or gastrointestinal radiologist with knowledge of GISTs  All imaging studies should be presented in a standardised and consistent format and reviewed and discussed by the MDT  Tumor size, mitotic rate, and location can be used to predict the risk of recurrence in GIST patients. Tumors <2 cm with a mitotic rate of <5/50 HPF have been shown to have lower risk of recurrence than larger or more aggressive tumors. Nevertheless, all GIST tumors should be considered to have malignant potential and no GIST tumor can be correctly classified as "benign." Histopathology and immunohistochemistry:  Macroscopic examination of the resected tumour, with adequate sampling for histological examination and for immunohistochemistry, should be performed  Biopsies should sample multiple sites and include some normal tissue  Laparoscopic biopsies may be considered if a biopsy cannot be obtained by other means  Control slides should include normal gastric body wall as a staining control and tumour to allow for direct comparison of staining  The diagnosis of GIST is supported by positive CD117 staining as part of an adequately controlled immunohistochemical panel in a spindle cell tumour of the GI tract when morphologic and clinical features of the tumour are consistent with GIST.  DOG1 is a useful marker of GIST and should be used in conjunction with CD117 and CD34 staining.

Treatment: Surgery is the mainstay of therapy for nonmetastatic GISTs. Lymph node metastases are rare, and routine removal of lymph nodes is typically not necessary.  Until recently, GISTs were notorious for being resistant to chemotherapy, with a success rate of <5%. Recently, the c-kit tyrosine kinase inhibitor imatinib, was found to be useful in treating GISTs, leading to a 40-70% response rate in metastatic or inoperable cases.  The two year survival of patients with advanced disease is 75–80% following imatinib treatment.  Adjuvant treatment with imatinib following surgical resection of GIST tumors can significantly reduce the risk of disease recurrence.  Patients who develop resistance to imatinib may respond to the multiple tyrosine kinase inhibitor sunitinib.  The effectiveness of imatinib and sunitinib depend on the genotype.

Management of primary localized GIST:

THE WEST BENGAL UNIVERSITY OF HEALTH SCIENCES MS (General Surgery) Examination, 2014 PAPER II

Time Allowed: 3 Hours Full Marks: 100 Attempt all questions

1. Classify thyroid cancer. How will you prognosticate differential thyroid carcinoma? Describe management of a FNAC proved follicular neoplasm of right lobe of thyroid in a 45 years old female. 4+4+12 2. Discuss the etiopathology, diagnosis of acute pancreatitis and treatment. 20

3. Write short notes of the following: 5 x 6 (a) Desmoids tumour. (b) Gynaecomastia. (c) Superior mesenteric artery syndrome. (d) POEM. (e) Adrenal incidentaloma.

4. Answer briefly on the following: 4 x 71/2 (a) TAPP and TEP repair of groin hernia. (b) Surgical manifestations of abdominal tuberculosis. (c) Traumatic duodenal injury. (d) Sterilization of laparoscopic instruments.

THE WEST BENGAL UNIVERSITY OF HEALTH SCIENCES

MS (General Surgery) Examination, 2014

April 2014

PAPER II

Time Allowed: 3 Hours Full Marks: 100 Attempt all questions

1. Classify thyroid cancer. How will you prognosticate differential thyroid carcinoma? Describe management of a FNAC proved follicular neoplasm of right lobe of thyroid in a 45 years old female. 4+4+12

Answer.

Classification and Incidence of Thyroid Malignancies

Tumors of Follicular Cell Origin Tumors of Parafollicular Other or C-cell Origin Differentiated Undiffere Medullary (5%|) Lymphoma (<1%) ntiated Papillary (75%) Anaplastic Follicular (10%) (5%) Hürthle Cell (5%)

Prognostication of differentiated thyroid carcinoma:

Low-Versus High-Risk Criteria in Papillary Thyroid Cancer

Low risk High risk 1. Women < 50 years 1. 2. Men < 40 years 2. 3. Well-or moderately 3. PoorlyWomen differentiated ≥ 50 yr tumors, differentiated tumors tall-cell,Men ≥ 40 columnar yr cell, or 4. Tumor < 4 cm in diameter oxyphilic variants. 5. Tumor confined to the thyroid 4. gland 5. Local invasion 6. No distant metastases 6. DistantTumor ≥metastases 4 cm in diameter

Prognostic risk Classification for patients with Well-Differentiated Thyroid Cancer (AMES or AGES)

MACIS is an abbreviation for the factors taken into account to predict survival:  Metastasis (distant) or spread of the cancer to areas outside the neck,  Age of the patient at the time the tumor was discovered,  Invasion into surrounding areas of the neck as seen by the naked eye,  Completeness of surgical resection (or removal) of the tumor, and  Size of the tumor.

Prognostic Scoring Systems for AGES, AMES, and MACIS: AGES Prognostic score = 0.05 × age (if age ≥40) + 1 (if grade 2) + 3 (if grade 3 or 4) + 1 (if extrathyroid) + 3 (if distant spread) + 0.2 × tumor size (cm maximum diameter) Survival by AGES score (20-yr):

4-4.99 = 80% 5-5.99≤3.99 == 99%67%

AMES Low risk: Younger patients (men ≤40, women ≤50) with≥6 no = 13% metastases Older patients (intrathyroid papillary, minor capsular invasion for follicular lesions) Primary cancers < 5cm No distant metastasis (Men >40, women >50) Survival by AMES risk-groups (20-yr): Low risk = 99% High risk = 61% MACIS Score = 3.1 (if age < 40 years ) or 0.08 x age (if age < 40years) + 0.3 × tumor size (cm maximum diameter) + 1 (if incompletely resected) + 1 (if locally invasive) + 3 (if distant spread) Survival by MACIS score (20-yr): <6 = 99% 6-6.99 = 89% 7-7.99 = 56%

Treatment of follicular≥8 = 24% carcinoma is primarily surgical: Current National Comprehensive Cancer Network (NCCN) guidelines recommend lobectomy plus isthmusectomy as the initial surgery for patients with follicular , with prompt completion of thyroidectomy if invasive follicular thyroid carcinoma (FTC) is found on the final histologic section. Therapeutic neck dissection of involved compartments is recommended for clinically apparent/biopsy-proven disease.  The diagnosis of the carcinoma cannot be determined by preoperative FNA or intraoperative frozen section diagnosis of a follicular lesion.  If the lesion is 2 cm or smaller and well contained within one thyroid lobe, thyroid lobectomy and isthmusectomy can be done.  If the lesion is larger than 2 cm, then total thyroidectomy should be done.  If the follicular lesion is larger than 4 cm, the risk for cancer is higher than 50%, and total thyroidectomy is an obvious choice.  Current recommendations support performing completion thyroidectomy if total thyroidectomy would have been offered if the diagnosis had been secured preoperatively.  The prognosis after treatment of follicular cancer depends on age. . Patients younger than 40 years have the best prognosis, with survival rates approaching 95% at 5 and 10 years. . Series comparing follicular cancer with PTC have shown a poorer prognosis for follicular cancer, although this disparity is more prominent after 10 to 15 years. . Poorly differentiated follicular cancer and well-differentiated follicular cancer have 60% and 80% 10-year survival rates, respectively. Postoperative Treatment: . Accepted postsurgical management of well-differentiated follicular thyroid cancers involves the use of radioiodine ablation and long-term monitoring of Tg. . 131I contains highenergy (gamma rays) and medium-energy (beta particles), which enhances the therapeutic effect. . Patients are usually withheld from thyroid replacement therapy so that TSH levels may become elevated, rendering the thyroid iodine avid and thus maximizing the effect of131I. . Several studies have suggested that 131I ablation reduces disease-specific mortality in patients with primary tumors measuring at least 1 cm. . If a patient has undergone complete thyroid ablation, Tg levels should be undetectable. . The recent development of human recombinant TSH has redefined the efficacy of monitoring stimulated Tg levels as evidence of recurrence. 2. Discuss the etiopathology, diagnosis of acute pancreatitis and treatment. 20 Answer.

Aetiopathology:

Acute pancreatitis has been attributed to a wide range of etiologic factors, some rare and rather obscure. Intra-acinar activation of trypsinogen, with subsequent activation of other pancreatic enzymes, is thought to play a central role in the pathogenesis of the disease. Furthermore, ischemia-reperfusion injury is believed to be critical to disease progression. A local inflammatory response in the pancreas is associated with the liberation of oxygen- derived free radicals and cytokines including interleukin 1 (IL-1), IL-6, and IL-8; tumor (TNF- ); and platelet-activating factor; these mediators play an important role in the transformation from a local inflammatory response to a systemic illness. necrosis factor α α Etiologic Factors in Acute Pancreatitis

Metabolic Mechanical Vascular Infection Alcohol Cholelithiasis Postoperative (cardiopulmonary bypass) Hyperlipoproteinemia Postoperative Periarteritis Coxsackie B nodosa Hypercalcemia Pancreas divisum Atheroembolism Cytomegalovirus Drugs Post-traumatic Cryptococcus Genetic Retrograde pancreatography Scorpion venom Pancreatic duct obstruction: pancreatic tumor, ascaris infestation Pancreatic ductal bleeding Duodenal obstruction

Investigations and diagnosis:

 Blood Investigations - Full blood count, Renal function tests, Liver Function, serum calcium, serum amylase and lipase, Arterial blood gas.  Imaging - Chest Xray (for exclusion of perforated viscus), Abdominal Xrays (for detection of "sentinel loop" dilated duodenum sign, and gallstones which are radioopaque in 10%) and CT abdomen  Amylase and lipase: Elevated serum amylase and lipase levels, in combination with severe abdominal pain, often trigger the initial diagnosis of acute pancreatitis. Serum lipase rises 4 to 8 hours from the onset of symptoms and normalizes within 7 to 14 days after treatment. Serum amylase may be normal (in 10% of cases) for cases of acute or chronic pancreatitis (depleted acinar cell mass) and hypertriglyceridemia. Reasons for false positive elevated serum amylase include disease (elevated salivary amylase) and macroamylasemia. If the lipase level is about 2.5 to 3 times that of amylase, it is an indication of pancreatitis due to alcohol. . Regarding selection on these tests, two practice guidelines state: "It is usually not necessary to measure both serum amylase and lipase. Serum lipase may be preferable because it remains normal in some nonpancreatic conditions that increase serum amylase including macroamylasemia, parotitis, and some carcinomas. In general, serum lipase is thought to be more sensitive and specific than serum amylase in the diagnosis of acute pancreatitis" "Although amylase is widely available and provides acceptable accuracy of diagnosis, where lipase is available it is preferred for the diagnosis of acute pancreatitis (recommendation grade A)  Computed tomography: Regarding the need for computed tomography, practice guidelines state:"Many patients with acute pancreatitis do not require a CT scan at admission or at any time during the hospitalization. For example, a CT scan is usually not essential in patients with recurrent mild pancreatitis caused by alcohol. A reasonable indication for a CT scan at admission (but not necessarily a CT with IV contrast) is to distinguish acute pancreatitis from another serious intra-abdominal condition, such as a perforated ulcer." "Patients with persisting organ failure, signs of sepsis, or deterioration in clinical status 6 10 days after admission will require CT (recommendation grade B)." – CT abdomen should not be performed before the 1st 48 hours of onset of symptoms as early CT (<48 h) may result in equivocal or normal findings. CT Findings can be classified into the following categories for easy recall:

Intrapancreatic - diffuse or segmental enlargement, edema, gas bubbles, pancreatic pseudocysts and phlegmons/abscesses (which present 4 to 6 wks after initial onset)

Peripancreatic / extrapancreatic - irregular pancreatic outline, obliterated peripancreatic fat, retroperitoneal edema, fluid in the lessar sac, fluid in the left anterior pararenal space

Locoregional - Gerota's fascia sign (thickening of inflamed Gerota's fascia, which becomes visible), pancreatic ascites, pleural effusion (seen on basal cuts of the pleural cavity), adynamic ileus, etc.

 Magnetic resonance imaging: While computed tomography is considered the gold standard in diagnostic imaging for acute pancreatitis, magnetic resonance imaging (MRI) has become increasingly valuable as a tool for the visualization of the pancreas, particularly of pancreatic fluid collections and necrotized debris.Additional utility of MRI includes its indication for imaging of patients with an allergy to CT's contrast material, and an overall greater sensitivity to hemorrhage, vascular complications, pseudoaneurysms, and venous thrombosis. Another advantage of MRI is its utilization of magnetic resonance cholangiopancreatography (MRCP) sequences. MRCP provides useful information regarding the etiology of acute pancreatitis, i.e., the presence of tiny biliary stones (choledocholithiasis or cholelithiasis) and duct anomalies. Clinical trials indicate that MRCP can be as effective a diagnostic tool for acute pancreatitis with biliary etiology as endoscopic retrograde cholangiopancreatography, but with the benefits of being less invasive and causing fewer complications. Classification by severity: Progression of pathophysiology: Acute pancreatitis can be further divided into mild and severe pancreatitis. Mostly the Atlanta classification (1992) is used. In severe pancreatitis serious amount of necrosis determine the further clinical outcome. About 20% of the acute pancreatitis are severe with a mortality of about 20%. This is an important classification as severe pancreatitis will need intensive care therapy whereas mild pancreatitis can be treated on the common ward. Necrosis will be followed by a systemic inflammatory response syndrome (SIRS) and will determine the immediate clinical course. The further clinical course is then determined by bacterial infection. SIRS is the cause of bacterial (Gram negative) translocation from the patient s colon. There are several ways to help distinguish between these two forms. One is the above ’ mentioned Ranson Score. Prognostic indices: In predicting the prognosis, there are several scoring indices that have been used as predictors of survival. Two such scoring systems are the and APACHE II (Acute Physiology, Age and Chronic Health Evaluation) indices. Most, but not all studies report that the Apache score may be more accurate. In the negative study of the Apache II, the Apache II 24 hr score was used rather than the 48 hour score. In addition, all patients in the study received an ultrasound twice which may have influenced allocation of co- interventions. Regardless, only the Apache II can be fully calculated upon admission. As the Apache II is more cumbersome to calculate, presumably patients whose only laboratory abnormality is an elevated lipase or amylase do not need prognostication with the Apache II; however, this approach is not studied. The Apache II score can be calculated. Pracactice guidelines state: "The two tests that are most helpful at admission in distinguishing mild from severe acute pancreatitis are APACHE-II score and serum hematocrit. It is recommended that APACHE-II scores be generated during the first 3 days of hospitalization and thereafter as needed to help in this distinction. It is also recommended that serum hematocrit be obtained at admission, 12 h after admission, and 24 h after admission to help gauge adequacy of fluid resuscitation." "Immediate assessment should include clinical evaluation, particularly of any cardiovascular, respiratory, and renal compromise, body mass index, chest x ray, and APACHE II score". Ranson criteria is a for predicting the severity of acute pancreatitis. At admission At 48 hours

Age in years > 55 years Calcium (serum calcium < 2.0 mmol/L (< 8.0 mg/dL) White blood cell count > 16000 cells/mm3 Hematocrit fall > 10% Blood glucose > 10 mmol/l (> 200 mg/dl) Oxygen (hypoxemia PO2 < 60 mmHg) Serum AST > 250 iu/l BUN increased by 1.8 or more mmol/L (5 or Serum LDH > 700 iu/l more mg/dL) after IV fluid hydration

Base deficit (negative base excess) > 4 mEq/L

Sequestration of fluids > 6 L

The criteria for point assignment is that a certain breakpoint be met at anytime during that 48 hour period, so that in some situations it can be calculated shortly after admission. It is applicable to both gallstone and alcoholic pancreatitis. Alternatively, pancreatitis can be diagnosed by meeting any of the following: APACHE II score necrosis according to contrast-enhanced CT) Apache score of ≥ 8 Organ failure Substantial pancreatic necrosis (at least 30% glandular pancreatitis is unlikely Or Score 0 to 2 : 2% mortality Score 3 to 4 : 15% mortality Score 5 toInterpretation 6 : 40% mortality If the Scorescore ≥7 3,to severe8 : 100% pancreatitis mortality likely. If the score < 3, severe "Acute Physiology And Chronic Health Evaluation" (APACHE II) score > 8 points predicts 11% to 18% mortality. Balthazar scoring Developed in the early 1990s by Emil J. Balthazar et al., the Computed Tomography Severity Index (CTSI) is a grading system used to determine the severity of acute pancreatitis. The numerical CTSI has a maximum of ten points, and is the sum of the Balthazar grade points and pancreatic necrosis grade points: Balthazar Grade:

Balthazar Appearance on CT CT Grade Grade Points

Grade A Normal CT 0 points

Grade B Focal or diffuse enlargement of the pancreas 1 point

Grade C Pancreatic gland abnormalities and peripancreatic inflammation 2 points

Grade D Fluid collection in a single location 3 points

Two or more fluid collections and / or gas bubbles in or adjacent to Grade E 0 points pancreas

Necrosis Score

Necrosis Percentage Points

No necrosis 0 points

0 to 30% necrosis 2 points

30 to 50% necrosis 4 points

Over 50% necrosis 6 points

CTSI's staging of acute pancreatitis severity has been shown by a number of studies to provide more accurate assessment than APACHE II, Ranson, and C-reactive protein (CRP) level. However, a few studies indicate that CTSI is not significantly associated with the prognosis of hospitalization in patients with pancreatic necrosis, nor is it an accurate predictor of AP severity.

Glasgow Imrie criteria 3 or more positive criteria within 48h of admission = severe attack

 Age > 55y  WCC > 15 000 Ã 109/L  Glucose > 7mmol/L  Blood urea > 7mmol/L—  Albumin < 35g/L  Corrected calcium < 2mmol/L  PaO2 < 10kPa

Treatment:  Pain control  Bowel rest: In the management of acute pancreatitis, the treatment is to stop feeding the patient, giving him or her nothing by mouth, giving intravenous fluids to prevent dehydration, and sufficient pain control. As the pancreas is stimulated to secrete enzymes by the presence of food in the stomach, having no food pass through the system allows the pancreas to rest.  Nutritional support: Recently, there has been a shift in the management paradigm from TPN (total parenteral nutrition) to early, post-pyloric enteral feeding (in which a feeding tube is endoscopically or radiographically introduced to the third portion of the duodenum). The advantage of enteral feeding is that it is more physiological, prevents gut mucosal atrophy, and is free from the side effects of TPN (such as fungemia). The additional advantages of post-pyloric feeding are the inverse relationship of pancreatic exocrine secretions and distance of nutrient delivery from the pylorus, as well as reduced risk of aspiration. Disadvantages of a naso-enteric feeding tube include increased risk of sinusitis (especially if the tube remains in place greater than two weeks) and a still-present risk of accidentally intubating the bronchus even in intubated patients.  Antibiotics: . Carbapenems: An early randomized controlled trial of imipenem 0.5 gram intravenously every eight hours for two weeks showed a reduction in from pancreatic sepsis from 30% to 12%. Another randomized controlled trial with patients who had at least 50% pancreatic necrosis found a benefit from imipenem compared to pefloxacin with a reduction in infected necrosis . The role of antibiotics is controversial. One recent expert opinion suggested the use of imipenem if CT scan showed more than 30% necrosis of the pancreas. . ERCP: Early ERCP (endoscopic retrograde cholangiopancreatography), performed within 24 to 72 hours of presentation, is known to reduce morbidity and mortality.

The indications for early ERCP are as follows : o Clinical deterioration or lack of improvement after 24 hours o Detection of common bile duct stones or dilated intrahepatic or extrahepatic ducts on CT abdomen The disadvantages of ERCP are as follows : o ERCP precipitates pancreatitis, and can introduce infection to sterile pancreatitis o The inherent risks of ERCP i.e. bleeding o It is worth noting that ERCP itself can be a cause of pancreatitis.

 Surgery: Surgery is indicated for (i) infected pancreatic necrosis and (ii) diagnostic uncertainty and (iii) complications. The most common cause of death in acute pancreatitis is secondary infection. Infection is diagnosed based on 2 criteria

 Gas bubbles on CT scan (present in 20 to 50% of infected necrosis)  Positive bacterial culture on FNA (fine needle aspiration, usually CT or US guided) of the pancreas. Surgical options for infected necrosis include: o Minimally invasive management - necrosectomy through small incision in skin (left flank) or stomach o Conventional management - necrosectomy with simple drainage o Closed management - necrosectomy with closed continuous postoperative lavage o Open management - necrosectomy with planned staged reoperations at definite intervals (up to 20+ reoperations in some cases)

 Other measures: o Pancreatic enzyme inhibitors are not proven to work. o The use of octreotide has not been shown to improve outcome.

The Complications of Acute Pancreatitis:

 Local :  Fluid collections  Pancreatic ascites/pleural effusion  Pancreatic pseudocyst  Pancreatic necrosis  Infected pancreatic abscess  Hemorrhage/pseudoaneurysm.

 Regional:  Venous thrombosis  Paralytic ileus  Intestinal obstruction  Intestinal ischemia/necrosis  Cholestasis  Systemic:  Systemic inflammatory response syndrome  Multiple-organ-dysfunction syndrome  ARDS/pulmonary failure  Renal failure  Cardiovascular complications  Hypocalcemia  Hyperglycemia  Disseminated intravascular coagulopathy  Protein calorie malnutrition  Encephalopathy

 Complications are treated accordingly.

3. Write short notes of the following: 5 x 6 (a) Desmoids tumour. (b) Gynaecomastia. (c) Superior mesenteric artery syndrome. (d) POEM. (e) Adrenal incidentaloma.

Answer. (a) Desmoids tumour. Desmoid tumour : it is a benign soft-tissue tumor that does not spread to other parts of the body. Desmoid tumors occur most often in young adults, and they usually involve the limbs or trunk, but they can also arise in the abdomen or thorax. Desmoid tumors are very difficult to remove because they adhere tenaciously to surrounding structures and organs.

A desmoid tumor is also called aggressive because it is locally aggressive and fibrous, like scar tissue. However, aggressive fibromatosis is locally aggressive. Despite their benign nature, they can damage nearby structures causing organ dysfunction. Most cases are sporadic, but some are associated with familial adenomatous polyposis (FAP). Approximately 10% of individuals with Gardner's syndrome, a type of FAP with extracolonic features, have desmoid tumors.

Histologically they resemble low-grade , but they are very locally aggressive and tend to recur even after complete resection. There is a tendency for recurrence in the setting of prior surgery; in one study, two-thirds of patients with desmoid tumors had a history of prior abdominal surgery.

Risk factors for desmoid disease amongst FAP patients include female gender, a 3' APC mutation, a positive family history and a history of previous abdominal surgery.

Desmoid tumors may be classified as extra-abdominal, abdominal wall, or intra-abdominal (the last is more common in patients with FAP). It is thought that the lesions may develop in relation to estrogen levels or trauma/operations.

A 3' APC mutation is the most significant risk factor for intra-abdominal desmoid development amongst FAP patients. FAP patients presenting with an abdominal wall desmoid pre-operatively are at an increased risk of developing an intra-abdominal desmoid post-operatively.

Desmoid tumours of the breast are rare. Although benign, they can mimic breast cancer on physical examination, mammography and breast ultrasound and can also be locally invasive. Even though they occur sporadically, they can also be seen as a part of Gardner's syndrome. A high index of suspicion and a thorough triple examination protocol is necessary to detect rare lesions like a desmoid tumour which can masquerade as breast especially where imaging studies are not conclusive and suggest a more ominous diagnosis. carcinoma. Desmoid tumour of the breast may present a difficulty in the diagnosis Treatment :Treatment may consist of watching and waiting, complete surgical removal, radiation therapy, antiestrogens and NSAIDs, or chemotherapy.

Management of these lesions is complex, the main problem being the high rates of recurrence.A biopsy is always Wide surgical indicated resection as the definitive with clear method margins to is determine the most widelynature practicedof the tumour. technique with radiation, chemotherapy, or hormonal therapy being used to reduce recurrence rates.

Current experimental studies are being done with Gleevec for treatment of desmoid tumors, and show promising success rates.

(b) Gynaecomastia.

Introduction: Gynecomastia refers to an enlarged breast in the male. Physiologic gynecomastia usually occurs during three phases of life: the neonatal period, adolescence, and senescence.  Common to each of these phases is an excess of circulating estrogens in relation to circulating testosterone.  Neonatal gynecomastia is caused by the action of placental estrogens on neonatal breast tissues, while in adolescence, there is an excess of estradiol relative to testosterone, and with senescence, the circulating testosterone level falls, resulting in relative hyperestrinism.  In gynecomastia, the ductal structures of the male breast enlarge, elongate, and branch with a concomitant increase in epithelium.  During puberty, the condition often is unilateral and typically occurs between ages 12 and 15 years. In contrast, senescent gynecomastia is usually bilateral.  In the nonobese male, breast tissue measuring at least 2 cm in diameter must be present before a diagnosis of gynecomastia may be made.  Mammography and ultrasonography are used to differentiate breast tissues.  Dominant masses or areas of firmness, irregularity, and asymmetry suggest the possibility of a breast cancer, particularly in the older male.  Gynecomastia generally does not predispose the male breast to cancer.  However, the hypoandrogenic state of Klinefelter's syndrome (XXY), in which gynecomastia is usually evident, is associated with an increased risk of breast cancer.

Pathophysiologic Mechanisms of Gynecomastia

I. Estrogen excess states A. Gonadal origin 1. True hermaphroditism 2. Gonadal stromal (nongerminal) neoplasms of the testis a. Leydig cell (interstitial) b. Sertoli cell c. Granulosa-theca

3. Germ cell tumors a. Choriocarcinoma b. Seminoma, teratoma c. Embryonal carcinoma

B. Nontesticular tumors 1. Adrenal cortical neoplasms 2. Lung carcinoma 3. Hepatocellular carcinoma

C. Endocrine disorders D. Diseases of the liver nonalcoholic and alcoholic cirrhosis E. Nutrition alteration states

II. Androgen deficiency states— A. Senescence B. Hypoandrogen states (hypogonadism) 1. Primary testicular failure a. Klinefelter's syndrome (XXY) b. Reifenstein's syndrome c. Rosewater, Gwinup, Hamwi familial gynecomastia d. Kallmann's syndrome e. Kennedy's disease with associated gynecomastia f. Eunuchoidal males (congenital anorchia) g. Hereditary defects of androgen biosynthesis h. ACTH deficiency

2. Secondary testicular failure a. Trauma b. Orchitis c. Cryptorchidism d. Irradiation

C. Renal failure III. Drug-related IV. Systemic diseases with idiopathic mechanisms

Clinical Classification of Gynecomastia

Grade I Mild breast enlargement without skin redundancy Grade Moderate breast enlargement without skin redundancy IIa Grade Moderate breast enlargement with skin redundancy IIb Grade Marked breast enlargement with skin redundancy and ptosis, which simulates a III female breast

Treatment: Generally, no treatment is required for physiologic gynecomastia. Pubertal gynecomastia resolves spontaneously within several weeks to 3 years in approximately 90% of patients. Breasts greater than 4 cm in diameter may not completely regress.

 Identifying and managing an underlying primary disorder often alleviates breast enlargement. If hypogonadism (primary or secondary) is the cause of gynecomastia, parenteral or transdermal testosterone replacement therapy is instituted. However, testosterone does have the potential to exacerbate gynecomastia through the aromatization of the exogenous hormone into estradiol.  For patients with idiopathic gynecomastia or with residual gynecomastia after treatment of the primary cause, medical or surgical treatment may be considered.  A major factor that should influence the initial choice of therapy for gynecomastia is the condition s duration. It is unlikely that any medical therapy will result in significant regression in the late fibrotic stage (a duration of 12mo or longer) of gynecomastia. As a result, medical’ therapies, if used, should be tried early in the condition's course.

Pharmacologic Therapy: I. With the administration of clomiphene, an antiestrogen, approximately 50% of patients achieve partial reduction in breast size, and approximately 20% of patients note complete resolution. Adverse effects, while rare, include visual problems, rash, and nausea. II. Tamoxifen, an estrogen antagonist, is effective for recent-onset and tender gynecomastia. Up to 80% of patients report partial to complete resolution. Nausea and epigastric discomfort are the main adverse effects. III. Other drugs used, less frequently, include danazol. Danazol, a synthetic derivative of testosterone, inhibits pituitary secretion of LH and follicle-stimulating hormone (FSH), which decreases estrogen synthesis from the testicles.

Breast Surgery:

I. Reduction mammoplasty is considered for patients with macromastia or long- standing gynecomastia or in persons in whom medical therapy has failed. It is also considered for cosmetic reasons (and for accompanying psychosocial reasons). II. More extensive plastic surgery may be required in patients with marked gynecomastia or who have developed excessive sagging of the breast tissue due to weight loss. If surgery is necessary for patients with pseudogynecomastia, liposuction may be warranted. III. A Chinese study indicated that endoscopic subcutaneous mastectomy, without skin excision, could be an effective treatment for gynecomastia. IV. Complications of surgery include sloughing of tissue due to a compromised blood supply, contour irregularity, hematoma or seroma formation, and permanent numbness in the nipple-areolar area.

(c) Superior mesenteric artery syndrome.

Introduction: Superior mesenteric artery (SMA) syndrome (also known as Wilkie syndrome) is a rare acquired vascular compression disorder in which acute angulation of superior mesenteric artery (SMA) results in compression of the third part of the duodenum leading to obstruction.

It should not be confused with nutcracker syndrome, also a superior mesenteric artery compression disorder, where the SMA compresses the left renal vein, although some authors use the terms interchangeably.

Epidemiology: It is an uncommon but well recognized clinical entity. About 400 cases have been described in English literature. Is seen more commonly in females than in males, and usually occurs in older children and adolescents.

Clinical presentation: Patients with SMA syndrome may present acutely, with chronic insidious symptomatology, or with an acute exacerbation of chronic symptoms:  Acute presentation is usually characterized by signs and symptoms of duodenal obstruction  Chronic cases may present with long-standing vague abdominal symptoms, early satiety and anorexia, or recurrent episodes of abdominal pain, associated with vomiting Pathophysiology: Normally, fat and lymphatic tissues around the SMA provide protection to the duodenum against compression. Under conditions of severe weight loss, this cushion around the SMA is diminished, causing angulation and reduction in the distance between the aorta and the superior mesenteric artery. This is usually associated with conditions causing significant weight loss such as:  Anorexia nervosa  Malabsorption  Hypercatabolic states (burns, major surgery, malignancy)  Severe congestive heart failure causing cachexia Other conditions may also precipitate this syndrome:

 Increased spinal lordosis  Application of a body cast  Short ligament of treitz  Unusually low origin of sma Radiographic features: The diagnosis of SMA syndrome is based on clinical symptoms and radiologic evidence of obstruction.

Plain film  Plain radiograph demonstrates a dilated, fluid- and gas-filled stomach Fluoroscopy  Barium radiography shows dilatation of the first and second part of the duodenum, extrinsic compression of the third part, and a collapsed small bowel distal to the crossing of the SMA CTA/MRA CT-Angiography or magnetic resonance angiography (MRA) enable visualization of vascular compression of the duodenum and measurement of aortomesenteric distance precisely.

 Normally, the aortomesenteric angle and aortomesenteric distance is 25° to 60° and 10 to 28 mm, respectively  In SMA syndrome, both parameters are reduced, with values of 6° to 15° and 2 to 8 mm Treatment and prognosis: Traditionally, treatment has consisted of conservative measures such as:  Nasogastric decompression and hyperalimentation followed by oral feeding with frequent small meals  Posturing maneuvers during meals and motility agents may be helpful in some patients Surgery may be considered if conservative treatment fails:

 Duodenojejunostomy is effective in the majority of patients  Laparoscopic duodenojejunostomy offers a new minimally invasive therapeutic approach to sma syndrome  Laparoscopic surgery involving lysis of the ligament of treitz with mobilization of the duodenum is another minimally invasive approach

(d) POEM.

Introduction: Polyneuropathy, organomegaly, endocrinopathy, monoclonal gammopathy, and skin changes (POEMS) syndrome is a rare multisystemic disease that occurs in the setting of a plasma cell dyscrasia.

Synonyms of POEMS Syndrome:  Crow-Fukase syndrome  Osteosclerotic myeloma  PEP syndrome  Polyneuropathy-organomegaly-endocrinopathy-M protein-skin lesions  Shimpo syndrome  Takatsuki syndrome  POEMS syndrome is an extremely rare multisystem disorder. POEMS is an acronym that stands for (P)Polyneuropathy, disease affecting many nerves; (O)Organomegaly, abnormal enlargement of an organ; (E)Endocrinopathy, disease affecting certain hormone-producing glands that help to regulate sexual function, and certain metabolic functions; (M)Monoclonal gammopathy or M proteins; and (S)Skin abnormalities.  Common symptoms include progressive weakness of the nerves in the legs and arms, an abnormally enlarged liver and/or spleen (hepatosplenomegaly), enlarged lymph nodes, abnormally darkening of the skin (hyperpigmentation), thickening of the skin and excessive hair growth (hypertrichosis). Causes:  The exact cause of POEMS syndrome is not known.  Studies have demonstrated abnormally high levels of vascular endothelial growth factor (VEGF) in the serum of individuals with POEMS syndrome. More research is needed to determine the exact role cytokines and VEGF play in the development of POEMS syndrome.  Other cytokines (non-antibody chemical that the body s cells to communicate with each other) have been showed to be elevated in patients with the disease, like interleukin-6, interleukin-1 and TNF-alpha. ’ Diagnosis: In most cases, POEMS syndrome is diagnosed based upon a thorough clinical evaluation, identification of characteristic symptoms and physical findings, a detailed patient and family history, and laboratory testing. Confirmation of certain immunologic abnormalities plays an essential role in establishing the diagnosis of POEMS syndrome. Laboratory tests conducted on the liquid portion of the blood (serum) or cerebrospinal fluid (CSF) may reveal elevated levels of M-proteins. Study of the blood plasma may show high levels of vascular endothelial growth factor. In many cases, surgical removal (biopsy) and microscopic examination of small samples of tissue from an osteosclerotic lesion or sometimes a simple bone marrow biopsy will reveal the abnormal presence of monoclonal plasma cells. Skeletal imaging may be performed to detect osteosclerotic lesions characteristic of POEMS syndrome. Treatment: The treatment of POEMS syndrome is two pronged. The first is directed at treating the underlying plasma cell disorder (e.g. the osteosclerotic myeloma). The second is directed toward ameliorating the specific symptoms that are apparent in each individual. Treatment may require the coordinated efforts of a team of specialists. The use of ionizing radiation (radiotherapy) or surgical removal (excision) or of osteosclerotic lesions that are localized (i.e., not widespread throughout the body) may temporarily (and sometimes permanently) lead to a remission of symptoms associated with POEMS syndrome. In many cases, including those with widespread osteosclerotic lesions or diffuse bone marrow involvement, therapy with certain anticancer drugs (chemotherapy), like corticosteroids with cyclophosphamide or melphalan may alleviate symptoms associated with POEMS syndrome. Many patients may be offered high-dose chemotherapy with peripheral blood stem cell transplant. Other treatment is symptomatic and supportive. Investigational Therapies: Because POEMS syndrome is a plasma cell disorder (i.e. due to an overgrowth of plasma cells), it is a cousin disease to multiple myeloma. In the 2000s, there have been several new drugs to treat myeloma. These may be tried only with extreme caution in patients with POEMS syndrome, because the major side effect of several of these new drugs is peripheral neuropathy, the dominant symptom in patients with POEMS syndrome. Additional therapies that have been used to treat POEMS syndrome include lenalidomide, thalidomide and bortezomib. The role of the following therapies is much less certain: anti- VEGF antibodies, interferon, intravenous immunoglobulin, plasmapheresis, and all-trans- retinoic acid.

(e) Adrenal incidentaloma.

Definition: It is an asymptomatic adrenal tumor that is discovered on an imaging test (CAT scan, MRI, etc) which was ordered to evaluate a problem that is unrelated to adrenal disease.

Types:

4. Answer briefly on the following: 4 x 71/2 (a) TAPP and TEP repair of groin hernia. (b) Surgical manifestations of abdominal tuberculosis. (c) Traumatic duodenal injury. (d) Sterilization of laparoscopic instruments. Answer. (a) TAPP and TEP repair of groin hernia. Introduction: The term laparoscopic inguinal herniorrhaphy can refer to any of the following 3 techniques:

 Totally extraperitoneal (TEP) repair  Transabdominal preperitoneal (TAPP) repair  Intraperitoneal onlay mesh (IPOM) repair

A number of studies have shown laparoscopic repair of inguinal hernias to have advantages over conventional repair, including the following:

 Reduced postoperative pain  Diminished requirement for narcotics  Earlier return to work

Laparoscopic repair has some disadvantages as well, including the following:

 Increased cost  Lengthier operation  Steeper learning curve  Higher recurrence and complication rates early in a surgeon s experience

Indications: ’  The general indications for laparoscopic inguinal , TAPP or TEP are the same as they are for open inguinal hernia repair. For young, active males with primary hernias, it may ofter decreased pain and an earlier return to activity. Laparoscopy may be ideal for bilateral groin hernias and recurrences from open approaches, but is also appropriate with unilateral primary hernias when the surgeon and patient are comfortable with the procedure.  Laparoscopic groin hernia classification is closely related to Nyhus, with all the anatomical aspects described according to this procedure Type 1: Congenital hernias with a narrow internal ring Type 2 : External oblique inguinal hernias with a dilated internal ring Type 3: a. The posterior wall of the inguinal canal is damaged b. External oblique hernias with a dilated internal ring. c. Femoral hernias identified in the medial inguinal region below the inguinal ligament Type 4: Recurrent hernias The laparoscopic procedure can be considered for all adult patients, regardless of the type of hernia. The best indications according to this classification are;

•Type 3 hernias,hernias Obesity,•Bilateral strenuous hernias activities (strenuous working and sports). •TypeRelative 4 indications: bilateral hernias;•Type I or voluminous II hernias exceptsliding whenhernias; associated with another type of hernia or in cases of nosed early

• Strangulated hernias, diag

Contraindications:  There are very few contraindications for these procedures. Some of them are listed below;  Contraindication to general anesthesia;  Extensive intra-abdominal adhesions;  Extremely voluminous sliding hernias with the bowel attached to the hernia sac;  Late diagnosis of strangulated hernias with advanced bowel obstruction.  Prior lower abdominal surgery or pelvic radiation are strong relative contraindications.

TEPP vs TAP:

 The TEPP repair differs from the TAP in that the abdominal cavity is not entered with the laparoscope.  All the surgery is performed in the layers outside the peritoneum.  The risk of bowel injury is therefore less with this technique.  The TEPP repair is particularly recommended for the repair of bilateral inguinal hernias and recurrent inguinal hernias. The procedure is performed under . Three small incisions are made in the midline between the umbilicus and the pubic bone.  Complications that can occur

. Pain . Wound infection . Scar . Atrophy of the testicle

. Bleeding . Mesh infection . Difficulty passing . Hernia urine recurrence

 The Trans-Abdominal Pre-peritoneal Repair (TAPP), is an alternative keyhole technique for inguinal hernia repair. With this technique a camera is inserted into the abdominal cavity. Gas is introduced, enabling the surgeon to see the hernia sac. The hernia is repaired using a piece of mesh which is inserted behind the muscle of the abdominal wall.

o The principle advantages of this technique over open surgery are: o Less post operative pain, o A quicker recovery o A quicker return to full physical activities. o There is no significant difference in the recovery after TEP or TAPP repair.

Complications: . Pain . Wound . Scar . Atrophy of . Deep Vein . Bleeding infection . Difficulty the testicle Thrombosis, . Seroma . Mesh passing urine (Shrinkage) Pulmonary infection . Hernia embolus recurrence . Bowel injury

(b) Surgical manifestations of abdominal tuberculosis. Abdominal tuberculosis Common surgical differential diagnosis in our daily practice.

Epidemiology:– Both gender: equally affected Most common age: 35-45 years – – Risk factors Alcoholic liver disease • HIV infection: 9% of all new TB cases are related to HIV – Advanced age – Low socioeconomic status – –

Intestinal tuberculosis:

Ulcerative type: Hyperplastic type:  Formation of mucosal ulcers  Extensive inflammatory changes

o Bleeding o Fistula o Obstruction. o Perforation o Stricture o Mass

Peritoneal tuberculosis:

Nodal/glandular tuberculosis:  Less common  Enlargement of  Mesenteric lymph nodes.  Retroperitoneal lymph nodes  Complications abscess formation

Solid visceral tuberculosis:–

Clinical presentations: Acute form (41%) Combined form (9%) Chronic form (50%)

(c) Traumatic duodenal injury. Introduction: Duodenal injuries are uncommon after blunt and penetrating trauma but can be challenging to diagnose and manage. Most are caused by penetrating mechanisms occurring in 6.7% of penetrating abdominal cases, most of which the result of gunshot wounds. The associated mortality is significant. In those that present with a blunt hollow visceral injury, 12% are located in the duodenum. The mortality after blunt duodenal injury ranges from 11.4% to 14.8%. Mechanism of injury: Blunt injuries are presumably caused by a blow to the epigastrium by a narrow object, resulting in contusion of the wall or a blowout secondary to acute elevation of intraluminal pressure. The classic description is the abdomen being struck by a steering wheel or, in children, a bicycle handlebar. Clinical features and diagnosis:  Although duodenal injuries after penetrating trauma are found at laparotomy, their identification after a blunt mechanism can be challenging and therefore require a high index of suspicion to avoid missed injuries.  Because of the retroperitoneal location of a significant portion of the duodenum, physical examination findings may be limited.  Even full-thickness perforations of the duodenum may not demonstrate peritoneal signs unless the perforation involves an intraperitoneal segment.  The mainstay of evaluation for duodenal injury has become abdominal CT, with a low threshold for operative exploration.  Findings on CT that reflect possible duodenal injury include thickened duodenal wall, air or fluid outside the bowel lumen, and contrast extravasation if oral contrast was administered.  Low-grade injuries resulting in a duodenal hematoma can be identified by CT, although it is important also to evaluate the pancreas because of a high rate of concomitant injury.

Duodenum injury scale

Grade Type of injury Description of injury

I Hematoma Involving single portion of duodenum Laceration Partial thickness, no perforation II Hematoma Involving more than one portion Laceration Disruption <50% of circumference III Laceration Disruption 50%-75% of circumference of D2 Disruption 50%-100% of circumference of D1,D3,D4

IV Laceration Disruption >75% of circumference of D2 Involving ampulla or distal common bile duct V Laceration Massive disruption of duodenopancreatic complex Vascular Devascularization of duodenum

Management:  Any indication of duodenal perforation on examination or imaging should prompt operative exploration.  At times, the findings are subtle but a low threshold for exploration should be maintained because of the potential for false-negative interpretations of the CT scan.  Upper GI contrast studies, DPL, and laboratory studies such as serum amylase level determination, have at most a limited role in the evaluation of duodenal injuries.  Management of duodenal injuries depends on the severity and location of the injury.  Hematomas of the duodenal wall typically require no treatment unless they are large and result in a gastric outlet obstruction. Treatment of obstructing hematomas consists of gastric decompression and initiation of total parenteral nutrition, with reevaluation of gastric emptying with a contrast study after 5 to 7 days. If after 2 weeks of upper GI and bowel rest the obstruction persists, exploration is warranted to evaluate for perforation, stricture, or associated pancreatic injury.  Duodenal hematomas identified at the time of laparotomy for another indication require careful evaluation for perforation. Frequently, they decompress during duodenal mobilization, although intentionally opening the serosa to drain an incidentally identified hematoma should generally be avoided in the absence of a full- thickness injury.  Most full-thickness injuries of the duodenal wall can be repaired primarily using a single- or double-layer approach, depending on the amount of tissue available. Adequate mobilization of the duodenum with a wide Kocher maneuver is required to provide necessary exposure and ensure a tension-free repair.  Duodenal transection can be managed with primary anastomosis as long as the ampulla is not involved and the segment is short. Larger segments of duodenal destruction may require more complex reconstruction, frequently using bypass around the injured duodenum. Any repair can be protected from the enteric contents by performing a pyloric exclusion and creating a .  In the damage control setting, the use of a duodenostomy tube or resection leaving the GI tract in discontinuity is highly effective for controlling contamination temporarily.

(d) Sterilization of laparoscopic instruments.

Introduction:

Laparoscopic surgery requires sophisticated and precisely calibrated instruments. The essential difference between instruments used in open surgery and people utilized for laparoscopic surgery would be that the latter are more complex in design and yet delicate in construction. Thus the laparoscopic instruments are more vulnerable to lodging of bioburden (micro-organisms and debris) within their crevices. Thus, the LI are difficult to clean, sterilize adequately and maintain as compared to their counterparts used in open surgery. Moreover, owing to their delicate design, gentlest methods have to be used for cleaning in addition to sterilization. Also, meticulous cleaning, maintenance in addition to sterilization are necessary so that not to compromise the safety from the patient, the surgeon or other operating room personnel. The rise in complexity of the laparoscopic procedures as also the emergence of resistant strains of bacteria, mycobacteria, fungi and viruses makes it imperative to effectively clean and disinfect instruments. Sterilization is the absolute elimination or destruction of forms of microbial life. It may be achieved with steam, gas or chemicals. However, disinfection is the relative removal of pathogenic organisms except spores.

Disinfection can be: a) High level - where all life forms except the spores are destroyed, b) Intermediate level - where some fungi, viruses and spores are spared, or c) Low-level - where fungi, viruses, spores and mycobacteria remain undestroyed. For laparoscopic instruments ideally sterilization or at best higher level disinfection should be used.

Cleaning and Sterilization Optimal processing of LI involves several steps that reduce the risk of transmitting infection from used instruments along with other what to healthcare personnel.

They are

1) Dismantling, 2) Decontamination, 3) Precleaning, 4) Cleaning and rinsing, 5) Drying 6) Sterilization and 7) Storage. For proper processing, it is essential to perform the steps in correct order.

Most major hospitals have a Central Sterile Supplies Department where the instruments are transported in the operating room for processing. Even in hospitals or nursing homes that do not have an elaborate CSSD, the fundamental steps in processing of instruments can be followed provided a well-established protocol is in place, and designated personnel receive the responsibility for the same. Proper processing of instruments forms an intrinsic aspect of their care which should undoubtedly significantly help in increasing their life time and trouble-free service.

Dismantling

The look of LI should be so that they ought to allow easy dismantling. Instruments that can't be dismantled completely are prone to harbour blood / debris within the shafts and compromise safety of the patients in whom they are utilised subsequently

Decontamination

Decontamination is the procedure used to reduce bioburden on reusable medical devices. The procedure begins in the theatre itself using the nursing staff wiping off visible blood tissue and body fluids in the instruments with a damp sterile sponge. At the conclusion of this all soiled or contaminated instruments should be placed in a container containing a disinfectant solution such as 0.5% chlorine and allowing them to soak for Ten minutes.

The instruments shouldn't be left on this solution for longer period of time as they could get damaged. Once the instruments get to the CSSD, a purpose-built bath is used for decontamination of their decontamination just before proceeding using the next step in the cycle. Modifications of the standard cleaning processes have to clean rigid endoscopic instruments effectively. Instruments designed with an external gasket, an internal seal that does not totally occlude the internal space, or no gasket ought to be placed in the vertical position in enzymatic cleaning and rinsing solutions, instead of the standard horizontal position, so the air trapped within the instrument is permitted to escape and replaces using the solution. All solutions should be irrigated through cleaning ports of instruments. During the process of manual cleaning, special attention ought to be given to intricate and delicate operating mechanisms located at the distal end of many instruments. An ultrasonic cleaner will boost the cleaning of hard-to-reach places. At the end of decontamination, the instrument should be safe for handling without contact with blood-borne pathogens.

Precleaning

Following the instruments reach the sterile supplies processing area, which is preferably a controlled environment, a pre-cleaning treatment with an enzymatic method is recommended. Numerous enzymatic products are available, viz. protease, lipase, amylase, which are effective in enhancing the cleansing process for difficult-to-clean instruments. These break up blood and other protein soil and facilitate cleaning. These enzymes are proteins, and must be removed by thorough cleaning.

Cleaning

Any instrument designed for autoclaving requires specialised cleaning just before sterilization. Users need to ensure that no residual, proteinaceous material or organic residue remains about the instrument surface. This is particularly important where the instrument has several small moving parts and crevices; build up of residues may eventually result in corrosive damage and pathogenic colonization (bioburden). Many hospitals adopt the technique of washing their instruments in soap scrubs. Although physical cleaning is partially effective, enzymatic and detergent based cleaners which dissolve and lift organic material from the surface of instruments are better suited to making certain instrument surfaces do understand of blood along with other body fluids and proteinaceous material before the sterilization process. For laparoscopic instruments this really is best completed using soft brushes that allow the inner surfaces from the instruments to be cleaned thoroughly.

Laparoscopic instruments are best rinsed in running water to ensure that all of the particulate matter in addition to residues of chemicals employed for contamination and cleaning are completely cleared from them. It is useful to possess cleaning guns with fine, pointed nozzles to wash theshafts from the laparoscopic hand instruments. The jet of water has the capacity to clean these instruments much better than rinsing“ them in stagnant” water.

A method of cleaning that's growing in popularity is ultrasonic cleaning. This method is, by far, the most efficient and effective available today. Its simplicity of use and superior efficiency is quickly making ultrasonic cleaning the preferred choice. Actually, ultrasonic cleaning is 16 times better than hand-cleaning. The instruments are placed in the ultrasonic unit for 10-15 minutes and use a neutral pH solution. Attention should be directed at the next points during ultrasonic cleaning:

 Before placing into the ultrasonic unit, the instruments are cleaned of all visible debris.  It s preferable to not mix instruments made of dissimilar metals (such as aluminum and stainless) in the same cycle. ’  It is important to ensure that the instruments have ample room. The ultrasonic cleaner shouldn't be overloaded.  As with all types of cleaning, all instruments ought to be opened so ratchets and box locks are fully subjected to the cleaning process.  Upon completion of the cycle, the instruments are removed immediately and rinsed.

Drying

The instruments should be dried at the end of the cleaning and rinsing cycle before they are packed for sterilization. This really is ideally achieved by using an air gun that blows all the water droplets off the surfaces of instruments or by using an oven. The second, however, may be available only in CSSD units.

Sterilization

The Centers for Disease Control (CDC) recommends that rigid laparoscopic instruments be sterile or, in the event that isn't feasible, they be high-level disinfected. There are three sterilization processes available to us - steam, ethylene oxide and peracetic acid. Because of product knowledge and proprietary design information, the instrument manufacturer may be the just one who can provide sterilization recommendations.

Steam sterilization

Steam sterilization in an autoclave is among the most typical forms of sterilization used in healthcare facilities. Autoclaving at 121 0C for 15minutes is ideal for all reusable metal instruments. It's effective, cheap and non-toxic. Laparoscopes may be sterilized by flash or vacuum steam sterilization. Before sterilization, all instruments that are insulated, all silicone tubing, and all sorts of cords ought to be doubly covered with a cloth to prevent connection with the hot metallic container. They are then put into the autoclave. Flash sterilization is carried out at 135 0C at 30 psi pressure for 60 minutes. This process requires post-vacuum and dry cycles. The instruments should rest on the sterilizer rack for 45 minutes to prevent water condensation about the lens all cords should be doubly wrapped in a cloth to avoid contact with the hot metallic container. They're then put into the autoclave. Flash sterilization is completed at 135 0C at 30 psi pressure for An hour. This method requires post-vacuum and dry cycles. The instruments should rest on a sterilizer rack for 45 minutes to prevent water condensation about the lens.

Gas sterilization

Using ethylene oxide (EO) would work for all disposable instruments, insulated hand instruments and tubings employed for gas, suction and irrigation. Endoscopic instruments may be sterilized with either cold or warm EO gas, with respect to the manufacturer s instructions. With cold gas, the temperatures are set at 85 0C and also the instruments are subjected for 4 hours and 30 minutes. Aeration must then follow for 12 hours. Warm’ gas sterilization happens at 145 0C for 2 hour 30 minutes, followed by 8 hours aeration. The benefits of EO are how the items aren't damaged, it's non-corrosive to optics also it permeates porous material. Its main disadvantages are its cost, toxicity, the requirement for aeration and being a longer process.

High level disinfection

When sterilization isn't available or feasible, high-level disinfection (HLD) is used for instrument processing. HLD eliminates bacteria, viruses, fungi, and parasites but doesn't reliably kill all bacterial endospores, which cause diseases such as tetanus, gas gangrene and atypical mycobacterial infections. HLD would work for items which will come in connection with broken skin or intact mucous membranes. The effectiveness of HLD depends on (a) the amount and kind of microorganisms, organic material (blood, other fluids, tissues), and other matter (for example dirt) present on the instrument or other item and (b) the quantity of protection them provides the microorganisms (such as if the item has grooves or the areas by which microorganisms can hide). Therefore it is important to decontaminate and thoroughly clean instruments along with other items before HLD.

Agents that are employed for HLD include 2% glutaraldehyde, 6% stabilized hydrogen peroxide and per acetic acid (acetic acid/hydrogen peroxide). Glutaraldehyde has got the benefits of having good biocidal activity, non-corrosive to optics and it is active in the presence of protein. Glutaraldehyde is irritating towards the skin, eyes, and respiratory system, especially at concentrations of 0.3 parts per million (ppm). The length of time that commercially available glutaraldehyde solutions may be used varies, usually from 14-30 days. It ought to be tested daily with the manufacturer s test strip. Always stick to the manufacturer s instructions regarding proper storage temperatures and expiration date. Solutions should be replaced any time they become cloudy.’ The efficiency of glutaraldehyde’ is influenced by the organic load, contact time and use pattern, concentration, physical configuration of instruments, temperature and pH. OSHA s established maximum allowable exposure limit for glutaraldehyde is 0.2ppm. Fibreoptic light cords and telescopes have to be soaked in 2% glutaraldehyde not less than Ten’ minutes. Soaking should not exceed twenty minutes. The endocamera could also disinfected by 10 minutes submersion in 2% glutaraldehyde. Care must be come to leave the plug end of the cord away from solution. Alternately, sterile drape over the camera and cord may be used. Soakage of other metallic instruments, including trocars, and hand instruments, has become recommended for An hour, to avoid infection with atypical mycobacterial infection. Formaldehyde, glutaraldehyde from phenolic derivatives, iodophors, hypochlorites, phenolics and quartery ammonium compounds are unpopular and it has been condemned. Formaldehyde is potentially cancercausing and very irritating to the skin, eyes, nose, and respiratory tract. Furthermore, its efficacy is found wanting, and for that reason, routine utilization of formaldehyde for sterilizing instruments and other items isn't recommended.

Newer methods of sterilization

Important for hospitals with high workload is the rapid turnaround times for instruments that can't be sterilized satisfactorily with steam or dry heat. One of the newer sterilizer system - STERRAD (Johnson & Johnson) - uses hydrogen peroxide vapor and low- temperature gas plasma to sterilize most devices quickly with no toxic residues. Usually, the process takes about 75 minutes for wrapped and dry instruments and devices. Within the chamber, a deep vacuum is drawn. Fifty-nine percent aqueous peroxide is vaporized into the chamber. The product will be enveloped within the peroxide vapor. Following a diffusion of the gaseous peroxide with the load, chamber pressure is reduced, permitting the generation of low-temperature gas plasma. Rf (RF) energy is put on the chamber via an RF amplifier, inducing the plasma state. Reactive species are generated in the peroxide on this state, reacting with materials and every other. When the high-energy species have reacted, they recombine to form water vapor, oxygen, along with other non-toxic byproducts. Upon completion of sterilization, instruments are dry for immediate use or sterile storage. Thus, recontamination risk is minimized, and given that they remain sterile until their next use, money and time is saved by avoiding reprocessing instruments when the case in canceled or delayed. This system occupies minimal space and requires no venting or water hookup. The only utility requirement is electrical hookup.

Storage

Items ought to be used or properly stored soon after sterilization or HLD so they do not become contaminated. Proper storage is as important as proper decontamination, cleaning, sterilization, or HLD. If items aren't stored properly, all of the effort and supplies used to properly process them will have been wasted, and the things is going to be contaminated. Specific instructions for proper storage rely on whether sterilization or HLD continues to be performed, the method used, and whether the items are wrapped or unwrapped.

The shelf-life of a wrapped item is suffering from numerous factors, including:

 The kind of packing material used  The number of times those is handled  The number of people who handle the pack  The cleanliness, humidity, and temperature from the storage space  Whether the packs are stored on open or closed shelves  Whether dust covers (for example sealed plastic bags) are utilized

For optimal storage, sterile packs are put in closed cabinets in areas that aren't heavily trafficked, have moderate temperatures, and are dry or of low humidity. Under optimal storage conditions and with minimal handling, properly wrapped items can be considered sterile as long as they remain intact and dry. Storage time and the handling of sterile packs ought to be kept to a minimum, because the probability of contamination increases over time and with increased handling. When in doubt about the sterility of a pack, consider it to become contaminated and resterilize the item before use.

THE WEST BENGAL UNIVERSITY OF HEALTH SCIENCES MS (General Surgery) Examination, 2013 PAPER II

Time Allowed: 3 Hours Full Marks: 100 Attempt all questions

1. How will you manage bile duct injury detected during laparoscopic chplecystectomy?20 2. Discuss the etiology, diagnosis and outline the management of chronic ulcer foot. 20

3. Write short notes of the following: 5 x 6 (a) Myopectineal orifice of Fruchand. (b) ARDS. (c) Tumour markers. (d) Paraneoplastic syndrome. (e) Benign oesophageal stricture.

4. Answer briefly on the following: 4 x 71/2 (a) Skin grafting and flaps. (b) Thromboprophylaxis. (c) Parotid fistula. (d) Discharging lesions of umbilicus.

THE WEST BENGAL UNIVERSITY OF HEALTH SCIENCES

MS (General Surgery) Examination, 2013

April 2013

PAPER II

Time Allowed: 3 Hours Full Marks: 100 Attempt all questions

1. How will you manage bile duct injury detected during laparoscopic chplecystectomy?20

Answer. LC has been associated with a higher incidence of IA bile duct injuries  LC 0.4 to 0.8%  Traditional OC 0.1-0.3%  Association: —  Increased mortality— and morbidity  Reduced long-term survival  Reduced quality of life  Infrequent but among the leading sources of malpractice claims against surgeons.  Between 34% and 49% of surgeons are expected to cause such an injury during their career. —  Awareness and preventative methods are of clinical importance to surgeons.

 Risk Factors ◦ Anatomical ◦ Anatomical variations (biliary and vasculature) ◦ Bleeding, scarring, obesity ◦ Laparoscopic ◦ Lack of Depth Perception, Tactile Feedback, Full Manual Maneuverability ◦ Improper surgical approach ◦ Improper Lateral retraction (insufficient or excessive) ◦ 0 degree scope ◦ Approach plane too deep Lack of conversion to OC during difficult cases.

Mechanism of injury: s Learning Curve Steady  Anatomical Misidentification: excision, incision, or transection of biliary anatomy o Injuries:Initially…Surgeon’ common bile duct, common hepatic– duct, right and left hepatic ducts, right hepatic artery, ducts draining hepatic segments o Anatomical variations (biliary and vasculature) o Electrocautery, thermal injury: stricture of CBD or hepatic ducts, bile leak\ o Mechanical trauma: stricture of the biliary ducts, bile leaks o Improper surgical approach.

Thermal injuries:

 Inappropriate use of electrocautery near biliary ducts  May lead to stricture and/or bile leaks  Mechanical trauma can have similar effects.

Strasburg Classification:

 Type A Cystic duct leaks or leaks from small ducts in the liver bed  Type B Occlusion of a part of the biliary tree, almost invariably the aberrant right hepatic ducts  Type C Transection without ligation of the aberrant right hepatic ducts  Type D Lateral injuries to major bile ducts  Type E Subdivided as per Bismuth classification into E1 to E5

E: injury to main duct (Bismuth) o E1: Transection >2cm from confluence o E2: Transection <2cm from confluence o E3: Transection in hilum o E4: Seperation of major ducts in hilum o E5: Type C plus injury in hilum

Management:

 Only 25-33% of injures are recognized intraoperatively  If experienced, convert to Open Procedure and perform Cholangiography (determine extent of injury)  If not experienced, perform the cholangiogram laparoscopically with intent of referring patient (placement of drains)  Consult an experienced hepatobiliary surgeon  Acute Management ◦ Biliary catheter for decompression of biliary tract and control of bile leaks ◦ Percutaneous drainage of intraperitoneal bile collection

2. Discuss the etiology, diagnosis and outline the management of chronic ulcer foot. 20

Answer.

Introduction: A foot ulcer is an open sore on the foot.

 A foot ulcer can be a shallow red crater that involves only the surface skin. A foot ulcer also can be very deep. A deep foot ulcer may be a crater that extends through the full thickness of the skin. It may involve , bones and other deep structures.  People with diabetes and people with poor circulation are more likely to develop foot ulcers. It can be difficult to heal a foot ulcer. In people with these conditions, even a small foot ulcer can become infected if it does not heal quickly.  If an infection occurs in an ulcer and is not treated right away, it can develop into:

o An abscess (a pocket of pus) o A spreading infection of the skin and underlying fat (cellulitis) o A bone infection (osteomyelitis) o Gangrene. Gangrene is an area of dead, darkened body tissue caused by poor blood flow

Among people with diabetes, most severe foot infections that ultimately require some part of the toe, foot or lower leg to be amputated start as a foot ulcer. Aetiology:

Causes of leg ulcers: Vascular Venous Neuropathic Diabetes Metabolic Diabetes Arterial Tabes Gout Mixed Syringomyelia Prolidase deficiency Haematological Sickle cell Trauma Pressure Tumors Basal cell disease Injury carcinoma Burns Squamous cell Cryoglobulinemia carcinoma Infection Bacterial Panniculitis Necrobiosis lipoidica Pyoderma Gangrenosum Fungal Fat necrosis Protozoal Special Hypertensive ulcer

Foot ulcers are especially common in people who have one or more of the following health problems:

 Peripheral neuropathy. This is nerve damage in the feet or lower legs. Diabetes is the most common cause of peripheral neuropathy. When nerves in the feet are damaged, they can no longer warn about pain or discomfort. When this happens, tight-fitting shoes can trigger a foot ulcer by rubbing on a part of the foot that has become numb.

People with peripheral neuropathy may not be able to feel when they've stepped on something sharp or when they have an irritating pebble in their shoes. They can injure their feet significantly and never know it, unless they examine their feet routinely for injury.

Many elderly people and diabetics with vision problems also can't see their feet well enough to examine them for problems.

 Circulatory problems. Any illness that decreases circulation to the feet can cause foot ulcers. Less blood reaches the feet, which deprives cells of oxygen. This makes the skin more vulnerable to injury. And it slows the foot's ability to heal.

Poor circulation in the leg arteries is called peripheral artery disease. It also causes pain in the leg or buttock during walking. It is caused by atherosclerosis. This is a disease in which fatty deposits of cholesterol build up inside arteries.

 Abnormalities in the bones or muscles of the feet. Any condition that distorts the normal anatomy of the foot can lead to foot ulcers. This is particularly true if the foot is forced into shoes that don't fit the foot's altered shape. Examples are claw feet, feet with fractures, and cases of severe arthritis. More than any other group, people with diabetes have a particularly high risk of developing foot ulcers. This is because the long-term complications of diabetes often include neuropathy and circulatory problems. Without prompt and proper treatment, a foot ulcer may require hospital treatment. Or, it may lead to deep infection or gangrene and amputation.

In addition to diabetes, other medical conditions that increase the risk of foot ulcers include:

 Atherosclerosis. This condition involves poor circulation to the legs.  Raynaud's phenomenon. This condition causes sudden episodes of decreased blood flow to the fingers and toes. During these episodes, the fingers and toes turn white as the blood supply diminishes. They turn blue, and red again as the circulation returns to normal.

It is rare for a foot ulcer to be unrelated to these risk factors and illnesses. A foot ulcer in a person who has none of these health problems may need to be checked for skin cancer, especially squamous cell carcinoma. This cancer occasionally looks like a foot ulcer.

Assessment of foot ulcer:

Investigations.

o The Ankle Brachial Pressure Index (ABPI) using a handheld Doppler ultrasound and sphygmomanometer can be carried out for more accurate assessment of arterial perfusion. The results are used to determine the likelihood of arterial insufficiency and can be used to guide the management plan.

When Doppler tests indicate arterial insufficiency, arterial duplex ultrasonography will (noninvasively) provide accurate anatomic and haemodynamic information on the site and extent of the arterial disease. When indicated, further detailed anatomic information for treatment planning can be obtained from magnetic resonance angiography, computer tomographic angiography, or digital subtraction angiography.

ABPI symptoms: management correlation guide:

o Accurate and regular measurement of the wound is important to give an objective assessment of the effectiveness of the current management plan. The Leg Ulcer Measurement Tool (LUMT) is a validated tool that has been developed to quantify leg ulcer assessment and can be used to track change in wound status over time. o Blood investigations such as complete blood count, erythrocyte sedimentation rate, blood sugar, lipid profile, renal function tests, and liver function tests are essential in patients with chronic leg ulcers. The plain radiography of the foot along with CT and MRI should be done to rule out osteomyelitis and malignancy. o Laboratory screening tests for vasculitis: urine analysis for proteinuria, hematuria, cylindruria, routine and immunohistopathology of skin biopsies, antinuclear antibodies, rheumatoid factor, complement C4, circulating immune complexes, paraproteins, immunoglobulin fractions, antineutrophil cytoplasmic antibodies, serological tests, and cultures for underlying infections. o Laboratory screening tests for clotting disorders: activated partial thromboplastin time, prothrombin time, thrombin time, factor V (Leiden) mutation (506R fi 506Q), factor II (prothrombin) mutation (20210G fi 20210A), antithrombin III, protein C and protein S, and lupus anticoagulant anticardiolipin. o Venography may be performed as an investigational procedure prior to valvular surgery. Lower extremities arteriography is indicated in patients with ischemic rest pain, intolerable claudication, impending gangrene, or the presence of nonhealing ulcers of suspected arterial origin. o Color duplex ultrasound scanning which is becoming the de facto standard for evaluation of venous obstruction is also used to assess the location and extent of reflux in venous ulcers. o Plethysmography and venous pressure data are important in determining the need for surgical bypass or valve replacement. Quantitative data on venous obstruction, calf muscle pump ejection fraction, and reflux are provided by air plethysmography, whereas venous pressure studies assess the physiological importance of anatomic obstruction because the collaterals may or may not provide adequate compensation for an obstructed pathway. o A quantitative bacterial culture is more specific and should be performed once wound infection is suspectd. This is performed by curetting or biopsying the bed of the ulcer. The quantitative biopsy is the current gold standard for assessing the quality and quantity of microbial pathogens within wound. Quantitative biopsies containing greater than 105 organisms per gram of tissue are considered significant, and systemic antibiotic therapy should be considered. If osteomyelitis is suspected, representative cultures need to be obtained from the bone or deepest tissue layers. o Ulcer biopsy is important in making a correct diagnosis and to rule out malignancy as these ulcers are prone to malignant transformation. This requires taking a deep wedge of tissue from the ulcer edge and can usually be performed under local anesthesia. Chronic ulcers are sometimes biopsied for experimental protocols: (A) to obtain information regarding the wound bed or the wound edge. (B) to grow cells in vitro from nonhealing wound. o The clinical application of gene variant analysis and evaluation in patients with venous leg ulcers implies that the high risk minority of patients could be identified in advance by means of a simple blood test that would act as a genetic screening device.

Treatment options for common foot ulcers:

Recent Advances in Management: Several researchers are still discovering other modalities of treatment.

 The discovery of miRNAs has opened up vast therapeutic opportunities. The knowledge of miRNA function in the regulation of wound healing and developing improved miRNA modulation techniques in the skin will help in translating this knowledge into more effective therapies.  The clinical practices could be strongly influenced by the results of the HFE genetic test. The presence of C282Y mutation would strengthen the indications and priorities for surgical correction of superficial venous insufficiency.  Chronic wounds are characterized by changes in cell receptors (integrins). The activation or inhibition of integrin receptors by various agents may provide an excellent means of influencing wound healing.  Venous leg ulcers can be healed with a spray formulation of allogeneic neonatal keratinocytes and fibroblasts without the need for tissue engineering, at an optimum dose of cells per mL every 14 days.  The regenerative medicine is utilizing therapeutic potential of the stem cells to promote skin regeneration. Stem cell-based therapies offer tremendous potential for skin regeneration following injury and disease. Functional stem cell units have been described throughout all layers of human skin, and the collective physical and chemical microenvironmental cues that enable this regenerative potential are known as the stem cell niche. Stem cells in the hair follicle bulge, interfollicular epidermis, dermal papillae, and perivascular space have been closely investigated as model systems for niche-driven regeneration. These studies suggest that stem cell strategies for skin engineering must consider the intricate molecular and biologic features of these niches. Innovative biomaterial systems that successfully recapitulate these microenvironments will facilitate progenitor-cell-mediated skin repair and regeneration.  According to Frade et al., the natural biomembrane of latex extracted from Hevea brasiliensis proved to be safe as a dressing, for it did not induce hypersensitivity reactions among the volunteers who underwent the patch test or among users of the natural biomembrane, as it was clinically and immunologically demonstrated by IgE levels.  The vegetal biomembrane was important for the induction of the healing, especially on the inflammatory stage, confirmed by the abundant exudation and debridement of the ulcers in relation of the control treatment of chronic venous ulcers, which seems to be directly related to the intense vascular formation followed by reepithelialization.  Authors report that a 115-aa fragment of -5) acts as an unconventional wound healing agent in mice. Topical application of F-5 peptide promoted acute and diabetic wound closure in mice far more effectivelysecreted than did Hsp90α PDGF- BB.(F

3. Write short notes of the following: 5 x 6 (a) Myopectineal orifice of Fruchand. (b) ARDS. (c) Tumour markers. (d) Paraneoplastic syndrome. (e) Benign oesophageal stricture.

Answer. (a) Myopectineal orifice of Fruchand.

 The term [myopectineal orifice] was coined originally by Dr. Henri Fruchaud, and refers to a "distinct area of weakness in the pelvic region". The term [myopectineal] arises from two root terms which are combined. The root term [-my-] means "muscle" and the term [-pect-] means "comb"or "pectinate". The word [pectineal] in this case refers to the pelvic bone area of origin of the pectinate muscle of the thigh.  Fruchaud postulated that the anterior abdominal wall has an area that is inherently weak, and that this area is genetically determined. As such, hernias are part of human nature, or as he stated, "a healthy man is, unknown to himself, a hernia bearer".  Fruchaud myopectineal orifice is an osseomyo- aponeurotic tunnel through which all the groin hernia comes out. This orifice is bounded by: Medially by the lateral border of the rectus sheath (1). Laterally by the iliopsoas muscle (2). Below by the pecten pubis and fascia covering it and the Cooper s ligament (3). Above by the arched fibres of internal oblique, transversus abominis muscle and the conjoint „ (4). ’

Schematic diagram of Fruchaud s myopectineal orifice

(b) ARDS. ’  Acute respiratory distress syndrome (ARDS), previously known as respiratory distress syndrome (RDS), acute lung injury, adult respiratory distress syndrome, or shock lung, is a severe, life-threatening medical condition characterized by widespread inflammation in the lungs. While ARDS may be triggered by a trauma or lung infection, it is usually the result of sepsis.  ARDS is a disease of the microscopic air sacs of the lungs (alveoli) that leads to decreased exchange of oxygen and carbon dioxide (gas exchange). ARDS is associated with several pathologic changes: the release of inflammatory chemicals, breakdown of the cells lining the lung's blood vessels, surfactant loss leading to increased surface tension in the lung, fluid accumulation in the lung, and excessive fibrous connective tissue formation.  The syndrome has a high mortality.The mortality rate with ARDS varies widely based on disease severity, a person's age, and the presence of other medical conditions. Signs and symptoms:  The signs and symptoms of ARDS often begin with two hours of a inciting event, but can occur after 1-3 days. Signs and symptoms may include shortness of breath, fast breathing, and a low oxygen level in the blood. A chest x-ray frequently demonstrates generalized infiltrates or opacities in both lungs, which represent fluid accumulation in the lungs.  Other signs and symptoms that occur in people with ARDS may be associated with the underlying disease process. For example, those with ARDS from sepsis may have low blood pressure and fever, while a person with pneumonia may have a cough.

Cause: The predisposing factors of ARDS are numerous and varied. Common causes of ARDS include sepsis, pneumonia, trauma, multiple blood transfusions, babesiosis, lung contusion, aspiration of stomach contents, and drug abuse or overdose. Other causes of ARDS include burns, pancreatitis, near drowning, or the inhalation of chemical irritants such as smoke, phosgene, or chlorine gas. Some cases of ARDS are linked to large volumes of fluid used during post-trauma resuscitation

Diagnosis:

ARDS is characterized by the following criteria:

 Lung injury of acute onset, within 1 week of an apparent clinical insult and with progression of respiratory symptoms  Bilateral opacities on chest imaging not explained by other lung pathology (e.g. Pleural effusion, pneumothorax, or nodules)  Respiratory failure not explained by heart failure or volume overload  Decreased arterial pao2/fio2 ratio: o mild ARDS: ratio is 201 - o moderate ARDS: 101 - o 300 mmHg (≤ 39.9 kPa) 200 mmHg (≤ 26.6 kPa) (a minimum positivesevere ARDS:end expiratory ≤ 100 mmHg pressure (≤ 13.3 (PEEP) kPa) of 5 cmH2O is required; it may be delivered noninvasively with CPAP to diagnose mild ARDS). A decreased PaO2/FiO2 ratio indicates reduced arterial oxygen content relative to that of the inhaled gas, indicating a failure of the lung to transport oxygen into the blood.

The above characteristics are the "Berlin criteria" of 2012 by the European Society of Intensive Care Medicine, endorsed by the American Thoracic Society and the Society of Critical Care Medicine. They are a modification of the previously used criteria:

 Acute onset  Bilateral infiltrates on chest radiograph sparing costophrenic angles  Pulmonary artery wedge pressure < 18 mmHg (obtained by pulmonary artery catheterization), if this information is available; if unavailable, then lack of clinical evidence of left atrial hypertensionif PaO2:FiO2 < 300 mmHg (40 kPa) acute lung injury (ALI) is considered to be present if PaO2:FiO2 < 200 mmHg (26.7 kPa) acute respiratory distress syndrome (ARDS) is considered to be present

An arterial blood gas analysis and chest X-ray allow formal diagnosis. Although severe hypoxemia is generally included, the appropriate threshold defining abnormal PaO2 has never been systematically studied. A severe oxygenation defect is not synonymous with ventilatory support. Any PaO2 below 100 (generally saturation less than 100%) on a supplemental oxygen fraction of 50% meets criteria for ARDS. This can easily be achieved by high flow oxygen supplementation without ventilatory support. While CT scanning leads to more accurate images of the lung tissue in ARDS, it has little use in the clinical management of patients with ARDS and remains largely a research tool.

Treatment:

Acute respiratory distress syndrome is usually treated with mechanical ventilation in the Intensive Care Unit. Ventilation is usually delivered through oro-tracheal intubation, or by tracheostomy possibilities of non-invasive ventilation are limited to the very early period of the disease or to prevention in individualswhenever prolonged with atypical ventilation pneumonias (≥2 weeks), lung contusion, is deemed or inevitable. major surgery The patients, who are at risk of developing ARDS. Treatment of the underlying cause is imperative. Appropriate antibiotic therapy must be administered as soon as microbiological culture results are available. Empirical therapy may be appropriate if local microbiological surveillance is efficient.

The origin of infection, when surgically treatable, must be operated on. When sepsis is diagnosed, appropriate local protocols should be enacted. Commonly used supportive therapy includes particular techniques of mechanical ventilation and pharmacological agents whose effectiveness with respect to the outcome has not yet been proven.

 Mechanical ventilation: The overall goal is to maintain acceptable gas exchange and to minimize adverse effects in its application. The parameters PEEP (positive end-expiratory pressure, to maintain maximal recruitment of alveolar units), mean airway pressure (to promote recruitment and predictor of hemodynamic effects) and plateau pressure (best predictor of alveolar overdistention) are used.

 Airway pressure release ventilation: No particular ventilator mode is known to improve mortality in airway pressure release ventilation (APRV).

Some practitioners favor airway pressure release ventilation when treating ARDS. Well documented advantages to APRV ventilation include: decreased airway pressures, decreased minute ventilation, decreased dead-space ventilation, promotion of spontaneous breathing, almost 24-hour-a-day alveolar recruitment, decreased use of sedation, near elimination of neuromuscular blockade, optimized arterial blood gas results, mechanical restoration of FRC (functional residual capacity), a positive effect on cardiac output (due to the negative inflection from the elevated baseline with each spontaneous breath), increased organ and tissue perfusion and potential for increased urine output secondary to increased kidney perfusion.

 Positive end-expiratory pressure: Positive end-expiratory pressure (PEEP) is used in mechanically ventilated patients with ARDS to improve oxygenation. In ARDS, three populations of alveoli can be distinguished. There are normal alveoli which are always inflated and engaging in gas exchange, flooded alveoli which can never, under any ventilatory regime, be used for gas exchange, and atelectatic or partially flooded alveoli that can be "recruited" to participate in gas exchange under certain ventilatory regimens. The recruitable aveoli represent a continuous population, some of which can be recruited with minimal PEEP, and others which can only be recruited with high levels of PEEP. An additional complication is that some alveoli can only be opened with higher airway pressures than are needed to keep them open, hence the justification for maneuvers where PEEP is increased to very high levels for seconds to minutes before dropping the PEEP to a lower level. PEEP can be harmful; high PEEP necessarily increases mean airway pressure and alveolar pressure, which can damage normal alveoli by overdistension resulting in DAD. A compromise between the beneficial and adverse effects of PEEP is inevitable.  Prone position: Distribution of lung infiltrates in acute respiratory distress syndrome is non-uniform. Repositioning into the prone position (face down) might improve oxygenation by relieving atelectasis and improving perfusion. If this is done early in the treatment of severe ARDS, it confers a mortality benefit of 26% compared to supine ventilation.  Fluid management: Several studies have shown that pulmonary function and outcome are better in patients that lost weight or pulmonary wedge pressure was lowered by diuresis or fluid restriction.  Corticosteroids: A study found improvement in ARDS using modest doses of corticosteroids. The initial regimen consists of methylprednisolone. After 3 5 days a response must be apparent. In 1 2 weeks the dose can be tapered. But high dose steroid therapy has no effect on ARDS when given within 24 hours of the onset of illness.– This study involved a small number of patients– in one center.  Nitric oxide: Inhaled nitric oxide (NO) potentially acts as selective pulmonary vasodilator. Rapid binding to hemoglobin prevents systemic effects. It should increase perfusion of better ventilated areas. Almitrine bismesylate stimulates chemoreceptors in carotic and aortic bodies. It has been used to potentiate the effect of NO, presumably by potentiating hypoxia- induced pulmonary vasoconstriction. In case of ARDS it is not known whether this combination is useful.

 Surfactant therapy: To date, no prospective controlled clinical trial has shown a significant mortality benefit of exogenous surfactant in adult ARDS.

 ECMO (Extracorporeal Membrane Oxygenation)

(c) Tumour markers.

Introduction:Tumor markers are indicators of cellular, biochemical, molecular, or genetic alterations by which neoplasia can be recognized. These surrogate measures of the biology of the cancer provide insight into the clinical behavior of the tumor. This is particularly useful when the cancer is not clinically detectable.

The information provided may be

• Diagnostic and distinguish benign from malignant disease • Correlate with the amount of tumor present (so-called tumor burden) • Allow subtype classification to more accurately stage patients • Be prognostic, either by the presence or absence of the marker or by its concentration • Guide choice of therapy and predict response to therapy

The ideal tumor marker has three defining characteristics: 1. The marker is expressed exclusively by the particular tumor 2. Collection of the specimen for the tumor marker assay is easy. 3. The assay itself is reproducible, rapid, and inexpensive.

Classification of tumour markers:

Potential Nonprotein Tumor Markers RNA-Based Markers Overexpressed/underexpressed transcripts Regulatory RNA (e.g., micro-RNA) DNA-Based Markers Single-nucleotide polymorphisms (SNPs) Chromosomal translocations bcr-abl (Philadelphia) Changes in DNA copy number — Microsatellite instability Epigenetic changes (e.g., differential promoter region methylation)

Protein Tumor Markers:  Carcinoembryonic Antigen: Carcinoembryonic antigen (CEA) is predominantly used clinically in patients with cancer of the colon and rectum. It is an oncofetal protein that is normally present during fetal life but can be seen in low concentration in healthy adults. Testing: Normal serum levels are less than 2.5 ng/mL, borderline if 2.5 to 5.0 ng/mL, and elevated if greater than 5.0 ng/mL. Borderline levels occur with benign disorders such as inflammatory bowel disease, pancreatitis, cirrhosis, and chronic obstructive pulmonary disease, and smoking can also increase CEA the upper limit of normal in smokers is considered 5 ng/mL. Screening: CEA is not useful as a screening— test because of its low sensitivity in early-stage disease elevated CEA levels occur in only 5% to 40% of patients with localized disease. Prognosis: Elevated CEA levels reflect the burden of tumor present. The degree of CEA elevation correlates— with increasing stage of disease, and therefore CEA levels have prognostic value. Preoperative serum CEA is an independent predictor of survival the higher the preoperative serum level, the poorer the prognosis. This effect persists even after patients are stratified for resectability and extent of local tumor invasion. — Monitoring: The most common application of CEA is to monitor patients for recurrent disease. CEA is most sensitive for hepatic or retroperitoneal metastasis and relatively insensitive for local, pulmonary, or peritoneal involvement.  α-Fetoprotein : -Fetoprotein (AFP) is used for the detection and management of HCC. It is an oncofetal antigen. Levels are elevated in the fetus, decrease sharply after birth, and are increased duringα pregnancy. Testing: AFP is measured with immunoassay kits, either enzyme-linked immunoassays or radioimmunoassays. The upper limit of normal for a healthy, nonpregnant adult is less than 25 ng/mL. Ten percent to 20% of HCCs do not have detectable levels of AFP. Levels are also raised in non-seminomatous testicular cancer, for which it is a valuable tumor marker (see discussion later). Twenty percent of patients with gastric or pancreatic cancer and 5% of patients with colorectal or lung cancer have significant elevations (>5 ng/mL) in serum AFP levels. Elevated levels are also seen in hepatitis, inflammatory bowel disease, and cirrhosis. Screening: AFP has an estimated sensitivity of 25% to 75%, a specificity of 76% to 94%, and a positive predictive value of 9% to 50%. Prognosis: The AFP concentration reflects tumor size, with levels higher than 400 ng/mL being associated with larger tumors. As a result, it has been shown that AFP correlates with stage and prognosis. The rate of increase, expressed as AFP doubling time, has also been associated with poorer prognosis. Monitoring: AFP has been shown to decline after resection or ablation. After complete resection, AFP levels should drop and remain at less than 10 ng/mL. Tumor regrowth after chemoembolization does not correlate with rate of increase in AFP or tumor burden. AFP levels usually decline in response to effective chemotherapy. Monitoring of AFP therefore avoids prolonged use of ineffective and potentially toxic chemotherapy.  Carbohydrate Antigen 19-9 : Carbohydrate antigen 19-9 (CA 19-9) is widely used as a serum marker for pancreas cancer, but its use is limited to monitoring response to therapy, not as a diagnostic marker. The CA 19-9 epitope is normally present within the biliary tree. Biliary tract disease, both acute and chronic, can elevate serum CA 19-9 levels. Testing: CA 19-9 is detected with an immunoassay, and the upper limit of normal for a healthy adult is 37 U/mL. Patients with negative Lewis blood group antigen cannot synthesize CA 19-9, and therefore it is not used as a serologic marker in these individuals, who make up about 10% of the population. Patients with benign biliary tract disease can have levels up to 400 U/mL. Significant numbers of patients with pancreatitis, either acute or chronic, also have elevated levels. Third, besides pancreatic cancer, CA 19-9 levels are also elevated in patients with other cancers, including those of the biliary tree (95%), stomach (5%), colon (15%), liver (HCC, 7%) and lung (13%). For colorectal cancer, CA 19-9 levels add little clinically useful information to determination of CEA levels. Screening: CA 19-9 is not useful as a screening modality because of its low sensitivity in early- stage disease. With increasing levels of CA 19-9, the diagnosis of pancreatic cancer becomes more accurate. Because of its frequent elevation in benign biliary tract disease, CA 19-9 is not useful in distinguishing benign from malignant distal common bile duct strictures. Prognosis: In patients with pancreatic cancer who have CA 19-9 detectable in their serum, the level has been shown to correlate with tumor burden. Monitoring: Serial measurement of CA 19-9 is used to monitor response to therapy. A rise in CA 19-9 after curative resection has been shown to precede clinical or computed tomographic evidence of recurrence by 2 to 9 months. In patients with unresectable/metastatic disease, failure of CA 19-9 levels to fall with chemotherapy reflects poor tumor response.

 Prostate-Specific Antigen: Prostate-specific antigen (PSA) is a serine protease that is formed in the prostatic epithelium and secreted into the prostatic ducts. PSA is considered a tissue-specific rather than a prostate cancer specific marker patients who have undergone curative radical prostatectomy, as well as females, have no detectable PSA. Testing: PSA is detected with an immunoassay.– Besides BPH, other— instances in which serum PSA levels may be elevated include prostatitis, prostatic massage, prostatic biopsy, and digital . Initial studies set the upper limit of normal for PSA at 4 ng/mL, with levels greater than 10 ng/mL being suspicious for malignancy and levels of 4 to 10 ng/mL being indeterminate. Screening: PSA is widely used as a screening tool for prostate cancer because it enables early detection and diagnosis of this disease. Screening detects prostate cancer earlier. However, much concern has been raised about the risk of overdiagnosis. Autopsy studies have found that prostate cancer can be found in 55% of men in their 5th decade of life and 64% in their 7th decade, thus indicating that a significant proportion of these cancers are not lethal. Monitoring Response to Therapy: After operative resection, the PSA level is expected to normalize after 2 to 3 weeks. In patients whose PSA level remained elevated 6 months after radical prostatectomy, recurrent disease eventually developed. In contrast, it takes 3 to 5 months for PSA to normalize after radiotherapy. However, failure of the PSA level to normalize after radiotherapy also predicts relapse. A rise in serum PSA is usually the first sign of either local recurrence or metastatic progression. In patients with advanced disease, PSA levels are also used to monitor response to systemic therapy.

 Carbohydrate Antigen 125: Carbohydrate antigen 125 (CA 125) is a carbohydrate epitope on a glycoprotein carcinoma antigen. It is present in the fetus and in derivatives of the coelomic epithelium, including the peritoneum, pleura, pericardium, and amnion. In healthy adults, CA 125 has been detected by immunohistochemistry in the epithelium of the fallopian tubes, endometrium, and endocervix. However, neither adult nor fetal ovarian epithelium expresses CA125.  Testing: CA 125 levels are measured with an immunoassay, with the upper limit of normal set at 35 U/mL. Elevated levels are detected in 80% of patients with ovarian cancer. It is also detectable in a high percentage of patients with cancer of the fallopian tube, endometrium, and cervix, as well as in nongynecologic malignancies of the pancreas, colon, lung, and liver. Benign conditions in which CA 125 is elevated include endometriosis, adenomyosis, uterine fibroids, pelvic inflammatory disease, cirrhosis, and ascites. As for CA 19-9 in patients with pancreatic cancer, CA 125 is an adjunct to diagnosis rather than being diagnostic by itself. Screening: Alone, CA 125 is not useful as a screening tool for ovarian cancer because of its poor specificity. Prognosis: Patients with elevated CA 125 levels at the time of diagnosis have a worse prognosis than patients with normal levels do. Absolute levels of CA 125 do not clearly correlate with tumor stage, although with increasing stage, greater percentages of patients have elevated CA 125 levels 50% of stage I patients, 70% of stage II patients, 90% of stage III patients, and 98% of stage IV patients. Monitoring— Response to Therapy: CA 125 is of value in monitoring the disease course. Partial or complete response to therapy is associated with a decrease in CA 125 levels in more than 95% of patients. Increasing levels of CA 125 correlate with disease recurrence and precede clinical or imaging evidence of recurrence by a median of 3 months. When rising CA 125 levels are used as an indication for second-look laparotomy, recurrent disease is found approximately 90% of the time. CA 125 levels in peritoneal fluid may be more sensitive than serum levels. Thus, in patients whose serum CA 125 level normalizes during therapy, peritoneal fluid CA 125 levels may better be able to distinguish patients with residual disease from those without. The upper limit of normal for peritoneal fluid CA 125 is 200 U/mL.

 α-Fetoprotein and Human Chorionic Gonadotropin in Testicular Germ Cell Tumors: Nonseminomatous testicular cancers comprise several different histologic types, including embryonal carcinoma, syncytiotrophoblasts (choriocarcinoma), yolk sac tumors, and teratomas. Marker expression can be predicted on the basis of the predominant histologic type human chorionic gonadotropin (HCG) is detected in more than 90% of choriocarcinomas, whereas AFP is expressed by 90% to 95% of yolk sac tumors, 20% of teratomas,— and 10% of embryonal carcinomas. Diagnosis: Of patients with proven nonseminomatous testicular germ cell tumors, about 50% will have elevated serum levels of HCG and 60% will have elevated AFP, with either marker being elevated in 90% of cases. Determination of both marker levels is very important because nearly half these tumors secrete only one of these substances. In addition to the high rate of marker positivity, there have been very few cases of spuriously elevated serum levels of HCG or AFP in patients without testicular cancer. The presence of a testicular tumor in combination with an elevated level of AFP or HCG is suggestive of testicular cancer, without being diagnostic. Elevated levels of these markers in a man younger than 40 years without signs of a testicular tumor may indicate extratesticular germ cell cancer. Prognosis: An absolute AFP concentration greater than 500 ng/mL or an HCG level higher than 1000 ng/mL is predictive of a poor prognosis. These tumor markers are useful in identifying biologically distinct categories of morphologically similar tumors. Monitoring: In the majority of patients with nonseminomatous germ cell tumors, tumor marker levels correlate with response to chemotherapy. After completion of primary therapy, increasing marker concentrations, even in the absence of other features of recurrence, may lead to salvage chemotherapy. Therefore, it is important to exclude false-positive results. The HCG level needs to be measured in urine, where the concentration is generally similar to that in serum; however, interfering substances are not excreted into urine. Intensive chemotherapy may induce hypogonadism with associated HCG levels of up to 5 to 10 IU/L. It can be differentiated from relapse by measurement of luteinizing hormone and follicle-stimulating hormone similar to the postmenopausal state in women, levels higher than 30 to 50 IU/L indicate that HCG is derived from the pituitary. —

DNA-Based Markers: Specific mutations in oncogenes, tumor-suppressor genes, and mismatch repair genes can serve as biomarkers. These mutations may be germline, such as the ret proto-oncogene of MEN 2 and the APC gene of FAP, or somatic mutations, such as the occurrence of p53 mutations in a wide variety of tumors. Chromosomal abnormalities such as the 9:22 translocation that creates the bcr-abl oncogene are also useful biomarkers. Specific single-nucleotide polymorphisms have been identified that are associated with increased risk for specific cancers, and haplotype assessment has been shown to predict susceptibility to several cancers, including prostate, breast, lung, and colon cancer. Epigenetic Changes: Testing for epigenetic changes is still at an early discovery stage and has not yet reached the clinic. However, it has great potential for a number of reasons. First, DNA assays for aberrant methylation are easier and more sensitive than those for point mutations. Second, cancer- specific DNA methylation patterns can be detected in tumor-derived free DNA in the bloodstream and in epithelial tumor cells shed into the lumen. This ease of access to sample medium may facilitate efforts at detection and monitoring of cancer. Third, DNA-methylation profiles are more chemically and biologically stable than RNA or most proteins. As a result, they may be more reliably detected in diverse biologic fluids. Methylation biomarker studies have been performed in a variety of cancers, including breast, esophageal, gastric, colorectal, and prostate cancer. Sources of the DNA have included plasma/serum, urine, sputum, and saliva. Combining DNA methylation assays may complement existing screening methods with high sensitivity but low specificity, such as PSA in prostate cancer. The use of panels of methylation targets in these studies improved the clinical sensitivity of the assay.

Potential Applications: 1. Early detection. Although abnormal epigenetic silencing of genes can occur at any time during carcinogenesis, it appears to occur most frequently early in the transformation process. Aberrant crypt foci that contain preneoplastic hyperplastic colonic epithelial cells have been found to demonstrate abnormal methylation in promoter regions of genes involved in abnormal activation of the Wnt signaling pathway. Detection of abnormal methylation patterns in histologically normal cells may emerge as a useful marker for assessment of cancer risk. 2. Predict response to therapy. Methylation of specific genes can be linked to the biologic behavior of the tumor. MGMT methylation was associated with prolonged survival in glioma patients treated with carmustine and in patients with large diffuse B-cell lymphoma who were treated with cyclophosphamide as part of multidrug regimens.Widschwendter and associates studied the correlation between methylation profiles and hormone receptor status in breast cancer. ESR1 methylation outperformed hormone receptor status as a predictor of clinical response in patients treated with tamoxifen. Individual methylation markers, such as the E-cadherin promoter, have also been linked to breast cancer metastasis. 3. Prognostication: Abnormal methylation of combinations of genes has been associated with a poor outcome. RNA-Based Markers: RNA-based markers have been identified in the context of global mRNA expression by high-throughput technologies. Proteomic Profiling: Proteomics is the study of all the proteins expressed by the genome. Proteomic profiling using mass spectrometry technologies generates complex fingerprints of ion peaks corresponding to protein concentrations, which can be correlated with disease states. Numerous studies using samples of blood (plasma or serum), urine, and pancreatic juice have demonstrated the feasibility of this technology for discovery of biomarkers and early detection of ovarian, breast, prostate, and pancreatic cancer.

Biomarkers and Biologically Targeted Therapies: CANCER BIOMARKER THERAPY Breast Estrogen receptor, Tamoxifen/aromatase progesterone receptor inhibitors Lymphoma CD20 Rituximab Chronic myelogenous bcr-abl Imatinib leukemia (CML) Gastrointestinal stromal c-kit Imatinib tumor (GIST) Non small cell lung cancer EGFR mutation Gefitinib Breast HER2/neu Trastuzumab –

(d) Paraneoplastic syndrome.

Introduction: A paraneoplastic syndrome is a syndrome (a set of signs and symptoms) that is the consequence of cancer in the body but that, unlike mass effect, is not due to the local presence of cancer cells. These phenomena are mediated by humoral factors (by hormones or cytokines) excreted by tumor cells or by an immune response against the tumor.

 Paraneoplastic syndromes are typical among middle-aged to older patients, and they most commonly present with cancers of the lung, breast, ovaries or lymphatic system (a lymphoma). Sometimes, the symptoms of paraneoplastic syndromes show before the diagnosis of a malignancy, which has been hypothesized to relate to the disease pathogenesis. In this paradigm, tumor cells express tissue-restricted antigens (e.g., neuronal proteins), triggering an anti-tumor immune response which may be partially or, rarely, completely effective in suppressing tumor growth and symptoms. Patients then come to clinical attention when this tumor immune response breaks immune tolerance and begins to attack the normal tissue expressing that (e.g., neuronal) protein.  Paraneoplastic syndromes can be divided into four main categories: endocrine, neurological, mucocutaneous and hematological paraneoplastic syndromes, as well as others that may not fit into any of the above categories:

Neurological: A particularly devastating form of paraneoplastic syndromes is a group of disorders classified as paraneoplastic neurological disorders (PNDs). These paraneoplastic disorders affect the central or peripheral nervous system; some are degenerative, though others (such as LEMS) may improve with treatment of the condition or the tumor. Symptoms of paraneoplastic neurological disorders may include ataxia (difficulty with walking and balance), dizziness, nystagmus (rapid uncontrolled eye movements), difficulty swallowing, loss of muscle tone, loss of fine motor coordination, slurred speech, memory loss, vision problems, sleep disturbances, dementia, seizures, sensory loss in the limbs.

The most common cancers associated with paraneoplastic neurological disorders are breast, ovarian and lung cancer, but many other cancers can produce paraneoplastic symptoms as well.

Treatment options include: Therapies to eliminate the underlying cancer such as chemotherapy, radiation and surgery.

Rapid diagnosis and treatment are critical for the patient to have the best chance of recovery. Since these disorders are relatively rare, few doctors have seen or treated PNDs. Therefore, it is important that PND patients consult with a specialist with experience in diagnosing and treating paraneoplastic neurological disorders

(e) Benign oesophageal stricture.

Introduction: Benign (noncancerous) esophageal stricture is a condition in which the diameter of the esophagus is made narrow by scar tissue. Even though the condition is not cancerous, it can still cause serious health issues. Narrowing of the esophagus can cause swallowing difficulty and also increases choking risk.

Causes of Benign Esophageal Stricture:

Benign esophageal stricture can be caused by damage to the esophagus that results in the formation of scar tissue. The most common cause is gastroesophageal reflux disease (GERD).

Other causes of the condition include:

 Extended use of a nasogastric tube (a tube inserted through the nose, into the esophagus and the stomach)  Ingestion of an acidic or corrosive substance, such as batteries or household cleaners  Damage caused by an endoscope (a thin, lighted tube used to see images inside the body for evaluation)  Treatment of esophageal varices (enlarged veins in the esophagus that can bleed and require repair, often with an endoscope)  Radiation therapy to the chest or neck

Risk Factors:

If one have experienced esophageal scarring, you are at an increased risk for benign esophageal stricture. GERD is the most common cause of scarring. Having GERD increases your risk of developing stricture.

Symptoms of Benign Esophageal Stricture

 Dysphagia: difficulty swallowing or feeling that food is not passing into the stomach normally  Painful swallowing  Unintended weight loss  Regurgitation of food or liquids: substances from the stomach flow back into the esophagus or up into the mouth  Heartburn

Potential Complications: o Problems with swallowing can prevent from getting the right amount of food and fluids necessary for good nutrition, which may lead to dehydration and malnutrition. o With the narrowing of the esophagus, denser solid foods, such as meat, can become lodged in the esophagus above the stricture, causing one to choke or have trouble breathing. o Regurgitating foods and liquids can result in pulmonary aspiration.This could result in choking or aspiration pneumonia, a condition where the lungs and the airways leading into the lungs become inflamed.

Diagnosis of Benign Esophageal Stricture: o Barium Swallow Test: In this test, a series of X-rays of the esophagus are taken during and after drinking a barium contrast material. This contrast material is a liquid that will temporarily coat the linings of esophagus to help it show up more clearly on the X-ray images. o Upper Endoscopy: In this test, an endoscope is inserted into the esophagus to allow for examination and biopsy of a stricture to determine its cause. o Esophageal pH Monitoring: In this test, a tube is inserted through your mouth into your esophagus to measure how much stomach acid enters your esophagus over a 24-hour period.

Treatment of Benign Esophageal Stricture:

Treatment will vary depending on the severity of the stricture as well as its underlying cause. Treatment options may include: o Esophageal Dilation: In most cases, esophageal dilation, or stretching, is the preferred option. This procedure is performed under general or local anesthesia for your comfort. An endoscope is passed through your mouth and into your esophagus, stomach, and small intestine. A small balloon on the end of the endoscope is inflated to stretch the esophagus.

This procedure may need to be repeated after a period of time to prevent the stricture from narrowing again. o Esophageal Placement: The insertion of esophageal can provide relief from stricture. A stent is a thin tube that may be made of plastic, expandable metal, or flexible mesh material. o Diet & Lifestyle: Daily lifestyle and dietary choices can be effective in managing chronic GERD, which is the primary cause of stricture. Recommendations can include:

 Avoiding tight clothes  Elevating the head of your bed to reduce backup of stomach contents  Fasting for three hours before bedtime  Losing approximately five percent of your body weight  Stopping smoking  Avoiding alcohol  Choosing smaller, instead of larger, meals  Avoiding foods that cause reflux, such as:

Spicy Fatty Carbonated Chocolate Coffee and Tomato- Citrus foods foods beverages caffeinated based products products foods

Drugs & Medication

Medications also can be an important part of your therapy. A group of acid-blocking drugs, known as protein pump inhibitors (PPIs), are the most effective options for managing GERD and preventing the backup of stomach acid that can damage your esophagus.These drugs act by blocking the so-called proton pump (enzymes), which leads to a reduced release of stomach acid. Your physician may prescribe these medications for short-term relief to allow your stricture to heal, or long-term therapy to prevent recurrence.

The PPIs used to control GERD include:

 Omeprazole  Lansoprazole  Pantoprazole  Esomeprazole

Other medications may also be effective in treating GERD and reducing your risk of stricture. They are:

 Antacids, which provide short-term relief by neutralizing the stomach s acids  Sucralfate, which provides a barrier that lines the esophagus and stomach to protect it from acidic stomach juices ’  Medications such as ranitidine and famotidine, which decrease the secretion of acid

Surgery: If medication and esophageal dilation are ineffective, your physician may recommend esophageal surgery. A surgical procedure can repair your lower esophageal sphincter, which is the valve between your esophagus and stomach. When working properly, it prevents the reflux of acid into the esophagus; however, this valve is dysfunctional in cases of GERD. By repairing the valve, this surgical technique allows for complete control of GERD symptoms.

Treatment can be difficult in the rare cases where stricture is caused by a non-functioning esophagus. In these extreme cases, an esophageal replacement may be the only effective option.

Part 8 of 9: Long-Term Outlook

Long-Term Outlook

Treatment, while effective, may not be permanent. You may need to take medication throughout your lifetime to control GERD and reduce your risk for stricture. For patients who undergo esophageal dilation, approximately 30 percent require a repeat of the dilation within one year (Banki).

Part 9 of 9: Prevention How to Prevent Benign Esophageal Stricture

You can help prevent benign esophageal stricture by avoiding substances that can damage your esophagus. Protect children and keep all corrosive household substances out of their reach.

Managing symptoms of GERD can greatly reduce your risk for stricture. Follow your physician s instructions regarding dietary and lifestyle choices that can minimize the reflux of acid into your esophagus, and take all medication as prescribed to control symptoms of GERD. ’

4. Answer briefly on the following: 4 x 71/2 (a) Skin grafting and flaps. (b) Thromboprophylaxis. (c) Parotid fistula. (d) Discharging lesions of umbilicus.

Answer.

(a) Skin grafting and flaps. Split-thickness skin grafts:  Thicker knife-gap settings give rise to fewer but brisker bleeding points on the donor site.  Thicker grafts heal with less contracture and are more durable.  Thinner donor sites heal better.  Grafts are hairless and do not sweat (these structures are not transferred). Full-thickness skin grafts:  Full-thickness grafts are harvested to incorporate the whole dermis, with the underlying fat trimmed away unless elements of fat (or even cartilage as well) are deliberately left attached to form a composite graft.  Full-thickness and– composite grafts require the most careful handling and postoperative nursing to help ensure that they take in their transplanted site.

Grafts are tissues that are transferred without‘ ’ their blood supply, which therefore have to revascularise once they are in a new site.

They include the following: Split-thickness skin grafts (of varying thickness). These are sometimes called Thiersch grafts. They are used to cover all sizes of wound, are of limited durability and will contract. They may be used to provide valuable temporary wound closure before better cosmetic secondary correction after rehabilitation. Full-thickness skin grafts (Wolfe grafts). Used for smaller areas of skin replacement where good elastic skin that will not contract is required (such as fingers, eyelids, facial parts). Composite skin grafts (usually skin and fat, or skin and cartilage). Often taken from the ear margin and useful for rebuilding missing elements of nose, eyelids and fingertips. Nerve grafts. Usually taken from the sural nerve, but smaller cutaneous nerves may be used. Tendon grafts. Usually taken from the palmaris longus or plantaris tendon (runs just anteromedial to the Achilles tendon) and used for injury loss or nerve damage correction.

Flaps Flaps are tissues that are transferred with a blood supply. They therefore have the advantage of bringing vascularity to the new area. Flaps can be raised to consist of any specific tissue; for skin flaps the following will illustrate the types that exist Random flaps. Three sides of a rectangle, bearing no specific relationship to where the blood supply enters; the length to breadth ratio is no more than 1.5:1. This pattern can be lengthened by delaying the flap, a process in which the cuts are partially made and the flap is part lifted at a first operation; it is then replaced, thus training the blood supply from a single border of‘ the rectangle.’ At a second procedure, it is raised further and finally transferred. Axial flaps. Much longer flaps, based on known blood ‘vessels supplying’ the skin. This technique was rediscovered in the 1960s and 1970s and enables many long thin flaps to be safely moved across large distances. Pedicled/islanded flaps. The axial blood supply of these flaps means that they can be swung round on a stalk or even fully islanded so that the business end of the skin being transferred can have the pedicle buried. Free flaps. The blood supply has‘ been ’isolated, disconnected and then reconnected using microsurgery at the new site Composite flaps. Various tissues are transferred together, often skin with bone or muscle (osseocutaneous or myocutaneous flaps, respectively). Perforator flaps. This description refers to a whole new subgroup of axial flaps in which tissues are isolated on small perforating vessels that run from more major blood vessels to supply the surface.

(b) Thromboprophylaxis: The prevention of clots forming in the veins can be achieved in three ways: chemical thromboprophylaxis; mechanical thromboprophylaxis; or, a combination of chemical and mechanical. Explanation of Risks which increase the risk of VTE  Age: There is an exponential increase in risk with age. In the general population: < 40years annual risk 1/10,000 60-69years annual risk 1/1,000 > 80 years – annual risk 1/100 This may reflect– immobility and coagulation activation.  Obesity :The– risk of VTE is increased by three times if BMI>=30kg/m2,again this may reflect immobility and coagulation activation.  Varicose Veins: The risk is increased by 1.5 times when a patient with varicose veins undergoes major orthopaedic or general surgery. However the risk of VTE is low after varicose vein surgery.  Immobility: Bed rest greater than 3 days. Lower limb immobility with plaster cast.  Dehydration: Increases the viscosity of the blood.  Previous VTE: Recurrence rate is 5% / year and this is increased by surgery.  Family history of VTE  Unidentified inherited thrombophilias will be risk factors which may be undiagnosed at time of admission.  Surgery:Without prophylaxis the rates of VTE after major orthopaedic surgery are 40 to 60%. Major surgery includes that which is greater than 90 minutes duration.  Pregnancy / post partum:There is a tenfold increase in thrombotic risk throughout pregnancy and the puerperium. The same factors that increase thrombotic risk in non- pregnant patients also increase the risk of thrombosis associated with pregnancy. During pregnancy additional risk factors such as pre-eclampsia and delivery, particularly operative delivery, must be considered. If a patient is admitted to a general ward and is pregnant, the obstetric team must be informed and the appropriate risk assessmentcarried out.  Thrombophilias . Low coagulation inhibitors (antithrombin, protein C or S) . Activated protein C resistance (e.g. factor V Leiden) . High coagulation factors (I, II, VIII, IX, XI) . Antiphospholipid Syndrome . High homocysteine All patients with thrombophilias having major surgery should have their care discussed with a Consultant Haematologist. There may be a need for extended thromboprophylaxis.  Other thrombotic states . Malignancy 7 x risk compared to the general population . Heart failure . Recent myocardial infarction / stroke . Acute infection . Inflammatory bowel disease, nephrotic syndrome . Polycythaemia, paraproteinaemia . Bechet s disease, paroxysmal nocturnal haemoglobinuria  Travel: Greater than 3 hours within four weeks (pre and post) of surgery  Lower limb cast’ or similar appliance :These inhibit venous return and increase the risk of pooling in the calf veins.  Medicines which increase the risk of VTE: . Oral contraceptives / Hormone Replacement Therapy (HRT) and hormone therapy. The use of oral contraceptives (oestrogen and oestrogen/progestogen) or HRT confers an increased risk of thromboembolism on all patients regardless of whether they are undergoing a surgical procedure. . Progestogen: Progestogen only contraception - No increased risk is conferred on the patient. Therefore, advise patient to continue the contraceptive and risk assess the patient according to procedure and other thromboembolic or clinical risk factors . High dose progestogens: There is evidence of an association of VTE with higher doses of progestogen used for other therapeutic indications (e.g. menstrual disorders) therefore the patient will need at least 20mg enoxaparin (dose dependent on other risk factors) . Combined oral contraceptives (COCP) and HRT . Oestrogen Receptor antagonists- Tamoxifen . Aromatase inhibitors . Thalidomide

The options available for thromboprophylaxis are:  Mechanical thromboprophylaxis o Anti-embolism stockings o Intermittent-pneumatic compression devices or foot impulse devices (may be used in Theatre) o Inferior vena caval filter  Pharmacological thromboprophylaxis o Low molecular weight heparin Enoxaparin o Oral anticoagulant Rivaroxaban for use in elective THR, TKR only o Unfractionated heparin – – (c) Parotid fistula. Introduction: A parotid fistula is a communication between the skin and a salivary duct or gland, through which saliva is discharged. Causes:  and duct injuries are rare complications following surgery of parotid gland and temporomandibular joint.  Various other causes of parotid injury are rupture of parotid abscess, inadvertent incision of parotid abscess, complication of superficial parotidectomy, gunshot wounds and trauma. Clinical features: Clinical features include salivary extravasations into the tissues causing swelling over or adjacent to parotid gland (sialocele), expanding neck mass and cutaneous fistula formation. In glandular fistulas discharge is less and tends to heal spontaneously with conservative treatment, where as ductal fistulas continuously discharge saliva and spontaneous healing is very rare. Classification of parotid injuries An injury classification system has been devised by Van Sickels. This system divides the parotid injuries into three regions: 1.Posterior to the masseter or intraglandular (site A), 2.Overlying the masseter (site B), and 3.Anterior to the masseter (site C). Diagnosis:  Injury to the parotid duct may be difficult to diagnose; therefore, the initial examining physician must have a high index of suspicion for injuries occurring in the parotid region. If not recognized will lead to salivary fistula and sialocele formation which will not heal spontaneously because of continuous flow of saliva. Successful treatment depends on early recognition and appropriate early intervention.  Examination of parotid injuries should include assessment of location , size ,shape , type (e.g., puncture, laceration, avulsion, crush, abrasion) , asymmetry, drainage (i.e., quality, character, odor) tenderness , surrounding erythema, oedema, cellulitis, or crepitation and facial nerve status.  The most straightforward way to diagnose a parotid duct injury is to cannulate the intraoral parotid duct papilla with a small silastic tube and observe if the tube is visible in the wound. This test does require patient cooperation; therefore, it may be difficult or impossible in children, individuals with intoxication, or individuals with mental disabilities. If any question regarding the diagnosis remains, a small amount of saline may be injected through the tube and observed for flow through the wound.  Methylene blue probably should not be injected through the tube because it terribly discolours tissues and makes subsequent operation even more challenging.  may be performed but is usually not necessary to establish the diagnosis of parotid duct injury. If performed, water-soluble contrast material should be employed because it is more easily drained and absorbed, and it does not remain as an irritant to the gland.  In doubtful cases fluid can be sent for laboratory analysis; raised salivary amylase levels confirm the diagnosis.  Computed tomography fistulography can be performed to look for the extent of the fistula. Treatment options

-Sialogogues

erization of the Fistulous

(d) Discharging lesions of umbilicus.

Causes of discharging lesions of umbilicus: Urachal abnormalities Vitello Intestinal Duct Inflammatory and related abnormalities disorders

 Urachal Sinus  Umbilical Adenoma /  Umbilical granuloma  Urachal Diverticulum Polyp  Omphalitis  Urachal Cyst  Vitelline Sinus  Vesicoumbilical fistula  Umbilical fistula  Alternating sinus

Vitelline Duct is connection between midgut and yolk sac  Usually involutes in 7th 9th weeks

Treatment: Symptomatic children with vitello-intestinal or omphalomesenteric duct remnants should be resuscitated before intervention. Those with significant haemorrhage should be transfused. Patients with obstructive symptoms should be resuscitated as rapidly as possible to obviate the need for ischaemic bowel resection. The incision chosen varies with the symptoms and the age of the patient. Children with faeculent umbilical drainage or prolapse of the omphalomesenteric duct remnant can be explored by a small infraumbilical incision. When connected to the ileum, a patent vitello-intestinal duct may have a wide lumen or be a high-output fistula. This may lead to fluid and electrolyte deficits, especially in neonates, and treatment should be expeditious. The treatment of a patent vitello-intestinal is wedge or segmental resection. Children with Meckel s diverticulitis or a bleeding Meckel s diverticulum are operated on by using a transverse incision with medial extension if necessary. Patients with suspected intestinal obstruction’ should be explored through’ a generous laparotomy incision. An open diverticulectomy includes the following steps: 1. A transverse appendectomy incision or subumbilical incision is made. 2. The caecum and ileum are identified. 3. The ileum is followed proximally to find Meckel s diverticulum, approximately 60 cm from the ileocaecal valve. 4. The diverticulum with the ileum are delivered into’ the wound. 5. The diverticulum is excised with the adjacent ileum and primary ileal end-to-end anastomosis is fashioned.

THE WEST BENGAL UNIVERSITY OF HEALTH SCIENCES MS (General Surgery) Examination, 2012 PAPER II Time Allowed: 3 Hours Full Marks: 100

1. Discuss the etiology, diagnosis and management of Melanoma. 20 2. What are the causes of enterocutaneous fistula? Classify them and give an outline of their management. 5+5+10

3. Write short notes of the following: 5x6 (a) Tuberculous peritonitis. (b) Achalasia cardia. (c) Short bowel syndrome. (d) Pheochromocytoma. (e) Gynecomastia.

4. Answer briefly on the following: 4x71/2 (a) Spigelian hernia. (b) Desmoid tumour. (c) Haemobilia. (d) Alvardo score.

THE WEST BENGAL UNIVERSITY OF HEALTH SCIENCES MS (General Surgery) Examination, 2012 May 2012 PAPER II Time Allowed: 3 Hours Full Marks: 100

1. Discuss the etiology, diagnosis and management of Melanoma. 20

Answer. Introduction: The incidence and outcome of melanoma are related to multiple factors.

 Melanoma is prevalent in whites.  This disease is less common in Asian and black populations.  Melanoma develops 20 times more frequently in whites than in blacks.  The disease occurs slightly more often in men than women, and the prognosis is slightly better for women.  The anatomic distribution of melanoma varies between the two genders. Melanomas arise more commonly on the lower extremity in women and more often on the trunk and head and neck in men.  Melanoma can occur at any age from birth to advanced age. The median age at diagnosis is in the range of 45 to 55 years. Tumors rarely develop before the age of puberty; however, there is a significant incidence in the third and fourth decades of life.

Risk factors -  Skin type: People with white racial background have ten times melanoma incidence of African Americans and seven times the melanoma incidence of American Hispanics.  Age: The incidence of melanoma increases with age.  Gender: Incidence of melanoma is higher in men than in women.  Tanning bed use: The use of a tanning bed more than ten times per year is associated with a doubling in the risk of melanoma for patients age 30 years or older.  Previous melanoma.  Sunlight exposure: Occasional or recreational exposure to sunlight, especially a history of severe blistering sunburn, has been associated with increased risk of melanoma. Sunburns after the age of 20 years may be associated with an increased risk of melanoma. The effects of sunlight have been attributed to UV-B radiation.  Benign nevi: The presence of large numbers of nevi has been consistently associated with an increased risk of melanoma. Persons with more than 50 nevi, all of which are greater than 2 mm in diameter, have 5 to 17 times the melanoma risk of persons with fewer nevi.  Family history: A family history of melanoma increases a person s risk of melanoma by 3 to 8 times. Persons who have two or more family members with melanoma are at a particularly high risk for developing melanoma. ’  Genetic predisposition: Specific genetic alterations have been implicated in the pathogenesis of melanoma. At least 4 distinct genes located on chromosomes 1p, 6q, 7 and 9 may play a role in melanoma. A tumour suppressor gene located on chromosome 9p21 is probably involved in familial and sporadic cutaneous– melanoma. Deletions or rearrangements– of chromosome10 and 11 are also well documented in cutaneous melanoma. Another tumour suppression gene p16 also plays a role in the development of melanoma. Another genetic alteration that may play role is mutation in the B-RAF gene.  Atypical mole and melanoma syndrome.

Diagnosis: Choice of Biopsy

Clinical management of melanoma begins with an accurate diagnosis. The classic signs of melanoma include a skin lesion with changing characteristics such as irregular borders, varying degrees of pigmentation, an irregular surface, bleeding, itching, and ulceration.

The decision to perform a biopsy is frequently based on clinical experience. The specific method of biopsy depends on the size of the lesion and its anatomic location. Regardless of the method, biopsy specimens are full thickness into subcutaneous tissue. For small lesions, an excisional biopsy is commonly performed that includes a narrow (1-2 mm) margin of surrounding skin.

Shave biopsies are commonly performed for benign-appearing lesions, this technique must not be used when melanoma is suspected. A shave biopsy may lead to a pathology report showing extension of tumor to the deep margins of excision. In these circumstances the most important prognostic factor, tumor thickness, will not be accurate and could lead to incorrect decisions regarding wide local excision (WLE), SLN biopsy, and adjuvant therapy.

The clinical appearance of melanoma may be deceptively benign, which is why the use of cautery or cryoablation may lead to a delay in the diagnosis of melanoma. If a skin lesion reappears at the site of a previously cauterized or frozen skin lesion, excisional biopsy and histopathologic analysis are required.

The technique of performing a surgical biopsy is straightforward. The biopsy removes the full thickness of skin, including a layer of the underlying fatty tissue and the entire visible tumor. Care must be taken to not crush or otherwise traumatize the specimen so that histologic interpretation is not compromised. The biopsy is a full-thickness specimen and includes a margin of adjacent normal skin if possible. When biopsy of a large lesion is performed, at least one punch is placed through the most elevated portion to accurately classify its thickness.

Early signs of malignant melanoma:

 A: Asymmetry.  B: Border irregularity.  C: Colour variegation.  D: Diameter greater than 6mm.  E: Evolving or enlarging lesion.

Prognostic Factors: Clark and associates described a classification of melanoma based on the extent of tumor invasion relative to the anatomic layers of the skin and showed that the level of invasion was related to survival.

 Level 1 tumors are limited to the epidermis, are in situ, and theoretically have no risk for metastasis.  Level 2 lesions extend into the papillary dermis and also have an excellent prognosis.  Clark level 3 tumors fill the papillary dermis and are associated with a significant risk for metastasis  Extension into the reticular dermis defines a Clark 4 lesion, and  Growth into subcutaneous fat characterizes Clark level 5, both of which impart high risk for mortality.

In some cases, determining the Clark level was found to be difficult, and readings of the same slides could differ between pathologists (especially in the level 3 to 4 range).

Breslow described a more straightforward system based on measuring the vertical thickness of the tumor in millimeters. This method was found to be accurately reproducible between pathologists, and there was excellent correlation with 5-year survival. The prognosis worsens with increasing thickness as a continuous logarithmic function without stair-step areas or natural breakpoints. The mortality rate begins to plateau at about 8 mm and never reaches 100%.

American Joint Committee on Cancer TNM Melanoma Classification—2002 Primary Tumor (T) TX Primary tumor cannot be assessed (e.g., shave biopsy or regressed melanoma) T0 No evidence of primary tumor Tis Melanoma in situ T1 T1a Melanoma ≤1.0 mm in thickness, with or without ulceration T1b Melanoma ≤1.0 mm in thickness and level II or III, no ulceration T2 Melanoma 1.01-2.0 mm in thickness, with or without ulceration Melanoma ≤1.0 mm in thickness and level IV or V or with ulceration T2a Melanoma 1.01-2.0 mm in thickness, no ulceration T2b Melanoma 1.01-2.0 mm in thickness, with ulceration T3 Melanoma 2.01-4.0 mm in thickness, with or without ulceration T3a Melanoma 2.01-4.0 mm in thickness, no ulceration T3b Melanoma 2.01-4.0 mm in thickness, with ulceration T4 Melanoma >4.0 mm in thickness, with or without ulceration T4a Melanoma >4.0 mm in thickness, no ulceration T4b Melanoma >4.0 mm in thickness, with ulceration Regional Lymph Nodes (N) NX Regional lymph nodes cannot be assessed N0 No regional lymph node metastasis N1 Metastasis in one lymph node N1a Clinically occult (microscopic) metastasis N1b Clinically apparent (macroscopic) metastasis N2 Metastasis in two or three regional nodes or intralymphatic regional metastasis without nodal metastases N2a Clinically occult (microscopic) metastasis N2b Clinically apparent (macroscopic) metastasis N2c Satellite or in-transit metastasis without nodal metastasis N3 Metastasis in four or more regional nodes, matted metastatic nodes, in-transit metastasis, or satellites with metastasis in regional node(s) Distant Metastasis (M) MX Distant metastasis cannot be assessed M0 No distant metastasis M1 Distant metastasis M1a Metastasis to skin, subcutaneous tissue, or distant lymph nodes M1b Metastasis to lung M1c Metastasis to all other visceral sites or distant metastasis at any site associated with elevated serum lactate dehydrogenase

Treatment: Surgery- Wide local excision is the primary treatment for most melanomas and premalignant lesions. Melanoma in situ (MIS) should be excised to clean margins. For all other malignant melanomas, the width of the surgical margin depends on the Breslow tumor thickness: Thin melanomas (Breslow <1 mm) should have a margin of 1 cm; lesions thicker than 1 mm and all scalp lesions should have a margin of at least 2 cm. Several prospective, randomized trials have investigated margin requirements. Recommended Margins for Surgical Resection of Primary Melanoma

TUMOR THICKNESS (mm) MARGIN RADIUS (cm) In situ 0.5 <1.0 1.0 1-2 1.0-2.0 >2.0

≥2.0 Mohs micrographic surgery has been advocated for areas where wide and deep excisions are difficult.

Elective lymph node dissection (ELND): The term elective refers here to lymph node dissection done in the absence of clinically evident, palpable nodes. ELND provided an element of local control and reasonably accurate staging for patients with occult“ lymph” node metastases.

Sentinel lymph node biopsy (SLNB): SLNB has greatly enhanced accurate staging of melanoma patients. This technique is based on the documented pattern of lymphatic drainage of melanomas to a specific, initial lymph node, termed the sentinel lymph node, before further spread. The histology of the SLN is highly (although not perfectly) reflective of the rest of the nodal basin. If the SLN is negative for metastases, a more radical and morbid lymph node dissection can be avoided.

SLNB appears to be most beneficial for intermediate-thickness melanomas (Breslow 1 to 4 mm) SLNB appears to be a better diagnostic procedure than clinical staging or ELND in terms of sensitivity and morbidity, but some controversy remains over its absolute benefit. o Therapeutic lymph node dissection should be performed for involved axillary and superficial inguinal lymph nodes unless unresectable distant metastases are present. Ideally, therapeutic LND provides optimal locoregional control of disease and a chance of cure, with 5-year survival rates of 20% to 40. Deep inguinal node dissection should be reserved for patients whose survival is thought to justify the potential morbidity of the procedure In some cases, nodal dissection will not benefit patients with advanced disease and should therefore be carefully considered. o Resection of metastases. The surgical options for patients with metastatic melanoma can be divided into two categories: curative or palliative.

 Curative interventions for metastatic melanoma should carefully weigh the risks of the surgery against the potential benefits.  Recent data on the surgical management of metastatic melanoma note that certain factors are associated with an improved overall survival: (1) ability to achieve a complete resection with negative margins, (2) the initial site of metastasis, (3) extent of metastatic disease (single or multiple sites), (4) disease-free interval after surgical removal of the primary melanoma, and (5) stage of initial disease.  Favorable sites for resection include the skin, subcutaneous tissue, lymph nodes, lung, and gastrointestinal tract. Skin and subcutaneous metastases demonstrated the best long-term results after resection. Unfavorable sites include metastases to the brain, adrenal, and liver.

 Isolated limb perfusion:

. (ILP) is used for recurrent limb melanoma that is locally advanced and cannot be resected by simple surgical means. . ILP delivers high-dose regional chemotherapy and establishes a hyperthermic environment to an extremity while its circulation has been isolated from the rest of the body. . Melphalan is commonly used. Adding tumor necrosis factor (TNF)- suggested to increase the complete response rate to 60% to 85%. . Patients who are elderly or who have medical comorbidities or systemicα to melphalan metastases has are been generally not suitable for this therapy.  Immunotherapy:

. Complete and durable regression of stage IV melanoma has been reported using interleukin-2 (IL-2) based immunotherapy alone. . Although patients can be cured of metastatic melanoma solely using high-dose IL-2, the response– rate is low. This has led to the use of IL-2 in conjunction with other treatments, including vaccines, monoclonal antibodies, and adoptive transfer of T lymphocytes. . Recently, a 50% response rate according to Response Evaluation Criteria in Solid Tumors (RECIST) criteria was reported in patients with metastatic melanoma treated with in vitro expanded tumor-infiltrating lymphocytes and IL-2 following a lymphodepleting nonmyeloablative preparative regimen of cyclophosphamide and fludarabine.

2. What are the causes of enterocutaneous fistula? Classify them and give an outline of their management. 5+5+10 Answer. See the Question 2 of Paper – II of 2015.

3. Write short notes of the following: 5x6 (a) Tuberculous peritonitis. (b) Achalasia cardia. (c) Short bowel syndrome. (d) Pheochromocytoma. (e) Gynecomastia. Answer.

(a) Tuberculous peritonitis. Introduction: Peritoneal tuberculosis is an uncommon site of extrapulmonary infection caused by Mycobacterium tuberculosis (TB). The risk is increased in patients with cirrhosis, HIV infection, diabetes mellitus, underlying malignancy, following treatment with anti-tumor necrosis factor (TNF) agents, and in patients undergoing continuous ambulatory .  Infection occurs most commonly following reactivation of latent tuberculous foci in the peritoneum that were established from hematogenous spread from a primary lung focus.  It can also occur via hematogenous spread from active pulmonary or miliary TB. Much less frequently, the organisms enter the peritoneal cavity transmurally from an infected small intestine or contiguously from tuberculous salpingitis. As the disease progresses, the visceral and parietal peritoneum become increasingly studded with tubercles. Ascites develops secondary to "exudation" of proteinaceous fluid from the tubercles, similar to the mechanism leading to ascites in patients with peritoneal carcinomatosis. Clinical features: Clinical findings in peritoneal TB are indistinguishable from those in bacterial peritonitis. Fever, abdominal pain, and cloudy fluid are the most common presenting symptoms. Cell count cannot be used to differentiate peritoneal TB from other forms of peritonitis.

Investigations:  More than 60% of peritoneal TB cases have a predominance of polymorphonuclear neutrophils.  Blood and sputum AFB cultures and PPD skin testing are often negative.  Specialized tests, with increased accuracy but not readily available at most hospitals, include spectrophotometric analysis of adenosine deaminase levels in ascitic fluid and serum and polymerase chain reaction (PCR) assays that amplify mycobacterial 16S ribosomal RNA.  CT scanning remains the most convenient and consistent diagnostic test with accuracy as high as 85%.  Previous studies have also shown ultrasound-guided fine needle aspiration (FNA) of abdominal ascitic fluid for analysis to be a reliable diagnostic modality. Treatment:  With suggestive clinical manifestations and bacteriologic proof of active tuberculosis anywhere in the patient, operation is not mandated; 1. In the presence features of peritonitis, and in the absence of definitive bacteriologic proof, is indicated for diagnostic purposes; 2. Antituberculous chemotherapy is highly effective, and is the treatment of choice.

(b) Achalasia cardia.

The literal meaning of achalasia is failure to relax, which is said of any sphincter that remains in a constant state of tone with periods of relaxation. “ ”  The incidence is 6 per 100,000 persons per year and is seen in young women and middle-aged men and women alike.  Its pathogenesis is presumed to be idiopathic or infectious neurogenic degeneration.  Severe emotional stress, trauma, drastic weight reduction, and Chagas' disease (parasitic infection with Trypanosoma cruzi) have also been implicated.  Regardless of the etiology, both the muscle of the esophagus and the LES are affected. Prevailing theories support the model that the destruction of the nerves to the LES is the primary pathology and that degeneration of the neuromuscular function of the body of the esophagus is secondary. This degeneration results in both hypertension of the LES and failure of the LES to relax on pharyngeal swallowing, as well as pressurization of the esophagus, esophageal dilation, and a resultant loss of progressive peristalsis. Achalasia is also known to be a premalignant condition of the esophagus. Over a 20-year period, a patient will have up to an 8% chance of developing carcinoma.

Symptoms:  The classic triad of presenting symptoms consists of dysphagia, regurgitation, and weight loss.  However, heartburn, postprandial choking, and nocturnal coughing are seen commonly.  The dysphagia that patients experience begins with liquids and progresses to solids.  As the water builds up pressure, retrosternal chest pain is experienced and can be quite severe until the LES opens and is quickly relieving.  Regurgitation of undigested, foul-smelling foods is common, and with progressive disease, aspiration can become life-threatening. Pneumonia, lung abscess, and bronchiectasis often result from long-standing achalasia.  The dysphagia progresses slowly over years, and patients adapt their lifestyle to accommodate the inconveniences that accompany this disease.  Patients often do not seek medical attention until their symptoms are quite advanced and will present with marked distension of the esophagus.

Diagnosis:

 Esophagram:The diagnosis of achalasia is usually made from an esophagram and a motility study. The findings may vary some depending on the advanced nature of the disease. The esophagram will show a dilated esophagus with a distal narrowing referred to as the classic bird's beak appearance of the barium-filled esophagus. Sphincter spasm and delayed emptying through the LES, as well as dilation of the esophageal body, are observed. A lack of “peristaltic waves” in the body and failure of relaxation of the LES are observed. Lack of a gastric air bubble is a common finding on the upright portion of the esophagram and is a result of the tight LES not allowing air to pass easily into the stomach. In the more advanced stage of disease, massive esophageal dilation, tortuosity, and a sigmoidal esophagus (megaesophagus) are seen.  Manometry:Manometry is the gold standard test for diagnosis and will help eliminate other potential esophageal motility disorders. In typical achalasia, the manometry tracings show five classic findings: two abnormalities of the LES and three of the esophageal body. The LES will be hypertensive with pressures usually above 35 mm Hg, but more importantly, it will fail to relax with deglutition. The body of the esophagus will have a pressure above baseline (pressurization of the esophagus) from incomplete air evacuation, simultaneous mirrored contractions with no evidence of progressive peristalsis, and low-amplitude waveforms indicating a lack of muscular tone.These five findings provide a diagnosis of achalasia.  An endoscopy is performed to evaluate the mucosa for evidence of esophagitis or cancer. It otherwise contributes little to the diagnosis of achalasia.

Treatment: There are both surgical and nonsurgical treatment options for patients with achalasia; all are directed toward relieving the obstruction caused by the LES.

Nonsurgical treatment options include medications and endoscopic interventions but usually are only a short-term solution to a lifelong problem.

 In the early stage of the disease, medical treatment with sublingual nitroglycerin, nitrates, or calcium channel blockers may offer hours of relief of chest pressure before or after a meal.  Bougie dilation up to 54 French may offer several months of relief but requires repeated dilations to be sustainable.  Injections of botulinum toxin (Botox) directly into the LES blocks acetylcholine release, preventing smooth muscle contraction, and effectively relaxes the LES. With repeated treatments, Botox may offer symptomatic relief for years, but symptoms recur more than 50% of the time within 6 months.  Dilation with a Gruntzig-type (volume-limited, pressure-control) balloon is effective in 60% of patients and has a risk for perforation less than 4%; however, perforation is life-threatening and must be weighed carefully in otherwise unhealthy patients.

Surgical treatment:

 Surgical esophagomyotomy - offers superior results and is less traumatic than balloon dilation. The current technique is a modification of the Heller . The modified laparoscopic is now the operation of choice. It is done open or with video or robotic assistance.

The decision to perform an antireflux procedure remains controversial. Most patients who have undergone a myotomy will experience some symptoms of reflux. The addition of a partial antireflux procedure, such as a Toupet or Dor fundoplication, will restore a barrier to reflux and decrease postoperative symptoms. This is especially true in patients whose esophageal clearance is greatly impaired.

Esophagectomy is considered in any symptomatic patient with tortuous esophagus (megaesophagus), sigmoid esophagus, failure of more than one myotomy, or an undilatable reflux stricture. Fewer than 60% of patients undergoing repeat myotomy benefit from surgery, and fundoplication for treatment of reflux strictures is even more dismal. In addition to definitively treating the end-stage achalasia, esophageal resection also eliminates the risk for carcinoma. A transhiatal with or without preservation of the vagus nerve offers a good long-term result.

(c) Short bowel syndrome.

Introduction: In the adult, the length of the small bowel varies from 300 to 600 cm and correlates directly with body surface area. Several factors determine the severity of short-bowel syndrome, including the extent of resection, the portion of the GI tract removed, the type of disease necessitating the resection, the presence of coexistent disease in the remaining bowel, and the adaptability of the remaining bowel. Generally, resection resulting in less than 120 cm of intact bowel leads to short-bowel syndrome (<150 cm if ending in an ileostomy, <75 cm if colon remains).

Infants may survive resection of up to 85% of bowel owing to the enhanced ability of the bowel to adapt and grow with the child. Because of its specialized absorptive function, resection of the ileum is usually not well tolerated. However, the entire jejunum can be resected without serious adverse nutritional sequela.

Etiology and pathophysiology:

 Short-bowel syndrome is characterized by dehydration, electrolyte derangements, acidic diarrhea, steatorrhea, malnutrition, and weight loss.  Congenital anomalies leading to short-bowel syndrome include intestinal atresia, midgut volvulus with intestinal necrosis, and necrotizing enterocolitis.  In middle-aged adults, inflammatory bowel disease and trauma are the leading causes of massive intestinal resection.  In the elderly, prominent causes include mesenteric ischemia and strangulated hernia.

Adaptation - The distal small intestine has the greatest adaptive potential and can assume many of the absorptive properties of the proximal GI tract. Cellular hyperplasia and bowel hypertrophy occur over a 2- to 3-year period, increasing the absorptive surface area. Fat absorption is the metabolic process most likely to be permanently impaired; other functions adjust and normalize fairly well.

 Fluid and electrolyte response. Of the 8 to 10 L of fluid presented daily to the small intestine, only 1 to 2 L are delivered into the colon. Significant quantities of electrolytes are absorbed in this process. With short-bowel syndrome, this physiology is altered. Strict intake and output records and close monitoring of serum electrolytes are critical in the early management of patients with short-bowel syndrome.  Malabsorption and malnutrition o Gastric hypersecretion, seen early in the postoperative period, can persist for prolonged periods. Increased acid load may injure distal bowel mucosa, leading to hypermotility and impaired absorption. The severity of hypersecretion correlates directly with the extent of bowel resection. This generally is more pronounced after jejunal than after ileal resection. Loss of an intestinal inhibitory hormone has been implicated. o Cholelithiasis. Altered bilirubin metabolism after ileal resection increases the risk of gallstones secondary to a decreased bile salt pool, which causes a shift in the cholesterol saturation index. Chronic total parenteral nutrition (TPN) also increases risk of cholelithiasis. o Hyperoxaluria and nephrolithiasis. Excessive fatty acids within the colonic lumen bind intraluminal calcium. Unbound oxalate, normally made insoluble by calcium binding and excreted in the feces, thus is absorbed readily, resulting in hyperoxaluria and calcium oxalate urinary stone formation. o Diarrhea and steatorrhea. Rapid intestinal transit, presence of hyperosmolar enteric contents in the distal bowel, disruption of the enterohepatic bile acid circulation, and bacterial overgrowth all promote diarrhea and steatorrhea. Fat absorption is most severely impaired by ileal resection. The delivery of bile acids into the colon produces a reactive watery diarrhea that may be severe. Unabsorbed fats in the colon further inhibit absorption and stimulate secretion of water and electrolytes. o Intestinal microflora. Loss of the ileocecal valve permits reflux of colonic bacteria into the small bowel. Intestinal dysmotility further promotes bacterial colonization. Bacterial overgrowth and changes in the indigenous microbial population result in pH alteration and deconjugation of bile salts, with resultant malabsorption, fluid loss, and decreased vitamin

B12 absorption. Infectious diarrhea (bacterial or viral) is a major cause of morbidity.

Acute cases: Acutely, the primary goal is to stabilize the metabolic, respiratory, and cardiovascular parameters related to the fluid shift and sepsis that frequently accompany massive small-bowel resection.

 Deranged motility patterns and changes in intraluminal milieu may produce a prolonged ileus. Parenteral nutrition should be provided until GI function resumes. If ileus persists for an unduly prolonged period, mechanical obstruction or sepsis may be the cause.

 Gastric hypersecretion requires H2-receptor antagonists or proton-pump inhibitors to reduce the hypersecretion response and protect against peptic ulceration. Antacids neutralize acid on contact and should be administered for nasogastric aspirate pH of less than 5.  Nutritional support should be instituted early to maintain positive nitrogen balance and to promote wound healing and adaptation of the remaining bowel. Enteral nutrition has a positive trophic effect on the bowel mucosa and should be started as soon as possible. Feeding tubes placed at laparotomy can be very helpful. Even if caloric goals are not met, enteral formula stimulates the remaining intestine and facilitates adaptation. Feeds should initially be low volume, low fat, and isosmotic.

Chronic Treatment:

 Diarrhea has many causes in short-bowel syndrome. Frequently, dietary modification

improves symptoms. H2-receptor antagonists reduce acid production and the volume of enteric contents. Chelating resins, such as cholestyramine, reduce intraluminal bile salts and subsequent diarrhea but affect the available systemic bile salt pools. Antisecretory medications, such as loperamide and somatostatin analog, may be beneficial. Low-dose oral narcotics, such as diphenoxylate hydrochloride and atropine (Lomotil) or codeine, are efficacious but addictive. Bacterial overgrowth should be evaluated by stool culture and prophylactic antimicrobials administered as needed.  Nutritional support with supplemental vitamins, trace elements and minerals, and essential fatty acids should be given parenterally until adequate enteral absorption is established. The absorption of fat-soluble vitamins A, D, E, and K is especially likely to be compromised.

Vitamin B12 and calcium absorption are also affected by altered fat absorption and should be supplemented. If required, chronic TPN can be administered nightly to permit normal daytime activities.

Late complications, mostly secondary to metabolic derangements, are common. Problems include nephrolithiasis, cholelithiasis, nutritional deficiency (e.g., anemia, bone disease, coagulopathy), liver dysfunction, TPN-related complications, and central venous catheter related problems, for example, sepsis or thrombosis. Anastomotic leak, fistula, stricture, and obstruction can also occur well beyond the early postoperative period. Late obstruction (partial– or complete) is fairly common, and reoperative rates are high.

Medical Therapy of Short Bowel Syndrome Standard Management Clinical Issue Medical Therapy

Hypergastrinemia/gastric hypersecretion H2 blocker or proton pump inhibitor First line of therapy: Loperamide, dyphenoxylate Diarrhea Second line of therapy: Codeine, DTO Avoid octreotide Cholestyramine (if colon is in continuity and <100 Bile salt-induced diarrhea cm ileum) Replete as needed (may need to give IV if increase Electrolyte losses (e.g., K+, Mg++, Ca++) in diarrhea with oral repletion) Monitor and replete as necessary; may require chronic administration Vitamin deficiency (e.g., A, D, E, K, B12) Liquid vitamins may be better absorbed than pill form Monitor and replete as necessary; may require chronic administration Mineral deficiency (e.g., zinc, selenium, iron) Liquid minerals (in a combined multivitamin/mineral preparation) may be better absorbed than pill form Specialized Treatment Clinical Issue Medical Therapy Bacterial overgrowth Metronidazole, tetracycline Large electrolyte and fluid loss not controlled Octreotide with standard management Inadequate adaptive response and TPN Consider growth hormone and glutamine dependence despite standard management DTO, Deodorized tincture of opium; IV, intravenously; TPN, total parenteral nutrition

Surgical therapy:  Various surgical procedures have been described for the management of short-bowel syndrome, although they have not been widely adopted. Most important is the prevention of complications by minimizing the extent of initial bowel resection.  A number of surgical strategies have been attempted in patients who are chronically TPN dependent with limited success; these include procedures to delay intestinal transit time, methods to increase absorptive area, and small bowel transplantation.  Methods to delay intestinal transit time include the construction of various valves and sphincters, with inconsistent results reported. Antiperistaltic segments of small intestine have been constructed to slow the transit, thus allowing additional contact time for nutrient and fluid absorption. Moderate successes have been described with this technique.  Other procedures, including colonic interposition, recirculating loops of small bowel, and retrograde electrical pacing, have been tried but were found to be unsuccessful in humans and were largely abandoned. Surgical procedures to increase absorptive area include the intestinal tapering and lengthening procedure originally described by Bianchi. This procedure improves intestinal function by correcting the dilation and ineffective peristalsis of the remaining intestine, as well as by doubling the intestinal length while preserving the mucosal surface area. Although beneficial in selected patients, potential complications can include necrosis of divided segments and anastomotic leaks.  Intestinal transplantation has improved with the introduction of the new immunosuppressive agent tacrolimus (FK506). Intestinal transplantation procedures have included primarily isolated small intestinal grafts and combined liver small intestinal grafts with a few more extensive cluster grafts in a large series reported from the International Intestinal Transplant Registry. –  An alternative to intestinal transplantation is mucosal stem cell transplantation, which involves transplanting enterocytes onto a biomatrix and achieving regeneration of intestinal mucosa. This procedure is, at best, preliminary but has shown some promise in experimental studies. (d) Pheochromocytoma. Introduction:  Pheochromocytoma affects approximately 0.2% of hypertensive individuals.  Males and females are affected equally.  The peak incidence in sporadic cases lies between the ages of 40 and 50, whereas familial cases tend to be manifested earlier.  A subset of patients have the classic triad of headache, diaphoresis, and palpitations, although almost all patients will display at least one of these symptoms.  Hypertension is present in 90% of cases and may be episodic or sustained.  The principal challenge in making the diagnosis of pheochromocytoma arises from the fact that essential hypertension is common and the clinical features suggestive of pheochromocytoma are nonspecific.  Previously, pheochromocytoma was dubbed the 10% tumor for the reason that 10% are bilateral, 10% malignant, 10% extra-adrenal, and 10% familial. “ ” Molecular Genetics of Pheochromocytoma: Pheochromocytoma was known to be associated with  multiple endocrine neoplasia type 2 syndromes (40%-50% penetrant),  von Hippel-Lindau syndrome (10%-20% penetrant),– and  neurofibromatosis type 1 (1%-5% penetrant).

The discovery that neuroendocrine cells of the carotid body proliferate in response to hypoxic stimuli led to the identification of mutations in the succinate dehydrogenase gene family in kindreds affected with pheochromocytoma/paraganglioma. o Germline mutations in the B and D subunits, which are inherited in an autosomal dominant fashion, have been identified in approximately 10% of apparently sporadic pheochromocytoma cases. o 21% to 30% of pheochromocytomas are familial. Familial cases occur at an earlier age and are more likely to be multifocal. o Succinate dehydrogenase B mutation carriers have high rates of extra-adrenal (abdominal or thoracic) pheochromocytomas and malignant disease, whereas succinate dehydrogenase D carriers tend to have multiple tumors and hormonally inactive paragangliomas of the head and neck. o Genetic counseling and testing is encouraged for patients in whom pheochromocytoma is diagnosed before the age of 50.

Symptoms and Signs o Headache, palpitations, and diaphoresis constitute the "classic triad" of pheochromocytomas. o Symptoms such as anxiety, tremulousness, paresthesias, flushing, chest pain, shortness of breath, abdominal pain, nausea, vomiting, and others are nonspecific and may be episodic in nature. o Cardiovascular complications, such as myocardial infarction and cerebrovascular accidents, may ensue. o These symptoms can be incited by a range of stimuli including exercise, micturition, and defecation. The most common clinical sign is hypertension. o The hypertension related to this tumor may be paroxysmal with intervening normotension, sustained with paroxysms, or sustained hypertension alone. o Sudden death may occur in patients with undiagnosed tumors who undergo other operations or biopsy.

Algorithm for the diagnosis, localization, and management of pheochromocytoma:

Perioperative Care:  The adverse perioperative hemodynamic changes that are most commonly observed with pheochromocytoma are intraoperative hypertension and postoperative hypotension.  Intraoperative hypertension may be caused by stimulation of catecholamine release by anesthetic induction agents, as well as direct manipulation of the tumor.  Postoperative hypotension may be profound. It results from a state of hypovolemia created by the presence of excess circulating catecholamines.  Sudden withdrawal of this stimulus after tumor removal leads to peripheral arteriolar vasodilation, in addition to a dramatic increase in venous capacitance, which together may precipitate cardiovascular collapse.  -adrenergic blockade is initiated to protect against hemodynamic lability. Start with phenoxybenzamine, 10 mgAs soon twice as daily. the biochemical The dosage diagnosiscan be titrated of pheochromocytoma upward every 2 to has 3 days been to confirmed, a maximum α of 40 mg three times daily to achieve normalization of heart rate and blood pressure. The period of preoperative conditioning lasts at least 2 weeks to allow adequate rev -adrenergic

ersal of α receptor down-regulation. Phenoxybenzamine is a nonspecific, noncompetitive (irreversible), long-acting (half- -adrenergic antagonist. Although its use is associated with the side effects of postural hypotension and significant nasal congestion, it is generally favored over a1-selective agentslife of 24 such hours) as prazosin α and doxazosin. Nasal congestion can actually serve as a useful indicator of adequate blockade. Furthermore, phenoxybenzamine provides the most -blockade among available agents, and its pharmacokinetics permits serum drug levels to decay in parallel with catecholamine levels postoperatively.  complete-Blockers α may be administered after -blockade has been achieved in the subset of patients with persiste -Blockers are never the first agent administered because β adequate- α -adrenergic tone, which may exacerbatent tachycardia. hypertension. β  aClinical decrease suspicion in peripheral of hypovolemia vasodilatory needs β receptor to remain stimulation high in the results postoperative in unopposed period, α and patients need to be aggressively resuscitated if they become hypotensive or oliguric.  - blockade is incomplete. Some patients may require vasopressors after tumor removal, especially if preoperative α Surgical Management and Outcomes:  Successful operative treatment of pheochromocytoma is dependent on close communication between the surgeon and anesthesiologist. Invasive hemodynamic monitoring is required, and fluid management must be meticulous. Manipulation of the tumor needs to be minimized, and - -blockers, as well as vasopressors, when necessary.  Surgerythe anesthetic is curative team in must greater be prepared than 90% to of administer pheochromocytoma supplemental cases. IV Althoughα and β these tumors are highly vascular and tend to adhere to adjacent structures, the great majority of them can be removed successfully via a laparoscopic approach. Laparoscopic resection is contraindicated when preoperative imaging demonstrates local invasion.  Functional image-guided focused exploration has replaced bilateral adrenal and retroperitoneal exploration and has led to diminished rates of solid organ injury.

(e) Gynecomastia. See the Question 3.b of Paper – II of 2014.

4. Answer briefly on the following: 4x71/2 (a) Spigelian hernia. (b) Desmoid tumour. (c) Haemobilia. (d) Alvardo score.

Answer. (a) Spigelian hernia.

Introduction: A Spigelian hernia (or lateral ventral hernia) is a hernia through the spigelian fascia, which is the aponeurotic layer between the rectus abdominis muscle medially, and the semilunar line laterally. These hernias almost always develop at or below the linea arcuata, probably because of the lack of posterior rectus sheath. These are generally interparietal hernias, meaning that they do not lie below the subcutaneous fat but penetrate between the muscles of the abdominal wall; therefore, there is often no notable swelling.

Clinical features:

 Spigelian hernias are usually small and therefore risk of strangulation is high.  Most occur on the right side.  Age: 4th 7th decade of life.  Compared to other types of hernias they are rare.  Patients– typically present with either an intermittent mass, localized pain, or signs of bowel obstruction.

Diagnosis: Ultrasonography or a CT scan can establish the diagnosis, although CT scan provides the greatest sensitivity and specificity.

Treatment:

 These hernias should be repaired because of the high risk of strangulation; fortunately, surgery is straightforward, with only larger defects requiring a mesh prosthesis.  Laparoscopic repair results in less morbidity and shorter hospitalization compared with open procedures.  Mesh-free laparoscopic suture repair is feasible and safe. This novel uncomplicated approach to small Spigelian hernias combines the benefits of laparoscopic localization, reduction, and closure without the morbidity and cost associated with foreign material.

(b) Desmoid tumour. See the Question 3.a Of Paper – II of 2014

(c) Haemobilia.

Introduction: Haemobilia:Implies bleeding into biliary tree. It can present as acute upper gastrointestinal(UGI) bleeding. It should be considered in upper abdominal pain presenting with UGI bleeding especially when there is a history of liver injury or instrumentation. Haemobilia occurs when there is a fistula between a vessel of the splanchnic circulation and the intrahepatic or extrahepatic biliary system.

Causes:  Trauma, accidental or iatrogenic e.g. due to procedures like cholecystectomy  Instrumentation e.g. after ERCP  Gallstone  Inflammatory conditions ranging from ascariasis to PAN  Vascular malformation  Tumor  Coagulopathy 

Clinical feature: Clinical feature "Quinke's" triad of upper abdominal pain, upper gastrointestinal haemorrhage and jaundice is classical but only present in 22% cases.

It can be immediately life threatening in major bleeding. However, in minor haemobilia, patient is haemodynamically stable despite significant blood loss being apparent.

Investigation: Combination of OGD, CT scan and angiography depending on clinical situation, bearing in mind that haemobilia may present many days after injury. Cholangiography is performed if there is a percutaneous access or if ERCP is undertaken.

Management:

 Most bleeding from instrumentation are minor and would settle spontaneously.  When indicated, management is directed towards stopping bleeding and relieving obstruction if present, which is achieved either by surgical ligation of hepatic artery or by endoscopic embolisation. Endoscopic trans-arterial embolisation (TAE) is preferred initially because of high success rate and less complication. TAE involves the selective catheterization of a hepatic artery followed by embolic occlusion.  Surgery is indicated when TAE has failed or sepsis present in biliary tree or drainage has failed.

(d) Alvardo score.

Introduction: The Alvarado score is a clinical scoring system used in the diagnosis of appendicitis. The score has 6 clinical items and 2 laboratory measurements with a total 10 points. The score: Elements from the patient's history, the physical examination and from laboratory tests:

 Abdominal pain that migrates to the right iliac fossa  Anorexia (loss of appetite) or ketones in the urine  Nausea or vomiting  Pain on pressure in the right iliac fossa  Rebound tenderness  Fever of 37.3 °C or more  Leukocytosis, or more than 10000 white blood cells per microliter in the serum  Neutrophilia, or an increase in the percentage of neutrophils in the serum white blood cell count.

The two most important factors, tenderness in the right lower quadrant and leukocytosis, are assigned two points, and the six other factors are assigned one point each, for a possible total score of ten points.

A score of 5 or 6 is compatible with the diagnosis of acute appendicitis. A score of 7 or 8 indicates a probable appendicitis, and a score of 9 or 10 indicates a very probable acute appendicitis.

A popular mnemonic used to remember the Alvarado score factors is MANTRELS - Migration to the right iliac fossa, Anorexia, Nausea/Vomiting, Tenderness in the right iliac fossa, Rebound pain, Elevated temperature (fever), Leukocytosis, and Shift of leukocytes to the left (factors listed in the same order as presented above). Due to the popularity of this mnemonic, the Alvarado score is sometimes referred to as the MANTRELS score.

Complementary value: The original Alvarado score describes a possible total of 10 points, but those medical facilities that are unable to perform a differential white blood cell count, are using a Modified Alvarado Score with a total of 9 points which could be not as accurate as the original score. The high diagnostic value of the score has been confirmed in a number of studies across the world. The consensus is that the Alvarado score is a noninvasive, safe, diagnostic method, which is simple, reliable and repeatable, and able to guide the clinician in the management of the case.

THE WEST BENGAL UNIVERSITY OF HEALTH SCIENCES MS General Surgery) Examination, 2011 PAPER II

Time Allowed: 3 Hours Full Marks: 100 Attempt all questions

1. Describe the management of Iatrogenic Bile duct injury. 20 2. Write on newer trends in Breast Conservation Therapy. 20 3. Write short notes of the following: 5 x 6 a) Pancreatic fistula – management. b) Venous ulcer. c) Seminoma testes.| d) H.A.L.S (Hand Assisted Lap Surgery ). e) Erisepelas.

4. Answer briefly on the following: 4 x 71/2 a) Colostomy and stoma care. b) Management of GERD. c) Solitary thyroid nodule – management. d) H.Pylori.

THE WEST BENGAL UNIVERSITY OF HEALTH SCIENCES

MS General Surgery) Examination, 2011

April 2011

PAPER II

Time Allowed: 3 Hours Full Marks: 100 Attempt all questions

1. Describe the management of Iatrogenic Bile duct injury. 20 Answer. Most benign strictures follow iatrogenic bile duct injury, most commonly during laparoscopic cholecystectomy. Most injuries are recognized intraoperatively or during the early postoperative period, and with appropriate management, the long-term results are acceptable. Long-term sequelae of unrecognized or inappropriately managed biliary strictures may lead to recurrent cholangitis, secondary biliary cirrhosis, and portal hypertension.

Pathogenesis: A number of factors may be involved in the occurrence of bile duct injuries during laparoscopic cholecystectomy. These include:

 Acute or chronic inflammation, obesity, anatomic variations, and bleeding.  Surgical technique with inadequate exposure and failure to identify structures before ligating or dividing them are the most common cause of significant biliary injury.  The bile duct injury rate is increased in patients with complications of gallstones, including acute cholecystitis, pancreatitis, cholangitis, and obstructive jaundice.  Surgeon training and experience were recognized as factors in early reports of laparoscopic bile duct injuries. As surgeon experience increases beyond 20 cases, the bile duct injury rate decreases.  Aberrant biliary anatomy is often cited as a factor in biliary injuries. The bile duct may be narrow and can be mistaken for the cystic duct. The cystic duct may travel parallel to the common bile duct before joining it, misleading the surgeon to the wrong place. Also, the cystic duct may enter the right hepatic duct, and the right hepatic duct may run aberrantly, coursing through the triangle of Calot and entering the common hepatic duct.  A number of other technical factors have been implicated in biliary injuries. The classic injury occurs when excessive cephalad retraction of the gallbladder may align the cystic duct with the common bile duct, allowing the latter to be mistaken for the cystic duct. Careless use of electrocautery may lead to thermal injury. Dissection deep into the liver parenchyma may cause injury to intrahepatic ducts, and poor clip placement close to the hilar area or to structures not well visualized can result in a clip across the bile duct. Presentation: Patients with bile duct injuries can present intraoperatively, in the early postoperative period, or months or years after the initial injury. About 25% of major ductal injuries are recognized intraoperatively because of bile leakage, an abnormal cholangiogram, or late recognition of the anatomy. The most common presentation of a complete occlusion of the common hepatic or bile duct is jaundice with or without abdominal pain. Patients may also present months or years after prior surgery with cholangitis or cirrhosis secondary to a biliary tract injury.

Diagnosis and Management

 The management of bile duct injury is dependent on the timing of diagnosis and extent and level of injury. Inappropriate management of biliary strictures may result in significant morbidity.  Isolated, small, non cautery-based partial lateral bile duct injury recognized at time of cholecystectomy can be managed with placement of a T tube. The T tube can be placed at the site of the injury– if this is similar in size to a choledochotomy.  However, if the biliary injury is more extensive, or if there is significant thermal damage owing to cautery-based trauma, or if the injury involves more than 50% of the circumference of the bile duct wall, an end-to-side choledochojejunostomy with a Roux- en-Y loop of jejunum should be performed.  Similarly, major bile duct injuries, including transections of the common bile or common hepatic duct, can be repaired if recognized at the time of cholecystectomy.  Isolated hepatic ducts smaller than 3 mm or those draining a single hepatic segment can be safely ligated.  Ducts larger than 3 mm are more likely to drain several segments or an entire lobe and need to be reimplanted.  If one is uncertain or underexperienced, and no colleague with sufficient expertise is immediately available, placing a drain followed by referral to an experienced center is the most appropriate course of action.

Management of the Bile Duct Injury Recognized After Cholecystectomy : Most large series report the incidence of ductal injury after laparoscopic cholecystectomy to be 0.3% to 0.85%. In general, patients with a bile leak will present early, whereas patients with postoperative biliary strictures alone often present with jaundice or cholangitis months to years after the initial injury. Diagnosis  Abdominal imaging with ultrasonography or CT should be performed in patients with signs of abdominal pain or peritonitis, sepsis, or any other clinical suspicion of biloma.  Such patients must be stabilized with immediate parenteral antibiotics and image- guided percutaneous drainage of any fluid collections.  Patients with signs and symptoms of cholangitis should undergo urgent cholangiogram with bile duct drainage.  Cholangiography should be performed to establish the presence of ductal stricture, identify the level of the stricture, and identify the nature of the injury when necessary ERCP may be easier to obtain in a patient with a biliary stricture and cholangitis who requires urgent cholangiography and biliary decompression. However, this is only useful in patients with bile duct continuity. Cystic duct leaks or small tangential injuries can be treated with endoscopic stenting. In situations in which the biliary stricture is too tight to pass with ERCP, PTC may be performed for proximal biliary decompression.  CT arteriography should be considered in the preoperative evaluation of patients with benign biliary strictures. Unrecognized injury to the hepatic artery or a portal vein branch occurs with a frequency of 12% to 47% concomitant with a bile duct injury. Certainly, if significant bleeding required urgent control at the time of the original operation, a vascular injury should be considered. The presence of a right hepatic artery disruption should not affect the surgical repair of a bile duct injury. In patients presenting with late strictures with evidence of liver dysfunction, a CT arteriogram should be performed to evaluate for evidence of portal hypertension. Intraoperative Considerations: The management of postoperative biliary strictures following ductal injury depends on the degree of injury, the presence of stricture-induced complications, and the operative risk of the patient. After recognition of a bile duct injury or stricture, a multidisciplinary team consisting of experienced interventional radiologists, endoscopists, and surgeons, coordinated by an experienced hepatobiliary surgeon, should plan the following specific goals: 1. Control the infection (abscess or cholangitis) 2. Drain the biloma 3. Complete the cholangiography 4. Provide definitive therapy with controlled reconstruction or stenting

Suspected injury to the biliary tract during laparoscopic cholecystectomy and treatment.

Suspected common bile duct injury during during laparoscopic cholecystectomy

Intraoperative cholangiogram

Partial injury

(<30%) to CBD Extensive injury Complete transection of Injury to isolated (>30%) CBD hepatic duct

>3mm in size <3mm in size

Primary repair over Roux – en – y T - tube Choledocho jejunostromy

Reimplantation or reconstruct with Ligate Roux – en – y hepatico jejunostromy

These goals do not mandate elaborate workup and delayed repair in all cases. Initial experience suggested that immediate repair of bile duct injury from cholecystectomy can give good results with low morbidity when performed properly.

Independent predictors of stricture recurrence after an initial operative repair include cholangitis before the initial repair, incomplete cholangiography, and primary repair within 3 weeks of the bile duct injury.

When a bile duct injury is recognized at the time of cholecystectomy (or other operation), the surgeon should optimize all aspects of the patient's physiologic situation. If immediate repair is to be attempted, consultation with more experienced surgeons should be made. In those cases in which expertise is not available at the time of a recognized injury, or difficult circumstances preclude elaborate reconstructive attempts, external biliary drains will allow patient recovery and transfer to a center of excellence.

Successful repair of biliary strictures after appropriate preoperative management requires adherence to specific surgical principles:

1. Use of proximal bile duct with minimal inflammation 2. Creation of a tension-free anastomosis with the use of a Roux-en-Y jejunal

limb 3. Direct mucosa-to-mucosa anastomosis

 An adequate incision permitting full visualization is necessary for a good biliary-enteric anastomosis.  The liver should be completely freed from the diaphragm, and adhesions from previous operations should be taken down to facilitate creation of a Roux-en-Y jejunal limb if necessary.  The Hepp-Couinaud approach to bile duct reconstruction is the best option in most circumstances. This technique requires dissection of the hilar plate to expose the left hepatic duct and allow for a side-to-side anastomosis of the left hepatic duct to the Roux-en-Y jejunal limb.  The use of a Roux-en-Y jejunal limb is favored over a direct choledochoduodenostomy or choledochojejunostomy because it also allows for the creation of an access loop of the proximal portion of the Roux-en-Y limb for future interventional radiologic access. “ ” Interventional Radiologic and Endoscopic Techniques

 Interventional radiologic techniques are useful in patients with bile duct injuries, leaks, or postoperative strictures. These techniques allow percutaneous drainage of abdominal fluid collections, preoperative identification of the ductal anatomy through percutaneous transhepatic cholangiography, and stricture dilation with or without placement of palliative stents for bile drainage in the patient whose overall physiologic status precludes a major operation.  Percutaneous transhepatic dilation can be employed in patients with intrahepatic ductal disease and in patients in whom ERCP is not possible. It is often used as an adjunct to operative repair in order to assist with identification of the proximal biliary tree for reconstruction and for the dilation of anastomotic strictures.

 In general, treatment of biliary strictures with this technique, as with interventional radiologic methods, requires multiple sessions of dilations, and nonischemic strictures (anastomotic strictures) respond best. Endoscopic dilation also has a low mortality rate, but it has a significant morbidity rate. The more common complications following endoscopic biliary interventions include hemobilia, bile leak, pancreatitis, and cholangitis Although endoscopic and interventional radiologic procedures are not ideal long-term treatments for unresponsive biliary strictures, endoscopic stenting and drainage is a successful treatment option for cystic duct leak or small common bile duct leaks following laparoscopic cholecystectomy.

Careful long-term follow-up of patients with biliary strictures treated with percutaneous or endoscopic dilation methods is required because ischemic biliary strictures will not respond permanently to dilation. Early retreatment (through repeat dilation or biliary-enteric recon-struction) of postdilation recurrent strictures is essential to prevent secondary biliary cirrhosis. The risk for additive morbidity from the required repeat sessions and the risk for late stricture recurrence should be considered and discussed with patients when treatment options for benign biliary strictures are being considered.

Outcomes

 Acceptable long-term results can be achieved in most patients undergoing operative repair of bile duct injuries. More than 90% of patients are free of jaundice and cholangitis after operative repair of a laparoscopic bile duct injury. The best results are obtained when the injury is recognized during the cholecystectomy and repaired by an experienced biliary surgeon.  Postoperative injuries identified in the presence of concomitant biliary leak should be repaired once the biliary leak has subsided and tissue planes are less inflamed, usually 6 weeks after initial laparoscopic cholecystectomy.  Complications of biliary reconstruction can be managed nonoperatively, and mortality rates have been less than 1%.Common complications include recurrent cholangitis, external biliary fistula, bile leak, and hemobilia.  Restenosis of a biliary-enteric anastomosis occurs in about 10% of patients, and may manifest up to 20 years later. About two thirds of recurrent strictures become symptomatic within 2 years after repair.  The more proximal strictures are associated with a lower success rate than are distal ones. Percutaneous balloon dilation with stenting has a significantly lower success rate (64%) than operative repair.  Although most patients are free of jaundice and cholangitis after operative repair of a bile duct injury, there appears to be a significant impact of the injury on the quality of life.

2. Write on newer trends in Breast Conservation Therapy. 20

Answer. Carcinoma Breast is a local manifestation of a systemic disease. Local radicality does not change survival. Cosmetic considerations and preservation of the nipple, an important sensate focus for younger women. Breast-conserving surgery is a less radical cancer surgery than mastectomy. Breast- conserving surgery, as in a lumpectomy removes part of the breast tissue during surgery, as opposed to the entire breast.

Indications: Stage I & II

Contraindications In the selection of patients for breast conservation treatment with radiation, there are some absolute and relative contraindications.

Absolute contraindications Relative contraindications

 Pregnancy is an absolute contraindication to  History of collagen vascular disease:Most the use of breast irradiation. However, in radiation oncologists will not treat patients with many cases, it may be possible to perform scleroderma or active lupus erythematosus, breast-conserving surgery in the third considering it an absolute contraindication. In trimester and treat the patient with contrast, rheumatoid arthritis is not a relative or irradiation after delivery. an absolute contraindication.  Women with two or more primary tumors in  The presence of multiple gross tumors in the same separate quadrants of the breast or with quadrant and indeterminate calcifications must be diffuse malignant-appearing carefully assessed for suitability. microcalcifications are not considered  Tumor size is not an absolute contraindication to candidates for breast conservation treatment. breast conservation treatment. However, a  A history of prior therapeutic irradiation to relative contraindication is the presence of a large the breast region that would require re- tumor in a small breast in which an adequate treatment to an excessively high total- resection would result in significant cosmetic radiation dose to a significant volume is alteration. In this circumstance, preoperative another absolute contraindication. chemotherapy should be considered.  Persistent positive margins after reasonable  Breast size can be a relative contraindication. surgical attempts: the importance of a single Treatment by irradiation of women with large or focally positive microscopic margin needs pendulous breasts is feasible if reproducibility of further study and may not be an absolute patient set-up can be assured and the technical contraindication. capability exists for greater than or equal to six MV photon beam irradiation to obtain adequate dose homogeneity.

Special considerations

• Family history • Primary tumour histology • Margin evaluation • Extensive intraductal component

METHODS Axillary Dissection NEW INVESTIGATION MODALITIES

• Lumpectomy +Axillary dissection • Better control of • MRI +RT locoregional • Intra-op ultra- • Lumpectomy+SLN biopsy +RT recurrence sound • QUART- Quadrantectomy +Axillary • Accurate staging of • Touch preparation dissection +RT disease cytology • CTART- Chemotherapy +RT • To decide adjuvant • Percutaneous therapy needle biopsy • Prognosis

Role of Neoadjuvant:

• Induction chemotherapy • Drugs • Selection and monitoring of induction chemotherapy patients

Sequencing of chemotherapy and RT:

• 6 Cycles of CMF followed by RT BCS : Procedures • RT followed by 6 cycles of CMF • 3 Cycles of CMF followed by RT followed • Lumpectomy with 2 cm clearance by 3 cycles of CMF (sandwich therapy) • Lumpectomy with Axillary Dissection • Quadrantectomy with Axillary Dissection

Indications for BCS Contraindications for BCS

• Small/Early Tumors in premenopausal  Very small Breast women  Very large Breast • Lateral than medial tumors • More important to know the • Advanced / High Grade Disease contraindications • Lactating Breast • Multicentricity • Disease in opposite Breast

Newer Frontiers

• Laparoscopic Axillary Dissection • Laparoscopic Int Mammary Clearance • Technically feasible • Clearance equal to standard technique • Acceptability only after it stands the test of time.

3. Write short notes of the following: 5 x 6 a) Pancreatic fistula – management. b) Venous ulcer. c) Seminoma testes.| d) H.A.L.S (Hand Assisted Lap Surgery ). e) Erisepelas

Answer. a) Pancreatic fistula – management.

Ans. A pancreatic fistula is an abnormal communication between the pancreas and other organs due to leakage of pancreatic secretions from damaged pancreatic ducts. An external pancreatic fistula is one that communicates with the skin, and is also known as a pancreaticocutaneous fistula, whereas an internal pancreatic fistula communicates with other internal organs or spaces. Pancreatic fistulas can be caused by pancreatic disease, trauma, or surgery.

Commonly used definitions used prior to the ISGPF recommendations:

Output > 10 mL/d of amylase rich fluid on post-operative day 5 or for > 5 d. Output > 10 mL/d of amylase rich fluid on post-operative day 8 or for > 8 d. Output between 25 mL/d and 100 mL/d of amylase rich fluid after post-operative day 8 or for > 8 d. Output > 50 mL/d of amylase rich fluid after post-operative day 11 or for > 11 d. ISGPF Definition: Output via an operatively placed drain (or a subsequently placed percutaneous drain) of any measurable volume of drain fluid on or after postoperative d 3, with an amylase content“ greater than 3 times the upper normal serum value .

Grade A Transient No clinical No peri-pancreatic Clinically well;” no sepsis; no fistula impact collections on CT scan; prolongation of hospital “ little/no change in stay; slow removal of ” management operatively placed drains Grade B Clinical Peri-pancreatic drains Clinically fairly well; degree impact in place or repositioned of infection requiring to drain collections; specific treatment; Change in management prolongation of hospital stay; patients often discharged with drains in situ and observed in outpatient setting Grade C Severe Worrisome peri- Clinically unwell; associated clinical pancreatic collections sepsis requiring aggressive impact that require antibiotics, octreotide and percutaneous drains; other intensive care major change in support; major prolongation management usually in of hospital stay; associated ICU setting; possible re- complications and surgery to salvage a possibility of mortality difficult situation (completion etc)

External Pancreatic Fistulas

An external pancreatic fistula is an abnormal communication between the pancreas (actually pancreatic duct) and the exterior of the body via the abdominal wall.

Loss of bicarbonate-rich pancreatic fluid via a pancreatic fistula can result in a hyperchloraemic or normal anion gap metabolic acidosis. Loss of a small volume of fluid will not cause a problem but an acidosis is common if the volume of pancreatic fluid lost from the body is large.

Internal Pancreatic Fistulas

Internal pancreatic fistulas can result in pancreatic ascites, mediastinital pseudocysts, enzymatic mediastinitis, or pancreatic pleural effusions, depending on the flow of pancreatic secretions from a disrupted pancreatic duct or leakage from a pseudocyst.

Clinical Characteristics

Marked recent weight loss is a major clinical manifestation, and unresponsiveness of the ascites to diuretics is an additional diagnostic clue.

Pathogenesis

Internal pancreatic fistulas are most commonly cause by disruption of the pancreatic duct due to chronic pancreatitis. The chronic pancreatitis is usually alcoholic in origin in adults, and traumatic in origin in children. They may also be caused by leakage from a pancreatic pseudocyst.

Anterior disruption of a pseudocyst or a pancreatic duct leads to leakage of pancreatic secretions into the free peritoneal cavity, leading to pancreatic ascites. If the duct is disrupted posteriorly, the secretions leak through the retroperitoneum into the mediastinum via the aortic or esophageal hiatus. Once in the mediastinum, the secretions can either be contained in a mediastinal pseudocyst, lead to enzymatic mediastinitis, or, more commonly, leak through the pleura to enter the chest and form a chronic pancreatic pleural effusion.

Diagnosis

Pleural or ascitic fluid should be sent for analysis. An elevated amylase level, usually > 1,000 IU/L, with protein levels over 3.0 g/dL is diagnostic. Serum amylase is often elevated as well, due to enzyme diffusion across the peritoneal or pleural surface. Contrast- enhanced computed tomography and endoscopic retrograde cholangiopancreatography (ERCP) may also assist in diagnosis, with the latter an essential component of treatment.

Treatment

The production of pancreatic enzymes is suppressed by restricting the patient's oral intake of food patient in conjunction with the use of long-acting somatostatin analogues. The patient's nutrition is maintained by total parenteral nutrition. This treatment is continued for 2 3 weeks, and the patient is observed for improvement. If no improvement is seen, the patient may be receive endoscopic or surgical treatment. If surgical treatment is followed, an ERCP– is needed to identify the site of the leak.

Fistulectomy is done in which the involved part of the pancreas is also removed.

b) Venous ulcer. Answer. See the Question 3.a of Paper – II of 2015. c) Seminoma testes.

Answer. Introductoion: Seminoma (also known as pure seminoma or classical seminoma) is a germ cell tumor (cancer) of the testis. It is one of the most treatable and curable cancers, with survival >95% in the early stages.Treatment usually requires removal of one testis, but this does not affect fertility or other sexual functioning.

Types of Seminoma: Classic (typical) Anaplastic (Atypical) Spermatocytic

Presentation: The average age of diagnosis is 40 years. This is about 5 to 10 years older than men with other germ cell tumors of the testes. In most cases, they produce masses that are readily felt on testicular self-examination; however, in up to 11 percent of cases, there may be no mass able to be felt, or there may betesticular atrophy. Testicular pain is reported in up to one fifth of cases. Low back pain may occur after metastasis to the retroperitoneum. Diagnosis and treatment Blood tests may detect the presence of placental alkaline phosphatase (PLAP) in fifty percent of cases. Human chorionic gonadotropin (hCG) may be elevated in some cases, but this correlates more to the presence of trophoblast cells within the tumour than to the stage of the tumour. Serum alpha fetoprotein is not elevated in classical seminoma. Serum LDH concentrations are elevated in 30% to 80% of men with pure seminoma and in 60% of those with nonseminomatous tumors. The cut surface of the tumour is fleshy and lobulated, and varies in colour from cream to tan to pink. The tumour tends to bulge from the cut surface, and small areas of hemorrhage may be seen. These areas of hemorrhage usually correspond to trophoblastic cell clusters within the tumour. Microscopic examination shows that seminomas are usually composed of either a sheet- like or lobular pattern of cells with a fibrous stromal network. The fibrous septa almost always contain focal lymphocyte inclusions, and granulomas are sometimes seen. The tumour cells themselves typically have abundant clear to pale pink cytoplasm containing abundant glycogen, which is demonstrable with a periodic acid-Schiff (PAS) stain. The nuclei are prominent and usually contain one or two large nucleoli, and have prominent nuclear membranes. Foci of syncytiotrophoblastic cells may be present in varied amounts. The adjacent testicular tissue commonly shows intratubular germ cell neoplasia, and may also show variable spermatocytic maturation arrest. POU2AF1 and PROM1 have been proposed as possible markers.

A radical inguinal orchiectomy with high ligation of the spermatic cord near the internal inguinal ring is performed to permit histologic evaluation of the primary tumor and to provide local tumor control. Scrotal violation through a scrotal incision or an attempt to biopsy the testicle must be avoided because of concern for changing the lymphatic channels available to the testis tumor and potential for a poorer outcome.

The serum half-lives of HCG and AFP are 18 to 36 hours and 5 to 7 days, respectively. If testicular cancer produces any of these serum markers, following their progressive change after radical orchiectomy is an important consideration in determining the adequacy of therapy. As an example, rapid normalization after orchiectomy for stage I disease suggests elimination of the tumor, whereas persistence of tumor markers after the period during which normalization is expected to occur may be the only evidence of persistent occult disease.

Seminomas metastasise via the lymphatics and haematogenous spread is uncommon. After determination of the histologic subtype of the testis cancer, several parameters may identify patients at high risk for metastasis to the retroperitoneum despite absence of lymphadenopathy on the staging CT scan. Treatment should be aimed at one stage above the clinical stage

 Seminomas - Radio-Sensitive. Treat with Radiotherapy.  Advanced Disease or Metastasis - Responds well to Chemotherapy  Radical INGUINAL ORCHIDECTOMY is Standard first line of therapy  Lymphatic spread initially goes to RETRO-PERITONEAL NODES  Early hematogenous spread RARE  Bulky Retroperitoneal Tumours or Metastatic Tumors Initially DOWN-STAGED with CHEMOTHERAPY “ ” Treatment of Seminomas

Stage I, IIA, ?IIB Radical Inguinal Orichidectomy followed by radiotherapy to Ipsilateral Retroperitonium & Ipsilateral Iliac group Lymph nodes (2500-3500 rads) – Bulky stage II and III Seminomas - Radical Inguinal Orchidectomy is followed by Chemotherapy

THERAPY OF PATIENT WITH SEMINOMA

Stage I, IIA, ? IIB Stage IIB, IIC, III

B - Bleomycin Abdominal Radiotherapy E - Etoposide (VP-16)  4 cycles P - cis-platin

Follow Up Stable/Regress Relapse/Growth

F/U ? RPLND ? Chemotherapy ? XRT

Prognosis for Seminoma: Stage I 99%., Stage II 70% to 92% and Stage III 80% to 85% d) H.A.L.S (Hand Assisted Lap Surgery ).

Answer. Hand-assisted laparoscopic surgery is an alternative laparoscopic approach in which aminilaparotomy is planned and performed to enable the surgeon to introduce his or her hand while the pneumoperitoneum is maintained and the dissection maneuvers are performed under videoendoscopic control. The insertion of the hand restores the tactile feeling and the sensation of depth, and facilitates the exposure, traction, and retraction maneuvers during the procedure.

HALS DEVICES The simplest way to perform HALS is to insert the hand through a minilaparotomy performed for that purpose. However, it is difficult to keep the seal tight and to avoid loss of gas; in addition, the movements of the arm and the hand are limited. Three different devices are available for HALS. One type is a glove fixed to a circular platform that adheres to the surface of the skin, around the incision . This device has the drawback of requiring adherence to the skin, which must be well prepared; adhesive substances are needed, and the device itself is easily lifted from the skin by the wound fluid. There are also devices with 2 elements (Hand Port; Smith & Nephew, London, England). The inflatable circular base adapts to the inner contour of the abdominal wall wound and is attached to a sleeve that is fixed to the surgeon s arm, which allows the insertion and withdrawal of the hand. This device is comfortable and easy to install, but the fact that the sleeve cannot be changed prevents the use of the other’ arm or the hand of the assistant. Third, there are single-piece devices that adapt to the inner contour of the wall incision and permit the interchange of hand insertion, either mechanically (LapDisc; Hakko-Medical, Tokyo, Japan) or by inflation (Omniport; Advanced Surgical Concepts, Dublin. Some special surgical instruments have been designed for HALS. These instruments may help some delicate steps of surgical procedures such as those used in splenic or kidney vessel dissection.

A procedure derived from HALS is finger-assisted LS (fingerscopy). By introducing a finger through a trocar wound the surgeon can free adherences or palpate and identify structures in situations such as appendectomy.

ADVANTAGES The obvious advantage of HALS is that it recovers the tactile feeling and improves hand-eye coordination despite the fact that the operation is performed under videoscopy. The recovery of tactile feeling shortens certain dissection maneuvers, avoids unnecessary movements, favors the smooth traction and exposure of structures, and facilitates the control of unexpected or difficult situations such as hemorrhage or the handling of a voluminous or adherent specimen. All of these advantages enhance the efficiency of the endoscopic procedure.

DISADVANTAGES The main drawback of HALS is that it requires an additional incision, thus increasing trauma. For this reason, the best indications are those that involve the performance of a minilaparotomy to extract the specimen. The HALS technique requires a new operative strategy to capitalize on the presence of the hand being inside the abdomen, and the scope and the trocars must be correctly placed. Furthermore, the hand takes up space inside the abdomen and may hamper certain maneuvers, particularly if the patient is thin or if the surgeon s hand is large. Hand-assisted laparoscopic surgery may also induce hand fatigue in long or complicated procedures. ’ ELECTION OF INCISION SITE FOR PLACEMENT OF THE HALS DEVICE The choice of incision site is likely to depend on whether the intra-abdominal hand is the surgeon s or the assistant s. If it is the surgeon s, the nondominant hand is used; if it is the assistant s, the dominant hand is used. If it is the surgeon s hand that is introduced, it should not’ impair visual field’ of the scope, so as’ to permit adequate triangulation’ over the target organ manipulated by the hand.’ In addition, the hand should not be placed over the structure to be dissected because this may impair the manipulation of the organ. When the procedure includes an accessory incision to extract the specimen (eg, in splenectomy for splenomegaly) or to perform the anastomosis (eg, in ), the hand may be introduced through the anatomical site at which the incision is performed. If it is the assistant who introduces the hand, the accessory incision may be made far away from the introduction points of the operative trocars (a Pfannenstiel incision). In some situations the incision should be made in a multifunctional site (for instance, in the periumbilical midline, for a subtotal colectomy).

INDICATIONS Hand-assisted laparoscopic surgery has been applied in many clinical situations, and its safety and efficacy have been amply demonstrated . Two multicenter series that included multiple diagnosis and complex procedures have underlined its efficacy and the low incidence of conversion; surgeons interviewed in those studies stated that HALS definitely facilitated the procedure (58% of the surgeons considered that HALS reduced operation time and 88% that the intra-abdominal hand was helpful). Furthermore, immediate postoperative evolution was similar to conventional LS procedures. This suggests that HALS maintains the advantages of LS. However, few comparative studies with conventional LS or open surgery have been performed, and more are needed before we are able to confirm the advantages or drawbacks of this new procedure.Hand-assisted laparoscopic surgery should be considered a priori as an aggressive surgery because (1) it requires a minilaparotomy incision at the beginning of the procedure, (2) this incision is stretched by the HALS device, and (3) the area of manipulation and traction is greater than in other procedures that use 5- to 10-mm instruments. In a prospective, randomized trial comparing HALS with laparoscopic colectomy, increases in Creactiveprotein and interleukin 6 used as tissue injury markers after HALS indicated that HALS is more invasive than conventional laparoscopic colectomy. — — Laparoscopic Procedures Performed With Hand-Assisted Laparoscopic Techniques

Transhiatal esophagectomy Preparation gastric tube Paraesophageal hernia Partial or total Gastric bypass Gastroplasty Adhesiolysis Colorectal cancer Colonic polyps Diverticulitis Crohn disease Rectal prolapse Hepatectomy Cryoablation liver metastasis Pancreatoduodenectomy Distal pancreatectomy (70% or 80%) Splenectomy Splenectomy for massive splenomegaly Staging hematological diseases Adrenalectomy Exeresis retroperitoneal tumor Nephrectomy Living-donor nephrectomy Vertebral fusion Aortobifemoral bypass

Hand-assisted laparoscopic surgery may be an interesting alternative to conventional LS or open surgery. It simplifies the performance of difficult procedures for experienced surgeons and can initiate less experienced surgeons in advanced LS. Comparative trials with wellestablished laparoscopic techniques are required to define the absolute advantages of HALS. However, it may well be a useful resource before conversion to open surgery now that it has been demonstrated that it maintains the advantages of LS. e) Erisepelas.

Answer. Erysipelas is an acute streptococcus bacterial infection of the deep epidermis with lymphatic spread.

Risk factors:This disease is most common among the elderly, infants, and children. People with immune deficiency, diabetes, alcoholism, skin ulceration, fungal infections and impaired lymphatic drainage (e.g., after mastectomy, pelvic surgery, bypass grafting) are also at increased risk.

Signs and symptoms

 Patients typically develop symptoms including high fevers, shaking, chills, fatigue, headaches, vomiting, and general illness within 48 hours of the initial infection. The erythematous skin lesion enlarges rapidly and has a sharply demarcated raised edge. It appears as a red, swollen, warm, hardened and painful rash, similar in consistency to an orange peel. More severe infections can result in vesicles, bullae, and petechiae, with possible skin necrosis. Lymph nodes may be swollen, and lymphedema may occur. Occasionally, a red streak extending to the lymph node can be seen.  The infection may occur on any part of the skin including the face, arms, fingers, legs and toes, but it tends to favor the extremities. Fat tissue is most susceptible to infection, and facial areas typically around the eyes, ears, and cheeks. Repeated infection of the extremities can lead to chronic swelling (lymphadenitis).

Etiology

 Most cases of erysipelas are due to Streptococcus pyogenes (also known as beta- hemolytic group A streptococci), although non-group A streptococci can also be the causative agent. Historically, the face was most affected; today the legs are affected most often. The rash is due to an exotoxin, not the Strep. bacteria itself and is found in areas where no bacteria are present - e.g. the infection may be in the nasopharynx, but the rash is found usually on the face and arms.  Erysipelas infections can enter the skin through minor trauma, eczema, surgical incisions and ulcers, and often originate from strep bacteria in the subject's own nasal passages. Infection sets in after a small scratch or abrasion spreads resulting in toxaemia.  Erysipelas does not affect subcutaneous tissue. It does not release pus, only serum or serous fluid. Subcutaneous edema may lead the physician to misdiagnose it as cellulitis, but the style of the rash is much more well circumscribed and sharply marginated than the rash of cellulitis.

Diagnosis

 This disease is diagnosed mainly by the appearance of well-demarcated rash and inflammation. Blood cultures are unreliable for diagnosis of the disease, but may be used to test for sepsis. Erysipelas must be differentiated from herpes zoster, angioedema, contact dermatitis, and diffuse inflammatory carcinoma of the breast.  Erysipelas can be distinguished from cellulitis by its raised advancing edges and sharp borders. Elevation of the antistreptolysin O (ASO) titer occurs after around 10 days of illness.

Treatment

 Depending on the severity, treatment involves either oral or intravenous antibiotics, using penicillins, clindamycin or erythromycin. While illness symptoms resolve in a day or two, the skin may take weeks to return to normal.  Because of the risk of reinfection, prophylactic antibiotics are sometimes used after resolution of the initial condition. However, this approach does not always stop reinfection.

Complications

 Spread of infection to other areas of body through the bloodstream (bacteremia), including septic arthritis and infective endocarditis (heart valves).  Septic shock.  Recurrence of infection Erysipelas can recur in 18 30% of cases even after antibiotic treatment.  Lymphatic damage — –  Necrotizing fasciitis commonly known as "the flesh-eating bug". A potentially deadly exacerbation of the infection if it spreads to deeper tissue. — 4. Answer briefly on the following: 4 x 71/2 a) Colostomy and stoma care. b) Management of GERD. c) Solitary thyroid nodule – management. d) H.Pylori. a) Colostomy and stoma care. Answer. A colostomy is a surgical procedure in which a stoma is formed by drawing the healthy end of the or colon through an incision in the anterior abdominal wall and suturing it into place. This opening, in conjunction with the attached stoma appliance, provides an alternative channel for feces to leave the body. It may be reversible or irreversible depending on the circumstances.

Indications There are many reasons for this procedure. Some common reasons are:

. A section of the colon has been removed, e.g. due to colon cancer requiring a total mesorectal excision, diverticulitis, injury, etc., so that it is no longer possible for feces to exit via the anus. . A portion of the colon (or large intestine) has been operated upon and needs to be 'rested' until it is healed. In this case, the colostomy is often temporary and is usually reversed at a later date, leaving the patient with a small scar in place of the stoma. Children undergoing surgery for extensive pelvic tumors commonly are given a colostomy in preparation for surgery to remove the tumor, followed by reversal of the colostomy. Options Placement of the stoma on the abdomen can occur at any location along the colon, but the most common placement is on the lower left side near the sigmoid where a majority of colon cancers occur. Other locations include the ascending, transverse, and descending sections of the colon. Types of colostomy:

. Loop colostomy: This type of colostomy is usually used in emergencies and is a temporary and large stoma. A loop of the bowel is pulled out onto the abdomen and held in place with an external device. The bowel is then sutured to the abdomen and two openings are created in the one stoma: one for stool and the other for mucus. . End colostomy: A stoma is created from one end of the bowel. The other portion of the bowel is either removed or sewn shut (Hartmann's pouch). . Double barrel colostomy: The bowel is severed and both ends are brought out onto the abdomen. Only the proximal stoma is functioning. Colostomy surgery that is pre-planned usually has a higher rate of long-term success than surgery performed in an emergency situation. Colostomy with irrigation:People with who have ostomies of the sigmoid colon or descending colon may have the option of irrigation, which allows for the person to not wear a pouch, but rather just a gauze cap over the stoma, and to schedule irrigation for times that are convenient. To irrigate, a catheter is placed inside the stoma, and flushed with water, which allows the feces to come out of the body into an irrigation sleeve. Most colostomates irrigate once a day or every other day, though this depends on the person, their food intake, and their health. Colostomy without irrigation : Colostomies are not viewed positively due to the misconception that it is difficult to hide the pouch and the smell of feces, or to keep the pouch securely attached. However, modern colostomy pouches are well-designed, odor- proof, and allow stoma patients to continue normal activities. Latex-free tape is available for ensuring a secure attachment. People with colostomies must wear an ostomy pouching system to collect intestinal waste. Ordinarily the pouch must be emptied or changed several times a day depending on the frequency of activity; in general the further from the anus (i.e., the further 'up' the intestinal tract) the ostomy is located the greater the output and more frequent the need to empty or change the pouch. Alternatives The preferred option by the surgical community, wherever possible, is now an internal colo-anal pouch which eliminates the need for an external pouch. In place of an external appliance, an internal ileo-anal pouch is constructed using a portion of the patient's lower intestine, to act as a new rectum to replace the removed original. Stoma care:

Stoma care is the first thing that a stoma patient learns even before the surgical procedure itself. Why the rush, you might wonder. Well, simply because if you are not able to handle the stoma care and everything it implies, than the operation becomes useless and tiresome. Therefore, in the same conversation where your attending physician explains why a stoma is necessary and how it can save your life, he or she will also give details about the stoma care. Some patients are simply horrified about everything that they hear and cannot even consider it. Others take some time to think it over and eventually agree. However, the myth about how stoma care is gruesome and nearly impossible to live with day by day is just that. A myth. Modern day stoma bags allow the wearer to continue living a normal and fulfilling life.

But before all this, maybe you should ask yourself what a stoma really is? First, the stoma procedure applies to a rather restrictive category of patients. Basically, only patients that suffer from some pretty life - threatening conditions are presented with this option. There are three main types of stoma, indicating the three areas in which a stoma operation could be the solution: ileostomy, colostomy and urostomy. The ileostomy stoma is introduced when the patient is suffering from Chron's disease, ulcerative colitis or any sort of upper digestive tract cancer. The colostomy stoma is used when colon or anus cancer is wreaking havoc on the patients' body, while the urostomy stoma comes in handy when the bladder has a problem.

As a physical procedure, the stoma procedure implies the following steps: the surgeon makes an incision in your abdomen in the appropriate spot. He or she then looks for the damaged parts of your digestive tract or urinary tract and eliminates them. When that is done, the surgical team will create an opening in your abdomen, aptly called a stoma (from the Greek word meaning "mouth"). The remaining part of your intestines or your bladder will be connected to this stoma, allowing for the wastes to leave your body without them passing through the large intestine, the anus or the bladder, accordingly to each type of stoma. The obvious conclusion about the stoma surgery would be that it is a by - pass operation. Perfectly true.

Stoma pictures

 Stoma care begins before the surgery itself. The first step towards stoma care is the psychological counseling: the patient must be made aware that the stoma will change his or her life for the better and it will probably prolong his or her life with decades. Also, the sufferer must understand that their life will continue as planned, that the fact that they have a stoma is not a hindrance.  The second step in the stoma care program is the pre - surgery treatment: a special diet, some physical exercises and vitamins and minerals are all included. The diet is meant to make the digestion process easier than usual, the physical exercises are meant to strengthen your body and the vitamins and minerals should energize your organism.  However, only after the surgery begins the real stoma care. In the first few weeks after the surgery, you will not be allowed to leave the hospital. The physician needs to keep an eye on you and the nurses need to teach you how to take care of your stoma. Several stoma complications can be avoided by remaining hospitalized: infection, fever, bleeding and skin rashes. The surgeon also monitories the healing process of your surgery and he or she is the only one that can give you a green light to resume your life.

After being released from the hospital, you will need some help for another few weeks. Afterwards, you can take manage your stoma care all on your own. There are a few things that you must respect, no matter the situation, when talking about the stoma care:

 Monitor your stoma bag carefully: in the first few months, your digestive system will undergo some changes. It needs them to get used to its new geography. Therefore, you need to monitor your stoma bag carefully, so as to avoid any leakage or bursting.  Keep in mind that your stoma does not work like an anus: the stoma does not have regulator muscles, thus you cannot control the flow of the wastes or the moment that your digestive system or urinary system decides to eliminate them. Unfortunately, in the first months after the surgery, you are not aware enough of your body so that you can detect bowel or bladder movement, making the digestive or excretion process a bit of a surprise for the patient.  Change the stoma bag as often as it is necessary, but not more often than you should: changing the bag more often than necessary will lead to the loosening of your stoma, to leakage and to skin problems. Also, every time you change to stoma bag, a little quantity of blood will flow out. Not to mention that stoma bags are not exactly cheap and they cannot be reutilized, they are "one - time" items.  Watch for any skin damage signs: the skin surrounding the stoma is the perfect barometer. It is highly sensitive to any sort of modifications inside your stoma and to your intestines and it will get red, blistered and itchy. If that happens, you must contact a physician as soon as possible.

b) Management of GERD. Answer. See the answer of question no. 4 (a), Paper II of 2009. c) Solitary thyroid nodule – management.

Answer. The solitary thyroid nodule is defined as a discrete palpable swelling in an otherwise impalpable gland. It is a clinical diagnosis. Many of these cases prove to be multinodular but presenting as a single thyroid nodule.

Diagnostic Tools of Solitary Thyroid Nodule  Clinical examination  Radiological studies:  Histopathological studies:

 Laboratory studies  Neck ultrasonography  Fine needle aspiration  Isotope scanning of the cytology thyroid  Frozen section  CT scan  Final histopathological examination

Treatment of the Solitary Cold Thyroid Nodule: Non-Surgical: Surgical:

 No treatment, just follow-up by  Isthmo-lobectomy FNAC  Near total thyroidectomy  Hormone suppressive therapy  Total thyroidectomy  Aspiration of a cyst  Ethanol injection  Recently, Laser photocoagulation

d) H.Pylori.

Answer. Helicobacter pylori is a Gram-negative, microaerophilic bacterium found in the stomach. It was identified in 1982 by Barry Marshall and Robin Warren, who found that it was present in patients with chronic gastritis and gastric ulcers, conditions that were not previously believed to have a microbial cause. It is also linked to the development of duodenal ulcers and stomach cancer.

Signs and symptoms

Most people (over 80%) infected with H. pylori show no symptoms.

Acute infection may appear as an acute gastritis with abdominal pain (stomach ache) or nausea. Where this develops into chronic gastritis, the symptoms, if present, are often those of non-ulcer dyspepsia: stomach pains, nausea, bloating, belching and sometimes vomiting.

Individuals infected with H. pylori have a 10 to 20% lifetime risk of developing peptic ulcers and a 1 to 2% risk of acquiring stomach cancer. Inflammation of the pyloric antrum is more likely to lead to duodenal ulcers, while inflammation of the corpus (body of the stomach) is more likely to lead to gastric ulcers and gastric carcinoma.

H pylori infection as an important cause of gastric malignancy. H. pylori causes chronic antigenic stimulation to stomach mucosa B lymphocytic proliferation MALT lymphoma. – – Prevention –

H. pylori is a major cause of diseases of the upper gastrointestinal tract. Eradication of the infection improves symptoms including dyspepsia, gastritis and peptic ulcers, and may prevent gastric cancer. Rising antibiotic resistance increases the need for a prevention strategy for the bacteria. Extensive vaccine studies in mouse models have shown promising results. Researchers are studying different adjuvants, antigens, and routes of immunization to ascertain the most appropriate system of immune protection, with most of the research only recently moving from animal to human trials. A number of foods may be useful to prevent colonization with H. pylori including: green tea, red wine, broccoli sprouts, garlic, probiotics and flavonoids.

Treatment

Once H. pylori is detected in patients with a peptic ulcer, the normal procedure is to eradicate it and allow the ulcer to heal. The standard first-line therapy is a one week "triple therapy" consisting of proton pump inhibitors such as omeprazole, lansoprazole and the antibiotics clarithromycin and amoxicillin. Variations of the triple therapy have been developed over the years, such as using a different proton pump inhibitor, as with pantoprazole or rabeprazole, or replacing amoxicillin with metronidazole for people who are allergic to penicillin. Such a therapy has revolutionized the treatment of peptic ulcers, and has made a cure to the disease possible; previously, the only option was symptom control using antacids, H2-antagonists or proton pump inhibitors alone.

An increasing number of infected individuals are found to harbour antibiotic-resistant bacteria. This results in initial treatment failure and requires additional rounds of antibiotic therapy or alternative strategies, such as a quadruple therapy, which adds a bismuth colloid, such as bismuth subsalicylate. For the treatment of clarithromycin- resistant strains of H. pylori, the use of levofloxacin as part of the therapy has been suggested.

Prognosis

H. pylori colonizes the stomach and induces chronic gastritis, a long-lasting inflammation of the stomach. The bacterium persists in the stomach for decades in most people. Most individuals infected by H. pylori will never experience clinical symptoms despite having chronic gastritis. Approximately 10-20% of those colonized by H. pylori will ultimately develop gastric and duodenal ulcers. H. pylori infection is also associated with a 1-2% lifetime risk of stomach cancer and a less than 1% risk of gastric MALT lymphoma.

In the absence of treatment, H. pylori infection once established in its gastric niche is widely believed to persist for life. In the elderly, however, it is likely infection can disappear as the stomach's mucosa becomes increasingly— atrophic and inhospitable to colonization.— The proportion of acute infections that persist is not known, but several studies that followed the natural history in populations have reported apparent spontaneous elimination.

The incidence of acid reflux disease, Barrett's esophagus, and esophageal cancer have been rising dramatically. In 1996, Martin J. Blaser advanced the hypothesis that H. pylori has a beneficial effect: by regulating the acidity of the stomach contents. H. pylori is a member of the normal flora of the stomach. He postulates that the changes in gastric physiology caused by the loss of H. pylori account for the recent increase in incidence of several diseases, including type 2 diabetes, obesity, and asthma.

THE WEST BENGAL UNIVERSITY OF HEALTH SCIENCES

MS (General Surgery) Examination, 2015

April 2015

PAPER III

Time Allowed: 3 Hours Full Marks: 100 Attempt all questions

1. Discuss the pathogenesis of incisional hernia. Discuss the steps of component separation technique of hernia repair. Give a brief comparison of it with pre- peritoneal mesh repair. 8+6+6

Answer. See the answer of question number 2 of Paper –III of 2012.

2. Describe the principles and steps of TME for carcinoma rectum. 20 Answer. See the answer of question number 1of Paper – IV of 2008 and 1 of Paper –I of 2013. Principle:Although TME has been modified over time, the basic principle of excising tumor and the mesorectum en bloc remains its foundation. This principle is based on the original observations of Moynihan in 1908 regarding potential pathways for lymphatic spread and also on the hypothesis of Heald that the mesorectum represents embryological advantages conferring pro- tection against tumor dissemination until the terminal stages. Lymphoscintigraphy further demonstrated this in an anatomical study of the lymphatics that drain the rectum.  There are some operative factors that are likely to affect outcomes. These predominately center on a fastidious approach to the technique, involving specimen-oriented surgery and allowing for additional time as needed.

 In today’s surgical practice, some may balk at the 3 to 5 hours initially advocated to complete a TME, but the low recurrence rates, confirmed by independent surgeons, are hard to dismiss.  A prospective study in Norway investigated the impact of adopting TME into surgical practice and identified several issues relating to the learning curve. The authors found that with sufficient instruction in TME procedures, comparable oncologic and technical efficacy could be reached in a relatively short time. However, the clinically significant anastomotic leak rate (16%) was more than twice that seen in the non-TME group. It is important to consider that augmented leak rates may indeed be independent of TME, and this difference may be accounted for by the lower level of anastomosis in the TME group as well as the author’s assertion that these predominately occurred early in the learning curve.  The principle holds that specimen-oriented surgery must be accompanied by specimen- oriented histopathology, which is essential for appropriate staging and audit. It also emphasizes careful evaluation of circumferential margins since macroscopic evaluation by the operating surgeon is not reliable.  A Quirke-based detailed histopathological approach as an audit method for surgeons. This not only provides objective data that may be extrapolated to define competence, but also may provide an immediate stimulus to surgeons regarding their technique.  Although perhaps controversial, the advent of TME has highlighted the previously disregarded inter-surgeon and inter-institution variability noted in locoregional recurrence following curative rectal cancer surgery, even when institutional selection bias is taken into account. Such inherent variability would not be tolerated in the realm of medical and radiological oncology.

3. Write short notes of the following: 5 x 6 (a) TRAM FLAP. (b) Subdural haematoma. (c) Abdominal compartment syndrome. (d) Staples in surgery. (e) Flail chest.

Answer.

(a) TRAM FLAP.

Autologous Reconstruction o Pedicled FlapToday, the gold standard in breast reconstruction with autogenous tissue is the transverse rectus abdominis myocutaneous (TRAM) flap because of the lower abdominal tissue’s similarities in consistency with breast tissue. o The first description of the TRAM flap used in breast reconstruction, by Robbins in 1979, was with a vertical skin island. The TRAM flap as we know it, with a horizontal lower abdominal skin paddle, was first described by Hartrampf in 1982. This orients the donor scar into a more acceptable abdominoplasty location. o Although this location of the skin paddle provides for a better arc of rotation,the ensuing blood supply to this large volume of tissue is more distal and therefore tenuous. o This donor area of skin and adipose tissue has a dual blood supply for the superior and inferior epigastric systems. Pedicled flaps are thus supplied by the proximal superior epigastric vessels and the inferior system must be divided for transfer. o The small vessels connecting the superior and inferior systems, known as choke vessels, are then dilated to increase perfusion once the deep system is ligated. o Studies by Moon and Taylor have further elucidated the perfusion zones of the lower abdomen skin territory. They found rich perforating blood vessels that arise out of the rectus to supply the overlying skin and fat. Perfusion is best overlying the rectus muscle (zone I) on the side (pedicle) used, followed by the region overlying the contralateral rectus muscle (zone II). Next is the ipsilateral outer region of tissue (zone III), and the region perfused the least is the farthest from the rectus pedicle(zone IV). o This type of reconstruction is advantageous in that it replaces like with like tissue and provides an acceptable donorm scar and improvement of abdominal contour.

Vascular territories of the abdominal wall provided by a unilateral TRA M flap (as determined by Moon and Taylor). Blood flow is best in zone I, followed by zones II , III , and IV, respectively.

Deep inferior Epigastric artery and vein

The limitations include the following: 1. This tissue has high metabolic demands that are sometimes not met, so that portions of the flap go on to form fat necrosis or die. 2. There is a longer recovery period after this surgery, with increased abdominal discomfort and the risk for abdominal weakness and/or hernia formation. 3. Its use can also be limited by previous abdominal operations and scars. 4. who are obese, smokers, or have medical comorbidities (especially diabetes) are at greater risk for these complications . BOX 37-4 Transverse R ectus A bdominis Muscle FlapReconstruction Indications: Breasts of all sizes Abdominal liposuction Abdominal liposuction Breast ptosis Previous abdominal surgery Previous abdominal surgery Relative Contraindications Pulmonary disease Pulmonary disease Smoking Obesity Obesity

Contraindications:  Previous abdominoplasty  Patient unable to tolerate a 4- to 6-week recovery period  Patient unable to tolerate a longer procedure

(b) Subdural haematoma. See the answer of 3(a) of Paper – III of 2007. (c) Abdominal compartment syndrome. See the answer of 2 of Paper –IV of 2011. (d) Staples in surgery. Answer. Introduction: Surgical staples are specialized staples used in surgery in place of sutures to close skin wounds, connect or remove parts of the bowels or lungs. A more recent development, from the 1990s, uses clips instead of staples for some applications; this does not require the staple to penetrate. o The first commercial staplers were made of stainless steel with titanium staples loaded into reloadable staple cartridges. o Modern surgical staplers are either disposable and made of plastic, or reusable and made of stainless steel. Both types are generally loaded using disposable cartridges.

Types and applications:

 The staple line may be straight, curved or circular.  Circular staplers are used for end-to-end anastomosis after bowel resection or, somewhat more controversially, in esophagogastric surgery. The instruments may be used in either open or laparoscopic surgery, different instruments are used for each application.  Laparoscopic staplers are longer, thinner, and may be articulated to allow for access from a restricted number of trocar ports.  Some staplers incorporate a knife, to complete excision and anastomosis in a single operation.  Staplers are used to close both internal and skin wounds. Skin staples are usually applied using a disposable stapler, and removed with a specialized staple remover. Staplers are also used in vertical banded gastroplasty surgery (popularly known as "stomach stapling").

 Vascular stapler for reducing warm ischemia in organ transplantation. With this model each stapler end can be mounted on donor and recipient by independent surgical teams without care for reciprocal orientation, being the maximal possible vascular axis torsion ≤30°. Activating guide-wire is connected just immediately before firing (video)

 While devices for circular end-to-end anastomosis of digestive tract are widely used, in spite of intensive research circular staplers for vascular anastomosis never had yet significant impact on standard hand (Carrel) suture technique.  Apart from the different modality of coupling of vascular (everted) in respect to digestive (inverted) stumps, the main basic reason could be that, particularly for small vessels, the manuality and precision required just for positioning on vascular stumps and actioning any device cannot be significantly inferior to that required to carry out the standard hand suture, then making of little utility the use of any device.  An exception to that however could be organ transplantation where these two phases, i.e.device positioning at the vascular stumps and device actioning, can be carried out in different time, by different surgical team, in safe conditions when the time required does not influence donor organ preservation, i.e. at the back table in cold ischemia condition for the donor organ and after native organ removal in the recipient..

. Although most surgical staples are made of titanium, stainless steel is more often used in some skin staples and clips. . Titanium produces less reaction with the immune system and, being non-ferrous, does not interfere significantly with MRI scanners, although some imaging artifacts may result. . Synthetic absorbable (bioabsorbable) staples are also now becoming available, based on polyglycolic acid, as with many synthetic absorbable sutures. . Titanium staples are not suspected of causing nickel reactions because nickel is rarely if ever used in titanium alloys.

(e) Flail chest. See the answer of 3.a of Paper –III of 2009.

1 4. Answer briefly on the following: 4 x 7 /2 (a) Recurrent varicose vein. (b) Management of urethral injury. (c) Cadaveric Renal Transplantation. (d) Management of stones of submandibular salivary gland and duct.

Answer. (a) Recurrent varicose vein.

Answer.

Introduction: Recurrence of varicose veins after conventional surgery is common. Recurrence rates of 25–50% are found following SSV surgery after a follow-up period of three to five years. Incomplete dissection, filling by tributaries or residual veins, neovascularization to a network of superficial veins and reflux from another origin may all contribute to varicose vein recurrence.

Definition: Recurrent varicose veins occur when you have previously had treatment for varicose veins. Treatment can include injections like sclerotherapy, avulsions, cuts in the groin or cuts behind the knee.

Causes: There are several reasons for veins coming back after treatment:

 Normal veins become varicose. If the person had varicose veins in the past then the patient is more likely to develop more varicose veins  Neovascularisation: When a cut has been made in the skin, the wound heals because living tissue has the ability to regenerate. When this happens, new blood vessels grow to provide a means for blood to get to the new skin. The same process is thought to happen in the area where varicose veins have been treated by surgery. The new veins (neo = new, vascularisation = vessel formation), appear to grow from the site of the vein ligation, most commonly the sapheno-femoral junction. The new veins are like weeds growing in the garden. If they connect up to an original main vein, then blood flows through them, runs into the area of varicose veins in the leg. This will cause recurrence of your varicose veins. If the long saphenous vein in your leg was not removed at the original operation then the new veins can connect with it and cause recurrence. People who have this type of recurrence may be suitable for VNUS treatment to remove their veins.  Inadequate initial treatment: In the past and rarely now, the surgeon who performed your original varicose vein surgery may not have treated all of the areas of varicose veins. There can be several reasons for this.  Pudendal and pelvic varicose veins:Some recurrent varicose veins are caused by connections to veins inside the pelvis. In some people the problem can arise in the veins that run from the testicles or ovaries (gonadal veins). If the blood runs in the wrong direction in these veins (gonadal reflux) then there can be a long column of blood pushing blood out of the pelvis and down the leg.

Treatment: In much the same way as primary varicose veins (those that have not been operated upon). It is important to know exactly where and how the recurrent veins are formed. o Surgery on the type of recurrence one have. o If one need another cut in the groin, he/she may be suitable for VNUS.If thforere is need another cut in the groin, the scarring from the previous can make the surgery more difficult. This means the operation can take longer and there are increased risks. Patient may be more likely to develop a wound infection, swelling in the groin and rarely permanent swelling of the leg. This can happen if the lymphatics have been damaged by the first operation and the redo or recurrent surgery causes further damage. At worst patient could develop lymphoedema.

 New techniques like VNUS, EVLT and Varicofoam can avoid these complication.

o If the patient needs another cut behind the knee, then he/she has recurrent short saphenous disease. He/she may need a much larger cut than before so that the surgeon can carefully identify the blood vessels and nerves behind the knee. With this operation, patient may be at increased risks of wound infection and damage to nerves.

 It might be worth considering varicofoam therapy if patient have recurrent short saphenous vein disease.

o In20% of the cases, inadequate surgery was considered to be the cause of recurrence and as soon as two weeks after surgery up to 23% of patients showed reflux on duplex scanning. o In the last decade, endovenous techniques, including endovenous laser ablation (EVLA) and radiofrequency ablation (RFA), have been successfully introduced. Various randomized trials have shown that EVLA of the GSV is at least comparable with conventional surgery of primary varicose veins.

(b) Management of urethral injury.See the answer of 4.b of Paper-III of 2009. (c) Cadaveric Renal Transplantation. Answer. Introduction: Kidney transplantation or renal transplantation is the organ transplant of a kidney into a patient with end-stage renal disease. Kidney transplantation is typically classified as deceased-donor (formerly known as cadaveric) or living-donor transplantation depending on the source of the donor organ.

Deceased donors can be divided in two groups:

 Brain-dead (BD) donors  Donation after Cardiac Death (DCD) donors

Although brain-dead (or "beating heart") donors are considered dead, the donor's heart continues to pump and maintain the circulation. This makes it possible for surgeons to start operating while the organs are still being perfused (supplied blood). During the operation, the aorta will be cannulated, after which the donor's blood will be replaced by an ice-cold storage solution, such as UW (Viaspan), HTK, or Perfadex. Depending on which organs are transplanted, more than one solution may be used simultaneously. Due to the temperature of the solution, and since large amounts of cold NaCl-solution are poured over the organs for a rapid cooling, the heart will stop pumping.

"Donation after Cardiac Death" donors are patients who do not meet the brain-dead criteria but, due to the unlikely chance of recovery, have elected via a living will or through family to have support withdrawn. In this procedure, treatment is discontinued (mechanical ventilation is shut off). After a time of death has been pronounced, the patient is rushed to the operating room where the organs are recovered. Storage solution is flushed through the organs. Since the blood is no longer being circulated, coagulation must be prevented with large amounts of anti- coagulation agents such as heparin. Several ethical and procedural guidelines must be followed; most importantly, the organ recovery team should not participate in the patient's care in any manner until after death has been declared.

Compatibility:

In general, the donor and recipient should be ABO blood group and crossmatch (HLA antigen) compatible. If a potential living donor is incompatible with their recipient, the donor could be exchange for a compatible kidney. Kidney exchange, also known as "kidney paired donation" or "chains" had recently gained popularity over the past few years.

In an effort to reduce the risk of rejection during incompatible transplantation, ABO- incompatible and densensitization protocols utilizing intravenous immunoglobulin (IVIG) have been developed, with the aim to reduce ABO and HLA antibodies that the recipient may have to the donor.

Procedure: In most cases the barely functioning existing kidneys are not removed, as this has been shown to increase the rates of surgical morbidities. Therefore, the kidney is usually placed in a location different from the original kidney, often in the iliac fossa, so it is often necessary to use a different blood supply:

 The renal artery of the kidney, previously branching from the abdominal aorta in the donor, is often connected to the external iliac artery in the recipient.  The renal vein of the new kidney, previously draining to the inferior vena cava in the donor, is often connected to the external iliac vein in the recipient.

Urological complications:

 Urine Leakage  Obstruction of the Upper Urinary Tract  Other Urological Complications  Urethral strictures  Urinary stone

(d) Management of stones of submandibular salivary gland and duct. Answer. Diagnosis and investigations:

 Radiographs of the submandibular gland and duct are helpful, although many calculi are not radio-opaque. o Lower occlusal X-ray of the teeth will show a stone in the submandibular duct. o A lateral oblique X-ray of the mandible will show a in the submandibular gland.  Submandibular duct radiography (sialography) is technically difficult and rarely gives more information than plain radiographs, although the flushing effect of the radio-opaque dye may give a therapeutic benefit.

Treatment: The traditional approach to obstructive salivary disorders suggests duct dilatation, incision and dissection in the case of distal stones (sometimes followed by marsupialisation, with the risk of post-operative stenosis), and sialadenectomy in the case of proximal, hylar or intraparenchymal sialoliths.

Although has been associated with a high incidence of chronic inflammation suggesting that the obstruction of the duct of the salivary glands led to irreversible parenchymal damage, recent scintigraphic and histopathological studies have shown that recovery of secretory function after stone removal is guaranteed in most cases.

 Stones in the intra-oral part of the ducts can be removed under general anaesthesia. Steady the stone with a Babcock's forceps and incise directly over it. Remove the stone; leave the duct marsupialized.  Stones in the submandibular gland itself are an indication for removal of the submandibular gland.  Sialolithotripsy(ESWL) is a non-invasive method of fragmenting salivary stones into smaller portions in order to favour their possible flushing out from the salivary duct system spontaneously or after salivation induced by citric acid or other sialogogues.

The exclusion criteria for ESWL are stones with a diameter of < 2 mm or which cannot be identified using an ultrasound probe, and the presence of complete distal duct stenosis; the procedure is contra-indicated in patients with acute or acute inflammation in the head and neck region, as well as in patients with cardiac pacemakers.

The main limitation of ESWL is that it does not always completely clear the calculus but leaves stone fragments inside the duct system that may subsequently become the nidus of recurrent sialolithiasis.

Other treatment modalities:

 Intra-corporeal lithotripsy: the shock-waves reach the stone surface through a lithotripsy probe placed inside the salivary duct system under endoscopic guidance.  Endoscopically guided intra-corporeal laser lithotripsy  Endoscopically controlled intra-corporeal electro-hydraulic lithotripsy  Endoscopically controlled intra-corporeal pneumatic lithotripsy  Initially used for diagnostic purposes, is now scheduled interventionally in the case of obstructive salivary gland disease  Fluoroscopically guided stone retrieval is currently the best therapeutic option in interventional radiology, and is indicated for mobile stones located in the middle and proximal portion of the sub-mandibular ductal system  Endoscopically assisted removal of sub-mandibular stones  Botulinum toxin therapy

THE WEST BENGAL UNIVERSITY OF HEALTH SCIENCES MS (General Surgery) Examination, 2014 April 2014 PAPER III Time Allowed: 3 Hours Full Marks: 100

1. Discuss the basic principles and steps of D2 subtotal gastrectomy. 20 Answer. Principle:  Radical resection of stomach and the relate d lymphatic drainage has been the way of treatment of gastric cancer (GC) in west and Japan. The debate has been carried out on what extent the resection must be carried out. Japanese investigators assert that the extended LND (D2) removes tumor in the regional lymph nodes (LNs) before it can metastasize. In addition, it is argued that extended LND improves staging accuracy. On the other hand, western surgeons have argued that the benefits promised by extended lymphadenectomy may be reversed by the high complication rate even if they have any. Theoretically, the removal of a wider range of LNs by extended LN dissection increases the chances for cure. In fact, the pattern of recurrence after extended surgery is completely different from that after limited surgery and involves locoregional recurrence in the majority of cases.  Gastric cancer surgical resection with curative intent is the only treatment providing hope for cure. Gastrectomy with lymph node dissection remains a challenging procedure, which should abide by well-defined oncological principles. Subtotal gastrectomy is the treatment of choice for middle and distal-third gastric cancer; however, due to the necessity of a more extended procedure at advanced stages and the trend for more conservative treatments in early gastric cancer, the indication for conventional subtotal gastrectomy depends on multiple variables.  Curative resection is the only chance for cure in patients with resectable gastric cancer. It aims to ensure complete removal of the tumor by providing adequate longitudinal and circumferential resection margins. Subtotal gastrectomy is the gold standard treatment for early-stage gastric cancer located in the distal third of the stomach. The results of two randomized studies carried out in European countries have shown that subtotal gastrectomy for distal-third gastric cancer entails similar long-term survival results as total gastrectomy, with lower morbidity and mortality rates and better postoperative quality of life. The extent of gastric resection is not a prognostic factor, whereas the adequate LN clearance of the LN stations beyond the perigastric ones is the most important surgical prognostic factor in both early and advanced gastric cancer.  According to the Japanese Gastric Cancer Association guidelines, a proximal margin of at least 3 cm is recommended for T2 or deeper tumors with an expansive growth pattern (Types 1 and 2) and 5 cm is recommended for those with infiltrative growth pattern.  Lymphatic spread is the most relevant prognostic factor in patients with gastric cancer resected for cure. LN status and ratio are the most important prognostic factors.  For absolutely curative resection, lymphatic dissection must be a level higher than the highest echelon of metastatic LNs, in addition to tumor free margin. Appropriate LN dissection is also important for accurate staging. The number of retrieved LNs has been validated as a method of evaluating the adequacy of LN dissection but data collection from each LN station needs considerable effort from both surgeons and pathologists.  The rate and number of metastatic LNs increases with the depth of tumor invasion through the gastric wall layers and shows a clear relationship with survival. Japanese Gastric Cancer Association recommends a D2 LN dissection in most gastric cancer. However, less extensive LN dissection was approved in patients with T1 cancer and clinical node-negative disease. The incidence of LN metastases in lower-third gastric cancer at each LN station according to the depth of tumor invasion was well described in a recent detailed report from the Seoul National University Hospital.  D2 lymphadenectomy was an independent prognostic factor and improved long-term survival in patients with stage-II tumors. The Intergroup randomized trial confirmed that local recurrence of cancer is reduced by 50% after D2 resection.  After standard D2 subtotal gastrectomy for advanced gastric cancer, the blood supply of the gastric stump is maintained up to the level of resection by the esophagocardiotuberal branches and the short gastric arteries.  The type of reconstruction will depend on the surgeon s choice and the remaining stomach length. Steps: ’ Incision: An upper midline incision from the xiphoid process to the umbilicus with an optional extension inferior to the umbilicus provides quick and bloodless access to the abdomen. Another option is a Chevron or a rooftop incision. This, combined with self- retaining subcostal retractors, provides excellent access to the upper abdomen.

 A thorough exploration of the abdominal cavity to look for metastasis in the liver, peritoneum, omentum, and pelvis is performed first.  Further elements of the procedure depend on the choice of reconstruction.  After resection of the stomach, continuity can be achieved with a gastroduodenal anastomosis (). A tension-free gastroduodenal anastomosis requires good duodenal mobilization.  The second option is to close the duodenal end and to perform a gastrojejunal anastomosis (Billroth II or its modifications). Several techniques have been described for the gastrojejunal anastomosis, and a description of each technique is beyond the scope of this topic. Two main variations to the gastrojejunal anastomosis include an end-to-side gastrojejunostomy using an uninterrupted loop of jejunum and an end-to- side gastrojejunostomy to a Roux loop.  Gastroduodenal anastomosis, when possible, is preferred because it maintains the physiologic route of gastric emptying. However, it is technically unfeasible if a more extensive gastric resection is necessary or if duodenal mobilization is difficult owing to inflammation or adhesions, in which cases a gastrojejunal anastomosis is recommended.

Duodenal mobilization: If the intention is to perform a gastroduodenal anastomosis, the first step is duodenal mobilization. The duodenum is mobilized by incising the peritoneum along its lateral border and then reflecting the duodenum to the left side until the inferior vena cava is exposed. This process is also referred to as kocherization. Care should be taken to avoid injury to the structures in the lesser omentum and middle colic vessels while mobilizing the duodenum.

Omental mobilization

 The greater omentum is freed from the transverse colon by dividing along the avascular plane between the transverse colon and the anterior leaf of the omentum. It is easier to identify an avascular window along the left half of the transverse colon. Further division of the omental reflection is extended toward the right side.  This step carries the potential of inadvertent injury to the middle colic artery. Such an injury can be avoided by spreading the gastrocolic omentum along the greater curvature to clearly visualize and demonstrate the vascular pattern of the gastroepiploic vessels and lifting the transverse colon intermittently to clearly visualize the vascular pattern of the transverse colon. As the lesser sac is entered, the posterior wall of the stomach is visible.  The posterior wall of the stomach is freed by dividing gastropancreatic folds of peritoneum. The lesser omentum is separated from the undersurface of the liver for malignancy and along the lesser curvature for ulcer disease.

Duodenal division:

 The right gastric artery is identified at the inferior end of the lesser curvature as originating from the proximal hepatic artery or the gastroduodenal artery. This is doubly ligated and divided. Similarly, the gastroepiploic artery is identified close to the inferior end of the greater curvature, doubly ligated, and divided.  At this point, about 1-2 cm of duodenum adjacent to the pylorus is cleared of all fat and vascular adhesions. Care is taken to avoid injury to the pancreatic tissue while clearing the duodenum. The duodenum is divided by a linear cutter or linear stapler at this level (see the image below).

Gastric division:

 Further mobilization of the stomach is easier after duodenal division.  The greater curvature of the stomach is mobilized further by dividing the gastrosplenic ligament. Depending on the extent of the planned gastrectomy, the greater curvature is mobilized to the point where the gastroepiploic artery is closest to the gastric wall (hemigastrectomy) or farther proximally to the second short gastric artery (subtotal gastrectomy). The first short gastric artery is left behind to supply the remnant stomach.  Similarly, the point of division of the stomach along the lesser curvature is marked at the level of the third prominent vein (hemigastrectomy) or about 1 cm inferior to the esophagogastric junction (subtotal gastrectomy) (see the image below).

Diagrammatic representation of proximal line of resection on stomach for hemigastrectomy and subtotal gastrectomy.

 The left gastric artery is divided as a part of subtotal gastrectomy. This artery divides into two branches close to the lesser curvature. The left gastric artery is secured via double ligation or ligation followed by a transfixing suture on the arterial side and a tie on the gastric side before being divided.  The lesser and greater curvatures should be prepared at the elected site and omental fat cleared at the line of section for a couple of centimeters.  The stomach is divided with a linear cutter at the site identified for proximal resection. At this stage, the specimen is delivered out.

Lymph node dissection:

According to Japanese Gastric Cancer Association, lymph node dissection can be categorized as D1, D2, D3, or D4 on the basis of the lymph node stations, as follows

 D1 lymphadenectomy - Removal of perigastric nodes (3 cm around tumor)  D2 lymphadenectomy - Removal of nodes along the left gastric artery, common hepatic artery, celiac trunk, splenic hilus, and splenic artery  D3 lymphadenectomy - Include dissection of lymph nodes along the hepatoduodenal ligament, posterior surface of the head of the pancreas, and the root of the mesentery (superior mesenteric vessels)  D4 lymphadenectomy - Dissection along the para-aortic and paracolic region

Resection of adjacent organs: Resection of adjacent organs (eg, distal pancreas, spleen, colon) is performed for lesions with direct involvement into these structures. Distal pancreatectomy and splenectomy are not performed as part of a conventional D2 lymphadenectomy, owing to increased postoperative morbidity and mortality.

Reconstruction: Either a Billroth I or a Billroth II reconstruction is performed. o Billroth I: A part of the staple line on the gastric side in the inferior aspect toward the greater curvature is opened up corresponding to the duodenal end diameter. Interrupted delayed absorbable sutures are taken from the middle of the posterior walls of the stomach and the duodenum. After all sutures are placed along the posterior layer, they are tied starting from the lesser curvature side. Once secured, the anterior layer is then similarly sutured. o Billroth II: The duodenal stump remains closed. A loop of jejunum is identified close to the duodenojejunal flexure. Care should be taken to keep the afferent loop reasonably short. The loop is anchored along its axis and isoperistaltic to the posterior wall of the stomach with delayed absorbable seromuscular sutures. An opening is made in the jejunum equal to about twice the diameter of the jejunal lumen. The staple line on the stomach is opened to correspond to this length. The anastomosis is performed with a continuous absorbable suture that starts from the middle of the posterior layer on either side and is continued to meet in the middle of the anterior layer. A fourth layer of seromuscular sutures is placed to bury the anterior continuous suture line.

The principles of gastrojejunal anastomosis remain similar for a Roux-en-Y anastomosis. Instead of a conventional Polya complete anastomosis, a Hofmeister-Finsterer modification can be performed, in which the part of the opening in the stomach on the lesser curvature side is closed and fixed to the jejunum with seromuscular sutures so as to narrow the anastomosis and thus exert a valvular effect. This is more useful in patients who are undergoing stomach resection for benign conditions when the conventional Polya anastomosis may be very wide.

Gastrojejunal anastomosis in progress. Stomach is not resected till last layer is completed so as to prevent retraction of stomach.

The abdominal wound is lavaged thoroughly. A right subhepatic drain is useful in early detection of a possible duodenal stump blowout. The abdominal wound is then closed in layers.

2. Discuss about selection criterias, techniques and complications of orthotopic liver transplant( OLT ). Answer. Indications for liver transplantation:

The selection of donors for adult-to-adult right hepatic lobe living donor liver transplantation (LDLT) is one of the most important aspects of this procedure. Criteria for donor selection are evolving as experience with the procedure increases. However, the two fundamental purposes of the donor evaluation are to ensure that (1) the donor operation may be per- formed safely, and (2) the donor is able to yield a suitable graft for the recipient. Phases of Living Donor Evaluation:

Body size compatibility between the donor and recipient is an important preliminary consideration in the donor evaluation. The donor and recipient should be within approximately 30% body weight of each other. Two formulas are used to assess graft size adequacy: (1) graft-recipient body weight ratio and (2) graft weight as a percentage of standard liver mass.

Special Considerations and/or Issues in Donor Evaluation  Obesity: Obesity (body mass index [BMI] - 28 kg/m2) in potential donors is an increasingly common problem.  Liver Biopsy:The role of liver biopsy in the donor evaluation process varies greatly from center to center.  Cost of Donor Evaluation:Financial considerations of the donor evaluation and hepatectomy are important to consider because the process is complex and expensive.  Donor Advocate: An important recent development in the donor evaluation process is the recommendation for a donor advocate.  Hepatitis B Core Antibody-Positive Donor: Donors who are hepatitis B virus (HBV) surface antigen positive are excluded from donation. However, donors who are HBV core antibody and HBV surface antigen negative have been successfully used as living liver transplant donors.

Patient Selection Criteria for Liver Transplantation

In both categories of liver diseases, factors that are listed below are often the precipitating reason for proceeding with liver transplantation:

 Severe fatigue  Unacceptable quality of life  Recurrent variceal bleeding  Intractable ascites  Recurrent or severe hepatic encephalopathy  Spontaneous bacterial peritonitis  Hepatorenal syndrome  Development of small hepatocellular carcinoma on hepatic imaging

Techniques:

 Before transplantation, liver-support therapy might be indicated (bridging-to- transplantation). Artificial liver support like liver dialysis or bioartificial liver support concepts are currently under preclinical and clinical evaluation.  Virtually all liver transplants are done in an orthotopic fashion, that is, the native liver is removed and the new liver is placed in the same anatomic location.  The transplant operation can be conceptualized as consisting of the hepatectomy (liver removal) phase, the anhepatic (no liver) phase, and the postimplantation phase.  The operation is done through a large incision in the upper abdomen. The hepatectomy involves division of all ligamentous attachments to the liver, as well as the common bile duct, hepatic artery, hepatic vein and portal vein.  Usually, the retrohepatic portion of the inferior vena cava is removed along with the liver, although an alternative technique preserves the recipient's vena cava ("piggyback" technique).  The donor's blood in the liver will be replaced by an ice-cold organ storage solution, such as UW (Viaspan) or HTK until the allograft liver is implanted.  Implantation involves anastomoses (connections) of the inferior vena cava, portal vein, and hepatic artery.  After blood flow is restored to the new liver, the biliary (bile duct) anastomosis is constructed, either to the recipient's own bile duct or to the small intestine.  The surgery usually takes between five and six hours, but may be longer or shorter due to the difficulty of the operation and the experience of the surgeon. . The large majority of liver transplants use the entire liver from a non-living donor for the transplant, particularly for adult recipients. A major advance in pediatric liver transplantation was the development of reduced size liver transplantation, in which a portion of an adult liver is used for an infant or small child. Further developments in this area included split liver transplantation, in which one liver is used for transplants for two recipients, and living donor liver transplantation, in which a portion of a healthy person's liver is removed and used as the allograft. Living donor liver transplantation for pediatric recipients involves removal of approximately 20% of the liver (Couinaud segments 2 and 3). . Further advance in liver transplant involves only resection of the lobe of the liver involved in tumors and the tumor-free lobe remains within the recipient. This speeds up the recovery and the patient stay in the hospital quickly shortens to within 5 7 days. . Many major medical centers are now using radiofrequency ablation of the liver tumor as a bridge while awaiting for liver transplantation. This technique has not been– used universally and further investigation is warranted.

Complications:

 Surgical complications requiring reoperation are common, and they relate to any of the five anastomoses between the donor and the recipient. Hepatic artery thrombosis is especially common in children (5%-10%) and often requires retransplantation. Biliary complications are also fairly common. Anastomotic strictures are more common than leaks, and these usually happen early. Late strictures can be related to rejection.  Post-transplant death is usually related to multiple organ failure. It is usually brought on by a combination of infection or rejection plus nephrotoxicity from cyclosporine or tacrolimus.  Recurrence of hepatitis. Hepatitis B recurrence can be treated with lamivudine. Recurrence of hepatitis C is universal and usually takes an insidious course. The long-term outcome of recurrent hepatitis C is mild or no hepatitis in 40%, moderate hepatitis in 45%, and recurrent cir  Rejection: Rejection is the most common complication following transplant surgery. It happens when the immune system, which defends the body against foreign agents such as viruses or bacteria, treats a transplanted organ as foreign and will try to attack it.  Acute rejection usually manifests as increased liver function tests and occurs in about 40% of recipients. Fever or jaundice can occur with rejection. Diagnosis is made by liver biopsy. Rejection is almost always reversible.  Chronic rejection, also called vanishing bile duct syndrome, represents immunologic attack on the bile ducts and the small arteries that nourish them. This is uncommon but may require retransplantation because it is generally untreatable.

 Infection: As a liver transplant recipient, the recipient will be more susceptible to infection because of intake of immunosuppressant medications to help prevent rejection. The risk of infection from bacteria and viruses is greatest in the early period after transplant, when immunosuppresant dosages are at their highest.  Hypertension: Many transplant recipients take medications to control hypertension or high blood pressure, since prednisone and cyclosporine, two of the medications used to prevent rejection, can raise blood pressure.  Renal dysfunction: Some of the medications that patients are required to take after transplant to prevent rejection can have an adverse effect of the kidney's filtering function. The transplant team will use caution when considering the use of other drugs that can also be toxic to the kidneys in light of the potential adverse effects of the transplant medications.  Hyperlipidemia: Hyperlipidemia can occur early post-transplant due to the effects of cyclosporine and Prograf. Patients may be placed on a cholesterol-lowering medication immediately following transplant for lipid control.  Post transplant Diabetes: Some of the immunosuppressant medications may increase the risk of developing diabetes.  CMV: Cytomegalovirus (CMV) is a very common virus. About 70 percent of adults have been exposed to CMV at some time. It usually causes a flu-like illness with fever, general body aches, and decreased appetite which lasts two or three days. After exposure to the CMV virus, your body forms antibodies in your blood to protect you from future exposures to CMV. This is similar to what happens after you have chicken pox. We are able to do blood tests to check both the transplant recipient and donor for the presence of CMV antibodies.

Because of the immunosuppressant medications, one will be at risk for infection with CMV after transplant. During the first few months, while the immunosuppressant doses are highest and immune system is especially weak, the CMV virus can reactivate or "wake up." A CMV infection can range from flu-like symptoms to more serious infections involving your stomach and lungs (pneumonia).

3. Write short notes of the following: 5 x 6 (a) Meningomyelocele. (b) Principles of cleft lip repair. (c) Thoracic inlet syndrome. (d) Surgical management of hypospadius. (e) Pituitary tumours. Answers a) Meningomyelocele is a type of spina bifida, a kind of birth defect in which the spinal canal and the backbone don't close before birth. This type of birth defect is also called a neural tube defect. The spinal cord and the meninges (the tissue that covers the spinal cord) may actually protrude through the child's back. Introduction: Failure of closure of the neural tube during the third week of gestation leads to the constellation of defects observed in patients with meningomyelocele (MMC). The open neural tube is continuous with the surface of the skin. For this reason, infants with MMC are at risk for bacterial meningitis due to the spinal defect. Leak of cerebrospinal fluid (CSF) leak is commonly observed. The major indication for early operative repair (within 48h of delivery) is prevention of infection.Although protection of the exposed neural tissue from trauma and drying is essential, the neurological deficit caused by MMC is fixed and rarely improves following repair. Deterioration, however, can occur. Diagnosis and pre-operative management: Currently, MMC is usually diagnosed prenatally by ultrasound during the second trimester. Positive screening for maternal serum alpha-fetoprotein may also prompt a fetal ultrasound. Although all MMC patients have a Chiari II malformation (hindbrain herniation) visualized On MR imaging, only a minority will be symptomatic at birth. This may consist of stridor and upper airway obstruction.

-Carefully examine MMC to estimate the anatomic level of lesion and whether sac is intact. A•If small infant amount did not of undergo CSF usually prenatal weeps repair from of the MMC: translucent edges of the neural placode. If the sac ruptures, it usually decompresses and drops to the level of the back. -Using sterile technique, cover ‘lesion with’ sterile Telfa dressing soaked in bacitracin (50,000 units/ 1,000 mL of 0.9% NaCl) and apply transparent dressing. -Once the dressing is in place and if repair is planned within 24-48h, do not change dressing unless it is soiled. If closure is delayed greater than 48h, change the dressing bid and keep it moist with bacitracin solution. -Avoid contamination of site and dressing from stool and urine.

-Examine operative site for evidence of breakdown, leakage of CSF, or inflammation. If•If operative infant did site have is well prenatal healed, repair no special of MMC: wound care is needed. -If there is leakage of CSF, breakdown of site, or evidence of inflammation, cover lesion with sterile Telfa dressing soaked in bacitracin (50,000 units/ 1,000 mL of 0.9% NaCl) and apply transparent dressing. -Consult with Neurosurgery and Pediatric Surgery regarding possible need for further surgery and/or special treatment. Perform a careful neurological examination to determine the levels of the sensory and motor defects. Note the presence of any orthopedic deformities such as clubfeet. hydrocephalus. Obtain a baseline head ultrasound. The decision to place a ventricuoperitoneal•Measure the head circumference and look carefully for clinical findings of shunt is individualized for each patient. In general, symptomatic hydrocephalus, progressive increase in head size, or leakage of CSF from the repaired defect site are indications for shunt placement. Usually a shunt is placed several days after the initial repair, although infrequently this may need to be done at the same time as the repair.

y for signs of meningitis. •Keep infant flat and either prone or on side. e.g., cranial and abdominal ultrasound,•Monitor infant radiograph closel of spine), timing of surgical repair and whether feedings can be•Check started. with Neurosurgery regarding further investigation ( Post-operative management: repair, a dry Telfa dressing should be applied to the incision daily or PRN if soiled. Gently clean the incision with sterile normal saline and apply a layer •Afterof bacitracin surgical ointment. Place Duoder around the incision and use paper tape to prevent skin breakdown. and timing of post-operative cranial ultrasounds. •Discuss orders with Neurosurgery regarding positioning of infant, antibiotics, feeding, evaluation of urinary function and associated orthopedic abnormalities. •Obtain abdominal and hip ultrasound and request Urology and Orthopedic consults for b) Principles of cleft lip repair.

Within the first 2 3 months after birth, surgery is performed to close the cleft lip. While surgery to repair a cleft lip can be performed soon after birth, often the preferred age is at approximately 10– weeks of age, following the "rule of 10s" coined by surgeons Wilhelmmesen and Musgrave in 1969 (the child is at least 10 weeks of age; weighs at least 10 pounds, and has at least 10g hemoglobin). If the cleft is bilateral and extensive, two surgeries may be required to close the cleft, one side first, and the second side a few weeks later. The most common procedure to repair a cleft lip is the Millard procedure pioneered by Ralph Millard. Millard performed the first procedure at a Mobile Army Surgical Hospital (MASH) unit in Korea.

Often an incomplete cleft lip requires the same surgery as complete cleft. This is done for two reasons. Firstly the group of muscles required to purse the lips run through the upper lip. In order to restore the complete group a full incision must be made. Secondly, to create a less obvious scar the surgeon tries to line up the scar with the natural lines in the upper lip (such as the edges of the philtrum) and tuck away stitches as far up the nose as possible. Incomplete cleft gives the surgeon more tissue to work with, creating a more supple and natural-looking upper lip.

The blue lines indicate incisions.

Movement of the flaps; flap A is moved between B and C. C is rotated slightly while B is pushed down.

Pre-operation Post-operation, the lip is swollen from surgery and

will get a more natural look within a couple of

weeks. See photos in the section above.

Important principles for repair of bilateral complete cleft lip are symmetry, primary orbicularis continuity, proper prolabial size and shape, median tubercle and mucocutaneous ridge formation from lateral lip tissue, and early construction of nasal tip and columella with anatomic placement of the alar cartilages. A two-stage repair employing techniques based on these concepts can be employed. At the initial procedure, the lateral crura are positioned and a tiny biconcave prolabium is shaped in anticipation of the changes with growth. The second stage (nasal correction) includes apposition of the alar genua, medial crural relocation, and intranasal transposition of banked forked flaps without disjunction of the columella-labial angle. The complete bilateral cleft lip is a four-dimensional problem. c) Thoracic Inlet Syndrome The thoracic inlet allows unobstructed passage of the neurovascular bundle (nerves, arteries and veins) from the root of the neck to the axilla (from the neck to the arm pit). Causes Signs and symptoms of thoracic inlet syndrome occur if there is some restriction of the passage of the neurovascular bundle between the shoulder and the neck. There are many factors that may be involved, including tight neck muscles or abnormal muscular attachment onto the ribs, cervical ribs (either calcific or fibrous), or postural causes. Presentation The patient will normally present with numbness and pins and needles through one or both arms. Neurological symptoms (pins and needles and pain) predominate in 90% of people and vascular in 10%. They may also complain of incoordination, muscle weakness and pain particularly in the hands and fingers. The symptoms are generally worse with sustained overhead activity and positions. It is quite common in weight lifters and carpenters, or in people who are overweight or with very poor posture. Observation: What is observed depends on whether there is a predominance of vascular or neurological causes. Typically one will observe some form of numbness, muscle weakness and in the later stages, wasting especially of the intrinsic finger muscles and thumb muscles. If the cause is primarily vascular a decrease of the radial pulse may be felt in sustained shoulder abduction, a blush discolouration in the hand when down by the side and venous symptoms may also be present such as oedema (swelling). Treatment Physiotherapy treatment will address faulty neck/back postures and joint mobility with mobilising of joints and posture correction exercises. Tightness of neck musculature, in particular the scalenes, can be decreased through stretching and retraining of shoulder girdle muscles. The stabilising muscles of the neck and thoracic spine (such as longus colli, lower trapezius and serratus anterior) are also retrained and strengthened. Modifying household activities and exercise routines is important for positive long-term outcomes. Initial manual physiotherapy can speed recovery and is best combined with a prescribed home exercise programme. Results from conservative treatment can be very successful with 70-90% resolution of symptoms. d) Surgical management of hypospadius. See the answer of Question 4.c of Paper –III of 2010 and 4.c of Paper –III of 2012. e) Pitutary tumours. Answer. Introduction: Almost all tumours of the pituitary gland are non-cancerous and do not spread. They are sometimes called adenomas. Pituitary tumours are either secreting (producing hormones) or non-secreting tumours (not producing hormones). Secreting tumours can release excess amounts of any of the pituitary hormones, and are named after the hormone that s being overproduced, for example a prolactin-secreting tumour. Causes: As with most brain tumours, the cause of pituitary’ tumours is unknown. Research is being carried out into possible causes. Signs and symptoms: Signs and symptoms of pituitary tumours are caused either by direct pressure from the tumour itself or by a change in the normal hormone levels. As the tumour grows, it puts pressure on the optic nerve (which leads to the eye) and this often causes headaches and sight problems. Symptoms caused by a change in hormone levels usually take a long time to develop. Prolactin secreting tumours: These are the most common type of secreting tumour. Women with this type of tumour may notice that their monthly periods stop and they may also produce small amounts of breast milk. Symptoms in men may include impotence (inability to have an erection). Infertility (inability to have children) is common in both men and women, and the tumour may be discovered during routine tests for infertility. Symptoms of other secreting tumours will relate to the hormones that are released. Growth hormone-secreting tumours: Excess production of growth hormones can cause a condition called acromegaly. This leads to abnormal growth and causes enlargement of the hands, feet, lower jaw and brows. It can also lead to high blood pressure, diabetes and excess sweating. TSH-secreting tumours: A tumour that releases too much TSH may cause symptoms like weight loss, palpitations and feeling shaky and anxious. These tumours are extremely rare. ACTH-secreting tumours: Overproduction of ACTH can produce a number of symptoms including Cushing s syndrome, which is characterised by a round face (known as moon face), weight gain, increased facial hair in women and mental changes such as depression. ’ Other anterior pituitary tumours:Tumours that secrete FSH or LH are very rare and are likely to cause infertility.

Posterior pituitary tumours:Tumours in the posterior pituitary are very rare and disturbances in this area are more likely to be caused by pressure being applied to the area from the surrounding tissues.

The most common symptom of a problem in the posterior pituitary is a condition called diabetes insipidus, which is different from the more common diabetes mellitus. The main symptom of diabetes insipidus is being very thirsty and passing large amounts of very weak urine.

Tests and investigations

Eye tests: By examining the eyes, pressure on the optic nerve can be detected, which may indicate that a tumour is present. A simple test may also be done to check the visual fields (range of vision).

Pituitary tumours are often discovered during a blood test. If high levels of pituitary hormones are found in the blood, a CT scan or MRI scan should be done. The scans will normally be able to confirm whether a pituitary tumour is present or not.

Imaging: CT scan and MRI of brain can detect and locate the tumour.

Treatment for pituitary tumours Treatment depends on the type and size of the tumour, and the person s age and general health. Generally, the treatment options include: ’  Surgery the pituitary gland is accessed via the nasal cavity and sinuses and no visible cuts are made. The tumour is removed using an endoscope or microscope. In rare cases, the tumour– may have to be removed through the front of the skull (craniotomy). Surgery is the first-line treatment for all tumours except .  Radiation therapy targets and destroys the tumour cells. Radiosurgery is focused radiotherapy that has improved the accuracy and minimised the risks of treating pituitary tumours. –Radiosurgery is used in conjunction with surgery and medications to control the growth of tumours or to stop the activity of hormone-releasing tumours.  Medication to shrink the tumour and stop it from producing hormones. Medication is the first line of treatment for . Hormone replacement therapy may also be required in –cases of pituitary hypofunction or post-operatively.

4. Answer briefly on the following: 4 x 71/2 (a) Surgical approach to tracheooesophageal fistula (TOF). (b) Video assisted thoracoscopic surgery (VATS). (c) Deep vein thrombosis. (d) MEN type II. Answer. (a) Surgical approach to tracheooesophageal fistula (TOF). Principle objective: closure of the fistula, maintain airway, and repair esophagus /trachea. Approach: Dependent on fistula size o Small fistulas - direct suture closure of the membranous trachea and esophagus, interposition of strap muscle. o Large fistulas - require tracheal resection and concomitant repair of the esophagus

•Lateral or Anterior cervical approache withright aposterolateral low cellar incision . Tracheal•Upper management: sternal spilt if distal exposure to the trachea is necessary o•Distal Most tracheal, common approach - Tracheal through resection th or membranous tracheal closure o Standard techniques for resection apply. o Long TEFs Alloderm - to recreate membranous trachea Esophageal management: o Small fistulas– may be repaired by 1or 2 layer closure. o Large fistulas might require end to end esophageal anastomosis or fill thickness skin graft.

(b) Video assisted thoracoscopic surgery(VATS). See the answer of Question 3.e of Paper – IV of 2011 (c) Deep vein thrombosis. Causes and features:  May develop in association with abnormalities of the vein wall, blood flow, or constituents of blood (Virchow's triad).  May be due to vein compression or stasis (immobility, trauma, mass, bed rest, surgery, paralysis, long distance travel including airline travel).  May be due to inherited hypercoaguability (factor V Leiden, protein C, protein S, or antithrombin insufficiency).  May be due to acquired hypercoaguability (surgery, malignancy, polycythaemia, smoking, hormone replacement therapy, OCP, dehydration).  Severity may vary from isolated asymptomatic tibial/calf thrombosis to severe iliofemoral segment thrombosis with phlegmasia caerulea dolens (venous gangrene).

Documented risk factors for DVT:

 Increasing Age  Cardiac Dysfunction  Cancer And Its  Obesity  Paralysis Treatment  Prolonged  Indwelling Central  Nephrotic Syndrome Immobility Venous Catheters  Major Surgery  Varicose Veins  Previous Venous  Pregnancy Or  Stroke Thromboembolism Estrogen Use.  Inflammatory Bowel  Trauma Disease

Clinical features:

 Clinical manifestations may be absent.  Local features of venous engorgement and stasis:  limb swelling;  pain;  erythema and warmth to the touch;  mild fever and tachycardia resulting from release of inflammatory mediators;  Homan's sign calf pain on dorsiflexion of the foot is very unreliable and should not be performed.  Complications:  pulmonary embolism;  venous gangrene (phlegmasia dolens).

Diagnosis and investigations:  Aim to confirm presence and extent of thrombosis (to decide on necessity and type of treatment, risk of embolization).  Fibrin, Fibrinogen Assays The basis of fibrin or fibrinogen can be assayed by measuring the degradation of intravascular fibrin. The D-dimer test measures cross- linked degradation products, which is a surrogate of plasmin's activity on fibrin. It is shown that in combination with clinical evaluation and assessment, the sensitivity exceeds 90% to 95%. The negative predictive value is 99.3% for proximal evaluation and 98.6% for distal evaluation. In the postoperative patient, D-dimer is causally elevated due to surgery, and, as such, a positive D-dimer assay for evaluating for DVT is of no use. However, a negative D-dimer test in patients with suspected DVT has a high negative predictive value, ranging from 97% to 99%.  Ascending venography: rarely used now.  Duplex scan: investigation of choice. Visualizes anatomy and gives extent of thrombosis. Relies on flow of blood and compressibility of vein. Is operator- dependent and has lower sensitivity for calf DVT.  VQ scan: If suspicion or evidence of pulmonary embolism.  CT pulmonary angiography (CTPA): safest, most sensitive, and most specific investigation for suspected pulmonary embolism.

Risk stratification for thromboembolism after surgery Abbreviations: DVT: deep vein thrombosis; PE: pulmonary embolism. Level of Age(years) Type Additional risk Incidence Incidenc risk of factors of e of surger proximal PE (%) y DVT (%) Low <40 Minor None 0.4 <0.5 Modera te A Any Minor Present B <40 Major None C 40 60 Nonma None jor High – 4 8 2 4 A >60 Nonma ±Other jor – – B >40 Major None C <40 Major Present Highest 10 20 4 10 risk B >40 Major Prior VTE – –

Cancer

Hypercoagulab le state

Recommendations:  Low Risk Patients No prophylaxis is needed other than early ambulation  Moderate Risk Low dose unfractionated heparin (LDUH) (5000 U) BID or low molecular weight heparin– (LMWH) (< 3,400 U) QD or intermittent pneumatic – compression stocking (IPCS). There is some data suggesting multiple modalities may be synergistic.  High Risk LDUH (5000U) BID or TID or LMWH (< 3,400 U) QD or IPCS  Highest Risk LDUH (5000U) BID or TID or LMWH (< 3,400 U) QD and IPCS – Arguments against of– heparin prophylaxis: 1) Bleeding: 2 3% increase risk compared with placebo 2) Heparin induced thrombocytopenia: Occurs in 1-5% of people and usually resolves with d/c of heparin.– Low molecular weight heparins have even a lower incidence. 3) Cost: Cost based analysis has shown heparin prophylaxis is far more cost effective when considering the decreased incidence of DVT and PE. Treatment:Prophylaxis. o Conservative measures: bed rest, elevation, and good hydration. o Uncomplicated DVT: Low molecular weight heparin (LMWH), initially in hospital; may be given on an outpatient basis via a dedicated DVT clinic. Subsequent treatment is with oral anticoagulation with warfarin for 3-6 months. o Complicated DVT:Initially with IV unfractionated heparin (UFH) whilst converting to oral anticoagulation with warfarin. Thrombolysis or surgical thrombectomy are reserved for severe thrombosis with venous gangrene.  Vena caval filter percutaneously inserted via jugular vein into infrarenal IVC to catch thromboemboli and prevent PE. Used for patients: with recurrent PEs despite treatment; at risk of major central PE; requiring urgent surgery despite high risk that DVT is present. Risks include IVC obstruction, renal vein thrombosis, complications of insertion.

(d) MEN type II. Answer. MEN 2A MEN2B Medullary thyroid carcinoma Medullary thyroid carcinoma Pheochromocytoma Pheochromocytoma Hyperparathyroidism Marfanoid body habitus Lichen planus amyloidosis Mucosal neuromas Ganglioneuromatosis of the Hirschsprung's disease gastrointestinal tract

MEN 2: Clinical Features of Sporadic MTC, MEN 2A, MEN 2B, and FMTC: CLINICAL FEATURES INHERITANCE ASSOCIATED SETTING OF MTC PATTERN ABNORMALITIES GENETIC DEFECT Sporadic Unifocal None None Somatic RET mutations MTC in >20% of tumors MEN 2A Multifocal, Autosomal Pheochromocytomas, Germline missense CLINICAL FEATURES INHERITANCE ASSOCIATED SETTING OF MTC PATTERN ABNORMALITIES GENETIC DEFECT bilateral dominant hyperparathyroidism mutations in extracellular cysteine codons of RET MEN 2B Multifocal, Autosomal Pheochromocytomas, Germline missense bilateral dominant mucosal neuromas, mutation in tyrosine megacolon, skeletal kinase domain of RET abnormalities FMTC Multifocal, Autosomal None Germline missense bilateral dominant mutations in extracellular or intracellular cysteine codons of RET

Treatment: Surgical treatment: MEN-2

 Total thyroidectomy (TT) indicated in patients identified by genetic screening. Symptomatic patients need TT and cervical nodal dissection for the lymph nodes on the involved side.  Laparoscopic adrenalectomy for phaeochromocytoma.  Parathyroidectomy for MTC in patients belonging to families in which hyperparathyroidism is frequently associated.

THE WEST BENGAL UNIVERSITY OF HEALTH SCIENCES

MS (General Surgery) Examination, 2013

April 2013

PAPER III

Time Allowed: 3 Hours Full Marks: 100 Attempt all questions

1. What are the indication, types and basic steps of Radical neck dissection. 20

Answer. Classification of Different Types of Neck Dissection with Clinical Indications

Comprehensive Nodal Levels Structures Indications Removed Preserved Radical neck Levels I V None N+ neck for SCC where dissection SAN involved Modified radical Levels I–V SAN N+ neck for SCC where neck SAN free of disease dissection type – type I

Modified radical Levels I V SAN, SCM N+ neck for SCC where neck IJV involved but SAN free dissection type II – of disease

Modified radical Levels I V SAN, SCM, IJV Metastatic differentiated neck thyroid carcinoma dissection type III –

Selective Supraomohyoid Levels I III SAN, SCM, IJV N0 neck for SCC of oral neck cavity and oropharynx dissection – (include level 4); N0 neck maliganant melanoma where primary site is anterior to ear (include parotidectomy for face and scalp) Extended Levels I IV SAN, SCM, IJV N0 neck for SCC of lateral supraomohyoid tongue – neck dissection

Lateral neck Levels II IV SAN, SCM, IJV N0 neck for SCC of larynx dissection and hypopharynx Posterolateral neck Levels II–V, SAN, SCM, IJV N0 neck malignant dissection subocciptial, melanoma where primary retroauricular– site is posterior to ear nodes

SAN, spinal accessory nerve; SCM, sternocleidomastoid muscle; IJV, internal jugular vein. Surgical procedure: Radical Neck Dissection:  Procedure is done under general anesthesia.  Position the patient in reverse Trendelenberg s position with neck extended at atlanto- axial joint and head elevated 10 degree above the table. Face should be turned to the opposite side of the dissection. ’  Neck skin should be cleaned with Betadine scrub and after that with 3 layers of Betadine solution. Drap the operating site with sterile towels over a polydrape sheet to minimize the infection rate.  Preferred incision for R.N.D. is Lahey s lateral utility incision in post-irradiated patients. Modified Schobinger s incision has been found to be useful in patients undergoing commando operation. ’  Mc fees double horizontal’ incision can be used in some selected post-irradiated cases.  Incision is marked with surgical marker pen, infiltrate with 10-15 ml of 1% xylocaine with 1:4 lacs adrenaline solution.  Wait for 5 minutes , make skin incision with 10 number surgical blade, raise the sub- platysmal flap superiorly till lower border of mandible, mastoid tip posteriorly, midline of neck anteriorly, anterior border of trapezius posteriorly, and till clavicle inferiorly. Then the lower part of sterno-cliedomastiod muscle is cut with electrocautery, 2cms above clavicle after dissecting it carefully from internal jugular vein.  Dissect the IJV from its fascial attachments with common carotid artery and vagus nerve. The lower end of IJV is ligated at level of common tendinous attachment of 2 bellies of omo-hyoid muscle crossing over IJV. Transfix the IJV after ligating with double ligatures. Pull the IJV up gradually with SCM muscle after holding with Babcock forceps. Dissect all lymph nodes, lymphatics,fat and fascia from the supra clavicular fossa including level 5 nodes.  Take care not to damage the brachial plexus, phrenic nerve, transverse cervical vessels. At the junction of upper 1/3 and lower 2/3 of SCM muscle, greater auricular nerve,can be seen exiting from cervical plexus crossing over external jugular vein along posterior border. GAN winds around the posterior border of SCM muscle and crosses obliquely upwards to enter into the tail of parotid gland. Spinal accessory nerve also exits at this point, known as Erb s point and runs in the posterior triangle to enter into trapezius muscle. ’  These nerves have to be dissected from cutaneous branches supplying the fascia and skin. Ligate middle thyroid vein at level of thyroid cartilage and remove all lymph nodes along the middle 1/3 of IJV thus clearing level 3&4.  Now, when reached at the upper end of IJV. Dissect at the level of posterior belly of digastric muscle which is the landmark for ligating the upper end. Bony landmark is the transverse process of atlas. Ligate with double ligatures, transfix with 3-0 silk suture and cut the IJV after ligating the venae commitante for hypoglossal nerve.  This will clear level 2a & 2b lymph nodes. Last step is removal of level 1a & 1b nodes along with submandibular gland. Remove the complete specimen enbloc. Irrigate the dissected field with normal saline and dilute betadine solution.  After securing hemostasis, put Romovac 14-16 FG size drain, fix it with braided silk sutures, and connect to the bellow.  After repositing the skin flap, first layer is sutured with 3-0 vicryl/ catgut suture and skin with staples /3-0 Ethicon monocryl sutures.  Apply pressure dressing and check the drain function before extubating the patient.  Post opetatively, patient is kept in fowler s position and give I.V. antibiotics for 5 days.  Remove drain when collection is < 10 ml.  Remove sutures on 7th post operative day.’ Discharge the patient on 7th day.  Follow up will be after 1 week, check the histopathology report to see how many lymph nodes were dissected and the number of positive nodes.  Refer for radiotherapy if needed. Thereafter at 1 month.  Contrast CT scan /PET-CT scan should be ordered at 6 month follow up for recurrent disease.  One monthly follow up will continue for 1 year ,thereafter 3 monthly for 2 years and then yearly for 10 years.

2. What are the current trends in the management of Urolithiasis. Discuss in detail. 20 Answer.

Risk Factors for the Development of Urinary Calculi

Risk factor Mechanisms Bowel disease Promotes low urine volume; acidic urine depletes available citrate; hyperoxaluria Excess dietary meat (including Creates acidic urinary milieu, depletes available citrate; poultry) promotes hyperuricosuria Excess dietary oxalate Promotes hyperoxaluria Excess dietary sodium Promotes hypercalciuria Family history Genetic predisposition Insulin resistance Ammonia mishandling; alters pH of urine Gout Promotes hyperuricosuria Low urine volume Allows stone constituents to supersaturate Obesity May promote hypercalciuria; other results similar to excess dietary meat Risk factor Mechanisms Primary hyperparathyroidism Creates persistent hypercalciuria Prolonged immobilization Bone turnover creates hypercalciuria Renal tubular acidosis (type 1) Alkaline urine promotes calcium phosphate supersaturation; loss of citrate

 Clinical Clues to the Diagnosis of Urinary Calculi

Evaluation Possible findings Laboratory evaluations Complete blood count Leukocytosis with struvite calculi Serum chemistry Elevation in creatinine levels with obstructing calculi; hypokalemia and hyperchloremia with renal tubular acidosis; elevated serum calcium levels with parathyroid disease Serum parathyroid Elevated in hyperparathyroidism hormone levels Urinalysis Microscopic or gross hematuria; acidic urine; alkaline urine (with struvite calculi); pyuria; crystals from involved calculi 24-hour urinalysis Elevated urinary calcium, oxalate, and sodium levels; decreased urinary volume and citrate levels Radiographic evaluations Abdominal, kidney, and Urinary calculi larger than 2 mm may be visible. upper bladder radiography CT (stone protocol) Nearly all calculi are visible on CT. Evaluates renal parenchyma, hydronephrotic changes, and surrounding organs for other etiologies of abdominal pain. Intravenous pyelography Calculi visible on scout film. Delay in contrast excretion if obstruction is present. Calculi may appear as filling defect. MRI Conventional MRI is not useful for imaging calculi. Ultrasonography Calculi appear as hyperechoic lesions that cast acoustic shadows. Not reliable for ureteral calculi. May demonstrate dilation of collecting system.

Treatment: When a stone causes no symptoms, watchful waiting is a valid option. For symptomatic stones, pain control is usually the first measure, using medications such as nonsteroidal anti-inflammatory drugs or opioids. More severe cases may require surgical intervention. For example, some stones can be shattered into smaller fragments using extracorporeal shock wave lithotripsy. Some cases require more invasive forms of surgery. Examples of these are cystoscopic procedures such as laser lithotripsy or percutaneous techniques such as percutaneous nephrolithotomy. Sometimes, a tube (ureteral stent) may be placed in the ureter to bypass the obstruction and alleviate the symptoms, as well as to prevent ureteral stricture after ureteroscopic stone removal. Conservative management:  Most ureteral calculi pass and do not require intervention. Spontaneous passage depends on stone size, shape, location, and associated ureteral edema (which is likely to depend on the length of time that a stone has not progressed). Ureteral calculi 4 5 mm in size have a 40 50% chance of spontaneous passage. In contrast, calculi >6 mm have a <5% chance of spontaneous passage. This does not mean that a 1-cm stone will not– pass or that a 1- to 2-mm– stone will always pass uneventfully.  The vast majority of stones that pass do so within a 6-week period after the onset of symptoms. Ureteral calculi discovered in the distal ureter at the time of presentation have a 50% chance of spontaneous passage, in contrast to a 25% and 10% chance in the mid- and proximal ureter, respectively. Medical agents:  Administration of tamsulosin and nifedipine is safe and effective in patients with distal ureteral stones with renal colic. However, tamsulosin is significantly better than nifedipine in relieving renal colic and facilitating and accelerating ureteral stone expulsion.  Chemolytic dissolution of stones:  Oral or percutaneous irrigation chemolysis of stones or their fragments can be useful first-line therapy. It may also be an adjunct to ESWL, percutaneous nephrolithotomy (PNL), ureterorenoscopy (URS) or open surgery to support elimination of small residual fragments, considering that its use as first-line therapy may take several weeks to be effective.  Combined treatment with ESWL and chemolysis is a minimally invasive option for patients with partial or complete infection staghorn stones who are not eligible for PNL.  Stone fragmentation leads to increased stone surface area and improved efficacy of chemolitholysis.  Chemolysis is possible only for the stone compositions listed below, therefore, knowledge of stone composition is mandatory before treatment.  Oral chemolysis:  Oral chemolitholysis is efficient only for uric acid calculi, and is based on alkalinisation of urine by application of alkaline citrate or sodium bicarbonate.  ESWL: Indications:

o Kidney stones of less than 2.5 cm in size o Proximal ureter stones o Distal ureter stones

 URS: Ureteroscopy can be employed to treat ureteral stenosis/stricture and ureteropelvic junction obstruction. It is useful for retrieval of distal ureteric and uretero-vesical junction stones. Ho:YAG laser lithotripsy is the preferred method for (flexible) URS.

 PCNL: Indications: o Stones larger than 1.5cm in the kidneys or the ureter. o Stones larger than 1cm in the lower pole of the kidney. o Staghorn shaped stones. o Patient choice.

 Open and laparoscopic surgery for removal of renal stones: o Open surgery: Advances in ESWL and endourological surgery (URS and PNL) have significantly decreased the indications for open stone surgery, which is now often a second- or third-line treatment option needed in only 1.0-5.4% of cases . The incidence of open stone surgery is ~1.5% of all stone removal interventions in developed countries, and in developing countries, it has dropped from 26% to 3.5% in recent years. o However, open surgery is still needed for the most difficult stones, which supports the importance of maintaining proficiency, skills and expertise in open renal and ureteral surgical techniques such as extended pyelolithotomy, pyelonephrolithotomy, anatrophic nephrolithotomy, multiple radial nephrotomy, partial nephrectomy, and renal surgery under hypothermia.

3. Write short notes of the following: 5 x 6 (a) Extradural haematoma. (b) Intussusception. (c) Cystic hygroma. (d) Raynaud’s phenomenon. (e) Ano-rectal anomalies.

Answers.

(a) Extradural haematoma. See the answer of Question 3.a of Paper –IV of 2009 (b) Intussusception. See the answer of Question 4.d of Paper –III of 2011. (c) Cystic hygroma. Answer.

Introduction:

 Cystic hygromas are abnormal growths that usually appear on a baby s neck or head. They consist of one or more cysts and tend to grow larger over time. This disorder most often develops while the baby is still in the womb. However, a cystic hygroma’ can also appear after birth.  Cystic hygromas are fluid-filled sacs caused by blockages in the lymphatic system. They usually form between the ninth and 16th week of pregnancy. Approximately half of all fetuses with a cystic hygroma have chromosomal abnormalities. The baby is unlikely to have a chromosomal abnormality if the cystic hygroma goes away by week 20.  Cystic hygromas can also increase the risk of miscarriage and may even be life- threatening.

Causes

 Cystic hygromas can develop due to genetic disorders or environmental factors. One or more growths may be present at the time of diagnosis.  Common environmental causes of cystic hygromas are:

o Viral infections passed from the mother to the baby during pregnancy o Exposure to drugs or alcohol during pregnancy

 Cystic hygromas are seen more often in infants with genetic diseases. They are particularly common in infants with chromosomal abnormalities. Some genetic conditions associated with hygromas include:

o Turner s syndrome o Trisomy 13, 18, or 21 o Noonan’ syndrome a disorder caused by a mutation (change) in one of seven particular genes — Symptoms

 Cystic hygromas may not be noticeable when the baby is born. They can become visible as they grow larger and the child gets older. Cystic hygromas will usually appear by the time the child is 2 years old.  The main symptom of a cystic hygroma is the presence of a soft, spongy lump. This lump most commonly appears on the neck. However, a cystic hygroma can also form in the armpits and groin area.  Cystic hygromas range in size from smaller than a quarter to as large as a baseball. Larger growths may interfere with movement or cause other difficulties.

Diagnosis:

 Amniocentesis if cystic hygroma is found during an ultrasound. An amniocentesis can check for genetic abnormalities in your fetus.  Other tests will be used to make a diagnosis if cystic hygromas aren t found until after the child is born. These include: ’ o Chest X-ray o Ultrasound o CT scan

Treatments:

 Cystic hygromas aren t treated while the baby is in the womb.  A cystic hygroma is usually treatable if it s found after birth. The first step of treatment is surgery. The’ entire growth must be removed to prevent it from coming back. ’  In some cases, other techniques may be used to shrink large cystic hygromas:

o Sclerotherapy Bleomycin is injected into the cyst o Chemotherapy o Radiation therapy— o Steroid medication

 These methods aren t very effective in treating small cystic hygromas, but they may be useful in shrinking large growths. Once the growth is small enough, it s easier to remove surgically. ’  One should never try to puncture or drain a cystic hygroma yourself. ’It may cause severe bleeding and lead to an infection.

Long-Term Outlook for Babies with Cystic Hygromas:

The main potential complications of cystic hygromas are:

 Recurring growths  Bleeding  Infection in the affected area  Damage to muscles, nerves, or tissues from surgery to remove the hygroma

However, the outlook for babies with cystic hygromas is generally good if the growths appear after birth. This is especially true if the growths can be completely removed. If they can t be removed, cystic hygromas may return or spread to other areas.

(d) ’Raynaud’s phenomenon. See the Question 3.e of Paper –II of 2015.

(e) Ano-rectal anomalies.

Answer. o Anorectal Embryology: By 6 weeks of gestation, the urorectal septum moves caudally to divide the cloaca into the anterior urogenital sinus and posterior anorectal canal. Failure of this septum to form results in a fistula between the bowel and urinary tract (in boys) or the vagina (in girls). o Complete or partial failure of the anal membrane to resorb results in an anal membrane or stenosis. o The perineum also contributes to development of the external anal opening and genitalia by formation of cloacal folds, which extend from the anterior genital tubercle to the anus. The perineal body is formed by fusion of the cloacal folds between the anal and urogenital membranes. o Breakdown of the cloacal membrane anywhere along its course results in the external anal opening being anterior to the external sphincter (i.e., anteriorly displaced anus).

Aetiology:  Unknown INCIDENCE 1 in 5000  1% chance of second child being affected SEX 60% male

Wingspread anatomical classification of Anorectal malformations (1984)

Associated Anomalies:  Sacrum and Spine Sacral deformities appear to be the most frequently associated defect.  Genitourinary Defects The frequency of associated genitourinary defects varies from 20% to 54%.  Anal atresia may occur as a part of the VACTERL group of anomalies V Vertebral body segmentation defect A Anal atresia C Cardiovascular (PDA, VSD) TE Tracheo esophagial fistula R unilateral Renal agenesis L Limb anomaly (radial ray hypoplasia)  CVS: Tetrology of Fallot or VSD  GI: Tracheo-esophageal abnormality , Duodenal atresia , Hirschsprung s .

Algorithm for management of ARM in newborn male

Posterior Sagittal Ano-RectoPlasty PSARP.

PSARP involves: –  Stimulation of muscles to demonstrate the midline and sphincter  Posterior sagittal incision - length depends on severity of abnormality and required extent of dissection  Rectum identified. Abdominal approach may be required in addition in 10% of males and 40% of cloacae Rectum dissected.  Separation from genitourinary tract - often the most difficult part  Repositioning the neoanus within the sphincteric mechanism.

Algorithm for management of ARM in newborn female

4. Answer briefly on the following: 4 x 71/2 (a) Brachytherapy. (b) Flail chest. (c) Median nerve injury. (d) Extrophy of bladder.

Answer.

a) Brachytherapy (from the Greek brachys, meaning "short-distance"), also known as internal radiotherapy, sealed source radiotherapy, curietherapy or endocurietherapy, is a form ofword radiotherapy βραχύς where a sealed radiation source is placed inside or next to the area requiring treatment.

Brachytherapy can be used alone or in combination with other therapies such as surgery, external beam radiotherapy (EBRT) and chemotherapy.

Brachytherapy represents an effective treatment option for many types of cancer. Treatment results have demonstrated that the cancer cure rates of brachytherapy are either comparable to surgery and EBRT or are improved when used in combination with these techniques. In addition, brachytherapy is associated with a low risk of serious adverse side effects.

Types: Different types of brachytherapy can be defined according to (1) the placement of the radiation sources in the target treatment area, (2) the rate or intensity of the irradiation dose delivered to the tumour, and (3) the duration of dose delivery. ‘ ’ Source placement

The two main types of brachytherapy treatment in terms of the placement of the radioactive source are interstitial and contact.

 In the case of interstitial brachytherapy, the sources are placed directly in the target tissue of the affected site, such as the prostate or breast.

 Contact brachytherapy involves placement of the radiation source in a space next to the target tissue. This space may be a body cavity (intracavitary brachytherapy) such as the cervix, uterus or vagina; a body lumen (intraluminal brachytherapy) such as the trachea or oesophagus; or externally (surface brachytherapy) such as the skin. A radiation source can also be placed in blood vessels (intravascular brachytherapy) for the treatment of coronary in-stent restenosis.

Dose rate:The dose rate of brachytherapy refers to the level or intensity with which the radiation is delivered to the surrounding medium and is expressed in Grays per hour (Gy/h). ‘ ’

 Low-dose rate (LDR) brachytherapy involves implanting radiation sources that emit radiation at a rate of up to 2 Gy·h . LDR brachytherapy is commonly used for cancers of the oral cavity, oropharynx, sarcomas−1 and prostate cancer.  Medium-dose rate (MDR) brachytherapy is characterized by a medium rate of dose delivery, ranging between 2 Gy·h to 12 Gy·h .  High-dose rate (HDR) brachytherapy−1 is when−1 the rate of dose delivery exceeds 12 Gy·h . The most common applications of HDR brachytherapy are in tumours of the cervix, esophagus−1 , lungs, breasts and prostate. Most HDR treatments are performed on an outpatient basis, but this is dependent on the treatment site.  Pulsed-dose rate (PDR) brachytherapy involves short pulses of radiation, typically once an hour, to simulate the overall rate and effectiveness of LDR treatment. Typical tumour sites treated by PDR brachytherapy are gynaecological and head and neck cancers.

Duration of dose delivery: Permanent brachytherapy is often performed for prostate cancer using "seeds" - small radioactive rods implanted directly into the tumour.

Clinical applications

Uses: o Cervical cancer

o Prostate cancer o Breast cancer

There are two methods that can be used to deliver breast brachytherapy:

 Interstitial breast brachytherapy using multiple catheters  Intracavitary breast brachytherapy using a balloon catheter

o Skin cancer o Coronary/vascular: Brachytherapy can be used in the treatment of coronary in-stent restenosis, in which a catheter is placed inside blood vessels, through which sources are inserted and removed. In treating In-stent restenosis (ISR) Drug Eluting stents (DES) have been found to be superior to Intracoronary Brachytherapy (ICBT). However, there is continued interest in vascular brachytherapy for persistent restenosis in failed stents and vein grafts. The therapy has also been investigated for use in the treatment of peripheral vasculature stenosis and considered for the treatment of atrial fibrillation.

Side effects

The likelihood and nature of potential acute, sub-acute or long-term side-effects associated with brachytherapy depends on the location of the tumour being treated and the type of brachytherapy being used.

Acute

Acute side effects associated with brachytherapy include localised bruising, swelling, bleeding, discharge or discomfort within the implanted region.

 Brachytherapy treatment for cervical or prostate cancer can cause acute and transient urinary symptoms such as urinary retention, urinary incontinence or painful urination (dysuria). Transient increased in bowel frequency, diarrhoea, constipation or minor rectal bleeding, may also occur. Acute and subacute side effects usually resolve over a matter of days or a few weeks. In the case of permanent (seed) brachytherapy for prostate cancer, there is a small chance that some seeds may migrate out of the treatment region into the bladder or urethra and be passed in the urine.  Brachytherapy for skin cancer may result in a shedding of the outer layers of skin (desquamation) around the area of treatment in the weeks following therapy, which typically heals in 5 8 weeks. If the cancer is located on the lip, ulceration may occur as a result of brachytherapy, but usually resolves after 4 6 weeks.  Most of the acute side– effects associated with brachytherapy can be treated with medication or through dietary changes, and usually –disappear over time (typically a matter of weeks), once the treatment is completed. The acute side effects of HDR brachytherapy are broadly similar to EBRT.

Long-term

 In a small number of people, brachytherapy may cause long-term side effects due to damage or disruption of adjacent tissues or organs. Long-term side effects are usually mild or moderate in nature. For example, urinary and digestive problems may persist as a result of brachytherapy for cervical or prostate cancer, and may require ongoing management.  Brachytherapy for prostate cancer may cause erectile dysfunction in approximately 15- 30% of patients. However, the risk of erectile dysfunction is related to age (older men are at a greater risk than younger men) and also the level of erectile function prior to receiving brachytherapy.  Brachytherapy for breast or skin cancer may cause scar tissue to form around the treatment area. In the case of breast brachytherapy, fat necrosis may occur as a result of fatty acids entering the breast tissues. This can cause the breast tissue to become swollen and tender. Fat necrosis is a benign condition and typically occurs 4 12 months after treatment and affects about 2% of patients. –

Safety around others

 If temporary brachytherapy is used, no radioactive sources remain in the body after treatment. Therefore, there is no radiation risk to friends or family from being in close proximity with them.  If permanent brachytherapy is used, low dose radioactive sources (seeds) are left in the body after treatment - the radiation levels are very low and decrease over time. In addition, the irradiation only affects tissues within a few millimeters of the radioactive sources (i.e. the tumour being treated). As a precaution, some people receiving permanent brachytherapy may be advised to not hold any small children or be too close to pregnant women for a short time after treatment.

Typical stages of a brachytherapy procedure.

Radiation sources: Commonly used radiation sources (radionuclides) for brachytherapy

Radionuclide Type Half-life Energy Cesium-131 (131Cs) 9.7 days 30.4 keV (mean) Cesium-137 (137Cs) - particles 30.17 years 0.662 MeV Electron Capture, ε Cobalt-60 (60Co) −- particles 5.26 years 1.17, 1.33 MeV β Iridium-192 (192Ir) -rays− 73.8 days 0.38 MeV (mean) β Iodine-125 (125I) Electron Captu 59.6 days 27.4, 31.4 and 35.5 keV γ Palladium-103 (103Pd) 17.0 days 21 keV (mean) re, ε Ruthenium-106 (106Ru) - particles 1.02 years 3.54 MeV Electron Capture, ε Radium-226 (226Ra) −- particles 1599 years β − β Electronic brachytherapy: Electronic brachytherapy involves placement of miniature low energy x-ray tube sources into a pre-positioned applicator within body/tumour cavities to rapidly deliver high doses to target tissues while maintaining low doses to distant non- target tissues.

(a) Flail chest. See the Question 3.a of Paper – III of 2009

(b) Median nerve injury. Answer. Injury can occur:  Occasionally in the elbow region in supracondylar fractures of the humerus  Commonly injured by stab wounds or broken glass just proximal to the flexor retinaculum Injury at Elbow(motor)  Here it lies between the tendons of flexor carpi radialis and flexor digitorum superficialis  Rarely injured due to its protected position beneath the biceps brachii muscle  If injured high up in the arm, the biceps & coracobrachialis are paralyzed & brachialis is weakened  Sensory loss along the lateral side of the forearm occurs  When the patient tries to make a fist, the index & middle fingers tend to remain straight  Only ring & little fingers flex  Flexion in these fingers is weakened by the loss of the flexor digitorum superficialis  Flexion of terminal phalanx of thumb is lost because of paralysis of flexor policis longus  The thumb is laterally rotated and adducted  Muscles of thenar eminence are paralyzed  The hand looks flattened and ape like Injury at Elbow (sensory)  Skin sensation is lost on the palmar aspect of the lateral 3 & ½ fingers  Sensory loss occurs on the skin of the distal part of the dorsal surfaces of the lateral 3 & ½ fingers  Total area of anesthesia is less Injury at Elbow(vasomotor changes)  The skin areas involved in sensory loss are warmer and drier than normal  Arteriolar dilatation and absence of sweating resulting from loss of sympathetic control Injury at Elbow(Trophic changes) In long standing cases:  Skin is dry and scaly  Nails crack easily  Atrophy of the pulp of the fingers Injury at Wrist  Almost all the clinical findings are same as injury of the median nerve at elbow  In addition a delicate pincer like movement is not possible Carpal Tunnel Syndrome:  The carpal tunnel is formed by the concave anterior surface of carpal bones and closed by flexor retinaculum  Clinically, the syndrome consists of a burning pain or pins & needles along the distribution of the median nerve  Lateral 3 & ½ fingers are involved  The exact cause is difficult to determine  Condition is relieved by decompressing the tunnel by making a longitudinal incision through the flexor retinaculum

(c) Extrophy of bladder. Answer.Bladder exstrophy (also known as ectopia vesicae) refers to a herniation of the urinary bladder through an anterior abdominal wall defect. The severity of these defects is widely variable. Epidemiology: The estimated incidence of bladder exstrophy is 1:10,000-50,000 live births. There is a recognised male predilection with a male to female ratio of ~3:1. Most cases are sporadic. Pathology: Bladder exstrophy is thought to be caused by a developmental defect of the cloacal membrane which results in a subsequent eversion of the bladder mucosa. This then protrudes out as a mass-like lesion. Associations

General In females

 Extension of the bladder defect into  Vaginal duplication the urethra  Clitoral cleft  Cryptorchidism  Bilateral inguinal herniation  Oeis complex  Epispadia

Serological markers

 Raised maternal alpha feto protein levels

Radiographic features

 Imaging findings include a soft-tissue mass extending from a large infra-umbilical anterior wall defect which may be close to the umbilical arterial exits. The absence of a normal urinary bladder and a low-lying umbilical cord insertion 5 may also indicate the diagnosis.  Failure of the pubic bones to meet in the midline (widened pubic symphysis). This appearance on AP plain radiograph of the pelvis has been likened to a manta ray swimming towards you (manta ray sign).  Amniotic fluid volumes are often normal.

Treatment and prognosis:Treatment is with surgical intervention (primary closure/excision with urinary diversion) and prognosis is generally good. Complications

 Urinary incontinence  Infertility  Acute pyelonephritis  Increased incidence of malignancy in the extruded bladder THE WEST BENGAL UNIVERSITY OF HEALTH SCIENCES

MS (General Surgery) Examination, 2012

April 2012

PAPER III

Time Allowed: 3 Hours Full Marks: 100 Attempt all questions

1. Enumerate causes of painless haematuria in a 60 year old aged man. Discuss its investigation and treatment protocol. 20

Answer. Haematuria is a common condition and one which must be taken seriously. While there are some spurious reasons for patients reporting blood in the urine (eating beetroot, dye ingestion), most patients reporting macroscopic haematuria will be correct. An exception may be in women after the menopause confusing vaginal bleeding with blood in the urine, but this is rare.

Haematuria is usually divided into macroscopic (where the urine is discoloured) and microscopic (where the blood is found only on dipstick or microscopy examination. Further clinically relevant distinctions can be made between painful and painless haematuria, and haematuria of glomerular and post-glomerular origin.

Causes of painless Haematuria:

 Renal Tumours: o The commonest primary renal tumour is renal cell carcinoma, an of collecting tubule origin. It commonly presents with haematuria although most are nowadays picked up incidentally by ultrasound scanning. Diagnosis is made by CT scanning and treatment is by surgical excision. o Transitional Cell carcinoma of the renal collecting system usually gives haematuria. Diagnosis may be difficult, requiring retrograde imaging and ureteroscopy. Treatment is by either local excision or, for high grade or larger lesions, nephro-ureterectomy. Immunotherapy is used for metastases with limited success. o Benign renal tumours may cause both bleeding and diagnostic difficulty. They are, with the exception of the incidental and usually asymptomatic renal cyst, rare. is a hamartomatous lesion, which may grow to great size and be associated with major haemorrhage; treatment is by radiological emobolisation or surgery, conserving normal renal tissue where possible.  Renal Stones: o Stone disease is very common, with concretions forming in the renal papillae, which then form a nidus for stone formation in the collecting system. While most stones may cause infection, one particular type (infection ormatrix stone) is thought to be caused by bacteria that are able to split urea to form ammonium. Renal stones tend to be asymptomatic but may cause haematuria by either infection or direct irritation of the mucosa. They may also cause renal pain if large enough or obstructing. Diagnosis is by imaging, usually intravenous urography. Renal stones can usually be treated by extracorporeal shock wave lithotripsy although large or complex stones may need percutaneuous or open surgical removal.

 Glomerulonephritis: Glomerulonephritis tends to present with microscopic haematuria. While pain may be associated, most cases will have either no symptoms or may show signs of renal failure.

 Pyelonephritis (ascending urinary tract infection): Acute bacterial pyelonephritis results from bacteria ascending from the bladder either by direct spread (vesico-ureteric reflux) or possibly by periureteric lymphatic extension. Painless haematuria may occur but the symptom complex usually includes loin pain, fever and possibly septicaemia.

 Papillary Necrosis: This condition occurs in diabetics and in patients with deficiencies of oxygenation, particularly sickle cell disease. It is characterised by a radiolucent filling defect on IVU and may usually be treated expectantly.

 Ureteric Stones: these nearly always present with pain but may have haematuria as the only symptom. The presence or absence of obstruction and the size of the stone dictates management.

 Cystitis: Typically cystitis is painful and in men is commonly associated with bladder outflow obstruction. Schistosomiasis, interstitial cystitis and drug related cystitis are rarer causes of bladder inflammation causing bleeding. Diagnosis is by urine microscopy and culture, assisted by cystoscopy. Bladder biopsies may be necessary.

 Bladder tumours:

o Most of the interest in painless haematuria stems from the desire to diagnose bladder tumours at an early stage. Nearly all are transitional cell cancers, with smoking and aromatic hydrocarbon exposure being risk factors. Rarer bladder tumours include adenocarcinoma (usually arising from the urachus) and squamous cancer (associated with chronic inflammation and schistosomiasis). o Diagnosis is as outlined above with management depending on the stage and grade: 70% are superficial at presentation and are managed by transurethral surgery with or without the use of intravesical therapy. For invasive tumours the choice lies between radical cystectomy or radiotherapy. Metastatic disease may respond to platinum based chemotherapy.

 Prostate tumours

o Benign prostatic hyperplasia is ubiquitous but can bleed on its own: it may also cause bacterial cystitis. Diagnosis is by urinary flow assessment and bladder residual volume measurement. Assuming that other causes of haematuria have been ruled out, treatment is tailored to the individual. o Prostate specific antigen levels should be checked (after any UTI has been treated) to rule out prostate cancer, which may cause haematuria directly or by urinary tract infection. Diagnosis is by prostatic biopsy, usually with ultrasound control. Treatment depends on the stage and outlook, but local disease may be suitable for radical prostatectomy or radiotherapy while advanced disease responds to hormonal manipulation.  Rare causes of haematuria:

o Arteriovenous malformations, tuberculosis and arteritis may all cause haematuria. Patients on anticoagulants who have control in the normal therapeutic range and who have haematuria must be fully investigated as above, since haematuria is not a normal consequence of anticoagulation.

Investigations of Haematuria:

 After a general physical examination (not forgetting blood pressure, the prostate in a male and the gynaecological organs in a female), the first investigation in a patient with reported haematuria is urinary examination. This must include microscopy for red and white blood cells and bacteria.  The presence of any crystals, ova or parasites should be noted and culture of a mid stream specimen carried out. If schistosomiasis or tuberculosis are suspected a first void urine sample is usually requested.  The level of protein in the urine must be assessed, but in heavy haematuria it may be difficult to be sure if light proteinuria is due to the haemoglobin present. If no red blood cells are found in the urine but haemoglobin is present the patient should be investigated for causes of haemoglobinuria.  All patients should have a full blood count with an erythrocyte sedimentation rate. Serum urea, creatinine and electrolytes should be measured, along with albumin, calcium and liver function tests if the patient is unwell or in renal failure. Coagulation studies are not routine.  Assuming microscopic amounts of blood are found, the presence or absence of proteinuria may guide initial investigation, since the combination of these favour a glomerular problem. In this event C reactive protein and24 hour urine protein excretion will be informative, as may serum electrophoresis and autoantibodies. Ultrasound will show renal cortical thickness and density.  In the majority of cases a renal biopsy with immunoglobulin histochemistry will be necessary to make a definitive diagnosis: the need for this will be balanced against the patient s state of health and the perceived risks of the possible disease or procedure.  In cases of microscopic haematuria without proteinuria, and all macroscopic cases, ’a surgical investigation plan can be followed. This is based around the exclusion of malignant or stone disease, and centres on cystoscopy to directly inspect the bladder.  “ Other imaging” may be done either by intravenous urography or a combination of plain abdominopelvic radiography and ultrasound of the urinary tract.  If no abnormality is found then a flexible cystoscopy under local anaesthetic may be performed, but if either the imaging or this endoscopic examination suggest a bladder lesion the patient will require a transurethral biopsy and examination under anaesthetic for both treatment and diagnosis.  In any of the above scenarios it is important to remember that if a particular investigation pathway leads to a negative result, consideration should be given to carrying out the test of the other pathways, thus flexible cystoscopy for a patient with persistent microscopic haematuria in whom no renal cause is found, and ultrasound in a patient with a normal bladder and intravenous urogram.

2. Describe the pathogenesis of Incisional Hernia. Discuss the steps of component separation technique of hernia repair. Give a brief comparison of it with pre-peritoneal mesh repair. 8+6+6. Answer. Incisional hernia is one of the common postoperative complication following general surgery. The cumulative incidence has remained constant despite several attempts to improve laparotomy closure. As well as surgical closure technique, individual biological and patient dependent risk factors play a key role. Factors affecting incisional hernia development:  Surgical technique o Type of incision o Suture material o Suture technique  Patient related factors o Wound healing o Local wound healing impairment o Age o Gender o Concomitant disease – Obesity – Anemia – Underlying malignancy – Diabetes mellitus – Abdominal aortic aneurysm  Exogenous toxins o Smoking o Certain medicines  Hereditary connective tissue disorders.

Pathogenesis: The development of an incisional hernia is multifactorial. There are two major areas which influence its pathogenesis: surgical technique, and patient-related factors.

Surgical technique: Most retrospective studies looking at surgical technique as a possible influence of incisional hernia risk have focused on the type of incision, type of suture material, and suture technique. Retrospective studies suggest that transverse incisions, Pfannenstiel incisions and other pararectal incisions are associated with reduced incidence of incisional hernia when compared with vertical incisions. There is currently no high quality evidence of an incision-related effect on the incidence of incisional hernia. The increasing use of minimally invasive procedures promise improvements in the incidence of incisional hernia, although incidences of hernia at the trocar site of 1 to 4% are reported. Regarding type of suture materials, several meta-analyses report the lowest incidences of incisional hernia for monofilamentous, nonresorbable or long-term resorbable suture materials . The theoretical advantage of nonresorbable suture material in hernia prevention must, however be balanced against a significantly higher incidence of fistula formation and higher rates of postoperative wound pain, due to permanent mechanical tissue irritation.

Patient related factors: A large number of biological factors related to the patient's preexisting disease and its systemic effects have a suspected role in the aetiology of incisional hernia. Normal wound healing Wound healing is a complex process involving the interaction of several different cell types with the extracellular matrix. The quantity and quality of connective tissue – produced is the primary determinant of the degree of normal physiological function and tissue stability achieved during scar formation. The ratio of type I to type III collagen is considered a predictor of connective tissue stability. Of the known collagen variants, type I collagen is, due to its tensile properties, particularly responsible for mechanical tissue resistance. By comparison, type III collagen, expressed primarily during the early wound healing phase, is mechanically less stable and is later replaced by type I collagen. In patients with incisional hernias, a significant decrease in the collagen type I/III ratio relative to normal fascias is observed, suggesting a dysfunction in collagen metabolism. Local disturbances of wound healing The development of wound infection or seroma significantly increases the risk of incisional hernia. It is established– that infections and seromas have an impact on local wound healing via an increase in the production of cytokines and proteases, leading in turn to a reduction in fibroblasts and diminished wound stability. Necrosis resulting from excessive suture tension can also disturb wound healing and stable scar formation. Age and gender age above 45 years emerged as a significant risk factor for incisional hernia. This can be explained by delayed wound healing in older individuals, with changes in fibroblast migration and structural– changes with reduced collagen formation as well as the high prevalence of concomitant disease. While many studies suggest that men were at greater risk than women of developing incisional hernia, other studies have not found significant effect of gender. Concomitant disease Many studies identify adiposity as a risk factor for incisional hernia development. Poor nutritional status and ill health can also promote incisional hernia formation. – Anemia,underlying malignant disorder and diabetes mellitus were all identified as independent risk factors. The incidence rate of incisional hernia following abdominal aortic aneurysm surgery is at 28 to 38% significantly higher than the rate observed after laparotomy performed for other indications. It has more recently been shown that these patients suffer in part from genetically determined type III procollagen disorder with concomitant impairment of new collagen synthesis. Environmental influences smoking promotes both hernia recurrence and hernia formation. For example, smokers have been shown to have not only a significantly increased rate of relapse after inguinal hernia repair, but –also a four-fold risk of developing incisional hernia. In addition, numerous other drugs such as ACE inhibitors, corticosteroids, non-steroidal anti- inflammatory substances and chemotherapeutic agents have an impact on the various stages of wound healing. Hereditary connective tissue disorders Connective tissue disorders such as Ehlers- Danlos syndrome, osteogenesis imperfecta, cutis laxa, but also congenital dislocations of the hip are generally associated with an increase in– the incidence of hernias or an increase in the relapse rate after primary repair. All these disorders have in common that they are caused by "non-physiologic" connective tissue composition .The association with hernias in these patients can be adequately explained by systemic impairment of connective tissue formation and wound healing, and confirms the biological basis underlying the pathogenesis of hernia formation. Complete prevention of incisional hernia cannot be achieved solely by optimizing the surgical technique for abdominal wall closure. The changes in the extracellular matrix observed in patients with incisional hernia, as well as the observed increased incidence ofincisional hernias in patients with additional "endogenous" or "exogenous" risk factors, suggest a multifactorial biological basis for the pathogenesis of incisional hernia.

The Component Separation Technique (CST), a type of rectus abdominis muscle advancement flap, was first used to reconstitute the linea alba, reduce abdominal wall tension, and provide a dynamic abdominal wall in patients with large abdominal wall defects. CST allows reconstruction of a large defect without requiring a free distant transposition flap. Despite the relatively high frequency of recurrent abdominal wall incisional hernias, there is no consensus on the optimal repair. The advantages of CST are that it restores structural support of the abdominal wall, provides stable soft tissue coverage, and optimizes esthetic appearance of complex abdominal wall defects and giant midline abdominal wall hernias. Basic Steps To Open CST

1. Enter the abdominal cavity and perform adhesiolysis. 2. Bilateral skin flaps raised. 3. Bilateral relaxing incisions made through external oblique 2-3 cm lateral to the linea semilunaris. 4. Midline closed. 5.Surgical Graft is secured into place against the abdominal wall with sutures, followed by skin closure.

Patient Preparation: Place the patient in a supine position with both arms abducted and appropriate padding on all pressure points. Apply sequential pneumatic compression devices to the lower extremities to prevent deep venous thrombosis. Utilize preoperative heparin products as appropriate.

Entering the Abdominal Cavity and Adhesiolysis:  Use sharp dissection to enter the abdominal cavity.  The incision is made adjacent to the prior surgical scar to allow for scar excision.

 Enter the abdominal cavity above or below the prior incision to gain access to the peritoneal cavity in an area free of adhesions.  Open the remainder of the incision.  Dissect bowel, omentum, and/or other adhered organs from the abdominal wall using direct visualization.

Bilateral Skin Flaps Raised:  Release a flap consisting of skin and subcutaneous tissue from the underlying anterior rectus sheath using cautery.  Extend laterally beyond the linea semilunaris to the level of anterior axillary line.  Extend caudally to the inguinal ligament and cephalad to a distance approximately 5 cm above the costal margins.  In most circumstances the releases are performed bilaterally.

Create Relaxing Incisions:  Create a vertically oriented incision in the external oblique aponeurosis located 2-3 cm lateral to the linea semilunaris.  The relaxing incision should extend from the inguinal ligament to at least the level of the costal margin.  In the event of an epigastric defect, the incision should extend several centimeters above the costal margin.

Standard Release:

Additional Advancement Options:

Posterior rectus sheath release Posterior p Extended release above the Posterior rectus sheath p costal margin

 After division of the external oblique, the plane between the external and internal oblique muscles is developed using blunt dissection and extending out to the anterior axillary line.  Further advancement can be obtained by elevating the rectus from the posterior rectus sheath by incising the posterior rectus sheath 1-2 cm from the midline. The anterior rectus sheath should be left intact for suture placement.  In most circumstances, the releases are performed bilaterally.

Midline Closure:

 healthy well-vascularized tissue as the midline is closed.  TheTo assess hernia the sac abdomen and attenuated for closure, • fascia Kocher are debrided clamps are from applied the midline to the toanterior ensure rectus apposition fascia of on each side and brought to the midline.  Monitor peak airway pressures during this maneuver.  Absent a significant increase in peak airway pressures, plan to close the fascia primarily with interrupted sutures.

Graft Placement:  Reinforce the hernia repair with a Surgical Graft in an onlay or underlay position.  To address larger defects, multiple Surgical Grafts may be sutured together with a running suture.  Orient the Surgical Graft in either a rectangular or diamond configuration to reinforce the entire abdominal wall and secure with a continuous suture.  For onlay mesh placement, suture the Surgical Graft bilaterally to the released lateral edges of the external oblique and sew it to the anterior rectus sheath superiorly and inferiorly.

Retro-Muscular Placement:

5 cm

Graft Placement continued:  For underlay mesh placement, place the Surgical Graft either as an intraperitoneal graft or in the retro-rectus space.  If placed in the retro-rectus space, additional dissection is needed to create a pocket for the graft. Once the retro-rectus space has been developed, close the posterior rectus sheath with a running absorbable suture. This will create a space for the Surgical Graft between the rectus muscle and the posterior rectus sheath.

Intraperitoneal Placement:

5 cm

 Reinforce the entire incision by overlapping the Surgical Graft by approximately 5 cm.  Suture the graft to the abdominal wall with a series of interrupted horizontal mattress sutures.  Place the sutures laterally on the abdominal wall to allow for approximation of the midline fascia when the sutures are secured on each side.

 Irrigate the wound with sterile, normal saline.  Place closed suction drains beneath the flaps in the subcutaneous plane.  Maintain the drains until the output decreases to less than 30 milliliters per day for 2 consecutive days.  Close the skin in a layered fashion due to the extensive undermining of skin.

Points Component separation technique Pre peritoneal mesh repair

Indications i) Size of the defect: Defects larger 1. Infected abdominal wall with or than 3 cm and smaller ones in obese without mesh. patients, recurrent hernias and 2. Patients with hernias who are also special types of hernias such as the having a colostomy reversed. spigelian hernia, are best treated laparoscopically. There are no 3. Very large ventral hernias. Defects objectively defined selection criteria of up to 20 cm at the waistline can be for the upper limit but some experts bridged. have suggested limiting LVHR to 4. Multiple defects. cases where transverse separation of 5. Multiple failed attempts at previous the fascial edges is 10 cm. However repairs. there are many reports in the literature of LVHR of far much larger 6. Treatment or prevention of the hernias. abdominal compartment syndrome (ACS). May avoid open wound ii) Obese patients and recurrent management which usually results in a hernias. large ventral hernia and potential skin iii) Swiss-cheese defects. deficit.

“ ” 7. In those patients with Loss of Domain, component separation allows placement of a smaller piece of mesh, thereby minimizing eventration.

Contraindications i) Extremely large hernias i) Patients with stoma. ii) End-stage cardiac, liver and ii) Delaying the operation until pulmonary disease any risk factors (obesity, iii) Extremely dense adhesions like in smoking, COPD, malnutrition, previous multiple laparotomies, steroids) can be eliminated will peritonitis or end stage renal disease greatly improve the outcome of with peritoneal dialysis, as there may the operation. be obliteration of the peritoneal cavity for placement of cannulas and the attendant increased risk of enterotomy. iv) Liver cirrhosis and portal hypertension. v) General contraindications to laparoscopy e.g. coagulopathy and ascites. vi) Paediatric age group due to the potential of mesh migration. vii) Strangulated hernias. viii) Lack of abdominal domain. This refers to patients with insufficient space in the abdomen to accommodate the contents of the hernia that are also at great risk for pneumoperitoneum. ix) Hernias in which the fascial edges extend lateral to the midclavicular line may make trocar placement lateral to the defect impossible. Defects in close proximity to the bony margins of the abdomen, especially those near the xiphoid, pose significant challenges for mesh fixation, though this is also true with open incisional herniorrhaphy.

Complications Rates of wound sepsis, mesh infection, mesh explantation and gastrointestinal mesh erosion are low with CST. Recurrence is low.

Cost Low High

3. Write short notes of the following: 5 x 6 a) Central venous catheterization. b) Congenital mega colon. c) Vesicoureteric reflux. d) Imperfectly descended testis. e) Adjuvant therapy in carcinoma breast.

Answer. a)Central venous catheterization: In medicine, a central venous catheter ("central line", "CVC", "central venous line" or "central venous access catheter") is a catheter placed into a large vein in the neck (internal jugular vein ), chest (subclavian vein or axillary vein) or groin (femoral vein). It is used to administer medication or fluids, obtain blood tests (specifically the "mixed venous oxygen saturation"), and directly obtain cardiovascular measurements such as the central venous pressure.

Indications for the use of central lines include:

Drugs that are prone to Need for cause phlebitis in intravenous therapy peripheral veins when peripheral (caustic), such as: venous access is impossible

. Monitoring of . Calcium chloride . Plasmapheresis . Blood the central venous . Chemotherapy . Peripheral . Medication pressure (CVP) in . Hypertonic saline blood stem cell . Rehydration acutely ill patients to . Potassium chloride collections quantify fluid balance . Amiodarone . Dialysis . Long- . vasopressors (e.g. . Frequent blood term Intravenous antibi epinephrine, draws otics dopamine) . Frequent or . Long-term Parenteral persistent

nutrition especially in requirement chronically ill patients for intravenous . Long-term pain access medications

. Chemotherapy

Central venous catheters usually remain in place for a longer period of time than other venous access devices, especially when the reason for their use is longstanding (such as total parenteral nutrition in a chronically ill patient). Insertion : The skin is cleaned, and local anesthetic applied if required. The location of the vein is then identified by landmarks or with the use of a small ultrasound device. A hollow needle is advanced through the skin until blood is aspirated; the color of the blood and the rate of its flow help distinguish it from arterial blood (suggesting that an artery has been accidentally punctured), although this method is inaccurate. Ultrasound probably now represents the gold standard for central venous access and skills, within North American and Europe, with landmark techniques are diminishing. The line is then inserted using the Seldinger technique: a blunt guidewire is passed through the needle, then the needle is removed. A dilating device may be passed over the guidewire to slightly enlarge the tract. Finally, the central line itself is then passed over the guidewire, which is then removed. All the lumens of the line are aspirated (to ensure that they are all positioned inside the vein) and flushed. A chest X-ray is typically performed afterwards to confirm that the line is positioned inside the superior vena cava and, in the case of insertion through the subclavian vein, that no pneumothorax was caused as a side effect. Vascular positioning systems can also be used to verify tip placement during insertion without the need to a chest X-ray, but this technique is not yet a standard of practice. Videos are available demonstrating placement of a central venous catheter without and with ultrasound guidance. Complications :  Mechanical Complications: o Arterial puncture, hematoma, and pneumothorax are the most common mechanical complications during theinsertion of central venous catheters. o Frequency of Mechanical Complications, According to the Route of Catheterization: Overall, internal jugular catheterization and subclavian venous catheterization carry similar risks of mechanical complications. Subclavian catheterization is more likely than internal jugular catheterization to be complicated by pneumothorax and hemothorax, whereas internal jugular catheterization is more likely to be associated with arterial puncture. o Hematoma and arterial puncture are common during femoral venous catheterization. Because mechanical complications are most likely during catheterization at the femoral site, the internal jugular or subclavian venous route should be chosen unless contraindicated. However, the rate of serious mechanical complications (e.g., pneumothorax requiring insertion of a chest tube or hemorrhage requiring blood transfusion or surgery) associated with subclavian insertion is similar to that associated with femoral insertion.

 Infectious Complications: o Catheter-related infections are thought to arise by several different mechanisms: infection of the exit site, followed by migration of the pathogen along the external catheter surface; contamination of the catheter hub, leading to intraluminal catheter colonization; and hematogenous seeding of the catheter. Available evidence suggests that subclavian catheterization is less likely to result in catheter-related infection than internal jugular catheterization, although the two approaches have not been compared in randomized trials.

 Thrombotic Complications o Patients who require central venous catheterization are at high risk for catheter-related thrombosis. Used routinely, ultrasonography with color Doppler imaging detects venous thrombosis in 33 percent of patients in medical intensive care units and in approximately 15 percent of these patients the thrombosis is catheter-related. The risk of catheter-related thrombosis varies according to the site of insertion. o Subclavian venous catheterization carries the lowest risk of catheter-related thrombosis. The clinical importance of catheter-related thrombosis remains undefined, although all thromboses have the potential to embolize.

Answer. b) Congenital megacolon: A rare condition similar to Hirschsprung's disease where abnormalities in the bowel muscles prevent it from contracting normally and pushing the fecal matter through. Congenital megacolon differs from Hirschsprung's disease in that the whole colon tends to be dilated. An infant with congenital megacolon is born with a rectum and colon that do not work properly because the nerves that stimulate the bowel are absent. Congenital megacolon causes dilation of the colon and rectum, as well as a build-up of stool in the colon and rectum. Congenital megacolon is usually inherited and may be associated with Down's syndrome.

 Congenital megacolon is usually the result of absent nerves within the colon that normally control contraction of the colon. This condition is called Hirschsprung disease.

Signs and symptoms of Congenital megacolon are as follows:

Constipation Blood-streaked Intestinal colic Short anal canal Enlargement of from birth stool colon Large stool Difficult bowel Distended Dilated rectum Gas movements abdomen Hard stool Painful bowel Palpable fecal Dilated pelvic Stomach noises movements masses colon

Conditions mimicking Congenital megacolon:

 Hirschsprung disease  Intestinal immotility  Intestinal obstruction  Meconium  Constipation

Complications of congenital megacolon include:

 Bacterial enterocolitis  Intestinal perforation  Gastrointestinal bleeding

Tests that may be used to evaluate congenital megacolon include:

 Barium enema x-ray: o Can show findings of this condition  Intestinal biopsy: o Needed to confirm the diagnosis of congenital megacolon.

Treatment for congenital megacolon may include:

 Intravenous fluids for dehydration  Enemas for congenital megacolon  Surgery for congenital megacolon: o Colostomy

Answer. c)Vesicoureteral reflux

Vesicoureteral reflux (VUR) is an abnormal movement of urine from the bladder into ureters or kidneys. Urine normally travels from the kidneys via the ureters to the bladder. In vesicoureteral reflux the direction of urine flow is reversed (retrograde)

Symptoms

Vesicoureteral reflux may present before birth as prenatal hydronephrosis, an abnormal widening of the ureter or with a urinary tract infection or acute pyelonephritis. Symptoms such as painful urination or renal colic/flank pain are not symptoms associated with vesicoureteral reflux. Newborns may be lethargic with faltering growth, while infants and young children typically present with pyrexia, dysuria, frequent urination, malodorous urine and GIT symptoms, but only when urinary tract infection is present as the initial presentation of VUR.

Causes: In healthy individuals the ureters enter the urinary bladder obliquely and run submucosally for some distance. This, in addition to the ureter's muscular attachments, helps secure and support them posteriorly. Together these features produce a valvelike effect that occludes the ureteric opening during storage and voiding of urine. In people with VUR, failure of this mechanism occurs, with resultant retrograde flow of urine.

Primary VUR: Insufficient submucosal length of the ureter relative to its diameter causes inadequacy of the valvular mechanism. This is precipitated by a congenital defect/lack of longitudinal muscle of the intravesical ureter resulting in an ureterovesicular junction (UVJ) anomaly.

Secondary VUR: In this category the valvular mechanism is intact and healthy to start with but becomes overwhelmed by raised vesicular pressures associated with obstruction, which distorts the ureterovesical junction. The obstructions may be anatomical or functional.

Secondary VUR can be further divided into anatomical and functional groups as follows:

Anatomical: Posterior urethral valves; urethral or meatal stenosis.

These causes are treated surgically when possible.

Functional: Bladder instability, neurogenic bladder and non-neurogenic neurogenic bladder Urinary tract infections may cause reflux due to the elevated pressures associated with inflammation.

Resolution of functional VUR will usually occur if the precipitating cause is treated and resolved. Medical and/or surgical treatment may be indicated.

Age:Younger children are more prone to VUR because of the relative shortness of the submucosal ureters. This susceptibility decreases with age as the length of the ureters increases as the children grow. In children under the age of 1 year with a urinary tract infection, 70% will have VUR. This number decreases to 15% by the age of 12.

Sex: Although VUR is more common in males antenatally, in later life there is a definite female preponderance with 85% of cases being female.

International Classification of Vesicoureteral Reflux

 Grade I reflux into non-dilated ureter  Grade II reflux into the renal pelvis and calyces without dilatation  Grade III– mild/moderate dilatation of the ureter, renal pelvis and calyces with minimal blunting –of the fornices  Grade IV – dilation of the renal pelvis and calyces with moderate ureteral tortuosity  Grade V gross dilatation of the ureter, pelvis and calyces; ureteral tortuosity; loss of papillary –impressions – The younger the age of the patient and the lower the grade at presentation the higher the chance of spontaneous resolution. Most (approx. 85%) of grade I & II cases of VUR will resolve spontaneously. Approximately 50% of grade III cases and a lower percentage of higher grades will also resolve spontaneously.

Diagnosis

VCUG demonstrating bilateral Grade III vesicoureteral reflux.

The following procedures may be used to diagnose VUR:

 Nuclear cystogram (RNC)  Fluoroscopic voiding cystourethrogram (VCUG)  Ultrasonic cystography  Abdominal ultrasound

An abdominal ultrasound might suggest the presence of VUR if ureteral dilatation is present; however, in many circumstances of VUR of low to moderate severity, the sonogram may be completely normal, thus providing insufficient utility as a single diagnostic test in the evaluation of children suspected of having VUR, such as those presenting with prenatal hydronephrosis or urinary tract infection (UTI). VCUG is the method of choice for grading and initial workup, while RNC is preferred for subsequent evaluations as there is less exposure to radiation. A high index of suspicion should be attached to any case a where a child presents with a urinary tract infection, and anatomical causes should be excluded. A VCUG and abdominal ultrasound should be performed in these cases

Early diagnosis in children is crucial as studies have shown that the children with VUR who present with a UTI and associated acute pyelonephritis are more likely to develop permanent renal cortical scarring than those children without VUR. Thus VUR not only increases the frequency of UTI's, but also the risk of damage to upper urinary structures.

Treatment

Medical treatment is the preferred mode of management, but surgical interventions may be necessary. Medical management is recommended in children with Grade I-III VUR as most cases will resolve spontaneously. A trial of medical treatment is indicated in patients with Grade IV VUR especially in younger patients or those with unilateral disease. Of the patients with Grade V VUR only infants are trialed on a medical approach before surgery is indicated, in older patients surgery is the only option.

Endoscopic injection

Deflux is a gel that is used in endoscopic injections to treat Vesicoureteral Reflux. It is the material that the surgeon injects around the ureteral opening to create a valve function and stop urine from flowing back up the ureter. Deflux consists of two types of sugar-based molecules called dextranomer and hyaluronic acid. Both substances are well-known from previous uses in medicine. Both materials are also biocompatible, which means that they do not cause significant reactions within the body. In fact, hyaluronic acid is produced and found naturally within the body. Medical Treatment

Medical treatment entails low dose antibiotic prophylaxis until resolution of VUR occurs. Antibiotics are administered nightly at half the normal therapeutic dose. The specific antibiotics used differ with the age of the patient and include:

 Amoxicillin or ampicillin - infants younger than 6 weeks  Trimethoprim-sulfamethoxazole (co-trimoxazole) - 6 weeks to 2 months

After 2 months the following antibiotics are suitable:

 Nitrofurantoin{5 7 mg/kg/24hrs}  Nalidixic acid  Bactrim –  Trimethoprim  Cephalosporins

Urine cultures are performed 3 monthly to exclude breakthrough infection. Annual radiological investigations are likewise indicated. Good perineal hygiene, and timed and double voiding are also important aspects of medical treatment. Bladder dysfunction is treated with the administration of anticholinergics.

Surgical Management

A surgical approach is necessary in cases where a breakthrough infection results despite prophylaxis, or there is non-compliance with the prophylaxis. Similarly if the VUR is severe (Grade IV & V), there are pyelonephritic changes or congenital abnormalities. Other reasons necessitating surgical intervention are failure of renal growth, formation of new scars, renal deterioration and VUR in girls approaching puberty.

There are three types of surgical procedure available for the treatment of VUR: endoscopic (STING/HIT procedures); laparoscopic; and open procedures (Cohen procedure, Leadbetter- Politano procedure).

Follow-up

2010 guidelines from the American Urological Association recommend ongoing monitoring of children with VUR until the abnormality resolves or is no longer clinically significant. The recommendations are for annual evaluation of blood pressure, height, weight, urinalysis, and renal ultrasonography. d) Answer. Imperfectly descended testis.

Incompletely descended testis:

Incidence: About 4% of boys are born with one or both testes incompletely descended. About half of these reach the scrotum during the first month of life, but full descent after that is uncommon. The condition is sometimes missed at birth and discovered later in life. In a few cases, the presence of a hernia, testicular pain or acute torsion directs attention to the abnormality. In 10% of unilateral cases there is a family history.

Pathology: Incompletely descended testes are often macroscopically normal in early childhood but by puberty the testis is poorly developed compared with its intrascrotal counterpart. The epithelial elements are immature histologically and by late puberty irreversible destructive changes halt spermatogenesis and limit the production of androgens. An incompletely descended testis brought down in early childhood often functions satisfactorily.

Undescended testis

■ Testes absent from the scrotum after 3 months of age are unlikely to descend fully ■ An incompletely descended testis tends to atrophy as puberty approaches ■ Early repositioning of an incompletely descended testis can preserve function.

Clinical features

The condition is more common on the right and is bilateral in 20% of cases. Secondary sexual characteristics are typically normal.

The testis may be: -abdominal, lying extraperitoneally above the internal inguinal ring;

• intra the superficial inguinal pouch, in which case it must be distinguished from retractile testis. • inguinal, in which case it may or may not be palpable. During• in childhood the testes are mobile and the cremasteric reflex is active. In some boys, any stimulation of the skin of the scrotum or thigh causes the testis to disappear into the inguinal canal. Simply exposing the parts can also do this so that the testis is never seen in an intrascrotal position. The scrotum is normal as opposed to underdeveloped, which is usually seen with true incomplete descent. When the cremaster relaxes, the telltale bulge of the testis reappears only to vanish when the scrotal skin is touched again. A retractile testis can be gently milked from its position in the inguinal region to the bottom of the scrotum. A diagnosis of true incomplete descent should be made only if this is not possible. In infancy, most inapparent testes are retractile. They are normal and require no treatment.

Retractile testis: Retractile testes are more common than incompletely descended testes and require no treatment.

Hazards of incomplete descent are:

terility in bilateral cases (especially intra-abdominal testes); ain as a result of trauma; • Sn associated indirect inguinal hernia is often present and may cause symptoms; • Porsion; • Apididymo-orchitis in an incompletely descended right testis can mimic appendicitis; atrophy of •an T inguinal testis before puberty may possibly be caused by recurrent minor trauma; • Encreased liability to malignant disease; cancer is more common in an incompletely descended testes orchidopexy may or may not diminish the risk but it does improve the prospect of early diagnosis.• I – Surgical treatment:

Orchidopexy is usually performed after the age of 1 year to avoid the risks of operating on a tiny patient. Testes should be brought down into the scrotum before the boy starts school. In bilateral cases it is usual to operate on one side at a time.

Ectopic testis: The sites of ectopic testis are:

t the superficial inguinal ring; n the perineum; • At the root of the penis; • In the femoral triangle. • A An• I ectopic testis is usually fully developed. The main hazard is liability to injury. e)Adjuvant therapy in carcinoma breast. Answer. Adjuvant treatment of breast cancer is designed to treat micrometastatic disease, or breast cancer cells that have escaped the breast and regional lymph nodes but have not yet had an established identifiable metastasis. Depending on the model of risk reduction, adjuvant therapy has been estimated to be responsible for 35-72% of the reduction in mortality rate. Taxanes ( docetaxel, paclitaxel):  Taxanes are among the most active and commonly used chemotherapeutic agents for the treatment of early-stage breast cancer.  HER2 positivity irrespective of estrogen receptor status predicted a significant benefit from paclitaxel in terms of reduced disease recurrence. Patients with ER-positive, HER2-negative, node-positive breast cancer did not seem to benefit from the addition of a taxane.  However, the National Cancer Institute of Canada MA.21[19] and UK TACT trials,[20] which used taxane- and nontaxane-based chemotherapeutic regimens in early-stage breast cancer patients, did not demonstrate a benefit in using taxanes. Although the precise role of adjuvant taxane therapy remains controversial, the optimal scheduling of taxane administration has been determined.  The Eastern Coast Oncology Group (ECOG) 1199 study demonstrated that paclitaxel weekly and docetaxel every 3 weeks were superior to 2 other regimens in terms of disease-free survival after a median follow-up of 64 months. This trial randomized 4950 women with lymph node- positive or high-risk lymph node negative early-stage breast cancer to 4 cycles of AC followed by 4 different taxane regimens: paclitaxel at 175 mg/m2 q3wk; paclitaxel at 80 mg/m2 weekly; docetaxel (Taxotere) at 100 mg/m– 2 q3wk; and docetaxel at 35 mg/m2 weekly.  Similarly, the TAX 311 trial, performed by the US Oncology Group in patients with advanced breast cancer that had progressed after an anthracycline-containing chemotherapy regimen, showed that every-3-week docetaxel at 100 mg/m2improved time to progression (TTP) and overall survival when compared with paclitaxel at 175 mg/m2 given every 3 weeks.  Thus, taxane-based regimens still have use in the treatment of early-stage breast cancer and should be considered in treating women, especially those with HER2-positive disease, using either the weekly paclitaxel or every-3-week docetaxel dosing schedules.

Anthracyclines:  Anthracycline-containing adjuvant chemotherapy regimens have been used in the treatment of early-stage breast cancer for decades, although concerns regarding anthracycline-associated cardiotoxicity or leukemogenic potential remain. The US Oncology 9735 trial established TC (docetaxel/cyclophosphamide) as a viable option for treating women with early-stage breast cancer, especially those at high risk of cardiotoxicity or requiring only 12 weeks of therapyAdditionally, a meta-analysis of 8 trials, comprising 6564 women with early-stage breast cancer, of anthracycline-based versus nonanthracycline-based regimens, suggested a benefit with anthracycline administration only in patients with HER2-positive disease.  Biologically, anthracyclines inhibit topoisomerase IIa, whose gene (TOP2A) lies adjacent to the HER2 gene on chromosome 17. TOP2A is co-amplified in approximately 35% of HER2- overexpressing breast cancers.  The original trials demonstrating superiority of anthracycline-based regimens over CMF (cyclophosphamide, methotrexate, fluorouracil) did not include TOP2A orHER2 testing. The BCIRG 006 trial, which randomized women with HER2-positive disease to AC followed by T, AC followed by TH (docetaxel/trastuzumab [Herceptin]), or TCH (docetaxel and carboplatin plus trastuzumab), did test forTOP2A and HER2 co-amplification.  An anthracycline followed by or concurrent with a taxane is the optimal therapy for "triple- negative" breast cancer patients with no medical contraindications. However, it remains unclear what the optimal combination chemotherapy regimen is for ER-positive, HER2-negative tumors. Currently, CMF, TC, or an anthracycline-based regimen may all be reasonable options. Adjuvant Hormone Therapy:  In ER-positive early-stage breast cancer, hormone therapy plays a main role in adjuvant treatment, either alone or in combination with chemotherapy. Hormone treatments function to decrease estrogen's ability to stimulate existing micrometastases or dormant cancer cells.  Adjuvant hormone therapy can reduce the relative risk of distant, ipsilateral, and contralateral breast cancer recurrence by up to 50% in tumors with high ER expression. FDA-approved endocrine therapies for adjuvant treatment of breast cancer include tamoxifen and the aromatase inhibitors (anastrozole, letrozole , exemestane).  Tamoxifen is a selective estrogen receptor modulator (SERM) that binds to and inhibits estrogen receptor signaling in the breast. As a receptor antagonist, it is effective in both premenopausal and postmenopausal women. Tamoxifen has ER-stimulating effects in other tissues, including bone (resulting in preservation of bone density) and endometrium (leading to a 2- to 4-fold increased risk of endometrial cancer).  Tamoxifen has been approved for breast cancer treatment since the early 1980s and has been shown in multiple studies to decrease breast cancer associated mortality and recurrence. In an analysis of 55 trials evaluating tamoxifen versus placebo in the adjuvant treatment of breast cancer, 5 years of tamoxifen therapy resulted in a 47%– reduction in recurrence and a 22% reduction in mortality. Common adverse effects associated with tamoxifen included the following:  Hot flashes (up to 80%)  Vaginal bleeding (2-23%)  Vaginal discharge (13-55%)  Vaginal dryness (< 1%)  Dyspareunia (3-5%)  Urinary frequency or urgency (10%)  Mood changes (12-18%)  Depression (2-12%)

Although many patients attribute postdiagnosis weight gain to tamoxifen, the literature suggests only a 5% increase in weight is associated with tamoxifen use. Aromatase Inhibitors:  Aromatase inhibitors (AIs) function by inhibiting aromatase, the enzyme (found in body fat, adrenal glands, and breast tissue, as well as tumor cells) responsible for converting other steroid hormones into estrogen. Aromatase is the sole source of estrogen in postmenopausal women and likely the underlying reason that obesity (larger volume of body fat produces more estrogen) has been associated with a higher risk of breast cancer in postmenopausal patients.  As the AIs have no effect on ovarian estrogen production, these agents are effective only in postmenopausal women. Common side effects of AIs include hot flashes , arthralgia/arthritis, headach, vaginal drynes, and mood changes. Advances in Targeted Therapy: The molecular era in cancer therapy has flourished and our knowledge of cancer biology has expanded. As increased numbers of targeted therapies show promise for the treatment of breast cancer, the goal is to optimize these expensive and toxic therapies with existing anticancer approaches. Novel antiangiogenic agents: Small-molecule vascular endothelial cell growth factor receptor (VEGFR) tyrosine kinase inhibitors (TKIs) offer several potential advantages over antibody therapies, including oral administration, a shorter half-life, and multitargeted effects. An open-label, multicenter, phase II study evaluated sunitinib (Sutent), an oral multi-targeted kinase inhibitor of VEGFR and platelet-derived growth factor (PDGF)receptor, as monotherapy in 64 metastatic breast cancer patients previously treated with an anthracycline and a taxane. Sunitinib has also demonstrated activity in combination with paclitaxel and sequentially with docetaxel as first-line therapy in metastatic breast cancer. Axitinib, another oral TKI of VEGFR, showed promising antitumor activity in combination with docetaxel in a randomized phase II trial of first-line chemotherapy in 168 patients with locally recurrent/metastatic breast cancer. The most common adverse events included diarrhea , nausea , fatigue, and stomatitis, with an increase in febrile neutropenia with the combination arm. Other VEGFR TKIs such as pazopanib, vatalanib, cediranib, and motesanib are under investigation. Dual blockade by antiangiogenic/HER2 agents HER2-targeted therapies have been investigated in combination with angiogenesis inhibitors, with promising results. HER2 overexpression is associated with an increase in VEGF levels in primary breast cancers. Combination therapy targeting HER2, EGFR, and VEGF pathways produces greater inhibition of human breast cancer cell lines than inhibition of any single or dual pathway. The angiogenesis inhibitor, pazopanib, showed promise in a randomized phase II study in combination with lapatinib versus lapatinib alone in chemotherapy-naive, HER2-positive metastatic breast cancer. Additional trials of lapatinib in combination with endocrine therapy or other targeted agents are ongoing. Insulinlike growth factor 1 receptor inhibitors Insulinlike growth factor 1 receptor (IGF-1R) overexpression is associated with the development – of breast cancer and correlates with disease-free survival in patients with primary breast cancer. IGF-1R-mediated growth– and antiapoptotic signaling occurs through the phosphatidylinositol 3- kinase (PI3K)/Akt and mitogen-activated protein kinase (MAPK) pathways. To date, drug development targeting the IGF pathway has focused on IGF-1R monoclonal antibodies (mAbs). Studies have shown that these compounds yield antitumor activity in several breast cancer models in vitro and in vivo. Breast cancer cell lines with IGF-1R overexpression appear to be more resistant to trastuzumab and ER-targeted therapies; thus, clinical trials are focusing on hormone-refractory and/or HER2- positive disease. This strategy of dual targeting is being employed in an ongoing phase II trial of CP- 751,871, an mAb targeting IGF-1R, in combination with the AI exemestane, compared with exemestane alone in postmenopausal patients with hormone receptor positive advanced breast cancer. – PI3K/AKT/mammalian target of rapamycin (mTor) inhibitors Among signaling pathways, the PI3K/Akt/mTor pathway is thought to be highly active in human breast cancer development and progression. In breast cancer, this pathway can be activated through PI3K by membrane protein receptors, including the HER family of growth factor receptors, IGF-1R and ER. PI3K activates downstream Akt, leading to mTor phosphorylation and promotion of breast cancer cell survival, as well as resistance to chemotherapy and targeted agents such as trastuzumab and tamoxifen. Constitutively active mutations of PI3K have been described in up to 40% of primary breast cancer tumors, implicating a role for PI3K in breast cancer tumorigenesis. SF1126 is a chemical conjugate of LY294002, the most studied of the PI3K inhibitors. This compound has a broad spectrum of inhibition via Akt and mTor resulting in antitumor and antiangiogenic activity. The water-soluble SF1126 has a good pharmacokinetic profile and is well tolerated in murine systems. Phase I trials are ongoing with this compound and other novel PI3K inhibitors. In preclinical breast cancer models, mTor inhibitors substantially inhibit tumor growth. In 109 heavily pretreated LABC or metastatic breast cancer patients, single-agent activity of the mTor inhibitor, temsirolimus (Torisel), yielded an overall response rate of 9%, consisting of 10 partial responses. Based on preclinical data showing a synergistic effect between mTor inhibitors and endocrine therapy, clinical trials have focused on combination therapy. Initial randomized trials investigating letrozole alone or in combination with temsirolimus showed no significant clinical benefit in LABC/metastatic breast cancer. However, a phase I study of letrozole in combination with daily everolimus (RAD001, Afinitor), another selective mTor inhibitor with a good safety profile, proved promising, with 1 complete response and 1 partial response in 18 patients with advanced breast cancer. The most common adverse events included stomatitis (50%), fatigue (44.4%), and diarrhea (38.9%). Heat shock protein 90 inhibitors Heat shock protein 90 (HSP90) is a molecular chaperone required for the stability and function of several expressed and/or activated signaling proteins. Direct inhibition of HSP90 inactivates, destabilizes, and degrades numerous chaperone-dependent client proteins, resulting in antitumor activity, antiangiogenesis, and apoptosis in cancer cells. A preclinical study reported that HER2 is particularly susceptible to degradation when exposed to the HSP90 inhibitor tanespimycin (KOS- 953). ARP inhibitors Polyadenosine diphosphate ribose polymerase (ADP) polymerase (PARP) inhibitors were initially developed to investigate the role of PARP-1, a nuclear enzyme involved in DNA repair. However, these agents have also proven to have antitumor activity in breast cancer cells. BRCA1- and BRCA2- deficient cell lines are defective in homologous recombination, thus relying on PARP-1 for DNA repair. BRCA-deficient cells treated with PARP inhibitors demonstrate an increase in DNA fragmentation and cell death. Farnesyltransferase inhibitors The Ras oncogene family is a key component of the MAPK signaling pathway, which promotes mitogenic activity. Aberrant Ras expression resulting in downstream signaling has been reported in breast cancer, although Ras mutations are rare in this malignancy. Farnesyltransferase inhibitors (FTIs) such as tipifarnib and lonafarnib were developed to block farnesylation and subsequent membrane localization of Ras. Early clinical data suggested that FTIs may modulate endocrine response. Src kinase inhibitors Dasatinib (Sprycel) is a novel oral kinase inhibitor that targets the Src family kinases and BCR-abl. Dasatinib is approved by the FDA for the treatment of chronic myelogenous leukemia (CML). A preclinical study revealed a dasatinib-sensitive signature in the basal breast cancer subtype. Additionally, preliminary results from a phase I study of gemcitabine/dasatinib demonstrated antitumor activity in a patient with IBC.

4. Answer briefly on the following: 4 x 71/2 (a) Subdural haematoma. (b) Cleft palate. (c) Hypospadias. (d) Surgery of varicose veins.

(a) Subdural haematoma. Answer. See the answer of 3(a) of Paper – III of 2007.

(b) Cleft palate.

Answer. Cleft palate is a condition in which the two plates of the skull that form the hard palate (roof of the mouth) are not completely joined. The soft palate is in these cases cleft as well. In most cases, cleft lip is also present. Cleft palate occurs in about one in 700 live births worldwide.

Palate cleft can occur as complete (soft and hard palate, possibly including a gap in the jaw) or incomplete (a 'hole' in the roof of the mouth, usually as a cleft soft palate). When cleft palate occurs, the uvula is usually split. It occurs due to the failure of fusion of the lateral palatine processes, the nasal septum, and/or the median palatine processes (formation of the secondary palate).

The hole in the roof of the mouth caused by a cleft connects the mouth directly to the nasal cavity. A result of an open connection between the oral cavity and nasal cavity is called velopharyngeal inadequacy (VPI). Because of the gap, air leaks into the nasal cavity resulting in a hypernasal voice resonance and nasal emissions while talking. Secondary effects of VPI include speech articulation errors (e.g., distortions, substitutions, and omissions) and compensatory misarticulations and mispronunciations (e.g., glottal stops and posterior nasal fricatives).[8] Possible treatment options include speech therapy, prosthetics, augmentation of the posterior pharyngeal wall, lengthening of the palate, and surgical procedures.

Submucous cleft palate (SMCP) can also occur, which is a cleft of the soft palate with a classic clinical triad of a bifid, or split, uvula which is found dangling in the back of the throat, a furrow along the midline of the soft palate, and a notch in the back margin of the hard palate.

Problems :

Psychosocial: o Most children who have their clefts repaired early enough are able to have a happy youth and social life. Having a cleft palate/lip does not inevitably lead to a psychosocial problem. However, adolescents with cleft palate/lip are at an elevated risk for developing psychosocial problems especially those relating to self concept, peer relationships and appearance. Adolescents may face psychosocial challenges but can find professional help if problems arise. A cleft palate/lip may impact an individual s self-esteem, social skills and behavior. o Research has shown that during the early preschool years (ages 3 5), children with cleft lip and or cleft palate tend to have’ a self-concept that is similar to their peers without a cleft. Children with clefts tend to report feelings of anger, sadness, fear, and alienation– from their peers, but these children were similar to their peers in regard to "how well they liked themselves." o A child who is entering school should learn the proper (and age-appropriate) terms related to the cleft. The ability to confidently explain the condition to others may limit feelings of awkwardness and embarrassment and reduce negative social experiences. o As children reach adolescence, the period of time between age 13 and 19, the dynamics of the parent-child relationship change as peer groups are now the focus of attention. An adolescent with cleft lip and or cleft palate will deal with the typical challenges faced by most of their peers including issues related to self esteem, dating and social acceptance. Adolescent boys typically deal with issues relating to withdrawal, attention, thought, and internalizing problems and may possibly develop anxiousness-depression and aggressive behaviors. Adolescent girls are more likely to develop problems relating to self concept and appearance. Individuals with cleft lip and or cleft palate often deal with threats to their quality of life for multiple reasons including: unsuccessful social relationships, deviance in social appearance and multiple surgeries.

Complications: A baby being fed using a customized bottle. The upright sitting position allows gravity to help the baby swallow the milk more easily.

Cleft may cause problems with feeding, ear disease, speech and socialization.

Due to lack of suction, an infant with a cleft may have trouble feeding. An infant with a cleft palate will have greater success feeding in a more upright position. Gravity will help prevent milk from coming through the baby's nose if he/she has cleft palate. Individuals with cleft also face many middle ear infections which can eventually lead to total hearing loss. The Eustachian tubes and external ear canals may be angled or tortuous, leading to food or other contamination of a part of the body that is normally self cleaning. Hearing is related to learning to speak. Babies with palatal clefts may have compromised hearing and therefore, if the baby cannot hear, it cannot try to mimic the sounds of speech. Thus, even before expressive language acquisition, the baby with the cleft palate is at risk for receptive language acquisition. Because the lips and palate are both used in pronunciation, individuals with cleft usually need the aid of a speech therapist.

Cause

(a) Cleft of soft palate and incomplete cleft of hard palate. (b) Muscles of the The three mucosal zones of the hard palate. 1, palatal soft palate: left, cleft palate; right, normal anatomy. A, tensor palati; B, fibromucosa; 2, maxillary fibromucosa; 3, gingival levator palati; C, palatopharyngeus; D, palatoglossus; E, musculus uvulae. fibromucosa.

The development of the face is coordinated by complex morphogenetic events and rapid proliferative expansion, and is thus highly susceptible to environmental and genetic factors, rationalising the high incidence of facial malformations. During the first six to eight weeks of pregnancy, the shape of the embryo's head is formed. Five primitive tissue lobes grow:

a) one from the top of the head down towards the future upper lip; (Frontonasal Prominence) b-c) two from the cheeks, which meet the first lobe to form the upper lip; (Maxillar Prominence) d-e) and just below, two additional lobes grow from each side, which form the chin and lower lip; (Mandibular Prominence)

Incomplete cleft palate Unilateral complete lip and palate Bilateral complete lip and palate

If these tissues fail to meet, a gap appears where the tissues should have joined (fused). This may happen in any single joining site, or simultaneously in several or all of them. The resulting birth defect reflects the locations and severity of individual fusion failures (e.g., from a small lip or palate fissure up to a completely malformed face).

The upper lip is formed earlier than the palate, from the first three lobes named a to c above. Formation of the palate is the last step in joining the five embryonic facial lobes, and involves the back portions of the lobes b and c. These back portions are called palatal shelves, which grow towards each other until they fuse in the middle. This process is very vulnerable to multiple toxic substances, environmental pollutants, and nutritional imbalance. The biologic mechanisms of mutual recognition of the two cabinets, and the way they are glued together, are quite complex and obscure despite intensive scientific research.

Genetics o Genetic factors contributing to cleft lip and cleft palate formation have been identified for some syndromic cases, but knowledge about genetic factors that contribute to the more common isolated cases of cleft lip/palate is still patchy. o Many clefts run in families, even though in some cases there does not seem to be an identifiable syndrome present, possibly because of the current incomplete genetic understanding of midfacial development.

Syndromes:

 The Van der Woude Syndrome is caused by a specific variation in the gene IRF6 that increases the occurrence of these deformities threefold.  Another syndrome, Siderius X-linked mental retardation, is caused by mutations in the PHF8 gene (OMIM 300263); in addition to cleft lip and/or palate, symptoms include facial dysmorphism and mild mental retardation.[27]

In some cases, cleft palate is caused by syndromes which also cause other problems.

 Stickler's Syndrome can cause cleft lip and palate, joint pain, and myopia.  Loeys-Dietz syndrome can cause cleft palate or bifid uvula, hypertelorism, and aortic aneurysm.  Hardikar syndrome can cause cleft lip and palate, Hydronephrosis, Intestinal obstruction and other symptoms.  Cleft lip/palate may be present in many different chromosome disorders including Patau Syndrome (trisomy 13).  Malpuech facial clefting syndrome  Hearing loss with craniofacial syndromes  Popliteal pterygium syndrome  Treacher Collins Syndrome

Specific genes: Many genes associated with syndromic cases of cleft lip/palate have been identified to contribute to the incidence of isolated cases of cleft lip/palate. This includes in particular sequence variants in the genes IRF6, PVRL1 and MSX1. The understanding of the genetic complexities involved in the morphogenesis of the midface, including molecular and cellular processes, has been greatly aided by research on animal models, including of the genes BMP4, SHH, SHOX2, FGF10 and MSX1.

Diagnosis

Traditionally, the diagnosis is made at the time of birth by physical examination. Recent advances in prenatal diagnosis have allowed obstetricians to diagnose facial clefts in utero.

Treatment o Often a cleft palate is temporarily covered by a palatal obturator (a prosthetic device made to fit the roof of the mouth covering the gap). o Cleft palate can also be corrected by surgery, usually performed between 6 and 12 months. Approximately 20 25% only require one palatal surgery to achieve a competent velopharyngeal valve capable of producing normal, non-hypernasal speech. However, combinations of surgical– methods and repeated surgeries are often necessary as the child grows. One of the new innovations of cleft lip and cleft palate repair is the Latham appliance. The Latham is surgically inserted by use of pins during the child's 4th or 5th month. o If the cleft extends into the maxillary alveolar ridge, the gap is usually corrected by filling the gap with bone tissue. The bone tissue can be acquired from the patients own chin, rib or hip.

Speech and hearing o A tympanostomy tube is often inserted into the eardrum to aerate the middle ear.[44] This is often beneficial for the hearing ability of the child. o Children with cleft palate typically have a variety of speech problems. Some speech problems result directly from anatomical differences such as velopharyngeal inadequacy. Velopharyngeal inadequacy refers to the inability of the soft palate to close the opening from the throat to the nasal cavity, which is necessary for many speech sounds, such as /p/, /b/, /t/, /d/, /s/, /z/, etc. This type of errors typically resolve after palate repair. o However, sometimes children with cleft palate also have speech errors which develop as the result of an attempt to compensate for the inability to produce the target phoneme. These are known as compensatory articulations. Compensatory articulations are usually sounds that are non-existent in normal English phonology, often do not resolve automatically after palatal repair, and make a child s speech even more difficult to understand. o Speech-language pathology can be very beneficial to help resolve speech problems associated with cleft palate. In addition,’ research has indicated that children who receive early language intervention are less likely to develop compensatory error patterns later.

Cleft palate surgery:

Cleft palate closure can be achieved by one- or two-stage palatoplasty. The surgical principle is mobilisation and reconstruction of the aberrant soft palate musculature together with closure of the residual hard palate cleft by minimal dissection and subsequent scar formation. Excess scar formation in the palate adversely affects growth and development of the maxilla. The philosophy of two-stage closure encourages a physiological narrowing of the hard palate cleft to minimise surgical dissection at the time of the second procedure.

(a and b) Method of repair of cleft palate. First-stage (a and b) Schematic representation of closure of the hard palate. Second-stage palatoplasty achieved with palatoplasty to reconstruct muscles of the soft palate. Red lines represent incisions, and orange areas raw two-layered closure. Red lines represent incisions, surfaces. and orange areas raw surfaces.

Timing of primary cleft lip and palate procedures (after Delaire)

Cleft lip alone

Unilateral (one side) - One operation at 5 6 months

Bilateral (both sides) - One operation at 4– 5 months

Cleft palate alone – Soft palate only - One operation at 6 months

Soft and hard palate - Two operations Soft palate at 6 months Hard palate at 15 18 months

Cleft lip and palate –

Unilateral - Two operations - Cleft lip and soft palate at 5 6 months Hard palate and gum pad with or without lip revision at 15 18 months Bilateral - Two operations - Cleft lip and soft palate at 4 –5 months Hard palate and gum pad with or without lip revision at 15–18 months – (c) Hypospadias. –

Answer. Definition: Hypospadias is a congenital deformity where the opening of the urethra (the meatus) occurs on the underside (ventral) part of the penis, anywhere from the glans to the perineum. It is often associated with a hooded foreskin and chordee (ventral curvature of the penile shaft). It occurs in 1 in 250 live male births. There is an 8% incidence in off-spring of an affected male, and a 14% risk in male siblings. Classification Hypospadias can be classified according to the anatomical location of the urethral meatus;

 Anterior (or distal): glandular, coronal, and subcoronal (~50%)  Middle: distal penile, midshaft, and proximal penile (~30%)  Posterior (or proximal): penoscrotal, scrotal, and perineal (~20%) The anatomical classification of hypospadias according to the location of the urethral meatus

Aetiology: Hypospadias results from incomplete closure of urethral folds on the underside of the penis during embryological development. This is related to a defect in production or metabolism of fetal androgens, or the number and sensitivity of androgen receptors in the tissues. Chordee are caused by abnormal urethral plate development, and the hooded foreskin is due to failed formation of the glandular urethra and fusion of the preputial folds (resulting in a lack of ventral foreskin but an excess of dorsal tissue).

Diagnosis: A full clinical examination will make the diagnosis. However, it is also important to seek out associated abnormalities which will need treatment (undescended testes, inguinal hernias, and hydroceles). Patients with absent testes and severe hypospadias should undergo chromosomal and endocrine investigation to exclude intersex conditions.

Treatment: Surgery is indicated where deformity is severe, interferes with voiding, or is predicted to interfere with sexual function. Surgery is now performed between 6 -12 months of age. Local application of testosterone for 1 month pre-operatively can help increase tissue size. Surgery aims to correct penile curvature (orthoplasty), reconstruct a new urethra, and bring the new meatus to the tip of the glans using urethroplasy, glanuloplasty, and meatoplasty techniques. Severe cases may require staged procedures. Common operations for anterior hypospadias include meatal advancement and glanuloplasty (MAGPI), meatal-based flaps (Mathieu procedure), and tubularization of the urethral plate. Posterior defects require free grafts (buccal mucosa), onlay grafts, and preputial transfer flaps.

Complications: Bleeding, infection, urethral strictures, meatal stenosis, urethrocutaneous fistula, urethral diverticulum, and failed procedures requiring re-operation.

(d) Surgery of varicose veins. Answer. The surgical treatment of varicose veins have been under development for more than 2000 years, but until the present era, relatively little weight was given to the cosmetic outcome of treatment. Current therapies are becoming less invasive with improved recovery, but long-term outcomes are uncertain. Therapies aim to remove the superficial venous system either through surgery, endovenous ablation, or sclerotherapy ablation. In 90% of cases where venous hypertension is from superficial and perforator vein reflux, removal or obliteration of the GSV alone can resolve the venous hypertension. However, in the remaining 10%, additional treatment to the incompetent perforator veins may be needed. Additionally, if severe deep venous incompetence exists, treatment of the GSV alone usually does not resolve the venous hypertension. In both these cases, additional interventions with subfascial endoscopic perforating vein surgery (SEPS), perforator vein ablation, and/or venous reconstruction can be attempted, but these details are not further discussed in this article.  Open techniques  The Rindfleisch-Friedel procedure of the early 1900s involved one incision to the level of the deep fascia that wrapped around the leg 6 times, creating a spiral gutter that brought into view a large number of superficial veins, each one of which was ligated. This wound was left open to heal by granulation. The Linton procedureused a large linear medial leg incision that brought into view all the superficial and perforator veins of the leg. Incompetent superficial veins were removed, and perforating veins were interrupted. Friedrich Trendelenburg, introduced a midthigh ligation of the GSV. The outcomes were variable, and this procedure was later modified by Perthes, who advocated a groin incision and a ligation of the GSV at the saphenofemoral junction. Later, even better outcomes were found if saphenectomy (removal of the GSV) with ligation at the SFJ was performed over ligation alone.  GSV saphenectomy Surgical removal of the GSV has evolved from large open incisions to less invasive stripping. Original methods of stripping used different devices and variations of techniques. The Mayo stripper was an extraluminal ring that cut the tributaries as it was passes along the vein. The Babcock device was an intraluminal stripper with an acorn-shaped head that pleated up the vein as it pulled the vessel loose from its attachments. The Keller device was an internal wire used to pull the vein through itself, as is done today with perforation-invagination (PIN) strippers. Currently, the technique of PIN stripping begins with a 2- to 3-cm incision made at the groin crease. The femoral vein and SFJ are exposed with dissection and all tributaries of the SFJ must be identified and flush-ligated to minimize the incidence of reflux recurrence. After ligation and division of the junction, the stripping instrument (usually a stiff but flexible length of wire or plastic) is passed into the GSV at the groin and threaded through the incompetent vein distally to the level of the upper calf. The stripper is brought out through a small incision (5 mm or smaller) approximately 1 cm from the tibial tuberosity at the knee. An inverting head is attached to the stripper at the groin and is secured to the proximal end of the vein. The vessel is then inverted into itself, tearing away from each tributary and perforator as the stripper is pulled downward through the leg and out through the incision in the upper calf, as depicted in the image below. If desired, a long epinephrine-soaked gauze or ligature may be secured to the stripper before invagination, allowing hemostatic packing to be pulled into place after stripping is complete.

Perforation-invagination (PIN) stripping schematic.

An older technique of stripping to the ankle (rather than to just the knee) has fallen into disfavor because of a high incidence of complications, including damage to the saphenous nerve, which is closely associated with the vein below the knee.  SSV saphenectomy Removal of the short saphenous vein is complicated by variable local anatomy and risk of injury to the popliteal vein and peroneal nerve. The saphenopopliteal junction must be located by duplex examination before beginning the dissection, and adequate direct visualization of the junction is essential. After ligation and division of the junction, the stripping instrument (often a more rigid stripper that facilitates navigation) is passed downward into the distal calf, where it is brought out through a small incision (2-4 mm). The stripper is secured to the proximal end of the vein, which is invaginated into itself as it is pulled downward from knee to ankle and withdrawn from below.  Stab phlebectomy (or ambulatory phlebectomy) This procedure is extremely useful for the treatment of residual vein clusters after saphenectomy and for removal of nontruncal tributaries when the saphenous vein is competent. A microincision is made over the vessel using a tiny blade or a large needle, a phlebectomy hook is introduced into the microincision, and the vein is delivered through the incision. With traction, as long a segment as possible is pulled out of the body until the vein breaks or cannot be pulled any further. Another microincision is made and the process is begun again and repeated along the entire length of the vein to be extracted. Short segments of veins can be removed through tiny incisions without ligatures, and skin closure is not necessary.  Cryosurgery: A cryoprobe is passed down the long saphenous vein following saphenofemoral ligation. Then the probe is cooled with NO2 or CO2 to a temperature of -85o. The vein freezes to the probe and can be retrogradely stripped after 5 sec of freezing. It is a variant of Stripping. The only point of this technique is to avoid a distal incision to remove the stripper.  SEPS (Sub-fascial Endoscopic Perforator Surgery):The role of the incompetent perforating vein in the varicose vein surgery is one of the more complex issues and many surgeons treating varicose veins do not understand either their role or their treatment. SEPS involved inserting a endoscope (a surgical "telescope") between the muscle and the layer of fat under the veins, allowing the surgeon to see the incompetent perforating veins. These could then be clipped or burnt in an attempt to close them.

THE WEST BENGAL UNIVERSITY OF HEALTH SCIENCES

MS (General Surgery) Examination, 2011

April 2011

PAPER III

Time Allowed: 3 Hours Full Marks: 100 Attempt all questions

1. Describe different surgical procedures for morbid obesity. 20

Answer. See the answer of question no. 1 of Paper – IV, 2010.

2. Discuss the current trend in management of Prostatic Carcinoma with special reference to robotic surgery. 20

Answer. Introduction: The diagnosis of prostate cancer is on the increase, probably as a result of increasing use of serum Prostatic Specific Antigen (PSA) testing for both symptomatic and asymptomatic men, and the use of more extensive prostatic biopsy protocols.

PSA and prostate cancer:PSA has revolutionized the diagnosis and management of prostate cancer, although its use in screening and early detection remains controversial. In addition to its use as a serum marker for the diagnosis of prostate cancer, PSA elevations may help in staging, counselling, and monitoring prostate cancer patients. PSA generally increases with advancing stage and tumour volume, although a small proportion of poorly differentiated tumours fail to express PSA.

 PSA is used, along with clinical (DRE),T stage and Gleason score, to predict pathological tumour staging and outcome after radical treatments using statistically derived nomograms and artificial neural networks.  >50% of patients have extra-prostatic disease if PSA >10ng/ml.  <5% of patients have lymph node metastases and only 1% have bone metastases if PSA <20ng/ml.  66% of patients have lymphatic involvement and 90% have seminal vesicle involvement if PSA >50ng/ml.  PSA should be virtually undetectable following radical prostatectomy for gland-confined disease.  PSA rise after radical prostatectomy precedes the development of clinical metastatic disease by a mean time of 8 years.  PSA falls to within the normal range in 80% of patients with metastatic disease on hormone therapy within 4 months; the PSA rises in a mean time of 18 months after starting hormone therapy, signalling progressing disease. In the presence of infection or instrumentation, PSA should not be requested until at least 28 days after the event, to avoid a false +ve result causing unnecessary concern to doctor and patient. Ideally, PSA should not be requested within 2 days of ejaculation or DRE, but in practice it makes negligible difference to the result.

PSA derivatives: free-to-total ratio, density, and velocity: . Measurement of the free-to-total (F:T) PSA ratio increases the specificity of total PSA because the ratio is lower in men with prostate cancer than in men with benign hyperplasia. This may be helpful in deciding whether to re-biopsy a patient with previous benign biopsies. . Consideration may be given to the prostate volume, since large benign prostates are the most common cause of mildly elevated PSA. Serum PSA/prostate volume = PSA density, and serum PSA/prostate transition-zone volume = PSA-TZ density. Various cut-off densities have been proposed to raise the specificity of total PSA, possibly to reduce the need for prostatic biopsy, but the issue remains controversial. . Short-term variations in serum PSA occur in the presence or absence of cancer, the cause of which may be technical or physiological. Longer term, the PSA tends to rise slowly due to BPH and faster due to prostate cancer PSA velocity. A PSA velocity >0.75ng/ml per year over at least 18 months in total PSA range 10ng/ml is suggestive of the presence of PC, given that only 5% of men without cancer exhibit such a velocity. A PSA velocity >20% per year should also prompt the recommendation of a biopsy, although a slower velocity does not exclude the presence of cancer. The use of PSA velocity is an option for the patient who wishes to avoid an initial or repeat prostatic biopsy.

PROSTATE CANCER: Transrectal Ultrasonography and Biopsies:

The most common diagnostic modality for prostate cancer is currently transrectal ultrasonography (TRUS) with guided biopsies.TRUS provides imaging of the prostate and seminal vesicles using a 7.5mHz biplane intra-rectal probe measuring approximately 1.5cm in diameter.

A DRE precedes insertion of the probe. If biopsies are planned, an antiseptic rectal wall cleansing is also undertaken. Broad-spectrum antimicrobials are given before and after the procedure.

Transrectal ultrasonography can image the outline of the prostate, cysts, abscesses, and calcifications within the prostate. Hypoechoic and hyperechoic lesions in the peripheral zone may be due to prostate cancer or inflammatory conditions, although most prostate cancers are isoechoic and are not seen.

Indications for transrectal ultrasonography alone

 Accurate measurement of prostate volume.  Male infertility with azospermia, to look for seminal vesicle and ejaculatory duct obstruction due to calculus or Müllerian cyst.  Suspected prostatic abscess (can be drained by needle aspiration).  Investigation of chronic pelvic pain, looking for prostatic cyst or calculi.

Indications for transrectal ultrasonography with biopsies

 An abnormal DRE and/or an elevated PSA (exceptions include very elderly men with massively elevated PSA and abnormal DRE, or those in whom a TURP is indicated for BOO with severe LUTS/retention where histology will be obtained).  Previous biopsies showing isolated PIN or ASAP.  Previous biopsies normal, but PSA rising or DRE abnormal.  To confirm viable prostate cancer following treatment if further treatment is being considered.

PROSTATE CANCER STAGING: Tumour staging uses the TNM classification.  T stage: is assessed by digital rectal examination and imaging (TRUS, MRI).  N stage: is assessed by imaging (MRI) or biopsy as necessary. Pelvic lymph node dissection is the gold-standard assessment of N stage. MRI or CT scanning may image enlarged nodes and most radiologists report nodes of >8mm in maximal diameter. However, nodes larger than this often contain no cancer, while micro-metastases may be present in normal-sized nodes.  M stage: is assessed by physical examination, imaging (MRI or isotope bone scan, chest radiology) and biochemical investigations (including creatinine and alkaline phosphatase).

Partin's nomograms:based on several thousand radical prostatectomies, are used widely to predict pathological T and N stage by combining clinical T stage, PSA, and biopsy Gleason score. Higher pathological stage (i.e. pT3 disease) found at radical prostatectomy may also be predicted by:

higher percentages (>66%) of positive biopsies cancer invading adipose in the biopsies (there is no fat in the prostate) possibly the presence of perineural cancer invasion within the prostate.

TNM (1992) staging of adenocarcinoma of the prostate

T0 No tumour (pT0 if no cancer found by histological examination) Tx T stage uncertain T1a Cancer non-palpable on digital rectal examination (DRE), present in <5% of TURP specimens (in up to 18% of TURPs) T1b Cancer non-palpable on DRE, present in >5% of TURP specimens T1c Cancer non-palpable on DRE, present in needle biopsy taken because of elevated PSA T2a Palpable tumour, feels confined, in half of one lobe on DRE T2c Palpable tumour, feels confined, in both lobes on DRE T3a Palpable tumour, locally advanced into periprostatic fat, uni- or bi-lateral and mobile on DRE T3b Palpable tumour, locally advanced into seminal vesicle(s) on DRE T4a Palpable tumour, locally advanced into adjacent structures, feels fixed on DRE T4b Palpable tumour, locally advanced into pelvic side-wall, feels fixed on DRE Nx Regional lymph not assessed N0 No regional lymph node metastasis N1 Tumour involves regional (pelvic) lymph nodes Mx Distant metastases not assessed M0 No distant metastasis M1a Non-regional lymph node metastasis M1b Tumour metastasis in bone M1c Tumour metastasis in other sites

Prostate cancer grading:  Adenocarcinoma of the prostate is graded using the Gleason system. Microscopically, adenocarcinoma is graded 1 to 5 according to its gland-forming differentiation at relatively low magnification. Since most are multifocal, an allowance is made by adding the two dominant grades to give a sum score between 2 and 10. If only one pattern is observed, the grade is simply doubled. The system is used with needle biopsies, TURP, and radical prostatectomy specimens.  Gleason scores 2 - 4 are considered well differentiated; 5 - 7 are moderately differentiated; and 8 - 10 are poorly differentiated.  The importance of the Gleason score is that it correlates well with prognosis, stage for stage, however the patient is managed. For example, a Gleason 3+3 = 6 adenocarcinoma carries a worse prognosis than a 3+2 = 5 cancer of equivalent stage. Moreover, cancers of the same Gleason score have a worse prognosis if the predominant grade is higher (for example, 4+3 = 7 is worse than 3+4 = 7). Some men with low-grade tumours develop high-grade tumours after several years. This is probably due to clonal expansion of high- grade cells rather than de-differentiation of low-grade tumour cells. In general, large- volume tumours are more likely to be high-grade than low-volume tumours, but occasionally exceptions are seen.  Finally, caution must be taken when Gleason-scoring prostate tissue that has been subjected to certain interventions, most notably hormone therapy. It is well recognized that prostate cancer treated with androgen ablation exhibits changes very similar to those seen in Gleason scores 8 - 10. It is possible that even treatment of BPH with 5-alpha- reductase inhibitors could adversely affect the Gleason score of cancer present in the gland. Pathologists are therefore keen to know relevant clinical details and are reluctant to provide Gleason scores for such patients.

Management of localized prostate cancer: watchful waiting:

The risks of developing metastatic disease and of death due to prostate cancer after 10 to15 years of can be considered using published data, according to biopsy grade.

Selection of patients for watchful waiting Watchful waiting is the best option for patients with localized prostate cancer and:

 Gleason score 2 - 4 disease (in which the results of the more aggressive treatments described below are no better); any age  Gleason score 5 and 6 disease; >75 years old  Significant comorbidity; life expectancy considered to be <10 years  Stage T1a disease with normal PSA (only 17% T1a will progress, compared to 68% with T1b)

However, WW should be considered and discussed with all who have Gleason score <7, when small-volume disease is predicted by DRE and the biopsy report. Watchful waiting protocols Most men with localized prostate cancer on WW are seen every 6 months for clinical history, examination (including a DRE), and a serum PSA test (before or after DRE). If the disease progresses during follow-up, palliative treatment (e.g. androgen ablation therapy) is recommended. The threshold for treatment was traditionally when symptoms and signs of advanced disease appeared (e.g. back pain and metastases on the bone scan). However, use of PSA, evidence of benefit with earlier use of hormone therapy, and involvement of patient choice have driven earlier thresholds for treatment. Hence, an asymptomatic patient with a rising PSA may choose whether to treat his disease and accept the side-effects, or whether to maintain his current quality of life while leaving the disease untreated.

Management of localized prostate cancer: radical prostatectomy: Radical (total) prostatectomy (RP) is excision of the entire prostate, including the prostatic urethra, with the seminal vesicles. It may be performed by open retropubic, perineal, or laparoscopic approaches. The perineal approach does not allow a simultaneous pelvic lymph node dissection. Following excision of the prostate, reconstruction of the bladder neck and vesico-urethral anastomosis completes the procedure. RP is indicated for the treatment of fit men with localized prostate cancer whose life expectancy exceeds 10 years, with curative intent. It is not considered to be an appropriate treatment for locally advanced disease. Patients with Gleason score 2 - 4 disease appear to do as well with WW as with any other treatment. The patient should consider all available treatment options and the complications of RP prior to proceeding. The surgeon should take part in multidisciplinary team discussion of each case; there may be local guidelines on age and upper PSA cut-off for offering RP, perhaps 70 years and 20ng/ml respectively.

Post-operative course after radical prostatectomy: complications  Day 1: mobilize; check FBC; C&E; transfuse if required; antimicrobials; physiotherapy if required.  Day 2: free fluids and diet; remove drains if possible; teach catheter care; encourage bowel.  Day 3 - 4: home with catheter and instructions.

Catheter time varies between 7 and 21 days; a cystogram is required only if there has been a documented urine leak or other catheter problem.

Complications of radical prostatectomy General complications (rare) Those of any major surgery: bleeding requiring re-operation and/or transfusion, infection, thromboembolism, and cardiac disturbance. These are minimized by attention to haemostasis, prophylactic antimicrobials, pneumatic calf compression, low-dose heparin post-operatively, and early mobilization. Chest infection may be prevented by physiotherapy and encouragement of deep breathing, especially in smokers. Post-operative death is estimated to occur in 1 in 500 cases. Specific complications early

 Per-operative obturator nerve, ureteric, or rectal injury (all rare): these should be managed immediately if recognized end-to-end nerve anastomosis; ureteric re- implantation; primary rectal closure with or without a loop colostomy.  Post-operative catheter displacement (rare): managed with careful replacement if within 48h, while later urethrography may reveal no anastomotic leak.  Post-operative urine or lymphatic leak (distinguished by dipstick glycosuria or creatinine concentration) through drains (occasional): managed by prolonged catheter and wound drainage; lymphatic leaks may require sclerotherapy with tetracycline.

Specific complications late

 Erectile dysfunction (ED) affects 70 - 90% of patients; spontaneous erections may return up to 3 years post-operatively. Men >65 years or with pre-existing ED are more likely to suffer long term. 40 - “70% respond to oral PDE5 inhibitors at 6 months, while others require intraurethral or intracavernosal prostaglandin E1 treatments, a vacuum device, or (rarely) a prosthesis.  Incontinence (stress-type) requiring >1 pad/day affects 5% of patients beyond 6 months; this is due to injury of the external urethral sphincter during division and haemostatic control of the dorsal vein complex. Predisposing factors include age >65 years and excessive bleeding. Pre-operative teaching of pelvic floor exercises helps to regain continence; periurethral bulking injections or implantation of an artificial urinary sphincter are occasionally necessary. Incontinence may also develop secondary to bladder neck stenosis or detrusor instability; flow rates, post-void residual measurement, urodynamics, and cystoscopy may help.  Bladder neck stenosis affects 5 - 8% of patients; typically occurs 2 - 6 months post- operatively, rarely becoming a recurrent problem. Predisposing factors include heavy bleeding, post-operative urinary leak, and previous TURP. Patients complain of new voiding difficulties and treatment is by endoscopic bladder neck incision.

Management of biochemical relapse post-RP:

The definition of rising PSA is controversial, though most agree >0.2ng/ml. DRE should be performed in case there is a nodule. Biopsy of the vesicourethral anastomosis is not widely practised unless there is a palpable abnormality. Studies have shown that MRI and bone scans are rarely helpful in searching for metastatic disease unless the PSA is >7ng/ml. Current management options include observation, pelvic radiotherapy, or hormone therapy. A good response to pelvic radiotherapy is likely if:

 the PSA rise is delayed >1 year  the PSA doubles in >10 months  the PSA is <1ng/ml  the disease was low-grade and low-stage  the radiation dose exceeds 64Gy

If the PSA never falls below 0.2, or it rises in the first year with a doubling time of less than 10 months, the response to pelvic radiotherapy is disappointing. It is likely in these circumstances that metastatic disease is present, and some form of hormone therapy is usually recommended.

The choice is between non-steroidal anti-androgen monotherapy: (e.g. bicalutamide 150mg daily) or androgen deprivation by bilateral orchidectomy or LH-RH analogues. There are no comparative outcomes data, so discussion focuses on the side-effects. Most patients choose the anti-androgen, wishing to preserve physical and sexual capabilities. Management of localized prostate cancer: radical external beam radiotherapy (EBRT) Since the early 1980s advances in radiotherapy for localized prostate cancer have included the advent of linear accelerators conformal and intensity-modulated techniques to minimize toxicity to the rectum and bladder. EBRT is administered with curative intent, often accompanied by 3 months of neoadjuvant hormone therapy in high-risk cases. A small, randomized study has demonstrated benefit in terms of progression and survival for patients treated with 6 months (2 months each of neoadjuvant, concurrent, and adjuvant) androgen ablation, in addition to radiotherapy, compared with radiotherapy alone.

Indication clinically localized prostate cancer, life expectancy >5 years. Patients with Gleason score 2 - 4 disease appear to do as well with WW as with any other treatment with 15-year follow-up. Contraindications

 Severe lower urinary tract symptoms  Inflammatory bowel disease  Previous pelvic irradiation

Protocol a 6-week course of daily treatments amounting to a dose of 60 - 72Gy. Side-effects

 Transient moderate/severe filling-type LUTS (common, rarely permanent)  Haematuria, contracted bladder  Moderate to severe gastrointestinal symptoms, bloody diarrhoea, pain, rectal stenosis  Erectile dysfunction (ED) gradually develops in 30 - 50%  The risk of a second solid pelvic malignancy is estimated to be 1 in 300, falling to 1 in 70 long-term survivors.

Outcomes of EBRT Definition of treatment failure: the ASTRO (American Society of Therapeutic Radiation Oncologists) definition is 3 consecutive PSA increases measured 4 months apart for 2 years, thereafter 6-monthly. Time to failure is midway through the 3 PSA measurements. Pre-treatment prognostic factors: PSA, Gleason score, clinical stage, percentage of positive biopsies.

Treatment of PSA relapse post-EBRT: Hormone therapy, either with anti-androgens or androgen deprivation, is currently the mainstay of treatment in this setting. However, local salvage treatment appears attractive, potentially offering another chance of cure if metastases cannot be demonstrated at repeat staging. Salvage radical prostatectomy is seldom undertaken because it is technically demanding, highly morbid, and outcomes are poor. Other local salvage treatments include cryotherapy and high-intensity focused ultrasound (HIFU), but outcomes data and access to these treatments are currently limited. Management of localized prostate cancer: brachytherapy (BT): This is ultrasound-guided transperineal implantation of radioactive seeds, usually I125, into the prostate. It is currently popular, having failed in the 1970s, prior to transrectal ultrasonography. BT is minimally invasive, requires general anaesthesia, and is completed in one or two stages. Either way, approximately 150Gy is delivered, and this may be augmented by an EBRT boost. Another approach is to use Iridium192 wires, left for several hours in situ in a series of applications, either before or after EBRT. The treatment is expensive due to the cost of the consumables. Indications for BT localized T1 - 2, Gleason <7, PSA <10ng/ml prostate cancer; life expectancy >5 years. Patients with Gleason score 2 - 4 disease appear to do as well with WW as with any other treatment. Indications for BT with EBRT T1 - 3, Gleason <8, PSA <20ng/ml prostate cancer. Contraindications to BT previous TURP (risk of incontinence); large volume prostate (>60ml) causes difficulty with seed placement; moderate to severe lower urinary tract symptoms (risk of retention). Complications

 Perineal haematoma (occasional)  Lower urinary tract symptoms (common), due to prostatic oedema post implant  Urinary retention  Incontinence (5%), if TURP is required to treat urinary retention  ED affects up to 50% of patients; gradual onset

LH-RH analogues are often used to reduce prostatic volume prior to treatment. Alpha-blockers are often used to treat LUTS and to improve the chance of successful trial without catheter in patients with urinary retention.

Management of localized and radio-recurrent prostate cancer: cryotherapy and HIFU: Two minimally invasive treatments for localized prostate cancer are in development. Proponents claim they are viable alternatives to radical surgery or radiotherapy, and that they are the only current options for salvage treatment of organ-confined recurrent disease following radical radiotherapy.

Cryotherapy Transperineal ultrasound-guided cryoprobes delivering argon or liquid nitrogen at a temperature of -20°C to -40°C. When applied in two cycles of freeze and thaw, cellular necrosis occurs. The diameter of the ice-ball is monitored using ultrasound; precautions must be taken to protect the urethra, external sphincter, and rectal wall, such as warming devices. An anaesthetic is required; this is a day-case procedure which can be repeated. Complications: ED ; incontinence ; LUTS due to urethral sloughing; pelvic pain; transient penile numbness; recto-urethral fistula (rare). In the salvage setting, good short-term PSA responses are reported in 66% of men, at the expense of significant morbidity, including incontinence and urinary retention (70% each). High-intensity focused ultrasound (HIFU) HIFU allows the selective destruction of tissues at depth without damaging intervening structures. Tissue is heated to the point of coagulative necrosis by high-energy ultrasound transmitted to the prostate using a transrectal device. The tissue temperature is raised locally at this point (over 85°C). With each firing of the probe a cigar shaped volume of damage is produced (a lesion). After one lesion is created, the focus is repositioned in order to create the next lesion with the same heating process. Lesions are placed side by side to create a continuous volume in which the tissue is necrosed. The rectal wall and the surrounding tissues are undamaged. An anaesthetic is required; this is a day-case procedure which can be repeated. Over 2000 patients have been treated to date in clinical trials of HIFU for the treatment of primary prostate cancer. The likelihood of morbidity is increased in the salvage treatment setting. Long- term results are awaited. Complications: ED; urinary retention; stress incontinence; recto-urethral fistula (rare).

Management of locally advanced non-metastatic prostate cancer (T3 - 4 N0M0): EBRT in combination with hormone therapy has consistently demonstrated better outcomes compared to EBRT alone, which is associated with a 15 - 30% 10-year survival. Hormone therapy alone is another option in elderly patients or those unwilling to consider radiotherapy. In this setting, a non-steroidal anti-androgen (e.g. bicalutamide 150mg) has equivalent efficacy to androgen deprivation by orchidectomy or LH-RH analogue, with potential advantages in terms of side-effects. However, discussion should include the point that hormone therapy is not a treatment offered with curative intent. A randomized trial of hormone therapy alone versus EBRT plus hormone therapy is underway. Watchful waiting is also an option for non-metastatic T3 disease in an elderly asymptomatic man who may wish to avoid side-effects of treatment. Palliative treatment of locally advanced disease Palliative TURP or medical therapy for LUTS or retention may be necessary. Incontinence can be due to sphincter involvement, though bladder outflow obstruction and instability should be considered: a urinary convene sheath or catheter may be required. Percutaneous nephrostomies or ureteric stents are occasionally necessary for ureteric obstruction. Rarely, a colostomy is necessary to bypass a rectal stenosis.

Management of advanced prostate cancer: hormone therapy I Metastatic disease is the cause of nearly all prostate cancer-related death. Currently incurable, 5- year survival is 25%; 10% survive <6 months, while <10% survive >10 years. The mainstay of treatment is hormone therapy, with cytotoxic chemotherapy in reserve and novel treatments such as growth factor inhibitors, angiogenesis inhibitors, immunotherapy, and gene therapy in development. Hormone dependence of prostate cancer 95% of circulating androgen, mainly testosterone, is produced by the Leydig cells of the testes under the influence of luteinizing hormone (LH). The anterior pituitary synthesises LH, stimulated by LH-releasing hormone (LH-RH) produced by the hypothalamus. The remaining 5% of circulating androgen is synthesized by the adrenal cortex from cholesterol, under the influence of pituitary ACTH. Testosterone is metabolized to the more potent dihydrotestosterone (DHT), by types 1 and 2,5 alpha reductase (5AR) enzymes. DHT binds to the androgen receptor, travels to the cell nucleus, and exerts its positive effect on cell growth and division. All prostate epithelial cells are dependent on androgens and fail to grow or undergo programmed cell death in their absence. Similarly, most previously untreated prostate cancer cells are dependent on androgens. Androgen deprivation results in a reduction in PSA and clinical improvement in >70% of patients. However, most will still die within 5 years due to the development of androgen- independent growth. This is considered to be due to growth of androgen-independent cell clones rather than a de-differentiation of previously androgen-dependent cells. The mean time to disease progression after androgen deprivation is 14 months in men with metastatic disease. Prognostic factors Predictors of poor hormone therapy response include:

 5 metastatic lesions at presentation  Elevated alkaline phosphatase at presentation  Anaemia at presentation  Poor performance status (level of activity) at presentation  Low serum testosterone at presentation  Failure of bone pain to improve within 3 months of treatment  Failure of PSA to normalize within 6 months of treatment (conversely a PSA nadir (= lowest value) of <0.1ng/ml predicts a long-term response) Management of advanced prostate cancer: hormone therapy II Mechanisms of androgen deprivation

 Surgical castration: bilateral orchidectomy  Medical castration: luteinizing hormone-releasing hormone (LH-RH) agonists, oestrogens; also termed androgen ablation or androgen deprivation  Anti-androgens (steroidal or non-steroidal): androgen receptor blockade at target cell  Maximal androgen blockade (MAB): medical or surgical castration plus anti-androgen  5alpha reductase inhibition (5ARI) with finasteride or dutasteride

Both forms of castration have equivalent efficacy, so patients should be given the choice. Oestrogens are no longer used first-line, due to the significant cardiovascular morbidity observed when they were the only alternative to orchidectomy. Anti-androgens alone are less effective in treating metastatic disease, but equivalent for non-metastatic disease. MAB has a theoretical advantage over castration in blocking the effects of the adrenal androgens, but significant clinical advantages have not been demonstrated in trial meta-analyses. 5ARIs are not licenced for the treatment of prostate cancer, but appear to have a role in prevention. Bilateral orchidectomy A simple procedure, usually carried out under general anaesthesia. Through a midline scrotal incision, both testes may be accessed. The tunica albuginea of each testis is incised and the soft tissue content is removed, after which the capsule is closed. The epidiymes and testicular appendages are preserved. Post-operative complications include scrotal haematoma or infection (both rare). Serum testosterone falls within 8h to <0.2nmol/l. LH-RH agonists Developed in the 1980s, giving patients an alternative to bilateral orchidectomy, with which they are clinically equivalent. They are given by subcutaneous or intramuscular injection, as monthly or 3-monthly depots. Examples include goserelin, triptorelin, and leuprorelin acetates. If the anterior pituitary is overwhelmed with an analogue of LH-releasing hormone (LH-RH), it switches off LH production, although serum testosterone rises in the first 14 days due to a surge of LH. This can result in tumour flare, manifest in 20% patients with increased symptoms, including catastrophic spinal cord compression. To prevent this, cover with anti-androgens is recommended for a week before and two weeks after the first dose of LH-RH agonist. An LH-RH antagonist is in development which should rapidly reduce serum testosterone.

Side-effects of bilateral orchidectomy and LH-RH agonists

 Loss of sexual interest (libido) and ED  Hot flushes and sweats can be frequent and troublesome during work or social activity  Weight gain  Gynaecomastia  Anaemia  Cognitive (mood) changes  Osteoporosis and pathological fracture secondary to osteoporosis may occur in patients on long-term (>5 years) treatment Anti-androgens These are administered as tablets. Examples include bicalutamide (150mg daily as monotherapy; 50mg daily for MAB, in combination with LH-RH analogues or orchidectomy), flutamide, and cyproterone acetate. The first two raise the serum testosterone slightly, so sexual interest and performance should be maintained, although many such patients have pre-existing ED due to the advancing disease. Side-effects include frequent gynaecomastia, breast tenderness, and occasional liver dysfunction; flutamide also causes frequent GI upset. At its full dose of 100mg tds cyproterone acetate may cause reversible dyspnoea; it may be used at 50mg bd for treatment of castration-induced hot flushes.

Management of advanced prostate cancer: hormone therapy III Monitoring treatment Typically, patients will have baseline PSA, full blood count, renal and liver function tests, a renal ultrasound, and a bone scan. The PSA is repeated after 3 months, 6 months, and 6-monthly thereafter until it rises. Liver function is checked 3-monthly if anti-androgen monotherapy is used. Renal function should be checked on disease progression, and bone imaging if clinically indicated. While PSA is very useful as a marker for response and progression, 15% of patients show clinical progression without PSA rise. This may occur in anaplastic tumours that fail to express PSA. Advice on exercise, diet, and treatment of erectile dysfunction is often sought by patients during treatment. Early versus delayed hormone therapy Traditionally, hormone therapy was reserved for patients with symptomatic metastatic disease. Arguments against early hormone therapy revolve around its side-effects and cost. However, studies of patients with locally advanced and metastatic disease have demonstrated slower disease progression and reduced morbidity when treated with androgen deprivation early (i.e. before the onset of symptoms). Improved survival has also been reported in patients without bone metastases but including node-positive disease, when treated immediately. Trials have also demonstrated slower disease progression in patients given bicalutamide 150mg daily (compared with placebo) for 2 years after treatment of high-risk, clinically localized prostate cancer with RP or RT. This benefit is not seen in patients managed by watchful waiting. Intermittent hormone therapy The potential advantages of stopping hormone therapy when the disease has remitted, then re- starting it when the PSA has risen again are the reduced side-effects and cost. However, there are no randomized trials yet demonstrating survival equivalence or advantage. None of the LH-RH analogues or anti-androgens are licensed for intermittent therapy. Moreover, it can take up to 6 months after stopping treatment for the serum testosterone to recover, hence side-effects may persist into the off-treatment periods.

Management of advanced prostate cancer: androgen-independent disease Second-line hormone therapy When the PSA rises from its lowest (nadir) value, or if symptomatic progression occurs despite a favourable biochemical response to first-line hormone therapy, the disease has entered its androgen-independent phase. In these circumstances, further treatment is usually considered. Most patients receiving anti-androgen monotherapy respond after switching to androgen ablation (orchidectomy or LH-RH analogue). If there is relapse during androgen ablation, 25% respond by adding an anti-androgen (e.g. bicalutamide 50mg daily) to establish maximal androgen blockade (MAB). If MAB was used from initiation of hormone therapy, withdrawal of the anti- androgen paradoxically elicits a favourable response in 25% of patients. A further rise in PSA may require third-line hormonal therapy such as the addition of oestrogens or corticosteroids. For example, diethylstilboestrol 1mg daily with 75mg aspirin for thromboembolic prophylaxis elicits a response in up to 60% of these patients. The mean duration of response is 4 months.

Cytotoxic chemotherapy Systemic chemotherapy is offered to appropriate patients with androgen-independent metastatic disease, by the medical oncologist. Men with low-volume disease who have failed radical local treatment and hormone therapy are also candidates for chemotherapy. Elderly, frail, and infirm patients with significant bone disease, renal impairment, haematological and clotting abnormalities are unsuitable. Correction of renal and bone marrow dysfunction is necessary prior to treatment. Symptom palliation Symptom improvements are reported with cytotoxic chemotherapy. In a randomized trial of mitoxantrone plus prednisolone versus prednisolone alone, 29% in the combination group experienced a reduction in pain and analgesic use compared with 12% in the prednisolone alone group. PSA response did not predict palliative response. In another study, docetaxel plus prednisolone produced a pain reduction in 35% compared to 22% of patients given mitoxantrone and prednisolone, resulting in improved quality of life scores.

Cancer control Ureteric obstruction:There are several reports of single-agent chemotherapy in the PSA era, most defining response as >50% decrease in PSA.

Palliative management of prostate cancer The involvement of the acute pain team, palliative care physicians, and nurses is often necessary in the terminal phase of the illness, to optimize quality of life. Pain is undoubtedly the most debilitating symptom of advanced prostate cancer. The pathogenesis of this pain is poorly understood, but there is known to be increased osteoclastic and osteoblastic activity. Spinal cord compression Lower urinary tract symptoms/urinary retention A TURP may be required for bladder outflow obstruction (BOO) or retention. Instrumentation can be difficult if there is a bulky fixed prostate cancer. The bladder may be contracted due to disease involvement, causing misery even after relief of BOO. This may perhaps respond to anticholinergic therapy. A long-term urethral or suprapubic catheter may be required for difficult voiding symptoms or recurrent retention. BOO This is a uro-oncological emergency. Locally advanced prostate cancer and bladder cancer may cause bilateral ureteric obstruction. The patient presents either with symptoms and signs of renal failure, or anuric without a palpable bladder. Renal ultrasound will demonstrate bilateral hydronephrosis and an empty bladder. After treating any life-threatening hyperkalaemia, the treatment options include bilateral percutaneous nephrostomies or ureteric stents. A clotting screen is required prior to nephrostomy insertion. Insertion of retrograde ureteric stents in this scenario is usually unsuccessful because tumour on the trigone obscures the location of the ureteric orifices. Antegrade ureteric stenting following placement of nephrostomies is usually successful. Hormone therapy should be commenced if not previously used; even in patients with androgen- independent disease, there are reports of high-dose intravenous oestrogens (Honvan) unobstructing ureters. Unilateral ureteric obstruction is occasionally observed at presentation or on progression. If asymptomatic, this may be managed conservatively provided there is a normal contralateral kidney. Anaemia, thrombocytopaenia, and coagulopathy For some patients, their haemoglobin levels drop rapidly and they become symptomatic on a regular basis. This tends to be normochromic and normocytic, and often occurs without other symptoms and with normal renal function. Such patients require regular transfusions. Platelet transfusions are rarely required for bleeding. Terminal patients may develop a clinical picture similar to disseminated intravascular coagulation (DIC) leading to problematic haematuria. Prostate cancer: prevention; complementary and alternative therapies.

Robotic radical prostatectomy is the most preferred way of treating localized prostate cancer. Robotic prostatectomy is performed with the help of a Surgical Robot (da Vinci) through 4 to 5 small 1-cm keyhole incisions across the abdomen. Through these small incisions, the robotic arms enter the abdomen. The operating surgeon sitting at the operating console controls the robotic instruments to precisely dissect the prostate gland, seminal vesicles, and vasa deferentia from the urethra and bladder.

Once the prostate gland is dissected free from the bladder, rectum, and urethra, it is placed in a small plastic bag and eventually removed intact by extending one of the keyhole incisions. The

complete prostate along with the lymph nodes are then sent to the histopathologist for detailed examination. The bladder is sewn back to the urethra to restore continuity of the urinary tract using robotic instruments and absorbable suturing material. A Foley catheter is placed through the penis to drain the bladder and allow healing of the bladder- urethra connection. In addition, a small drain is placed around the surgical site, exiting one of the keyhole incisions. Robotic surgery is performed adhering to the same anatomic principles of open surgery, but without the surgeon’s hands entering into the patient’s body cavity. Specially designed endowrist instruments allow seven degrees of motion that mimic the dexterity of the human hand and wrist. The result is a better anastomosis when the bladder and the urethra are sewn together after removal of the prostate. Thus the quality of open surgery is achieved without giving a big surgical incision and its associated complications. In fact, during robotic surgery, a high- resolution 3-D vision along with 10-12 folds magnification provides true-to-life view of the surgical field and gives the surgeon an excellent visualization and details of the prostate gland and the surrounding neurovascular structures, allowing for precise dissection of the prostate and suturing of blood vessels. The average length of surgery is 2.5 – 3 hours but it varies from patient to patient (2.5 - 4,5 hours) depending on the size of the prostate gland, shape of the pelvis, weight of the patient, and presence of scarring or inflammation within the pelvis due to infection or prior abdominal/pelvic surgery.

Blood loss during robotic prostatectomy is routinely less than 300 cc. Transfusions are rarely required. Donation of blood prior to surgery for autologous blood transfusion can be arranged if the patient desires, but is not recommended.

Pain Another advantage of robotic surgery is much less pain compared to open surgery. Immediate post surgery pain medication can be controlled and delivered by the patient via an intravenous patient-controlled analgesia (PCA) pump or by injections by the nursing staff. Minor transient shoulder pain may be experience related to carbon dioxide gas used to inflate the abdomen during surgery.

Bladder spasms are experienced after prostatectomy as a cramping sensation in the lower abdomen. These spasms are transient and decrease over time. If severe, medication can be prescribed by your doctor to decrease such episodes.

Gas Cramps You may experience sluggish bowels and gas cramps for initial 2-3 days after surgery. Pain medications and effect of anesthetic drugs also prolong the recovery of bowel functions. Getting out of bed, walking, more fluids and stool softeners help in the recovery. Hospital Stay Typically a patient stays for 1-2 nights in the hospital.

Back to normal activities One of several advantages of robotic surgery is earlier return to normal activity. Patient is expected to get out of bed and begin walking with the supervision and help of nurse on the evening of surgery. Gradually the physical activity is increased. Walking is strongly advised. In fact, prolonged sitting or lying in bed should be avoided to decrease the risk of forming blood clots in legs.

Driving should be avoided for 2 weeks after surgery. Most patients return to full activity at an average of 3-4 weeks after surgery.

Lifting heavy weights, strenuous exercises (jogging, biking, contact sports etc) should be deferred for six weeks or until instruction by doctor.

Diet & Bowel functions Oral liquids are allowed once you recover from the effect of anesthetic drugs. It is advised to take liquids and avoid high fiber diet in the beginning (till you have a normal bowel movement). It may take 2-3 days for the intestines to recover from the surgery and anesthesia. Pain medication, decreased physical activity and dehydration may cause constipation. Therefore, pain medication should be discontinued as soon as possible. Along with lots of fluids and mild laxatives or stool softeners may help in avoiding constipation. Once you have normal bowel movements, you may return to your normal diet. Follow up Appointment Patient will be discharged with a urinary catheter for draining the bladder and supporting the anastomosis (where the bladder and urethra is reconnected). The catheter is typically removed after 7-10 days following surgery.

Pathology results The result of the examination of the radical prostatectomy specimen is usually available approximately 7 days following surgery. The results will be discussed with you in the office along with the implications of the findings on future treatment and follow up.

Need for further treatment The decision regarding the need for any additional treatment such as radiation or hormonal therapy is based upon the pathologic stage of the cancer found on radical prostatectomy specimen as well as the trend of PSA values following surgery.

Long-term follow up First PSA test is drawn 6 weeks after surgery. Subsequently PSA is evaluated every 3 months.

Urinary incontinence Most men will experience some degree of stress urinary incontinence (small urine leak with coughing and sneezing etc). This generally improves with time. Most men experience improvement within 3-6 months. Approximately 70% of the patients are dry at 6 months and more than 90% at 12 months. Pelvic floor muscle exercise (kegel exercise) may help in faster recovery of bladder control.

Avoid excessive caffeine, alcohol or fluid intake during the recovery period as this will exacerbate urinary leakage.

Erectile function The return of erectile function following surgery depends upon age of patient, preoperative potency, nerve sparing surgery. Whether one or both side’s erectile nerve bundles can be spared depends upon the tumor volume, Gleason score and proximity of the tumor to prostate capsule. Discuss the issues related to return of erectile function with your doctor.

Robotic prostatectomy is a radical prostatectomy that is also known as Robotic Assisted Laparoscopic Prostatectomy (RALP). It is a minimally invasive prostate removal and cancer treatment surgery. Dr. Samadi developed his signature SMART (Samadi Modified Advanced Robotic Technique) Surgery, which incorporates oncologic principles, combines it with open radical prostatectomy skills and builds upon it with a robotic approach. SMART is a revolutionary and advanced prostate cancer surgery and prostate cancer treatment method, which drastically reduces blood loss, pain, hospital stay, recovery time and the side effects of other types of prostate cancer treatments. The SMART Technique is also known as bloodless prostate surgery. The SMART Technique does not require opening the endopelvic fascia or cutting the dorsal vein complex. This results in no sutures and less damage to the neurovascular bundle.

3. Write short notes of the following: 5 x 6 a) D2 gastrectomy. b) Partial amputation of penis - steps. c) Day care surgery. d) Surgical audit. e) M.R.I in cancer management.

a) D2 gastrectomy. Answer. In D2 Gastrectomy level(tier) N2 i.e distant perigastric node and nodes along the main arteries supplying stomach (hepatic and splenic arteries) are removed. o D2 lymphadenectomy can be performed with low morbidity-mortality, and a 5-year survival of more than 50%. The procedure offers benefit in terms of survival for a certain percentage of patients with positive level N2 lymph nodes. o The rationale for extended lymphadenectomy has been enhanced local control. The clearance of the possible metastatic nodes in the region outside of the perigastric nodes is presumed to impact on overall patient survival rates. The proponents of systemic LND point to numerous anatomical/ lymphographic studies, histopathologic analyses, statistical analyses of LN metastases, clinical prognostic observations, and advances in surgical techniques to support their argument that ELND is the most effective procedure to treat the lymphatic spread to GC. Steps:  Abdomen is opened from the Bilateral Subcostal incision or Midline Incision.  Omni retractor, Thomson's or similar retractor is used to lift up the rib cage. There should be good exposure of the stomach, lesser omentum, greater omentum and duodenum  Abdominal cavity is explored for metastasis especially liver, pouch of douglous, ovaries, base of mesentary, mesocolon, pelvis.  Note the site of tumor, extent and depth of the malignancy. Is there involvemnt of the adjacent organs  Bring down the hepatic flexure of colon and do a wide Kocher's maneuvre .  Omentum is now taken down from the transverse colon with the anterior layer of mesocolon. The dissection goes toward the right towards Right gastroepiploic artery and sweeping the sub pyloric group of lymph nodes up.  The hepatoduodenal ligament is cleared preserving the accessory left hepatic artery if present. Dissection in the porta might be done but not required because for D2 gastrectomy the other lymph nodes that need removal are Hepatic artery, left gastric artery, splenic artery and coeliac artery group.  Right gastric artery is ligated and duodenum cut and stapled and owersewn.  Transected part of stomach and duodenum is now lifted up and tissue inferior to the common hepatic artery with the pancreatic capsule is taken off. Coeliac trunk will now need to be exposed.  Splenic hilum will now be approached from below and all the lymphatic tissue will now need to be swept towards the Left gastric artery. o Many surgeons in the West argue against the therapeutic value of D2 dissection and are clearly against the routine use of D2 dissection for Western patients. In their opinion, D2 dissection increases postoperative morbidity and mortality rates and does not improve long-term survival. This argument has been based on retrospective studies, which have failed to demonstrate any survival benefit in favour of D2 dissection. o In the Dutch trial, D2 dissection did not improve longterm survival or decrease the risk of relapse. However, the D1 and D2 groups were not well balanced. Resection of the spleen was an independent risk factor for reduced survival but splenectomy and pancreatectomy was significantly more often performed in the D2 than the D1 group.

Contraindications of proceeding with a D2 gastrectomy: . Liver Secondaries . Serosal Deposits . Positive cancer cells in peritoneal fluid . Lymph nodes in para aortic region or base of mesocolon

b) Partial amputation of penis - steps.

Answer. This operation is indicated when the growth is confined to the glans and prepuce only and has not spread to the body of the penis.

 Spinal anaesthesia.  Patient is in supine position.  ASD.  A fine catheter is applied at the base of the penis as a tourniquet.  A long ventral flap is made whose breadth is equal to the half of the circumference of the penis and the length is equal to the diameter of the penis.  A short flap, 1.25 cm long is made on the dorsal aspect. The flaps are dissected back to their bases. The corpus spongiosum is isolated from the corpora cavernosa by inserting a fine scalpel on either side of the corpus spongiosum and divided1.25 cm distal to the proposed level of section of the corpora cavernosa.  The corpora cavernosa are now divided at the level of the flaps. The tourniquet is now removed and bleeding points are secured.  A small opening is made on the ventral flap and the corpus spongiosum is brought out through the opening.  The two flaps are now sutured together at the base of the penis.  The sutures should be well spaced for adequate drainage of the haematoma,which may be formed beneath the flap.  The end of the emerging urethra is split for a distance of 1 cm and each half is sutured to the skin of the flap. This is done to prevent subsequent stricture formation. A self – retaining catheter is then introduced.

c) Day care surgery. Answer. See the answer of question no. 4(a) of Paper IV, 2008.

d) Surgical audit. Answer. Clinical audit is a process used by clinicians who seek to improve patient care. The process involves comparing aspects of care(structure, process and outcome) against explicit criteria. Keeping track of personal outcome data and contributing to clinical database ensures that a surgeon’s own performance is monitored continuously and can be compared with a national data set to ensure compliance with agreed standards. If the care falls short of the criteria chosen, some change in the way that care is organised should be proposed.This change may be required at one of many levels. It might be an individual who needs training or an instrument that needs replacing. At times, the change may need to take place at the team level. Sometimes, the only appropriate action is change at an institutional level (e.g.a new antibiotic policy), regional level (provision of a tertiary referral centre) or, indeed, national level (screening programmes and health education campaigns). Audits are formal processes that require a structure. The following steps are essential to establish an audit cycle:  Define the audit question in a multidisciplinary team.  Identify the body of evidence and current standards.  Design the audit to measure performance against agreed standards based on strong evidence. Seek appropriate advice (local audit department in UK).  Measure over an agreed interval.  Analyse results and compare performance against agreed standards.

Research Clinical audit Service evaluation The attempt to derive generalisable Designed and conducted to Designed and conducted new knowledge including studies that produce best care information solely to define or judge aim to generate hypotheses as well as to inform the delivery of best current care. studies that aim to test them. care. Quantitative research – designed to test Designed to answer the Designed to answer the a hypothesis question: ‘Does this service question: ‘What standard Qualitative research – reach a predetermined does this service achieve?’ identifies/explores themes following standard?’ established methodology Addresses clearly defined questions, Measures against a standard Measures current service aims and objectives without reference to a standard Quantitative research – may involve Involves an intervention in use Involves an intervention in evaluating or comparing interventions, ONLY. (The choice of use ONLY. (The choice of particularly new ones. treatment is that of theclinician treatment is that of the Qualitative research – usually involves and patient according to clinician and patient studying how interventions and guidance, professional according to guidance, relationships are experienced standards and/or patient professional standards and/or preference) patientpreference) Usually involves collecting data that Usually involves analysis of Usually involves analysis of are additional to those for routine care existing data but may include existing data but may include but may include data collected administration of simple administration of simple routinely. May involve treatments, interviews or questionnaires. interviews or questionnaires. samples or investigations additional to routine care Quantitative research – study design No allocation to intervention No allocation to intervention may involve allocating patients to groups: the health-care groups: the health-care intervention groups. professional and patient have professional and patient have Qualitative research uses a chosen intervention before chosen intervention before clearlydefined sampling framework clinical audit clinical service evaluation underpinned by conceptual or theoretical justifications.

May involve randomisation No randomisation No randomisation

e) M.R.I in cancer management. Answer. Magnetic resonance imaging (MRI), nuclear magnetic resonance imaging (NMRI), or magnetic resonance tomography (MRT) is a technique used in radiology to visualize detailed internal structures. MRI makes use of the property of nuclear magnetic resonance (NMR) to image nuclei of atoms inside the body. Magnetic resonance imaging (MRI) is highly sensitive for cancer staging, problem-solving, post treatment surveillance, and other indications. I. It can detect primary breast cancers and additional foci of cancer that are occult to standard imaging. MRI has been shown to be useful in:

. Staging biopsy-proven primary breast carcinoma

. Detecting an occult primary breast cancer in a patient with proven axillary node involvement but negative results on mammography and ultrasonography

. Ascertaining the extent of disease after lumpectomy with positive margins or close margins

. Investigating suspected pectoralis muscle invasion

. Assessing response to chemotherapy, including preoperative chemotherapy

. Looking for suspected recurrent disease, such as in a postsurgical scar

. A compelling clinical presentation with negative or equivocal imaging results

. Problem solving, ie, workup of uncertain imaging findings that could not be resolved even after special mammographic and ultrasonographic techniques were used

. Needle localization and guided biopsy

. Known or suspected rupture of breast implants

Screening patients with certain well-defined risk factors for breast cancer.

II. MRI in hepatic imaging:

The role of MRI in liver imaging has advanced greatly with the development of faster sequences incorporating breathold techniques, which significantly reduce motion artefact. This allows information on the vascularity of liver lesions to be obtained of similar quality to dual phase contrast enhanced spiral CT. Recent development of specific MRI hepatic contrast agents has shown great potential to improve the accuracy and sensitivity of hepatic imaging.

In an assessment of liver MR imaging in oncology, several distinct groups of patients must be considered. The first group is those suspected of having a primary hepatic neoplasm, such as hepatocellular carcinoma (HCC) or a . Studies have shown little significant difference in the detection or staging of these tumours between CT and MRI, although the present perception is that dynamic contrast-enhanced MRI is marginally superior. Combination of CT and ultrasound will characterise a majority of these lesions as either cysts or haemangioma, it can be extremely difficult to differentiate small metastases from these lesions or from areas of focal fatty infiltration.

In areas of suspected focal fatty infiltration, specific opposed phase and in-phase sequences can demonstrate the presence of focal fatty change and aid in the differentiation from neoplasms. In patients with equivocal hepatic lesions, we advocate MRI for further evaluation and characterisation prior to biopsy.

New hepatic specific contrast agents aimed at improving the accuracy and sensitivity of hepatic MR have shown a significant improvement in lesion detectability. Two specific groups have been developed: reticuloendothelial agents; and hepatocellular agents. Reticuloendothelial agents consist of superparamagnetic iron oxide particles (SPIO) or ultra small superparamagnetic iron oxide particles (USPIO). Using state of the art equipment, these agents have begun to challenge the role of CTAP in this patient group in their ability to detect small metastatic liver lesions. In addition, at the time of the MRI examination, gadoliniu- enhanced gradient echo sequences in the oblique coronal plane and axial plane can elegantly demonstrate the vascular anatomy of the liver to allow surgical planning.

III. MRI in tumour angiogenesis

Angiogenesis is defined as the formation of new blood vessels through the development of capillaries from pre-existing micro-vessels. This mechanism is normally strictly regulated, but neoangiogenesis can play a key role in the development of certain diseases. In tumours, neoangiogenesis has been shown to occur when a tumour reaches 2mm in diameter. The new vessels that are formed encourage tumour growth by increasing the local delivery of oxygen and nutrients to the lesion. These new vessels form rapidly and lack normal structure with no smooth muscle, and with larger than normal gaps between the endothelial cells. This results in rather a fragile and leaky vascular network throughout the tumour, which allows rapid movement of small molecules from the blood stream into the extra-cellular space surrounding the tumour.

In addition, the presence of these new vessels within tumours is one of the factors, which enable abnormal cells to enter the circulation, thus encouraging metastatic spread.

IV. Lymph node evaluation: Overall, the difference in the accuracy, sensitivity and specificity between CT and MRI in the detection of metastatic lymphadenopathy is minimal. Recent studies have demonstrated the potential of two specific applications of MRI to improve our detection of metastatic nodes. Dynamic contrast enhanced imaging using gadolinium-DTPA has demonstrated a rapid and increased

V. MRI in suspected recurrent disease

The diagnosis of relapse is usually due to a combination of clinical findings, tumour markers and appropriate imaging. There are specific situations where MRI may be helpful. The distinction between post-operative fibrosis/scar tissue and recurrent tumour can be difficult. MRI using fast dynamic gadolinium-enhanced sequences and signal intensity curves can be used in these clinical settings. This technique has been useful in the assessment of pre-sacral masses in patients following surgery for colorectal carcinoma.

VI. With the advent of powerful gradient coil systems and high-resolution surface coils, magnetic resonance imaging (MRI) has recently extended its role in the staging of rectal cancer. MRI is superior to endorectal ultrasound, the most widely used staging modality in patients with rectal tumors, in that it visualizes not only the intestinal wall but also the surrounding pelvic anatomy. The crucial advantage of MRI is not that it enables exact T- staging but precise evaluation of the topographic relationship of a tumor to the mesorectal fascia. MRI is currently the only imaging modality that is highly accurate in predicting whether or not it is likely that a tumor-free margin can be achieved and thus provides important information for planning of an effective therapeutic strategy, especially in patients with advanced rectal cancer. VII. The primary indication for MRI of the prostate is the evaluation of prostate cancer. The test is commonly used after a prostate biopsy has confirmed cancer in order to determine if the cancer is confined to the prostate, or if it has spread outside of the prostate gland. VIII. Magnetic resonance imaging (MRI) uses a powerful magnet linked to a computer to create detailed pictures of areas inside the body. MRI scans can aid in diagnosis, particularly in helping to distinguish soft tissue sarcomas from benign tumors, as well as showing the extent of the tumor. MRIs are also used to monitor the patient after treatment to see if the tumor has recurred.

1 4. Answer briefly on the following: 4 x 7 /2 a) Nephron sparing surgery. b) Completion thyroidectomy. c) Femoropopliteal Bypass. d) Intussusceptions in a child.

a) Nephron sparing surgery.

Answer.  Nephron-sparing surgery has become an established surgical treatment for patients with renal cell carcinoma (RCC), particularly in situations in which preservation of renal parenchyma is critical. However, due to the fear of local renal fossa recurrence with nephron-sparing surgery, radical nephrectomy has historically been the treatment of choice for patients with unilateral RCC and a normal contralateral kidney.  Recently, increased incidence of low-stage, localized, solitary RCC has led to renewed interest in partial nephrectomy. With excellent disease-specific survival and recurrence rates comparable to that achieved with radical nephrectomy, nephron-sparing surgery can be confidently utilized in treating patients with stage T1 RCC lesions (<7 cm) and a normal contralateral kidney. The utility of nephron-sparing surgery in the context of adjunctive systemic immunotherapy remains to be explored.

Indications:

 Historically, nephron-sparing surgery has been the accepted mode of treatment in cases in which performing a radical nephrectomy would render the patient anephric and require subsequent dialysis.Such indications include patients with only a solitary functioning kidney secondary to unilateral renal agenesis, prior removal of a kidney, or malfunction of a kidney due to a benign disease process.  Another such indication is seen in patients with bilateral RCC in whom bilateral radical nephrectomy would otherwise be required. Also, partial nephrectomy has been historically indicated in patients with unilateral RCC and a normal contralateral kidney, if the contralateral kidney is at risk for dysfunction in the future due to calculous disease, chronic pyelonephritis, renal artery stenosis, ureteral reflux, or systemic disease such as diabetes and nephrosclerosis.  Recently, the improved sensitivity of computed tomography (CT) and ultrasound has led to the increased incidence of incidentally diagnosed ,small, low stage RCC lesions.Usually well localized and easily amenable to partial nephrectomy, these cases have created a controversy of whether nephron-sparing surgery is indicated when the patient has a normal functioning contralateral kidney.  However, despite very encouraging results (discussed later in this review) obtained with partial nephrectomy in treating patients with this indication, fear of renal fossa recurrence and subsequent metastasis has led radical nephrectomy to remainn as the gold standard of treatment for this indication.

Operative considerations:

 Preoperative evaluation of patients with RCC should include assessment of the location and extent of the local tumor as well as the presence of any local or distant metastasis. Renal arteriography depicts the intrarenal vasculature and, thus, allows for a surgical approach that results in the least blood loss and destruction of adjoining renal parenchyma. For patients with large or centrally located tumors, selective renal venography should also be performed to determine the presence of intrarenal venous thrombosis, a condition consistent with an advanced RCC stage and a more complex surgical excision. Preoperative hydration and mannitol administration are also imperative to ensure optimal renal perfusion at the time of operation.  Recently, intraoperative sonography has served as a useful adjunct in nephron-sparing surgery by helping to guide the surgical approach and excision of the tumor. Intraoperative ultrasound can provide a critical view of the extent of the tumor as well as the relationship of the tumor to renal vessels and other critical adjacent structures.This intraoperative imaging modality is particularly useful in cases of deep intrarenal lesions that are nonpalpable and visually imperceptible during surgery. Ultrasonography can help delineate a surgical plane between the tumor edge and crucial hilar structures to facilitate excision. Also, in cases of large tumors invading the hilum, the surgeon may switch intraoperatively to more aggressive measures, such as radical nephrectomy, if ultrasonography displays critical involvement by the tumor that preoperative CT failed to clearly demonstrate.  Once the extent of the tumor has been delineated, several surgical techniques can be employed, depending on the tumor characteristics . Tumors confined to the upper or lower poles of the kidney are generally excised by means of a polar segmental nephrectomy. A transverse resection is employed for large tumors extending throughout the upper or lower pole of the kidney. For tumors not located in either the upper or lower poles of the kidney, a wedge resection is employed. Regardless of the surgical technique, the common principles of a 1 cm tumor-free margin, early vascular control, proper hemostasis, closure of the collecting system, and avoidance of renal ischemia should be followed.  Once the tumor is excised, the control of hemorrhage becomes imperative. Given the increased complexity of nephron-sparing surgery, intraoperative and postoperative bleeding is a common complication. Hence, the control of bleeding is an integral aspect of the surgical procedure. The basic means of controlling severe bleeding requires clamping the renal artery temporarily while the source of bleeding is controlled. If such measures are anticipated for a prolonged period, surface cooling of the kidney with ice slush and mannitol infusion can provide safe ischemia and help avoid permanent ischemic injury to the kidney. Other methods of hemostasis found to be effective include ultrasonic aspiration, laser photocoagulation, microwave coagulation, and fibrin glue.  Aside from this in situ approach to nephron-sparing surgery, some physicians support an extracorporeal approach, particularly for RCC tumors requiring very complex excision. Such a procedure involves complete nephrectomy, tumor excision and renal repair, and subsequent autotransplantation of the kidney. This procedure is beneficial in cases of large RCC lesions invading the hilum or in cases with concomitant renal artery disease, because it provides better access for the surgeon. However, the procedure suffers from drawbacks such as longer operative time and the increased risk of postoperative renal failure.

Disadvantages:

 Tumor Recurrence in the Preserved Kidney The major disadvantage of NSS for treating RCC is the risk of recurrence in the ipsilateral kidney (ipsilateral recurrence). The incidence of local recurrence is reported to be between 0% and 10%, but is lowest in patients who undergo elective NSS for low-stage lesions of less than 4 cm in diameter. Ipsilateral recurrence is more often associated with tumor multifocality than with incomplete resection of the tumor that leaves positive surgical margins. To avoid positive surgical margins, frozen-section biopsies of the tumor bed have been performed. One study of 301 patients who underwent NSS for tumors with a mean diameter of 3.6 cm analyzed frozen-section biopsies for the presence of tumor cells. The authors concluded that the accuracy of frozen-section analysis is low, and that the results of such analyses have minimal clinical significance and should, therefore, not be incorporated in routine urologic practice. Although the disease-specific and overall survival rates of patients undergoing NSS are comparable to those of patients undergoing RN, the presumed higher risk of postoperative local recurrence after NSS mandates a more-intensive surveillance than that proposed in the European Association of Urology guidelines. Regular ultrasound and/or annual CT scan of both the ipsilateral and contralateral kidney is considered appropriate.  Complications and Morbidity Associated With Nephron-sparing Surgery Another argument that was initially used against expanding the indications for NSS was the greater risk of complications with NSS compared with RN. Partial nephrectomy is a more complicated operation than RN. Hemorrhage is the most common complication, with an incidence between 0% and 5% in NSS. Less-frequent complications include urinary leakage or fistula formation, renal artery thrombosis and acute renal failure. Complications in patients undergoing open NSS are related to the level of experience the surgeon has with the procedure. Complications in these patients are rare, therefore, in experienced hands, and when they do occur they can mostly be managed conservatively. Laparoscopic partial nephrectomy is an emerging minimally invasive approach for the treatment of RCC. The laparoscopic approach had a longer renal warm-ischemia time compared with the open approach ,and was associated with more frequent and more severe intraoperative complications and more postoperative urological complications than open NSS. The authors concluded that open partial nephrectomy still remains the standard technique for NSS.

(b) Completion thyroidectomy.

Answer. Completion thyroidectomy plays an important role in the treatment of patients with thyroid cancer. Recently, not all patients have undergone total thyroidectomy,which is the preferred operation for cancer treatment . One of the reasons is the difficulty of peri-operative diagnosis of cancer which can lead to inappropriate procedures. Completion thyroidectomy is still believed to have increased complication rates , but other authors in the literature describe no higher incidence of complications. Several studies that are designed to decrease the complication risk have focused on the timing of completion surgery and the care for, or attention to, the surgical technique. A completion thyroidectomyCompletion thyroidectomy - when a lobectomy is performed and the pathology comes back that the lobe had cancer, the remaining lobe is surgically removed as well. involves removing the remaining thyroid tissue after a patient has had a previous partial thyroid resection (i.e. lobectomy). It may be done years later or it may be done soon after a lobectomy (as early as the next week). The reasons for completion thyroidectomy are the same as for a lobectomy or total thyroidectomyTotal thyroidectomy - removal of the whole thyroid. All patients who undergo a completion thyroidectomy will need to be on life-long thyroid hormone replacement after surgery.

(c) Femoropopliteal Bypass. Answer. Femoropopliteal bypass surgery is a procedure in which surgeons reroute femoral and popliteal blood flow either through a blood vessel transplanted from a healthier part of the leg or through a prosthetic vessel. It is a sort of detour around blocked or damaged blood vessels near the knee. The transplanted vessel then serves as a conduit between the femoral and popliteal arteries.  Femoropopliteal bypass surgery begins with a consultation where the surgeon determines whether to redirect the flow through an existing vein, or to use a graft material. Most doctors prefer to use an alternate vein. Studies suggest that using alternate veins produce better results. Doctors use general anesthesia or an epidural injection to put patients to sleep before theprocedure in order to reduce pain and discomfort.  Patients can expect to remain in the hospital for 2 to 4 days after the procedure, although initial recovery (sitting up and walking around) may begin as early as 24 hours later. Although femoropopliteal bypass surgery is typically successful in alleviating blockage symptoms, it is usually viewed as a last resort when more conservative treatments have not worked. The surgery is reserved for patients whose symptoms are so severe that amputation is a potential consideration. Like most procedures, fem-pop bypass surgery carries a number of risks. Patients should discuss potential side effects such as infection, failed or blocked grafts, excessive bleeding, heart attack, and swelling.  During the operation, the blocked native artery is bypassed using either a section of the patient's own vein (autologous vein), human umbilical vein (HUV), or an artificial graft e.g. Dacron or polytetrafluoroethylene (PTFE).  Reasons a femoral popliteal bypass surgery or PTA of the femoral artery may be performed include, but are not limited to, the following: . Medical management that has not improved symptoms, or worsening of symptoms despite medical management . Severe disability (intermittent claudication that causes interference with lifestyle and/or ability to work) . Nonhealing wounds . Infection or gangrene . Ischemic rest pain (lack of oxygen and nutrients to the leg resulting in pain at rest) . Danger of losing the limb due to decreased blood flow

 Not all peripheral arterial disease can be treated with PTA. Your doctor will decide the best treatment of your PAD based on your individual circumstances.  There may be other reasons for your doctor to recommend femoral popliteal bypass surgery or PTA.

Risks of the procedure: As with any surgical procedure, complications can occur. Some possible complications may include, but are not limited to, the following:

Myocardial infarction (heart Wound infection Pulmonary edema (fluid Restenosis (blockage in attack) in the lungs) the blood vessels after PTA procedure) Cardiac arrhythmias (irregular Leg edema (swelling Bleeding at the catheter Nerve injury heart beats) of the leg) insertion site (usually the groin) after PTA procedure Hemorrhage (bleeding) Thrombosis (clot in Blood clot or damage to Graft occlusion the leg) the blood vessel at the (blockage in the graft insertion site (PTA) used in bypass surgery)

 Patients who are allergic to or sensitive to medications, contrast dyes, iodine, shellfish, or latex should notify their doctor.  There may be other risks depending on your specific medical condition. Be sure to discuss any concerns with your doctor prior to the procedure.

(d) Intussusceptions in a child.

Answer. Intussusception is a serious disorder in which part of the intestine — either the small intestine or colon — slides into another part of the intestine. This "telescoping" often blocks the intestine, preventing food or fluid from passing through. Intussusception also cuts off the blood supply to the part of the intestine that's affected. Intussusception is the most common cause of intestinal obstruction in children. Intussusception is rare in adults. Most cases of adult intussusception are the result of an underlying medical condition. In contrast, most cases of intussusception in children have no demonstrable cause.

Intussusception:

 Occurs most often in babies between 5 and 10 months of age (80% of cases occur before a child is 24 months old)  Affects between 1 and 4 infants out of 1,000  Is more common in boys than in girls.  With prompt attention, intussusception can often be successfully treated without lasting problems.

Causes For most cases of intussusception in children, the cause is unknown. Possible triggers may be:

 Viral infection  A noncancerous growth or a cancerous tumor in the intestine.  Meckel's diverticulum  Polyp  Duplication  Appendix  Hyperplasia of Peyer's patches  Idiopathic Risk factors for intussusceptions include:  Age. Children are much more likely to develop intussusception than adults are. It's the most common cause of bowel obstruction in children between the ages of 3 months and 6 years, with the majority of cases occurring in children younger than 1 year.  Sex. Intussusception affects boys more often than girls.  Abnormal intestinal formation at birth. Malrotation, a condition present at birth (congenital) in which the intestine doesn't develop correctly, also is a risk factor for intussusception.  A prior history of intussusception. Once you've had intussusception, you're at increased risk to develop it again.

Illustration of an intussusception showing the invaginated intussusceptum (blue) and the invaginating intussuscipiens (red) . (A) demonstrates a direct or normograde intussusception occurring in the direction of normal peristalsis. (B) demonstrates an indirect or retrograde intussusception occurring against the normal direction of peristalsis.

Classification:

 Ileoileal - 5%  Ileocolic - 77%  Ileo-ileo-colic - 12%  Colocolic - 2%  Multiple - 1%  Retrograde - 0.2%  Others - 2.8% Symptoms: Children: Signs and symptoms of intussusception in children include:

 Severe abdominal pain that comes and goes (intermittent pain)  Stool mixed with blood and mucus (sometimes referred to as "currant jelly" stool because of its appearance)  A lump in the abdomen  Swollen (distended) abdomen  Vomiting, possibly vomiting bile (yellow-brown or greenish fluid)  Diarrhea  Fever  Dehydration  Lethargy The first sign of intussusception in an otherwise healthy infant may be sudden, loud crying caused by abdominal pain. Infants who have abdominal pain may pull their knees to their chests when they cry. The pain of intussusception comes and goes, usually every 15 to 20 minutes at first. These painful episodes last longer and happen more often as time passes. Complications Intussusception can cut off the blood supply to the affected portion of the intestine. If left untreated, lack of blood causes tissue of the intestinal wall to die. Tissue death can lead to a tear (perforation) in the intestinal wall, which can cause peritonitis, an infection of the lining of the abdominal cavity. Peritonitis is a life-threatening condition that requires immediate medical attention. Signs and symptoms of peritonitis include:

 Abdominal pain  Abdominal swelling  Fever  Thirst  Low urine output Peritonitis may cause your child to go into shock. Signs and symptoms of shock include:

 Cool, clammy skin that may be pale or gray  A weak and rapid pulse  Abnormal breathing that may be either slow and shallow or very rapid  Lackluster eyes that seem to stare blankly  Profound listlessness A child who is in shock may be conscious or unconscious. If you suspect your child is in shock, seek emergency medical care right away.

Tests and diagnosis To confirm a diagnosis of intussusception, your child may need:

 A physical exam. The doctor may suspect intussusception or another type of intestinal obstruction if your child has a lump in the abdomen, along with intermittent pain or inconsolable crying.  Blood or urine tests. Your child's doctor may order blood and urine tests, as well as a test, which checks for blood in the stool.  Ultrasound or other abdominal imaging. Taking images of your abdomen or your child's abdomen with an ultrasound, X-ray or computerized tomography (CT) scan may reveal intestinal obstruction caused by intussusception. Abdominal imaging also can show if the intestine has been torn (perforated).  Air or barium enema. An air or barium enema is basically a colon X-ray. The doctor will likely use air first and use barium only if necessary as a second choice. During the procedure, the doctor will insert air or liquid barium into your colon or your child's colon through the rectum. This makes the images on the X-ray clearer. Sometimes an air or barium enema will correct intussusception, and no further treatment is needed. A barium enema can't be used if the intestine is torn. Treatments and drugs Emergency medical care is required to treat intussusception to avoid severe dehydration and shock, as well as prevent infection that can occur when a portion of intestine dies due to lack of blood. Initial care When child arrives at the hospital, the doctors will first stabilize his or her medical condition. This includes:

 Giving your child fluids through an intravenous (IV) line.  Helping the intestines decompress by putting a tube through the child's nose and into the stomach (nasogastric tube). Correcting the intussusception To treat the problem, your child's doctor may recommend:

 A barium or air enema. This can correct the telescoping intestine and successfully treat intussusception. If an enema works, further treatment is usually not necessary.  Surgery. If the intestine is torn or if an enema is unsuccessful in correcting the problem, surgery is necessary. The surgeon will free the portion of the intestine that is trapped, clear the obstruction and, if necessary, remove any of the intestinal tissue that has died. In some cases, intussusception may be temporary and improve on its own without treatment. If no underlying medical condition is found that caused the intussusception, no further treatment is necessary. Prevention: Because its cause is unknown in most cases, intussusception generally can't be prevented.

THE WEST BENGAL UNIVERSITY OF HEALTH SCIENCES MS (General Surgery) Examination, 2015 PAPER IV

Time Allowed: 3 Hours Full Marks: 100

1) a) Which objective criteria should be used preoperatively to identify patients with borderline resectable pancreatic cancer? 5+5+5+5 b) Is there enough evidence for operative exploration and resection in the presence of mesenteric-portal axis or arterial involvement? c) How should the pathologic specimen be examined and which definition of tumour free resection margins should be used? d) Which prognostic markers should be considered regarding patient selection? 2) What are the types of intestinal failure? Describe their causes and outline management. 5+5+10. 3) Write short notes of the following: 5x6 a) Stem cell research in surgery. b)Pharmacological advances in local anaesthetics in surgical practice. c) Current management of Fistula – in – ano. d) Stents in surgical practice. e) Management of patients with open abdomen.

4) Answer briefly on the following. 4x71/2 a) Classification and management algorithm of post laparoscopic cholecystectomy bile duct injury. b) Recent updates of phyllodes tumour of breast. c) Roux syndrome following gastrectomy. d) Evaluation of biliary enteric anastomosis for benign biliary disorder.

THE WEST BENGAL UNIVERSITY OF HEALTH SCIENCES

MS (General Surgery) Examination, 2015

April 2015

PAPER IV

Time Allowed: 3 Hours Full Marks: 100 Attempt all questions

1) a) Which objective criteria should be used preoperatively to identify patients with borderline resectable pancreatic cancer? 5+5+5+5 b) Is there enough evidence for operative exploration and resection in the presence of mesenteric-portal axis or arterial involvement? c) How should the pathologic specimen be examined and which definition of tumour free resection margins should be used? d) Which prognostic markers should be considered regarding patient selection?

Answer. a) Borderline resectable disease: 1) No extrapancreatic disease. 2) The following possible tumor-vessel relationships: an SMV-PV confluence that can be reconstructed even is short segment venous occlusion is present (ie, a suitable portal vein above, and a suitable SMV below the area of occlusion); tumor abutment of the SMA of < 180 o ; or short segment encasement of the hepatic artery amenable to resection and reconstruction (this is usually at the origin of the gastroduodenal artery and reconstruction may or may not require interposition grafting with a short segment of reversed saphenous vein). Definition of respectability: Borderline Resectable NCCN Version 2.2012 SMA, Celiac < 180 degree abutment Hepatic Artery (HA) 1. Gastro-duodenal artery encasement up to the Hepatic artery or 2. Direct abutment of hepatic artery without extension to celiac axis

SMV/PV 1. Impingement and narrowing of the lumen 2. Encasement or short segment venous occusion.

b) The MD Anderson borderline resectable categories included three patient subsets as defined by the following clinical and radiographic character istics. Type A: patients with borderline resectable tumor anatomy as defined on CT images to include one or more of the following findings: tumor abutment (180° of the circumference of the vessel) of the SMA or celiac axis; tu- mor abutment or encasement ( 180° of the circumfer- ence of the vessel) of a short segment of the hepatic artery, typically at the origin of the gastroduodenal artery; or short- segment occlusion of the SMV, PV, or SMV-PV confluence that was amenable to vascular resection and recon- struction because of a patent SMV and PV below and above the area of tumor-related occlusion. Type B: patients with borderline resectable disease owing to a concern for possible extrapancreatic metastatic disease. This subgroup of borderline resectable patients included those with CT findings suspicious for, but not diagnostic of, metastatic disease and those with known N1 disease from either prereferral laparotomy or endoscopic ultrasonography- guided fine-needle aspiration. Type B patients may have had a technically resectable. or a borderline resectable pri- mary tumor as defined on CT images. Type C: patients with borderline resectable disease owing to a marginal performance status, or those with a better performance status and severe preexisting medical comorbidity thought to require protracted evaluation that precluded immediate operation. By definition, type C patients with a marginal performance status were thought to have reversible causes of their current symptoms (such as hyperbilirubinemia-induced anorexia and fatigue); pa- tients judged to have Type C patients may have a radiologically resectable or a bor- derline resectable primary tumor.

Pathologic Assessment of the Surgical Specimen The modifications to the TNM staging system in the 6th edition of the AJCC Ca ncer Staging Manual allow the accurate staging of patients even if they do not undergo pancreatic resection. The T4 (and Stage III) designation is reserved for locally advanced unresectable primary tumors in the absence of distant metastases. In addition to TNM staging, when the pancreaticoduodenectomy specimen is evaluated pathologically, the retroperitoneal margin (the soft tissue margin directly adjacent to the proximal 3-4 cm of the SMA) must be evaluated on permanent sections by inking the margin and sectioning the tumor perpendicular to the margin. It is critical that the surgeon identify this margin at the time of resection because it cannot be assessed retrospectively and many pathologists cannot accurately identify the retroperitoneal margin on a pancreaticoduodenectomy specimen. The surgeon and pathologist should classify the retroperitoneal margin after integrating the operative findings and the histologic assessment of this margin. All pancreatic resections should be classified according to residual d residual disease; R1, microscopic residual disease (microscopically positive surgical margins with no gross residual disease); and R2,isease grossly status evident (termed residual “R” factor): disease. R0, The no pathologistgross or microscopic cannot usually differentiate an R1 (microscopically positive) from an R2 (grossly positive) retroperitoneal margin in the absence of information regarding the retroperitoneal dissection, which should be included in the operative note. The R designation should appear in the final pathology report if possible and should always be listed in the dictated operative note The difficulty in differentiating R1 from R2 resections has significant implications for the conduct of clinical trials examining the potential advantage of nonsurgical therapies, especially in patients with borderline resectable tumour c) Standardized pathologic evaluation of the surgical speci- men was performed as previously described. The SMA margin was defined as the soft tissue margin directly adja- cent to the proximal 3 to 4 cm of the SMA. In all patients, the SMA margin was evaluated according to the AJCC Cancer Staging Manual (6th edition) guidelines.

This margin was identified and inked by the surgeon and pathologist immediately on specimen removal and was evaluated by permanent-section microscopic examination; when tumor extended to the inked margin, the margin was considered positive. The technique for assessment of the SMA margin was the same regardless of whether vascular resection was performed. The pancreatic transection margin and the common bile/hepatic duct transection margins were evaluated by examining a complete en face section of each margin. At the discretion of the surgeon, these two margins were usually evaluated intraoperatively using frozen-section analysis, and if positive, additional bile duct or pancreatic parenchyma was usually resected. An operation was designated R0 if all final margins were negative (no tumor cells were identified at any of the three resection margins) and R1 if any of the final margins were micro- scopically positive (tumor cells were present at one or more of the margins). Tumor size was calculated by the pathologist by measuring the maximum gross transverse diameter of the tumor after resection. This measurement was difficult to determine in some patients after preoperative therapy because the tumor was often hard to distinguish from uninvolved adjacent pancreatic parenchyma by gross examination. The grade of neoadjuvant treatment effect was assessed on permanent sections by a faculty gastrointestinal pathologist and scored using a previously published grading system.

A minimal pathologic response was defined as a treatment effect score of either grade I (90% or more viable tumor cells r emaining after induction therapy) or IIa (50% to89% remaining viable tumor cells). A partial pathologic response was defined as a treatment effect score of IIb (10% to 49% remaining viable tumor cells) or III (less than 10% remaining viable tumor cells). A treatment effect score of IV, indicating no remaining viable tumor cells, was used to designate a complete pathologic response. d) The Importance of Pretreatment Staging to define the extent of disease as a necessary component of the conduct of clinical trials and outcome reporting Evaluation of the potential value of nonsurgical therapies (chemotherapy and radiation therapy) in improving local disease control and survival of patients with pancreatic cancer requires accurate pretreatment staging (to define the study population) and a standardized system for the pathologic evaluation of surgical specimens (to determine the completeness of resection). This is routinely performed in most other solid tumors yet rarely completed in an organized fashion in pancreatic cancer making the interpretation of the published literature difficult or impossible. For example, the definition of resectable pancreatic cancer used in most studies is based upon whether or not the surgeon has removed the pancreatic head, often with no system of margin analysis. Multidetector (multislice) computed tomography (CT) is used to objectively define (anatomically) potentially resectable disease, borderlin e resectable disease, locally advanced disease, and metastatic disease. Although contrast enhanced CT is widely available, accurate interpretation and reporting of the tumor related findings remains inconsistent. For optimal pretreatment staging and assessment of operability, a CT report in a patient with suspected periampullary or pancreatic cancer should include the following information: 1.Commentary on the presence or absence of a primary tumor in the pancreas; 2.Commentary on the presence or absence of peritoneal and hepatic metastases; 3.Description of the patency of the superior mesenteric vein-portal vein confluence and the relationship of these veins to the tumor; 4.Description of the relationship of the tumor to the superior mesenteric artery (SMA), celiac axis, and hepatic artery. Specific, objective radiographic criteria can be used to create the following definitions: Potentially resectable disease: 1) no extrapancreatic disease, 2) a patent SMV-PV confluence (assuming the technical ability to resectand reconstruct this venous confluence), and 3) a definable tissue plane between the tumor and regional arterial structures including the celiac axis, common hepatic artery and SMA.

Borderline resectable disease: 1) no extrapancreatic disease, 2) the following possible tumor-vessel relationships: an SMV-PV confluence that can be reconstructed even is short segment venous occlusion is present (ie, a suitable portal vein above, and a suitable SMV below the area of occlusion); tumor abutment of the SMA of < 180 degree; or short segment encasement of the hepatic artery amenable to resection and reconstruction (this is usually at the origin of the gastroduodenal artery and reconstruction may or may not require interposition grafting with a short segment of reversedsaphenous vein).

Locally advanced disease: 1) no extrapancreatic disease, 2) tumor encasement of the SMA or celiac axis defined as tumor involvement of > 180 degree of the arterial circumference. Metastatic disease: Radiographic or clinical evidence of distant organ or peritoneal metastases. Despite clear evidence that high quality cross-sectional imaging predicts resectability accurately, many patients undergo laparotomy for pancreatic cancer without adequate preoperative assessment. Some patients are fo und to have unresectable tumors intraoperatively when such a conclusion might have been possible prior to surgery. Conversely, because of a lack of adequate preoperative imaging and surgical expertise, many patients who are resected have been left with gross residual disease not recognized by the surgeon intraoperatively, or documented in the operative note. with “curative intent” 2) What are the types of intestinal failure? Describe their causes and outline management. 5+5+10. Answer. Introduction: Intestinal failure occurs when a person's intestines can't digest food and absorb the fluids, electrolytes and nutrients essential to life and normal development. Patients must then receive TPN, which provides liquid nutrition through a catheter or needle inserted into a vein in the arm, groin, neck or chest. Types: Intestinal Failure can be categorised into three types: • Type1: This type of intestinal failure is short term, self limiting and often peri -operative in nature. This type is common and these patients are managed successfully in a multitude of healthcare settings, especially surgical wards, including– all units which perform major, particularly abdominal surgery. Some patients on high dependency units (HDU) or intensive care units (ICU) will also fall into this category. • Type 2: This occurs in metabolically unstable patients in hospital and requires prolonged parenteral nutrition over periods of weeks or months. It is often associated with sepsis, and may be associated with renal impairment. These patients often need the facilities of an intensive care or high dependency unit for some or much of their stay in hospital. This type of intestinal failure is rare and needs to be managed by a multi-professional specialist intestinal failure team. Poor management of type 2 intestinal failure increases mortality and is expected to increase the likelihood of later development of type 3 intestinal failure. • Type 3: This is a chronic condition requiring long term parenteral feeding. The patient is characteristically metabolically stable but cannot maintain his or her nutrition adequately by absorbing food or nutrients via the intestinal tract. These are, in the main, the group of patients for which HPN or electrolyte (HPE) are indicated.

Causes of Intestinal Failure:

The most common cause of intestinal failure is short bowel syndrome where at least half or more of the small intestine has been removed. Short bowel syndrome is typically a postsurgical condition for treatment of conditions such as trauma or necrotizing enterocolitis.

➢ Pedatric vs. Adult Causes

Intestinal failure may also be caused by functional disorders such as Crohn's disease, a digestive disorder, or chronic idiopathic intestinal pseudo-obstruction syndrome. The conditions leading to intestinal failure are age-dependent. That is, some conditions are more closely associated with pediatric intestinal failure while others are more common with intestinal failure in adults. o Pediatric Conditions Causing Intestinal Failure

• Congenital malformations such as small bowel atresia, gastroschisis, aganglionosis • Infections of the gastrointestinal tract such as necrotizing enterocolitis) • Short bowel syndrome following extensive bowel surgeries secondary to mesenteric ischemia (e.g., midgut volvulus) • Absorptive impairment (e.g., intestinal pseudo-obstruction, microvillus inclusion disease) o Adult Conditions Causing Intestinal Failure • Short bowel syndrome following extensive surgeries secondary to mesenteric ischemia (following thrombosis, embolism, volvulus, or trauma) • Inflammatory bowel disease such as Crohn's disease • Small bowel tumors such as Gardner's syndrome (familial colorectal polyposis) • Tumors of the mesenteric root and retroperitoneum (desmoid tumor)

Treatment:

The clinical features of intestinal failure include intractable diarrhoea or a high stomal output, weight loss, dehydration, vitamin and mineral deficiency, and malnutrition. Acute intestinal failure is usually temporary and often attributable to infection or perioperative complications. Chronic intestinal failure results from intestinal resection(s), gastrointestinal disease or small bowel dysfunction.

Management Specific management depends on the amount, type and integrity of the remaining bowel, as the presence of even part of the large bowel can significantly increase absorption. The aim is to maximise gastrointestinal function and, where necessary, supplement fluid and nutrient intake by the least invasive means, thus providing the individual with the best possible quality of life. o Maximising gastrointestinal function There are a number of sites in the gastrointestinal tract that affect the rate of gastric emptying and thus influence gastrointestinal motility. These sites (or brakes) occur in the stomach, the proximal small bowel, the distal small bowel, and the colon and rectum. o Absence of any or all of these sites will lead to decreased intestinal transit time, high intestinal fluid loss and reduced absorption of fluid and nutrients. (Intestinal transit time refers to the time that food and fluids remain in the gastrointestinal tract). Increasing intestinal transit time is therefore an important consideration in optimising gastrointestinal function as it will increase fluid and nutrient absorption and decrease intestinal losses.

▪ Medication Anti-diarrhoeal medications such as loperamide and codeine phosphate are commonly used to reduce motility and enhance absorption. To maximise their effectiveness they should be taken half an hour before food. An additional dose before bedtime can be of enormous psychological benefit by reducing the fear of incontinence or leakage from a stoma. ▪ Anti-secretory medications such as histamine 2 (H2)-receptor antagonists or proton pump inhibitors can lead to a reduction in output from the gastrointestinal tract by reducing the secretion of gastric acid. Large doses of both anti-diarrhoeal and anti-secretory medications may be necessary to compensate for the decreased intestinal transit time and impaired absorption.

Nutritional Support in Intestinal Failure: o Reducing the intake of hypotonic fluids Arguably, the most important intervention in maximising gastrointestinal function is to reduce the amount of hypotonic fluid (such as water, tea, coffee and squash) entering the intestine, as this will significantly reduce the amount of fluid the intestine has to assimilate. Without an appreciation of the mechanisms affecting absorption in the jejunum, this intervention may appear to be illogical. o The sodium concentration in the jejunum is maintained at 90mmol/litre. Drinking hypotonic fluid dilutes the sodium concentration causing a net efflux of sodium from the plasma in order to restore the concentration to 90mmol/litre. o Facilitating small bowel adaptation A final consideration in maximising intestinal function is facilitating small bowel adaptation. Following intestinal resection the remaining bowel undergoes structural and functional changes. The small bowel dilates and lengthens and there is a reduction in motility. These changes result in increased intestinal function. The controlling mechanisms behind adaptation are not yet fully understood, but it is believed that the presence of nutrients within the intestine is an important factor. o Patients with intestinal failure are encouraged to eat for a number of reasons. In addition to being important in small bowel adaptation, the presence of nutrients within the intestine is also important in maintaining normal gastrointestinal flora and the gut barrier function, for example, preventing infection - factors that can impact on intestinal function. o Supplementing fluid/nutrient intake The small intestine transports between 7-9 litres of fluid a day, absorbing about 80 per cent of this so that only about 1-2 litres enters the colon. The colon absorbs fluid, so that only 100-200ml is actually excreted each day. Although there will be alteration in fluid and nutrient absorption in all patients with intestinal failure, the most significant will occur in those in whom there has been a significant loss of intestine. A reduction in absorptive area, such as in someone with a proximal small bowel stoma or enterocutaneous fistula, will result in a greater volume of fluid excreted and subsequent fluid and electrolyte imbalance and a degree of malabsorption. o The need for fluid and nutrient replacement is determined by a number of factors, namely, the amount of available intestine, the volume of daily intestinal losses, and the ability to maintain a satisfactory weight. Individuals with less than 75cm of jejunum, an intestinal output of greater than 2500ml/day, and absorption below about one-third of all energy taken orally will usually be dependent on long-term parenteral nutrition. Those with 75-100cm of jejunum, and average daily outputs of about 1200-2500ml/day, may require parenteral supplementation of fluid while being able to maintain their nutritional status with an enteral regime including food and/or supplemental enteral nutrition. o While long-term parenteral supplementation of fluids and nutrients will help prevent

volumes usually mean infusion times of at least 12 hours, and while cyclical infusion overnight permitsdehydration freedom and malnutrition,from the infusion it imposes pump duringrestrictions the day, on anthe individual’s patient may daily suffer life. disrupted Large fluid sleep due to the need to pass urine frequently during the night. o It is important to note that even though a patient may be dependent on parenteral nutrition or fluid, this does not mean that other aspects of their treatment regimen can be relaxed. Fluid and

significant shift in this - for example, by drinking excessive amounts of hypotonic fluid or not takingnutrient medication needs are -determined they will still by become a patient’s dehydrated. average daily fluid balance and if there is a

New Therapeutic Agents:

▪ Growth hormone, for example, has received approval from the US Food and Drug Administration (FDA) for the management of short bowel syndrome. Various treatment strategies with growth hormone and other agents are being tried in patients with short bowel syndrome. In one randomized controlled trial, a combination of an adequate diet plus growth hormone plus glutamine produced results that were superior to dietary therapy alone. ▪ Another hormone, glucagon-like peptide (GLP)-2, may have a role in stimulating intestinal mucosal adaptation in patients with short bowel syndrome, although its mechanism of action is not completely clear. This agent is now undergoing evaluation in a phase 3 trial.

➢ Intestinal Failure and Transplantation: An intestinal transplant is a last-resort treatment option for patients with intestinal failure who develop life-threatening complications from total parenteral nutrition (TPN). Long-term TPN can result in complications including bone disorders, catheter-related infections and liver failure. Over an extended period of time, TPN also can damage veins used to administer the nutrition via the catheter.

An Intestinal transplant is a complex procedure requiring a highly skilled multidisciplinary transplant team. An isolated Intestinal transplant surgery takes approximately three to four hours to complete whereas a multivisceral (multi-organ) transplant operation can take up to twelve hours.

There are three major types of intestinal transplants:

❖ Isolated intestinal (Small Bowel) Transplantation:

In an isolated intestinal transplant, the diseased portion of the small intestine is removed and replaced with a healthy small intestine from a donor. In an isolated intestinal transplant, the disease limited to the small bowel only without liver failure. This procedure can be lifesaving for patients with irreversible intestinal failure that has become life-threatening.

❖ Combined Liver and Intestinal Transplantation:

Combined liver and intestine transplantation is done for patients with both liver and intestinal failure. IN In this procedure, the diseased liver and intestine are removed and replaced with a healthy liver and intestine from an organ donor. Complications of intravenous nutrition (TPN) are the main cause of liver failure attendant to intestinal failure. Without a transplant, patients with intestinal and liver failure have an expected median survival of 6 - 12 months while continued on TPN.

❖ Multivisceral Transplantation:

Multivisceral transplantation is performed where two or more intra-abdominal organs (including the intestines) are failing. The transplanted organs may include the stomach, duodenum, pancreas, intestine, and liver. This complex procedure can be life-saving for patients with combined abdominal organ failure resulting diseases such as Gardner's syndrome (familial colorectal polyposis), a pre-malignant colorectal condition and intestinal pseudo-obstruction (decreased ability of the intestines to push food through).

Evaluation for Intestinal Transplantation

During the evaluation process, a detailed medical history is taken and numerous studies (tests) are performed to determine whether an intestinal transplant is likely to benefit the patient including:

• Abdominal CT scan • Barium enema • Blood tests for liver function, electrolytes, kidney function and antibodies to certain viruses • • ECG and echocardiogram • Endoscopy • Motility studies • Ultrasound of the circulatory system • Upper gastrointestinal and small bowel X-ray series

Intestinal Transplant Procedure

In an Intestinal transplant, the small intestine (small bowel) is surgically removed (along with any other diseased organs such as the liver), and replaced with healthy organ(s). Initially, the blood vessels of the patient and donor are connected to establish a blood supply to the transplanted intestine. The donor's intestine is then connected to the patient's gastrointestinal tract.

An ileostomy is then performed. An ileostomy is a surgically created opening through which the ileum, a section of the patient's small intestine, is brought up through the abdominal wall. This opening allows body waste to pass directly out of the body and empty into a pouch. The ileostomy also allows the transplant team to assess the health of the patient's transplanted intestine. In time, most patients are able to have the ileostomy closed. A feeding tube is also placed into the stomach to help the patient transition to an oral diet.

3) Write short notes of the following: 5x6 a) Stem cell research in surgery. b) Pharmacological advances in local anaesthetics in surgical practice. c) Current management of Fistula – in – ano. d) Stents in surgical practice. e) Management of patients with open abdomen.

Answer. a) Stem cell research in surgery.

Answer. Early stem cell research has traditionally been associated with the controversial use of embryonic stem cells. The new focus is on non-embryonic adult mesenchymal stem cells which are

Research and uses: found in a person’s own blood, bone marrow, and fat. Urology:

Interstitial Cystitis

Millions of patients suffer from interstitial cystitis/painful bladder syndrome. This severe and debilitating condition has historically been confused with other bladder pathology which must be

(pain, pressure, discomfort) perceived to be related to the urinary bladder. ruled out, making IC difficult to diagnose. Currently, IC/PBS is defined as “an unpleasant sensation Peyronies Disease

Peyronies Disease has been described by experts as a physically and psychologically devastating problem manifested by a fibrous inelastic scar of the fibrous chambers of the penis known as the hinging and shortening of the penis in the erect state. tunica albuginea. The scarring (known as “peyronies plaques”) can cause pain, bending, narrowing, Erectile Dysfunction

Erectile Dysfunction is defined as the inability to achieve or sustain an erection suitable for sexual intercourse. ED affects up to one third of men of men throughout their lives and has a substantial negative impact on intimate relationships, quality of life and self-esteem. Causes are multifactorial but can be related to loss of testosterone, surgical damage to the penile nerves, medications, or other medical illness.

Male Incontinence

In America alone, more than three million men are affected by loss of bladder control, a medical condition known as urinary incontinence. This problem has a great impact on health and quality of life for those who suffer with it. Male urinary incontinence is usually caused by a damaged sphincter, the circular muscle that controls the flow of urine out of the bladder

Orthopaedics:

Neck Arthritis and Spine Disease

Two common problems causing neck pain is degenerative disk disease and facet arthropathy (a type of arthritis) of the cervical spine. Not all disk problems are amenable to regenerative therapy and patient selection is very important to optimize outcomes. Many forms of spine pain are actually due to arthritis of the facet joints (small joints where vertebra connect to their adjacent neighboring vertebrae).

Back Arthritis and Spine Disease

Two common problems causing back pain is degenerative disk disease and facet arthropathy (a type of arthritis). Not all disk problems are amenable to regenerative therapy and patient selection is very important to optimize outcomes. Many forms of back pain are actually due to arthritis of the facet joints (small joints where vertebra connect to their adjacent neighboring vertebrae).

Knee Problems

Knee arthritis is a form of degenerative joint disease that affects millions of Americans. There are several alternatives to knee surgery such as injection of cartilage substitutes or steroids. Steroid injection is not favored as a viable long term treatment since the steroids actually damage local tissues over time.

Hip Problems

Hip arthritis is a form of degenerative joint disease that affects millions of Americans. Patients try to delay hip replacement surgery for as long as possible and often seek hip injections with steroids. Steroid injection is not favored as a viable long term treatment since the steroids actually damage local tissues over time.

Elbow & Hand Problems Arthritis of joints of the upper extremity is a form of degenerative joint disease that disables millions of Americans. Certain types of elbow and wrist joint problems including certain tendon problems and ligamentous injuries may not be amenable to cell therapy. It is important to evaluate each individual case to assess likelihood of success with attempting regenerative therapy.

Shoulder Problems

Shoulder problems come from chronic pain from rotator cuff injuries and arthritis of joints of the shoulder disables millions of Americans. Certain types of shoulder joint problems including certain tendon problems and ligamentous injuries may not be amenable to cell therapy. It is important to evaluate each individual case to assess likelihood of success with attempting regenerative therapy.

b) Pharmacological advances in local anaesthetics in surgical practice.

Answer. Local anesthetics (LAs) can be defined as drugs that reversibly block transmission of a nerve impulse, without affecting consciousness. All of the amide LAs harbor varying levels of cardiovascular (CVS) and central nervous system (CNS) toxicity that is still a major complication seen today. Ropivacaine is indicated for local anaesthesia including infiltration, nerve block, epidural and intrathecal anaesthesia in adults and children over 12 years. It is also indicated for peripheral nerve block and caudal epidural in children 1 12 years for surgical pain. It is also sometimes used for infiltration anaesthesia for surgical pain in children. – Ropivacaine is often coadministered with fentanyl for epidural analgesia, for example in pregnant women during labour.

Compared to bupivacaine, levobupivacaine is associated with less vasodilation and has a longer duration of action. It is approximately 13 percent less potent (by molarity) than racemic bupivacaine and has a longer motor block onset time.

Levobupivacaine is indicated for local anaesthesia including infiltration, nerve block, ophthalmic, epidural and intrathecal anaesthesia in adults; and infiltration analgesia in children.

Short acting local anaesthetics for Spinal Anaesthesia: Heavy Prilocaine: Prilocaine 2% hyperbaric solution for injection was associated with faster discharge times than a hyperbaric formulation of another local anaesthetic in one small single-centre, double- blind, randomised study. Use of this preparation may allow service improvement through benefits to individual patients or service delivery.

2-Chloroprocaine:

Chloroprocaine (2-chloroprocaine, CP) has been one candidate to replace lidocaine for short procedures, despite the fact that neurologic sequelae have been described following the intrathecal injection of large doses of preservative-containing CP intended for epidural use. It is also used in TURP. AAGBI Safety Guideline Management of Severe Local Anaesthetic Toxicity

c) Current management of Fistula – in – ano. Answer. See the answer of question no. 3.b of Paper –IV of 2014. Issues in mamagement of fistula in ano:

Complex fistula – management options:

Complex fistula in ano – evaluation:

d) Stents in surgical practice. Answer. • Coronary stents are placed during a percutaneous coronary intervention, also known as angioplasty. The most common use for coronary stents is in the coronary arteries, into which a bare-metal stent, a drug-eluting stent, a bioabsorbable stent, a dual-therapy stent (combination of both drug and bioengineered stent), or occasionally a covered stent is inserted. • Vascular stents are commonly placed as part of peripheral artery angioplasty. Common sites treated with peripheral artery stents include the carotid, iliac, and femoral arteries. Because of the external compression and mechanical forces subjected to these locations, flexible stent materials such as nitinol are used in a majority of peripheral stent placements. • A stent graft or covered stent is type of vascular stent with a fabric coating that creates a contained tube but is expandable like a bare metal stent. Covered stents are used in endovascular surgical procedures such as endovascular aneurysm repair. Stent grafts are also used to treat stenoses in vascular grafts and fistulas used for hemodialysis. • Ureteral stents are used to ensure the patency of a ureter, which may be compromised, for example, by a kidney stone. This method is sometimes used as a temporary measure to prevent damage to a blocked kidney until a procedure to remove the stone can be performed. • Prostatic stents are places from the bladder through the prostatic and penile urethra to allow drainage of the bladder through the penis. This is sometimes required in benign prostatic hypertrophy. • Esophageal stents are a palliative treatment for advanced esophageal cancer. • Biliary stents provide bile drainage from the gallbladder, pancreas and bile ducts to the duodenum in conditions such as ascending cholangitis due to obstructing gallstones. • Other types are duodenal stents, colonic stents, and pancreatic stents, the designations referring to the location of their placement. e) Management of patients with open abdomen. Answer. Introduction: abdominal viscera. Damage control surgery associated with trauma and abdominal compartment syndrome are theThe most term frequent “open abdomen” reasons for refers leaving to a andefect abdomen in the open.abdominal Open wall abdomen that exposes exposes the the viscera and leads to fluid and heat loss, which can be reduced with temporary abdominal closure techniques until the abdomen can be closed primarily, closed functionally, or graft coverage of the exposed viscera provided. Open Abdomen: 1. Mortality and complication rates remain high. 2. Abdominal wall / gut reconstruction after a period of open management, may be extremely technically challenging (especially in the setting of an intestinal fistula) and is best undertaken in specialist centres. Indications for and management of the open abdomen: o Intestine is a highly vascularized structure with a rich network of arteries, veins, capillaries and lymphatics. o In pathologic states, decreased fluid clearance from the extracellular space can result in swelling of intestinal wall several times the normal diameter, potentially interfering with bowel perfusion. o Furthermore, peritoneal surface is highly vascularized and possess inflammatory cells. o The net result is excretion of pro-inflammatory substances that increase the local inflammatory response as well as systemic absorption leading to a systemic inflammatory response syndrome (SIRS) and may progress to a multi-organ dysfunction syndrome (MODS). o Premature closure of the abdomen can exacerbate the inflammatory response and potentially accelerate the syndrome. Pathophysiology The open abdomen (OA) treatment: ▪ The open abdomen (OA) treatment can be defined as leaving the fascial edges of the abdominal wall intentionally unapproximated.

was• Aliases first popularized include: Laparostomy in the mid Temporary1990s. Abdominal Closure (TAC) Eteppenlavage Indications• Nowadays, to leave the OAthe procedure abdomen has open been: widely applied and improved clinical outcomes since it 1- Major abdominal trauma. 2- Massive bowel oedema. 3- Need for multiple explorations. 4- Extensive abd wall defects re-explorations. 5- Intra-abdominal hypertension (IAH). 6- Abdominal Compartment Syndrome. 7- Severe abdominal sepsis: as in =secondary peritonitis. =unsatisfactory source control cterial killing (improved oxygenation). • Theoretically, OA is assumed to improve drainage and ↑ ba -abdominal infections depending on the degree of peritoneal contamination per se. • Very little evidence is there to support this technique in treating: intra 1. Intestinal oedema from sepsis or resuscitation that may precipitate ACS if abdomen is closed. 2.• ThePlanned only reoperation.2 sufficient indications for OA management are Complications of OA management: Ileus EA fistula Bleeding Hernia Heat/fluid loss Infection Defective Defective ventilation Fascial retraction Fascial retraction ventilation These complications can be minimized by: 1. Understanding of the pathophysiology found in the OA . 2. Using materials that minimize trauma to the abdominal contents. 3. Prevention of severe intra-abdominal sepsis. 4. Facilitation of multiple re-explorations; if needed. 5. Prevention of abdominal wall loss. 6. Tension-free fascial closure. Management of the patient with OA: Management of OA: General management: Wound management IV Fluids Heat loss control Analgesia & sedation Nutrition Temporary abdominal closure Dressing Definitive closure (T.A.C.) ❖ I.V. fluid administration: timise preload. •The 1st 24 hrs in ICU after returning from OR frequently require large fluid infusions to op oedema. •It can be challenging to balance this need with the restrictive resuscitation-directed that fluid minimises therapy visceral followed by the use of vasopressors to maintain physiological parameters. •Literature favours early aggressive fluid resuscitation guided by goal ❖ Heat loss control:

•On arrival in ICU, patients are frequently cold. •The OA will furtherly increase heat loss. ❖•Active Analgesia and passive and sedation: methods of rewarming may be required.

which abdomen is closed. •Need for analgesia is, perhaps surprisingly, not significantly different to that after major surgery in OA esp. if associated with EAF. •Pain control will go hand in hand with management of agitation and stress of the striking sight of sympathetic psychological support. ❖•Established Nutrition: protocols involve the use of fentanyl and midazolam in combination, with expert and verely insulin resistant and profoundly catabolic). •OA may represent a source of protein/nitrogen loss (patients are inevitably se

•Intestinal failure develops in at least 15% of these patients. time before closure). •Enteral nutrition (EN) shouldly require be employed parenteral whenever nutrition possible (PN), but (↓ mayseptic also complications be satisfactorily and ↓fed by "fistuloclycis". •Patients with EAF usual

❖ Temporary•Nutritional abdominal support can closure be optimised (T. A. C.): by consultation of specialist dieticians.

C was first described by Ogilvie using a piece of canvas (1940). •OA is maintained by a TAC device. •TA -access to the abdominal cavity (patients with OA require multiple trips to the •Manyoperating techniques room. exist, ie, every the ideal 24 48technique hrs for asshould long asfulfill 2 or the 3 weeks following using goals: staged abdominal •Easyexplorations). re –

evisceration•Protection of(selective the abdominal tamponade). contents. •Preventionvation of of (or fascia. better lowering) bacterial contamination and infection. •Prevention of

•Preser •Prevention of fascial retraction and loss of domain. •Quantification of 3rd-risk space patients losses. can rapidly develop high abdominal pressures secondary•keeping the to bleeding,patient dry. pack •Providing placement, abdominal and rapid support accumulation for ventilation. of third space fluid) 1. T. A. C.: •Prevention IAH (high simple 3 layer system using negative pressure for abdominal support: 1.Vacuum Fenestrated Pack polyurethane Technique “ Vacuum sheet wrap Assisted around Closure the bowels “ or “ Negative (prevents Pressure bowel adherence Wound Therapy to the ant “ •A abd wall and facilitate fluid drainage ). 2. Sponge (granular foam dressing). if haemostasis is crucial) is applied to the sponge layer using sophisticated software giving specific alarms3. Adhesive when plastic NPWT sheet is impaired (applied or over interrupted. the sponge) •A uniform suction (100 to 150 mmHg or lower General indications of NPWT: 1- Chronic wounds (e.g. MRSA) 2- 2- Open Abdomen 3- 3- Partial thickness burns. 4- 4- Mechanism of action: Indulent ulcers (DM, bed sores, venous,….) wound responses: •ContinuousA- Physical negative response preesure (macrostrain)= applied to * the approximation wound surface of woundhelps wound edges. healing* removal by ofevoking excess 2 fluids. * reduction of tissue oedema. B- B- Biological response (microstrain)= = removal of exudates that may contain inhibitors of healing. = reduction of oedema with better tissue perfusion. = cell-surface strain

(microdeformation)epeated by negative dressing, pressure the GT induces gradually cellular fills thestretch gap allowing→ cells become subsequent more 2ry metabolicallyclosure . granulation active → tissue ++ fibroblast formation migration } ++ cell proliferation ++ extracellular matrix Commerciallydeposition available •With r devices such as VAC,KCI = $500 / dressing. perforations.

• Alternatives include bogota bag or 2 plastic sheet stuck together with • Sponges with 2 JP drains through the skin to suction.

❖• Adherent Dressing plastic : after sheetthe decision over the has sponges been made on the to abdominal leave the abdomen wall • $ 50 open, Poor dressing Man’s VAC should be:

•Available in the theatre. •Easy to apply. •Protecting the skin and viscera to ↓ fistulation. ❖ •Easy to remove for redressing. ▪ Plan a strategy for closing the abdomen at almost the same time as the decision to leave it open. ▪ DefinitiveTiming: In Closure general, at delayed a 2nd look primary laparotomy closure “Delayedwill be possible primary within closure”: several days, once haemodynamic stability and visceral oedema subside (IAH is avoided). ▪ When abdomen is left open for long periods, fascial retraction with loss of domain ensues. oIf it is not possible to close abdomen by day 10, it may not be possible at all. ▪ Delayed primary closure is more likely to be possible after DCS for surgery, or haemorrhage (e.g. ruptured AAA) than for treatment of abdominal sepsis or ACS. ▪ When Primary Closure is not possible Fascial closure rates of 60% at a 2nd look laparotomy have been reported although the rate of incisional hernia may be as high as 30%. ▪ If so, available options may include: 1.Planned Ventral Hernia 2.Skin graft 3.Component Separation 4.Canica ABRA closure system ✓ Planned Ventral Hernia

e some stability to the abdominal wall. • Collagenization of the wound converts-6 granulationmonths. tissue into scar. • Absorbable mesh may be used to provid become• Wound apparent. contraction will occur over 3 - Ventral• Over Megahernia the succeeding Surgical months correction to a year, carries the collagenextensive is risks slowly with removed at least and a 30% a hernia complication will rate including:

✓•Skin necrosis •Bowel ischaemia •Abdominal compartment syndrome •Prolongedranulate intubation and a •Wound infection •Recurrence •Death intervalSkin Grafting of at least• Easier 6-12 wound months) care. • ?? less EAF rates. • viscera are left to g ✓ skinComponent graft is separationundertaken. technique: • giant ventral hernia is inevitable but can be repaired after an 1. Abdominal wall flap is mobilized from the underlying fascia, this is done until lateral to the linea semilunaris. 2. Be careful to maintain the abdominal wall muscle perforators to avoid devascularization of the skin. 3. A small incision is then made in the external oblique just lateral to the linea semilunaris. 4. By the aid of a haemostat, the incision is taken cephalad till the lower ribs and caudad to the pubis using cautery. 5. Medial traction of rectus muscle by the assistant helps to accomplish the procedure. 6. This is done bilaterally. 7. Upto 14 cm can be gained in the abd wall per side. 8. Midline closure Component separation technique described techniques. • In rare circumstances, adequate skin cover is not possible despite application of the previously flaps have been described. • Complex myocutaneous flaps have been successfully used to close such defects. • Success with anterolat. Thigh and rectus abdominis -Tex Dual Mesh in initial stage. -towel clamping of the mesh or resuturing of the mesh every• Gore 72 hrs to 2 weeks. • Subsequent stages involve serial excision fascia is reapproximated with component separation and bioprosthetic• VAC may be reinforcement.placed over the mesh. • Mesh is then excised and the

• Allows ABRA for primary closure closure system inrestores ~90% thewithout primary planned closure ventral option hernia. for full-thickness abdominal • Averagedefects, time by to approximating definitive closure the skin46 days. and fascial margins for low- tension primary closure

with a more voluminous abdominal cavity. . • It acts by turning the ‘ flat abdomen ’ with eviscerating contents to a ‘ rounded abdomen ‘ mobile wound margins. • All necrotic-existing tissues, abdominal adhesions closure or unhealthy device should granulation be removed tissues prior are excised to ABRA to application.achieve clean ABRA Dynamic Closure System • Any pre 4) Answer briefly on the following. 4x71/2 a) Classification and management algorithm of post laparoscopic cholecystectomy bile duct injury. b) Recent updates of phyllodes tumour of breast. c) Roux syndrome following gastrectomy. d) Evaluation of biliary enteric anastomosis for benign biliary disorder. Answer. a) Classification and management algorithm of post laparoscopic cholecystectomy bile duct injury. Answer.

 Type A Cystic duct leaks or leaks from small ducts in the liver bed  Type B Occlusion of a part of the biliary tree, almost invariably the  aberrant right hepatic ducts  Type C Transection without ligation of the aberrant right hepatic  ducts  Type D Lateral injuries to major bile ducts  Type E Subdivided as per Bismuth classification into E1 to E5

 Type 1 Leaks from cystic duct stump or small ducts in liver bed  Type 2 Partial CBD/CHD wall injuries without (2A) or with (2B) tissue loss  Type 3 CBD/CHD transection without (3A) or with (3B) tissue loss  Type 4 Right/Left hepatic duct or sectoral duct injuries without (4A) or with (4B) tissue loss  Type 5 Bile duct injuries associated with vascular injuries CBD, common bile duct; CHD, common hepatic duct.

➢ Goals of therapyin iatrogenic bile duct injury:n Iatrogenic Bile Duct Injury 1. Control of infection limiting inflammation

2.• Parenteral Clear and thoroughantibiotics delineation of entire biliary anatomy • Percutaneous drainage of periportal fluid collections

3.• MR R e-establishment CP/PTC of biliary enteric continuity • ER CP (especially-free, mucosa- if cysticto-mucosa duct stump anastomosis leak suspected) -en-Y hepaticojejunostomy • T ension -term transanastomotic stents if involving bifurcation or higher. • R oux • L ong

b) Recent updates of phyllodes tumour of breast.

Answer. (PT) of the breast is a rare that constitutes <1% of all female breast tumors. Although it can occur at any age from adolescents to elderly, in most of the cases occurs between 35 and 55 years of age. It shows a wide range of clinical and pathological behavior. Phyllodes are frequently large in size and may be up to 40 cm in diameter. In most of the patients having unilateral disease, bilateral tumors are rare. Even though specific pathological criteria are available for the diagnosis, a definite preoperative diagnosis is still difficult. Clinically, it is very similar to other benign breast masses. Uncertain clinical behavior and preoperative diagnoses results in inappropriate management. Axillary lymph nodes metastases are found only in <1% of patients. The recurrence is the peculiar feature of the phyllodes and still there are no accurate clinicopathological factors that can predict recurrence and mortality. Surgery remains the mainstay of management and options vary from wide local excision (WLE) to mastectomy. Optimal surgery is still a matter of debate, particularly in case of BL and malignant PT. The role of adjuvant radiotherapy and chemotherapy has been controversial.

Diagnosis

• Phyllodes tumours account for less than 1% of breast neoplasms. • Median age of presentation is 45 years; 20 years later than that for fibroadenomas. • Diagnosis should be considered in all rapidly growing benign breast lesions. • No radiological indicators exist to differentiate benign and malignant lesions. • Fine needle aspiration cytology specimens require both epithelial and stromal elements to confirm the diagnosis.

Paddington Clinicopathological Suspicion Score.

Pathology

• Tumours can grow to over 20 cm in diameter. • Macroscopically small tumours usually resemble fibroadenomas. • • Microscopy shows epithelial lined cystic spaces with hypercellular stroma. • LargeOn histological tumours oftenappearance have a tumoursred “meaty” are classifiedappearance as withbenign, cystic borderline, and necrotic or malignant. areas. • Axillary lymph node metastases occur in approximately 10% of patients.

Prognosis

• Most malignant tumours do not metastasise. • Some benign tumours may show an unusually aggressive behaviour. • Incomplete excision increases the risk of local recurrence. • Histological type is the most important predictor for metastatic spread. • The five year survival for benign, borderline, or malignant tumours is 96%, 74%, and 66% respectively.

Azzopardi and Salvadori criteria for diagnosis of nature of phyllodes

Management

• All rapidly growing benign breast lesions require histological assessment. • Wide excision or mastectomy should be performed ensuring histological clear margins. • Mastectomy for malignant tumours offers no survival advantage. • Axillary nodal dissection is not required. • The role of adjuvant radiotherapy is unclear.

Recurrent and metastatic disease

• 15% of patients develop local recurrence. • Local recurrence is not associated with distant metastases or reduced survival. • Local recurrence can often be controlled with further surgery. • 20% of patients with malignant tumours develop distant metastases. • The commonest sites are lung, bone, and abdominal viscera. • Good palliation can be obtained with single agent or combination chemotherapy. • The role of hormonal therapy remains to be defined.

c) Roux syndrome following gastrectomy. Answer. Introduction:

d) Evaluation of biliary enteric anastomosis for benign biliary disorder. Answer. The ability to achieve steady, long-term results confirms hepaticojejunostomy as the best procedure in the treatment of benign biliary strictures, even if endoscopic procedures are gaining a new role in the treatment of a greater number of patients.

STANDARD surgical techniques offer a good chance of cure for the majority of patients affected by extrahepatic benign biliary stricture. Nevertheless, operative repair has a long-term recurrence rate of stricture in 10% to 30% of patients. Advances in endoscopic procedures have provided alternative options of relieving biliary obstructions, but prolonged length of treatment and rehospitalization have to be considered if endoscopy is performed. The purpose of this study was to compare the effectiveness of endoscopic and surgical treatments of benign stenosis of the bile duct.

Because a recurrent stricture, mostly occurring in the first 5 years, can arise at any time after primary treatment, 8 to 10 years of follow-up is needed to determine whether repair has been successful. Our analysis was focused on patients with a length of follow-up longer than 60 months. Long-term results following both endoscopic procedures and surgical treatment were compared. The long-term outcome in our series is as good as that of other reports, which confirm that more than 80% of patients can be treated successfully with endoscopic or surgical procedures.

Complications after biliary enteric anastomosis for a benign disease

Complications Local Wound infection Biliary leak Cholangitis/persistent jaundice Delayed gastric emptying Pancreatic fistula Hemorrhage Pancreatitis Systemic Chest infection Pulmonary edema Acute myocardial infarction Systemic sepsis Renal Failure Mortality

• Methods of evaluatinga biliary-enteric bypass for ob struction or other postoperative complications include hepatobiliary scintigraphy, ultrasonography, cross-sectional imaging, endoscopie retrograde cholangiopancreatography (ERCP) and percutaneous transhepatic cholangiography (PTC). • ERCP and PTC are used to define the anatomy of the biliary tree and when the need for possible intervene tion is anticipated. • The use of ultrasound as a noninvasive technique for detection of biliary-enteric bypass obstruct tion is limited by two factors. First, the presence of gas within the bowel or the biliary tree following surgery creates difficulties in the assessment of biliary duct size. Second, if the size of the biliary ducts can be measured, ductal dilatation does not necessarily indicate an obstructive pattern in this population as dilatation may persist after surgical relief of obstruction. • Hepatobiliary scintigraphy (HBS) using technetium-99 attached to iminodiacetic acid is a physiological imaging method that is useful for detecting biliary obstruction. • Baseline laboratory characteristics of the subjects: • Laboratory test: Following parameters are also evaluated: • Laboratory test: Following parameters are also evaluated: ➢ Bilirubin, mg/dL, median (range) ➢ AST, U/L, median (range) ➢ ALT, U/L, median (range) ➢ ALP, U/L, median (range) ➢ GGT, U/L, median (range) ➢ Change of bile duct size ➢ Stricture on the cholangiogram, A biliary enteric anastomosis is taken as successful if after five years of operations the LFT values of the patient is normal with normal result in imaging studies. –

THE WEST BENGAL UNIVERSITY OF HEALTH SCIENCES MS (General Surgery) Examination, 2014 PAPER IV

Time Allowed: 3 Hours Full Marks: 100

1. Describe the presentation and diagnosis of locally recurrent rectal cancer. Discuss the management strategy of locally recurrent rectal cancer. 20 2. Define metabolic Surgery. What are the different types of metabolic surgery and how do they influence metabolism? 20 3. Write short notes of the following: 5x6 a) Biofilms in wound management. b) Recent methods of surgical management of fistulae in ano. c) NOTES. d) Coagulopathy in trauma patients. e) Decompressive hemicraniectomy.

4. Answer briefly on the following. 4x71/2 a) Earfly Gastric carcinoma – recent advances in diagnosis.. b) Nephron sparing Surgery. c) Percutaneous management of benign breast lesions. d) Infection and carcinogenesis.

THE WEST BENGAL UNIVERSITY OF HEALTH SCIENCES

MS (General Surgery) Examination, 2014

April 2014

PAPER IV

Time Allowed: 3 Hours Full Marks: 100 Attempt all questions

1. Describe the presentation and diagnosis of locally recurrent rectal cancer. Discuss the management strategy of locally recurrent rectal cancer. 20 Answer. Recurrent Rectal Cancer: Local recurrence of rectal adenocarcinoma is seen in up to 30% of patients. Although recurrence may be seen at the distal margin of the anastomosis, most develop from residual cancer on the pelvic wall. The time course for recurrences to present through the anastomosis is approximately 18 months. By their nature, these tumors are fixed to the pelvic wall and surrounding viscera.  They cause significant symptoms, such as: o Intractable pelvic pain, o Bleeding, o Cramping or constipation, o Urinary tract dysfunction, and o Chronic pelvic sepsis.

Diagnosis:  When patients present with these symptoms or with a rising CEA level, a workup including CT scan of the abdomen and pelvis, ERUS, MRI of the pelvis, and PET scan may be helpful.  A careful pelvic examination is mandatory.  A biopsy, either via sigmoidoscopy or CT-guided, should be used to confirrm the diagnosis pathologically.  If external radiation has not been used before, it should be considered.  The surgeon should review the imaging studies and determine which organs are involved, such as the vagina, uterus, prostate, bladder, sacrum, and small intestine, which will require en bloc resection.  Urology consult should be obtained if there is any question of prostate or bladder involvement; ureteral stents should be placed preoperatively. Removal of the rectum and urinary bladder with surrounding lymphatic tissue results in a permanent colostomy and ileal conduit. Treatment:  Intraoperative Radiation Therapy:  Intraoperative radiation therapy (IORT) may be considered in patients with pelvic sidewall recurrence. This is performed in an operating room–radiation therapy suite.  Resection with negative microscopic margins and absence of vascular invasion independently predicts improved local control and survival after resection and IORT.  The major morbidities of IORT include peripheral neuropathy and ureteral stenosis.  Palliation:  These tumors are difficult to palliate.  Surgical resection combined with aggressive multimodality therapy is advocated to avert the morbidity of pelvic disease and to prolong survival in a subset of patients, with survival ratesup to 30%.  Most patients, however, will not be offered curative surgery on the basis of comorbidities, poor performance status, distant metastases, or locally unresectable disease on preoperative imaging. These patients may be offered palliative intervention.  Chemoradiation:  Patients with rectal cancer who undergo surgery with intention to cure and without evidence of gross disease postoperatively may still develop local recurrence or distant metastases.  Up to 10% of patients who undergo TME with tumor-free radial and distal margins may develop local failure.  The goal of adjuvant therapy is to eliminate the micrometastatic disease present at the time of surgery.  Adjuvant Chemoradiation: In 1990, the National Institutes of Health consensus statement concluded that “combined postoperative chemotherapy and radiotherapy improves local control and survival in stages II and III patients and is recommended.” . Most of the information regarding chemotherapy for colorectal cancer comes from trials of colon cancer rather than for rectal cancer. . The NSABP C-04 (National Surgical Adjuvant Breast and Bowel Project C-04) trial studied stages II and III colon cancer patients and demonstrated that 5-_uorouracil (5-FU) and leucovorin treatment had a significantly better 5-year survival rate (74 vs 69%) compared with 5-FU and levamisole. . Several trials have suggested a benefit for adjuvant chemoradiation for rectal cancer in patients with resected stage II -III cancers. . The GITSG (Gastrointestinal Tumor Study Group) trial demonstrated that combined chemoradiation resulted in an improvement in overall survival as well as a decrease in local recurrence. . NCCTG (North Central Cancer Treatment Group) trial demonstrated that the addition of chemotherapy to radiation reduced both local recurrence (13 vs 25%) and distant metastases (28 vs 46%) and improved survival.  Radiation therapy used alone as adjuvant therapy may improve local recurrence and survival rates. A theoretical reason to use postoperative radiation therapy is that more appropriate patient selection can be achieved because pathologic staging is performed prior to radiation. . Disadvantages include radiating the neorectum and small bowel and a lower tendency of patients to complete their radiation.  Neoadjuvant Chemoradiation: There are a number of potential advantages for using neoadjuvant chemoradiation. . They include the ability to deliver higher doses of chemotherapy with radiation. . Another advantage is not only to downstage the tumor, which has been noted in 60–80% of patients, but also to achieve a pathologic complete response, which occurs in 15–30% of patients. . The ability to “shrink” the tumor facilitates surgical resection, thereby allowing one to achieve negative margins and perform a operation in patients who otherwise would require an APR. . Additional advantages include radiating tissues with a greater oxygen supply, not radiating the anastomosis, and decreased likelihood of developing radiation enteritis because small bowel is less likely to enter the pelvis. . Finally, patients are more likely to complete the course of radiation therapy because it precedes their surgical resection. . A meta-analysis concluded that preoperative radiation therapy plus surgery compared with surgery alone significantly reduced the 5-year overall mortality rate, cancer-related mortality rate, and local recurrence rate. . In the German Rectal Cancer Trial published in the New England Journal of Medicine, Sauer et al randomly assigned patients with clinical stage II or III rectal cancer to preoperative (421 patients) or postoperative (402 patients) chemoradiotherapy based on a concurrent long course of radiotherapy (5040 cGy delivered in fractions of 180 cGy per day, 5 days per week) and 5-FU (120-hour continuous intravenous infusion during the first and fifth weeks). Six weeks later, TME was performed, followed by four cycles of 5-FU 1 month postoperatively. Despite the preponderance of distal tumors in the preoperative chemoradiation group, there was no difference in overall survival or disease-free survival at 4 years. On the other hand, patients receiving preoperative chemoradiotherapy had a 6% local recurrence rate as compared to a 13% local recurrence rate in those receiving postoperative chemoradiotherapy. Moreover, the group treated with preoperative chemoradiotherapy had a higher incidence of sphincter preservation and lower treatment- related toxicities (27 vs 40%). Differences in local recurrence, sphincter preservation, and treatment toxicities were all statistically significant.  Current practice is to recommend preoperative staging with ERUS or MRI to all patients with rectal adenocarcinoma and then to offer chemoradiation to medically fit patients with curative intent who have T3-T4 or N-positive rectal carcinoma. Some patients with bulky T2 lesions near the sphincters should also be considered for neoadjuvant chemoradiotherapy in order to improve sphincter preservation.  Neoadjuvant therapy then is followed by TME with APR or TME with an end-to-side or colonic J-pouch reconstruction. Postoperatively, patients who have had involved lymph nodes either by preoperative staging or on the final pathology report are encouraged to have additional postoperative chemotherapy.  Postoperative chemotherapy in node-negative patients or patients who have had a complete response is determined on a case by case basis.

2. Define metabolic Surgery. What are the different types of metabolic surgery and how do they influence metabolism? 20

Answer.  Bariatric surgery, from the Greek “baros” meaning “weight”, is synonymous with weight loss surgery. Yet, the benefits and mechanisms of gastrointestinal procedures extend beyond weight loss, questioning the appropriateness of a name and practice entirely based on weight- reduction.  Bariatric surgery indicates a variety of gastrointestinal (GI) surgical procedures originally designed to induce weight reduction in morbidly obese patients. See the answer of question no. 2 of Paper – IV of 2013, answer of question no. 1 of Paper – IV of 2010, question no. 1 of Paper – III of 2009.

3. Write short notes of the following: 5x6 a) Biofilms in wound management. b) Recent methods of surgical management of fistulae in ano. c) NOTES. d) Coagulopathy in trauma patients. e) Decompressive hemicraniectomy.

Answer. a) Biofilms in wound management. Introduction:  The term biofilm is used to describe a community of microorganisms encased within an extracellular polymeric matrix, which accumulates at a surface. Biofilms play a significant role in a large number of infections in humans, and due to the intrinsic resistance of these structures to an array of antimicrobial agents and host defense mechanisms, such diseases can be difficult to treat effectively.  The term biofilm is used to describe a community of microorganisms encased within an extracellular polymeric matrix, which accumulates at a surface. Biofilms play a significant role in a large number of infections in humans, and due to the intrinsic resistance of these structures to an array of antimicrobial agents and host defense mechanisms, such diseases can be difficult to treat effectively.  In the absence of specific antibodies, the polysaccharide component of the biofilm matrix also blocks complement activation. If antibodies are present, the polymeric matrix generally renders them ineffective.  It has been shown that the biofilm matrix is also able to inhibit chemotaxis and degranulation by polymorphonucleocytes (PMNs) and macrophages and also depress the lymphoproliferative response of monocytes to polyclonal activators.  Not only are host defenses unable to deal effectively with biofilms, but their persistence can cause tissue damage (e.g., lung tissue in cystic fibrosis). Contact with a surface triggers the expression of a panel of bacterial enzymes that catalyze the formation of sticky polymers that promote colonization and protection.  The structure of biofilms is such that immune responses may be directed only at those antigens found on the outer surface of the biofilm, and antibodies and other serum proteins often fail to penetrate into the biofilm.  In addition, PMNs are unable to effectively engulf bacteria growing within a complex polymer matrix attached to a solid surface. This causes the PMNs to release large amounts of pro-inflammatory enzymes and cytokines, leading to chronic inflammation and destruction of nearby tissues (i.e., chronic inflammation). Bacteria that may be embedded within the wound biofilm matrix are likely to be resistant to both immunological and non-specific defense mechanisms of the body.

 It has been estimated that biofilms are associated with 65 percent of nosocomial infections.So why are biofilm-related infections such a problem to treat? The challenge arises as a consequence of the following several factors:  Biofilms display innate resistance to antimicrobial agents, thus protecting associated bacteria.  Biofilms increase the opportunity for gene transfer between and among bacteria. This is important, since bacteria resistant to antibiotics may transfer the genes for resistance to neighboring susceptible bacteria. Also, gene transfer could convert a previous avirulent commensal organism into a highly virulent pathogen.  Antibiotics are used to treat bacterial infections. However, biofilm-related infections do not succumb so easily to this form of treatment, because they provide a protective mechanism that renders bacterial cells less susceptible to both antibiotics and biocides. However, on removal of these cells from the matrix of the biofilm, they are equally susceptible to biocides. There have been a number of models used to determine resistance in biofilms, and the results of these studies have highlighted a number of the factors thought to contribute to the ability of a biofilm to tolerate high concentrations of antibiotics. These include: 1. Impaired penetration of an antibiotic into the biofilm matrix:Many researchers have investigated the possible lack of antibiotic/biocide penetration as an explanation of biofilm resistance. It was suggested that the antimicrobial agent either reacted chemically with the extracellular components of the biofilm or attached to the anionic polysaccharides. However, since the exopolymer matrix does not form a complete impenetrable barrier to antimicrobial agents, other mechanisms must exist within biofilms aiding bacterial survival.

2. Reduced growth rate of bacteria in biofilms, which renders them less susceptible to antibiotics (they change from being physiologically active in the planktonic state to sessile in the biofilm state). Antibiotics are more effective in killing cells when they are growing actively. Antibiotics, such as ampicillin and penicillin, are not able to kill nongrowing cells.Cephalosporins and fluoroquinolones, however, are able to kill nongrowing cells but are nonetheless more effective in killing cells that are rapidly growing and dividing. Therefore, evidence of bacteria that are growing slowly in a biofilm may contribute to reduced susceptibility to antibiotics.

3. Altered micro-environment within the biofilm (e.g., pH, oxygen content), which reduces the activity of an antimicrobial agent. There is evidence of gradients of physiological activity within a biofilm in response to antibiotic treatment. This would suggest that the response to antibiotics will vary according to the location of specific cells within a biofilm ecosystem.

4. Altered gene expression. Altered gene expression by organisms within a biofilm or a general stress response of a biofilm have been documented as factors known to reduce susceptibility to antibiotics.

5. Quorum sensing (QS). QS has been documented as being involved in antibiotic resistance, but its role is currently unclear, which justifies the need for additional research in this area. QS involves the production of signalling molecules within a bacterial population that enables bacteria to communicate with each other and initiate a response to their surrounding environment once a critical population density (quorum) has been reached. In a wound environment, many different bacteria live together in often dense populations, and this provides an ideal situation for QS to occur. Communication in this way allows bacteria to coordinate their behavior and, if necessary, change physiologically to enable them to adapt to a new environment (e.g., a wound). Adaptive responses of bacteria within a wound environment may be associated with nutrient availability, competition with other microorganisms, and the avoidance of host defense mechanisms. Adaptation to a wound environment via QS may involve bacteria secreting protective EPS and increasing the production of enzymes that facilitate their tissue invasion.

6. Reduced biofilm-specific phenotype. It has been suggested that a biofilm-specific phenotype may be induced in a subpopulation of the biofilm.These subpopulations have been shown to express active mechanisms to reduce the efficacy of antibiotics.

Wounds are susceptible to microbial contamination from both exogenous and endogenous sources including the nose, skin, mouth, and the gut, and it is likely that such organisms are involved in the evolution of microbial communities in wounds. The development of microbial communities and their variation over time previously has been demonstrated in an acute full-thickness porcine wound model. In one study, the authors found that the microbial ecology changed from a predominantly Gram- positive bacterial population during the early days after wounding to a predominantly Gram-negative population by Day 6 with anaerobes becoming evident between Days 6 and 8. It is likely that a similar microbial progression and community development occurs in human cutaneous wounds, and biofilms are likely to play an important role in this. Microbial progression within a wound is thought to consist of a number of stages and if this process is not controlled, the probability of infection increases.

Early contaminants on a wound surface are most likely to be skin flora (e.g., Staphylococcus epidermidis) that adhere to the wound, proliferate, synthesize EPSs, and form a "healthy" biofilm. It is at this point that the host initiates a normal immune response and maintains a "homeostasis" at the site of contamination. It is conceivable, based on documented evidence, that Gram-negative bacilli may then colonize the biofilm -- these organisms utilize available oxygen and provide growth factors that enable anaerobes to establish within the biofilm thus forming a complex but stable polymicrobial "climax" community often termed microbial homeostasis. A climax community is a collection of microorganisms within a "quasi" steady state, implying stable associations and integrations of function between microbial populations. In this situation, the progression to wound healing may become compromised by the biofilm community, i.e., the microbial to host balance is weighted in favor of the microorganisms, and the term critically colonized has been used to describe this state. At this stage, the microorganisms, while interfering with the wound healing process, may not necessarily induce any clinical signs of infection, although there may be subtle signs that indicate bacterial imbalance (e.g., change in wound color or odor together with the presence of devitalized tissue and ischemia). It may be appropriate to consider the use of a broad-spectrum topical antimicrobial agent at this stage to control the microbial challenge. The combined effects of the antimicrobial agent and the host immune response are likely to improve conditions for healing where bacterial imbalance is evident. Without control of microbial progression, a transition from an early "healthy" biofilm to a "pathogenic" wound biofilm may develop and ultimately lead to clinical infection. While the net pathogenic effect of the biofilm community exceeds the host's immune response, wound healing is likely to be compromised.

Scientific and clinical research in the area of wound biofilms and the associated bacterial interactions is now warranted to better understand the impact on wound healing. b) Recent methods of surgical management of fistulae in ano. Answer. Surgical management:  Fistulotomy:  This is the standard treatment for lowsimple anal fistulas, submucosal and low inter- sphincteric fistulas.  The indications are low cryptoglandular fistulas, low chronic fistulas, simple fistulas where 30-50% of the tracts pass through the sphincter, which is not anterior in female, single tract, non recurrent, continent, non chronic.  Fistulotomy is usually a single stage procedure but in complex procedures it can be used as a staged procedure with adjuvant seton or glue therapy. The incontinence rate varies from 0- 40% in inter-sphincteric fistulas.  Radiation ablation of tract is another improvement in techniques of fistulectomy. It is known to have decreased gas continence as radiation frequency ablation causes minimal damage to surrounding tissue.  Ultrasound dissection of the fistulous tract is a future advancement. The recurrence rate of fistulotomy is 7-16% after 2 yrs follow up. This recurrence rate increases to 40% after 6 yrs.  Fistulectomy: Studies have shown that the Fistulectomy does not offer any additional advantage over the Fistulotomy procedure. Newer modalities:  Fibrin Glue - it is amixture of fibrinogen, thrombin and calcium ions which when combined form a soluble clot due to cleavage of fibrinogen to fibrin. This clot seals the fistula tract in 30- 60 seconds. Between days 7 and 14 the tract is replaced by synthesized collagen. The advantages of this procedure are that it is a simple procedure without any learning curve. There is no decrease in level of continence and also other treatment options remain open in case of a failure.  Fibrin Plug -The plug is made up of lypophilized porcine small intestinal submucosa shaped in a conical fashion which increases the mechanical stability thus avoiding dislodgement during straining. The reasons for failure of the plug are improper securing of the plug to the primary opening leading to dislodgement. Multiple fistula tracts have been associated with higher failure rates.  Adipose derived stem cells - have been used in the treatment of complex anal fistulas.  Mucosal Advancement Flap - this a sphincter sparing procedure where endorectal/ endoanal flaps are advanced to close the internal ring with or without closure of the tract..  LIFT procedure - It is ligation of the inter sphincteric fistula tract. c) NOTES. Answer.  Natural orifice transluminal endoscopic surgery (NOTES) is an experimental surgical technique whereby "scarless" abdominal operations can be performed with an endoscope passed through a natural orifice (mouth, urethra, anus, etc.) then through an internal incision in the stomach, vagina, bladder or colon, thus avoiding any external incisions or scars.  Portions of the gastrointestinal tract such as the stomach and colon commonly examined with endoscopy (such as for colon cancer screening) also reside in the peritoneal cavity. Perforation (inadvertent puncture creating a hole) of the gastrointestinal tract has been traditionally considered an undesirable event. Interventions using NOTES, however, intentionally perforate the stomach, bowel or vagina, thereby allowing the operator to access the peritoneal cavity using a normal body orifice as a conduit. Here, any peritoneal structure is theoretically approachable.  As an example, in natural orifice surgery the gallbladder might be removed through the mouth. The doctor would insert a tube down the esophagus, make a small incision in the stomach wall to gain access to the abdominal peritoneal cavity and take the gallbladder out by the same route. Potential advantages of the NOTES technique include reduced post- operative pain, shortened recovery times, and improved cosmesis (lack of surgical incision scars).  A range of procedures might be performed this way, such as gastric bypass, fallopian tube ligation, removal of the ovaries and diagnostic work. Some operations might be done via the rectum, vagina, urethra or bladder as well. Because NOTES is so new, research generally has been confined to animals, mainly pigs. Recently, however, human studies have emerged that report the procedures to be feasible. Further studies involving a variety of NOTES procedures, under close supervision by medical experts and Institutional Review Boards, are being performed. The first multicenter human clinical trial in the U.S. is currently underway.  The last surgical innovation with such radical changes was laparoscopic surgery which was introduced in the late 1980s. Laparoscopic surgery was initially associated with an increased rate of specific complications which threatened to discredit the technique at that time. As a result, extensive research regarding safety measures was conducted in the following years. Laparoscopy is a mature technique today and is the standard procedure for many abdominal operations. d) Coagulopathy in trauma patients. Answer. Introduction: Trauma remains a leading cause of death and disability in adults in spite of advances in resuscitation, surgical management, and critical car. Between 25 to 35 percent of injured civilian trauma patients develop a biochemically evident coagulopathy upon arrival in the emergency department, in spite of improved efficiency of trauma systems, military and civilian, in reducing the time interval between acute injury and treatment. Coagulopathy may be the result of physiologic derangements such as acidosis, hypothermia, or hemodilution related to fluid or blood administration; however, an acute coagulopathy can also occur in severely injured patients independent of, or in addition to, these factors. Several terms are used in the literature to refer to this condition, including acute traumatic coagulopathy (ATC), early coagulopathy of trauma (ECT), trauma-induced coagulopathy, and the acute coagulopathy of trauma-shock. Mechanism:  Acute intrinsic coagulopathy arising in severely injured trauma patients is now termed trauma-induced coagulopathy (TIC) and is an emergent property of tissue injury combined with hypoperfusion. Mechanisms contributing to TIC include anticoagulation, consumption, platelet dysfunction, and hyperfibrinolysis.  Hemorrhagic shock from blood loss is a critical cause of mortality in severely injured patients. Contributing to blood loss is an intrinsic dysregulation of the blood coagulation system now named trauma-induced coagulopathy (TIC). TIC arises in the presence of both tissue hypoperfusion from blood loss and severe anatomical tissue injury and, when present, is strongly and positively associated with mortality. The physiological environment in which TIC arises is a complex mixture of inflammation, anticoagulation, and cellular dysfunction of mixed etiology.  Several key processes, including dysfunction of natural anticoagulant mechanisms, platelet dysfunction, fibrinogen consumption, and hyperfibrinolysis, have been identified as primary components of TIC. In addition, specific effects of blood dilution from resuscitation fluids, environmental hypothermia, and acidosis can modulate clot formation, adding more layers of complexity to TIC. o Anticoagulation:

 Anticoagulation is a primary component of TIC. TIC was initially described as an increased international normalized ratio (INR) by Brohi et al in severely injured trauma patients measured on arrival at the emergency department.  Activated partial thromboplastin times (aPTT) were also elevated in patients with severe injury and there was tissue hypoperfusion as measured by the base deficit, although changes in this assay lagged behind changes in INR.  The investigators presumed that elevations in INR and aPTT were independent of dilution by fluid resuscitation or environmental influences due to the limited resuscitation fluids received by these patients before sampling.  The thrombin-thrombomodulin-protein C anticoagulant system was implicated as a potential primary mechanism of anticoagulation. The thrombin-thrombomodulin-protein C anticoagulant system was implicated as a potential primary mechanism of anticoagulation.  The anticoagulant properties of aPC are derived by its degradation of activated factors V and VIII, producing reduced concentrations and activities.  Other anticoagulant mechanisms may contribute to the pathomechanism of TIC. Endogenous autoheparinization, possibly related to shedding of the endothelial glycocalyx, has been suggested by the ability to reverse anticoagulation in the presence of heparinase in whole blood from TIC patients.  Measurement of fibrinopeptides liberated either by thrombin or from degradation by plasmin, have shown a relatively greater increase of plasmin relative to thrombin activation during the initial encounter with trauma patients.  It is clear that anticoagulation, either directly from the thrombin-thrombomodulin-aPC system or through thrombin activation and factor consumption, is an important component of TIC. o Platelet dysfunction:

 There is a rapidly growing body of support for a prominent role of platelet dysfunction in the pathophysiology of TIC. o Fibrinogen consumption and fibrinolysis:  After its activation by thrombin, fibrinogen spontaneously polymerizes from its individual monomers to form an insoluble polymeric fibrin mesh to stop blood loss at sites of vascular injury. This process, along with platelet-induced clot contraction, is the primary component of secondary hemostasis.  There is strong evidence that consumption of fibrinogen and fibrinolysis by the action of plasmin are key mechanistic components of TIC. o Combining injury and shock to produce TIC:  Observational clinical data have demonstrated that TIC is most common in severely injured trauma patients who are subjected to a high degree of both anatomical tissue injury and hypoperfusion from hemorrhagic shock. A multicenter retrospective study of over 3646 trauma patients confirmed that the severity of TIC is strongly associated with combined severe injury and shock.  A synergistic effect of materials released from tissue injury with inflammation and anticoagulant factors induced by endothelial injury and tissue hypoxia likely mediate this relationship to produce TIC.  There is evidence that circulating intracellular contents such as histones, known to be released from injured tissue, are associated with TIC.  In a small study of 37 multiple trauma patients, patterns of elevated histones and extracellular DNA derived from activated neutrophils followed leukocyte counts, IL-6 levels, and myeloperoxidase levels and were associated with subsequent sepsis, multiple organ failure, and death.  Neurohormonal and endothelial activation in response to acute blood loss may also be involved in the development of TIC.  Although specific mechanisms of shock-induced endothelial activation and dysfunction have not yet been elucidated, there is emerging evidence that disruption of the endothelial glycocalyx barrier (indicated by syndecan-1 release) plays an important role in the endothelial response to traumatic shock.  Three proinflammatory cytokines, IFN-γ, IL-1β, and fractalkine, were negatively associated with plasma syndecan-1 levels, presumably by their binding and activation of endothelium.  Glycocalyx shedding and associated loss of endothelial barrier function also presumably promotes leukocyte recruitment and activation, liberation of injurious reactive oxygen species, and endothelial cell damage, as demonstrated by the damaging effect of incubating leukocytes captured from trauma patients on endothelial progenitor cells in vitro.

e) Decompressive hemicraniectomy. Answer. Introduction: Decompressive hemicraniectomy and durotomy is a surgical technique used to relieve the increased intracranial pressure and brain tissue shifts that occur in the setting of large cerebral hemisphere mass, or space-occupying, lesions. In general, the technique involves removal of bone tissue (skull) and incision of the restrictive dura mater covering the brain, allowing swollen brain tissue to herniate upwards through the surgical defect rather than downwards to compress the brainstem. Indications: Hemicraniectomy has been used to treat brain swelling and mass effect secondary to a middle cerebral artery (MCA) territory infarction, hemispheric encephalitis, and large parenchymal intracerebral hemorrhage in subarachnoid hemorrhage. Procedure: Hemicraniectomy involves removal of a substantial portion of the skull on one side of the head over the area of cerebral infarction. The landmarks for the portion of bone removed are anterior—frontal to the midpupillary line; posterior—posterior to the external auditory meatus; superior—to the lateral edge of the superior sagittal sinus; and inferior—to the floor of the middle cranial fossa at the origin of the zygomatic arch. Surgeons avoid the sigmoid and superior sagittal sinuses to prevent the possibility of sinus thrombosis or hemorrhage. Complications: Complications associated with hemicraniectomy include infection, CSF leak, shunt-dependent hydrocephalus, subdural fluid collection, and seizure.

1/ 4. Answer briefly on the following. 4x7 2 a) Earfly Gastric carcinoma – recent advances in diagnosis.. b) Nephron sparing Surgery. c) Percutaneous management of benign breast lesions. d) Infection and carcinogenesis.

Answer. a) Earfly Gastric carcinoma – recent advances in diagnosis. Introduction: Early gastric cancer (EGC) is defined as tumour confined to mucosa & submucosa irrespective of lymph node involvement. Due to wide variation in the survival of lymph node + and – cases, the definition of EGC should be modified to gastric malignancy confined to the mucosa & submucosa without the involvement of lymph nodes.

Diagnosis: To determine the depth of invasion and the presence of lymph node metastasis, the following investigations have been used: – Virtual Endoscopy – Magnifying Endoscopy – Fluorescence Endoscopy – Endoscopic Ultrasonography Virtual Endoscopy:  Using Helical CT system for 3D reconstruction with the volume rendering technique  Elevated lesions (EGC I and IIa) were better depicted rather than non-elevated lesions (EGC IIb and IIc).  Fine mucosal details, colour changes, textures and hyperaemia evident by conventional gastroscopy are not well depicted by virtual gastroscopy. Magnifying Endoscopy:  Histopathological results were compared with findings of magnifying endoscopy regarding surface structures and microvessels.  There was a definitive correlation between the small, regular mucosal pattern of sulci and ridges and differentiated carcinoma. Fluorescence Endoscopy:  Exogenously applied sensitizers (5-aminolaevulanic acid) accumulate selectively in malignant lesions and induce fluorescence after illumination with light of adequate wavelength.  Better detection of non-visible malignant or premalignant lesions Endoscopic Ultrasonography:  Useful tool in differentiating early from late carcinoma of the stomach (accuracy of 91%)  Low accuracy rate in differentiating between mucosal & submucosal cancer (accuracy rate 63.7%).  Accuracy rates for detecting intramucosal cancer using endoscopy and endosonography were 84% and 78% respectively

b) Nephron sparing Surgery. See the answer of question 4.a of Paper – III of 2011. c) Percutaneous management of benign breast lesions.

Answer. Benign breast conditions:

 Fibrocystic changes - Changes in hormone balances during normal, monthly menstrual cycles can create, for some women, symptomatic breast changes that are referred to as fibrocystic changes. Cysts often are described as benign, tiny, fluid-filled sacs that may feel like lumps. Tenderness and lump size commonly increase the week before the menstrual period and lessen a week after. The lumps may be hard or rubbery and can appear as a single breast lump that may be large or small. Fibrocystic changes also can appear as thickening of the breast tissue. Fibrocystic changes can occur in one or both breasts and are the most common cause of benign breast lumps in women age 35 to 50. It is relatively uncommon for postmenopausal women to have symptomatic breast changes because of a lack of hormone stimulation of breast tissue.  Simple cysts - Simple cysts are benign fluid-filled sacs that usually occur in both breasts. They can be single or multiple and can vary in size. Tenderness and lump size often change with the woman's menstrual cycle. Cysts may also be affected by caffeine (coffee, tea, chocolate, energy drinks).  Fibroadenomas - Fibroadenomas are the most common benign solid tumors found in the female breast. They are round, rubbery, slippery lumps that move freely in the breast when pushed. They form as the result of excess formation of lobules (milk-producing glands) and stroma (connective tissue in the breast). Fibroadenomas are usually painless. They occur most often between the ages of 20 and 40 and are more common in African-American women.  Intraductal papillomas - These are small, wart-like growths in the lining of the mammary duct near the nipple. They usually affect women 40 to 50 years of age and can produce bleeding from the nipple or nipple discharge.  Traumatic fat necrosis - This condition occurs when there is trauma (sudden injury) or surgery to the breast. This causes fat to form in lumps. The lumps are usually round, firm, hard, single, painless and in the area of a surgical scar.

Diagnostic tests:

 Fine needle aspiration (FNA): This is a non-surgical form of breast biopsy in which a small needle is used to withdraw a sample of cells from the breast lump. If the lump is a cyst (fluid- filled sac), removal of the fluid will cause the cyst to collapse. If the lump is solid, cells can be smeared onto slides for examination in the laboratory.  Core biopsy: This is similar to FNA, but a larger needle is used because actual breast tissue is removed, rather than a sampling of cells. A sample of the lump is removed, but the whole lump is not removed. The types of core biopsies include ultrasound-guided core biopsy and stereotactic core biopsy.  Ultrasound-guided core biopsy: This is a tissue sampling technique that does not require surgery. A biopsy needle is placed into the breast tissue. Ultrasound helps confirm the needle placement using sound waves reflected off breast tissue so the exact location of breast tissue is biopsied. The type of echo varies with each type of tissue. Ultrasound can distinguish many benign lesions, such as fluid-filled cysts, from solid lesions. Tissue samples are then taken through the needle.  Stereotactic core biopsy: A biopsy needle is placed into the breast tissue. Computerized mammographic pictures help confirm the needle placement using digital imaging so the exact location of breast tissue is biopsied. Tissue samples are then taken through the needle.  Open excisional biopsy: This is the surgical removal of the entire mass. The tissue is then studied under a microscope. If a rim of normal breast tissue is taken all the way around a lump (lumpectomy), biopsy can also serve as part of breast cancer treatment.

Treatment:

 Fibrocystic breast changes do not require treatment.  Large, painful simple cysts can be treated through fine needle aspiration, which may take place during or after the evaluation. After fluid is drawn from the cyst, the cyst collapses, and the pain resolves.  Fibroadenomas of certain size and if painful are recommended to be removed surgically.  Intraductal papillomas are removed surgically.  Mastitis can be treated with warm compresses and in some cases, an antibiotic medication may be prescribed. d) Infection and carcinogenesis. Answer. Bacterial:  Heliobacter pylori is known to cause MALT lymphoma.  Salmonella Typhi and cancer development: Among the several risk factors for gallbladder cancer (cholelithiasis, obesity, exposure to certain chemicals), chronic infection with S. Typhiis of great importance.Chronic carriers of this pathogen have an approximately eightfold excess risk of developing gallbladder carcinoma than non carriers and an approximately 200 fold excess risk of developing hepatobiliary carcinoma, compared with people who have had acute typhoid and have cleared the infection.  Chlamydia species and cancer development: Chlamydia pneumoniae : C. pneumoniae is a Gram negative bacillus and a compulsory intracellular parasite. It causes respiratory infection in more than 50% of adults leading to a higher incidence of pneumonia. It has been suggested that persistent C. pneumoniae inflammation correlates with increased risk of lung cancer. According to some data, elevated C. pneumoniae antibody titers have been observed in lung cancer. It has been reported that individuals with elevated IgA antibody titers to this organism have up to a twofold increased lung cancer risk. Viral: Many cancers originate from a viral infection; this is especially true in animals such as birds, but less so in humans. 12% of human cancers can be attributed to a viral infection. The mode of virally induced tumors can be divided into two, acutely transforming or slowly transforming. In acutely transforming viruses, the viral particles carry a gene that encodes for an overactive oncogene called viral-oncogene (v-onc), and the infected cell is transformed as soon as v-onc is expressed. In contrast, in slowly transforming viruses, the virus genome is inserted, especially as viral genome insertion is obligatory part of retroviruses, near a proto-oncogene in the host genome. The viral promoter or other transcription regulation elements, in turn, cause over-expression of that proto-oncogene, which, in turn, induces uncontrolled cellular proliferation. Because viral genome insertion is not specific to proto-oncogenes and the chance of insertion near that proto-oncogene is low, slowly transforming viruses have very long tumor latency compared to acutely transforming virus, which already carries the viral-oncogene.

Viruses that are known to cause cancer such as HPV (cervical cancer), Hepatitis B (liver cancer), and EBV (a type of lymphoma), are all DNA viruses. It is thought that when the virus infects a cell, it inserts a part of its own DNA near the cell growth genes, causing cell division. The group of changed cells that are formed from the first cell dividing all have the same viral DNA near the cell growth genes. The group of changed cells are now special because one of the normal controls on growth has been lost.

Depending on their location, cells can be damaged through radiation from sunshine, chemicals from cigarette smoke, and inflammation from bacterial infection or other viruses. Each cell has a chance of damage, a step on a path toward cancer. Cells often die if they are damaged, through failure of a vital process or the immune system; however, sometimes damage will knock out a single cancer gene. In an old person, there are thousands, tens of thousands or hundreds of thousands of knocked-out cells. The chance that anyone would form a cancer is very low.

When the damage occurs in any area of changed cells, something different occurs. Each of the cells has the potential for growth. The changed cells will divide quicker when the area is damaged by physical, chemical, or viral agents. A vicious circle has been set up: Damaging the area will cause the changed cells to divide, causing a greater likelihood that they will suffer knock-outs.

This model of carcinogenesis is popular because it explains why cancers grow. It would be expected that cells that are damaged through radiation would die or at least be worse off because they have fewer genes working; viruses increase the number of genes working.

One concern is that we may end up with thousands of vaccines to prevent every virus that can change our cells. Viruses can have different effects on different parts of the body. It may be possible to prevent a number of different cancers by immunizing against one viral agent. It is likely that HPV, for instance, has a role in cancers of the mucous membranes of the mouth.

Helminthiasis

Certain parasitic worms are known to be carcinogenic

These include:

 Clonorchis sinensis (the organism causing Clonorchiasis) and Opisthorchis viverrini (causing Opisthorchiasis) are associated with cholangiocarcinoma.

 Schistosoma species (the organisms causing Schistosomiasis) is associated with bladder cancer.

Chronic bacterial infections and carcinogenesis:

THE WEST BENGAL UNIVERSITY OF HEALTH SCIENCES MS (General Surgery) Examination, 2013 PAPER IV

Time Allowed: 3 Hours Full Marks: 100 Attempt all questions

1. What are the guidelines for considering liver resection in colorectal liver metastasis? Discuss the management of colorectal liver metastasis. 20 2. What are the indications and preoperative evaluation for surgery for morbid obesity? Describe in short the different types of surgery in morbid obesity. 20 3. Write short notes of the following: 5x6 a) PAIR therapy for Hydatid cyst. b) Recent trends in management of varicose veins. c) Pancreatic necrosectomy. d) Perioperative fluid management. e) Composite mesh.

4. Answer briefly on the following. 4x71/2 a) Robotic Surgery. b) PET scan in Surgery. c) Modern surgical management of haemorrhoids. d) Surgery for chronic pancreatitis.

THE WEST BENGAL UNIVERSITY OF HEALTH SCIENCES

MS (General Surgery) Examination, 2013

April 2013

PAPER IV

Time Allowed: 3 Hours Full Marks: 100 Attempt all questions

1. What are the guidelines for considering liver resection in colorectal liver metastasis? Discuss the management of colorectal liver metastasis. 20 Answer. Introduction : The most common malignant tumors of the liver are metastatic lesions. The liver is a common site of metastases from gastrointestinal tumors presumably because of dissemination through the portal venous system. The most relevant metastatic tumor of the liver to the surgeon is colorectal cancer because of the well-documented potential for long- term survival after complete resection. Guidelines:  General:  Patients under consideration of treatment of hepatic metastases should be discussed at a multidisciplinary meeting which has experience in the management of liver metastases.  A hepatobiliary multidisciplinary team (MDT) which carries out liver resection should be based in a cancer centre serving a population of at least two million. When two or three networks cooperate to create a single joint team, there should be explicit arrangements for referral between networks. (Category of evidence II ; strength of recommendation B)  Consideration of patients for resection of liver metastases should be carried out at a single high volume centre. (Category of evidence II; strength of recommendation B)

 At time of presentation of primary colorectal cancer:

 Patients with primary colorectal cancer should have a computed tomography (CT) scan of the abdomen and pelvis performed with intravenous contrast and ideally a maximum collimation of 5 mm. This should be performed preoperatively or, in the case of an emergency, as soon as practical thereafter. (Category of evidence II; strength of recommendation B)  A chest CT is ideal to assess the presence of pulmonary metastases but a chest xray is considered satisfactory. (Category of evidence III; strength of recommendation C)  The whole of the colon should be visualised to ensure a “clean colon”. (Category of evidence II; strength of recommendation B)  A baseline measurement of carcinoembryonic antigen (CEA) should be performed. (Category of evidence III; strength of recommendation C)

 Following curative resection of primary colorectal cancer:  Following treatment of the primary disease, some patients will prove to be unfit or unwilling to have further treatment and in such cases follow up is inappropriate.  Other patients will prove to have metastatic disease at presentation. Some of these will have isolated liver metastases and should be managed as described in these guidelines.  Patients undergoing R0 colorectal resection may be candidates for adjuvant chemotherapy and a further abdominal contrast enhanced CT (or magnetic resonance imaging (MRI)) should be performed following completion of chemotherapy. (Category of evidence III; Strength of recommendation C)  Where possible patients over the age of 50 years should be considered for randomisation to the FACS trial, the UK NCRN trial of follow up strategies.  If the patient is ineligible for trial inclusion, does not wish to participate in the randomised trial, or if the unit is not recruiting to the trial, the following follow up schedule is appropriate: i. CT scan of the abdomen and pelvis should be undertaken as a minimum in the two years following completion of treatment of the primary disease. (Category of evidence III; strength of recommendation C) ii. Colonoscopy repeated after five years. iii. The case for routine serial CEA measurements is unproven. (Category of evidence III; strength of recommendation C)

 Further staging investigation to detect extrahepatic involvement in patients with colorectal liver metastases:

 For a patient discovered to have isolated liver metastases, CT of the chest, abdomen, and pelvis should be performed by the liver surgery unit or using protocols agreed with that unit. The liver surgery centre will also often perform liver specific imaging by local protocol.  Biopsy of hepatic lesions should not be performed without discussion with the regional hepatobiliary unit. (Category of evidence III; strength of recommendation C)  Patients with “high risk” primary disease (T4 (perforated); C2 (apical node)) should have careful preoperative investigations that might include positron emission tomography (PET) and laparoscopy. (Category of evidence III; strength of recommendation C).

Management of colorectal liver metastasis:

 The liver is the most common site of metastases for tumor sites that drain initially via the portal circulation. Metastatic liver disease is found in 10% to 25% of patients having surgery for primary colorectal cancer. If hepatic interrogation is specifically pursued with computed tomography (CT) and intraoperative ultrasonography, the incidence appears to be close to 35% in patients with otherwise curable primary disease.  It is important to acknowledge that not all hepatic lesions in a patient with a history of colorectal cancer are metastatic disease, although a majority are. Many other mass lesions are known to occur with some frequency, and it is important to rule these out.  The ability to distinguish between these disorders and colorectal metastases relies on an adequate history of the patient, laboratory studies, and imaging studies. This combination allows accurate preoperative diagnosis in most patients. Preoperative biopsy is rarely necessary in patients with resectable disease.  As the chemotherapeutic regimens in the adjuvant and metastatic settings currently differ, patients who are not candidates for resection usually require a tissue diagnosis or definitive imaging before beginning chemotherapy.  In this setting, percutaneous biopsy may be appropriate. The presence of a new mass lesion, increasing levels of carcinoembryonic antigen (CEA), and a history of colorectal cancer should provide enough evidence of disease to justify treatment. Biopsy carries risk for tumor dissemination (which should be enough to discourage unwarranted biopsies) as well as minor risks for bleeding and pneumothorax.  Goals for management of metastatic colorectal cancer are to provide the patient with an optimum quality of live for the longest duration possible. Resection of hepatic metastases is now associated with long-term survival and low mortality such that patients who would have been denied surgical treatment in previous eras are now routinely offered locally aggressive treatment options. Presentation and diagnosis:  Presentation of liver metastases may be either synchronous or metachronous. Synchronous disease, commonly defined as liver metastasis occurring within 12 months of the colon or rectal primary, represents 13% to 25% of newly diagnosed colorectal liver metastases.  Disease is found primarily on preoperative imaging or intraoperative exploration. In these patients, the original presentation is often related to the primary neoplasm, whereas liver metastases are incidental findings.  Common presenting symptoms generally include fever, fatigue, weight loss, and anorexia. Patients may also describe a sense of abdominal fullness or even upper abdominal and right flank pain.  Physical examination may reveal a palpable liver mass or hepatomegaly, jaundice, and ascites. More commonly, the physical examination is unremarkable.  The optimal timing for synchronous liver metastases is controversial. Although a staged approach with initial resection of the primary lesion followed by hepatic resection 3 months later has been practiced, an increasing number of hepatic surgeons are utilizing a simultaneous, collaborative approach with the colorectal surgeon during the initial operation. o Metachronous disease develops in 20% to 25% of patients. o The presentation of metachronous hepatic metastases varies according to the method by which they are detected. In unscreened populations, hepatic metastases can be detected only if the patient is having symptoms referable to the metastatic process. o Other more commonly used methods of early detection in patients who previously had curative treatment for primary colorectal cancer include serial serum CEA determination and serial surveillance with cross-sectional imaging and/or ultrasonography. o Presently, most metachronous presentations are not amenable to resection because of the late presentation or detection. Thus, dedicated, systematic follow-up for patients with a resected colorectal primary would lead to identification of patients who would benefit from hepatic resection.  Diagnostic tests for hepatic colorectal metastases include serum CEA levels, CT, magnetic resonance imaging (MRI), positron emission tomography (PET), ultrasonography, and laparoscopy.  CEA level has a sensitivity of ~75% and a specificity of 90% to 95% in detecting hepatic recurrence. CEA levels also have prognostic significance in patients undergoing evaluation for hepatic metastasectomy.  Patients with a preoperative CEA level less than 200 ng/mL had a median survival of 38 months, and those with a level greater than 200 ng/mL had a median survival of 24 months.  Patients with a preoperative CEA level less than 200 ng/mL had a median survival of 38 months, and those with a level greater than 200 ng/mL had a median survival of 24 months.  Patients with a preoperative CEA level less than 200 ng/mL had a median survival of 38 months, and those with a level greater than 200 ng/mL had a median survival of 24 months.  PET is an appropriate second test in patients with increasing levels of CEA and no clear abnormalities on CT of the chest, abdomen, or pelvis. Combined CT-PET devices have also been developed that provide both physiologic and anatomical detail in the same setting.  The most important use of ultrasonography is during surgery. Intraoperative ultrasonography can detect occult colorectal metastases not seen on CT or transabdominal ultrasonography and has an overall sensitivity of 96%.  Intraoperative ultrasonography is also useful in demonstrating segmental hepatic anatomy. This is particularly important when the tumor is in proximity to the inflow or outflow vessels. The value of intraoperative ultrasonography is operator dependent but in well- trained hands has been shown to alter the preoperative surgical plan in nearly 20% of patients.  Diagnostic laparoscopy is useful prior to planned hepatic resection for colorectal metastases. It can aid in identifying lesions that may have been missed on preoperative cross-sectional imaging studies. Performance of laparoscopy does add time, expense, and its unique morbidities and has, therefore, not been universally practiced. A clinical risk score (CRS) has been described to clarify the role of pre-resectional laparoscopy yield.

Treatment:

 Systemic chemotherapy: o With ~80% of patients with colorectal hepatic metastases presenting with nonresectable disease, systemic chemotherapy represents the main if not the only form of therapy for many patients. Chemotherapy may also play a role in transforming a portion of patients with unresectable disease into resection candidates. o Systemic agents have been introduced that may elicit a better response than standard 5-fluorouracil (FU)-leucovorin (LV)–based treatment for advanced disease. Irinotecan (IR) and oxaliplatin (OX) combined with FU-LV appear to have a higher response rate than FU-LV alone. o IR has also been shown to facilitate resectability in patients initially deemed unresectable. o Addition of the monoclonal antibody against vascular endothelial growth factor, bevacizumab, to the combination chemotherapy has been shown to improve survival significantly. o The success of the modern chemotherapeutic agents for metastatic colorectal cancer is creating new opportunities for long-term survival in nonresectable patients. The application of neoadjuvant therapy for resectable disease also warrants investigation in clinical trials.  Operative management: o Hepatic resection is currently the most effective form of therapy for colorectal metastases confined to the liver. It is important to have a definition of resectability to maintain a standard for evaluation. Resectability is defined as complete gross resection while retaining a sufficient liver remnant with intact biliary drainage and vasculature. o With nearly 80% of hepatic parenchyma being safe to remove, hepatic resection should be considered in many patients. It is equally important to identify patients who have conditions and/or factors that would preclude a resectional strategy. o Accepted contraindications to metastasectomy include poor overall health, inadequate liver reserve, inability to achieve margin-negative resection, and the presence of extrahepatic disease. o Anatomic or segmental resections are currently favored over large wedge resections, although there are currently scant data on this subject. The oncologic principle of getting an adequate negative margin should be applied. Anatomic resections offer the best chance for achieving negative margins while maintaining maximum liver parenchyma. o The resection margin is also critical in trying to achieve a cure. A positive histologic margin has been shown to be associated with poor long-term survival. The optimal surgical margin width, however, remains debatable. There have been no definitive studies showing that 1 cm or greater is favorable to a grossly negative margin. Thus, the importance of achieving a negative margin favors anatomic resections over wedge resections. It is likely, however, that a wedge resection is oncologically equivalent in the event that a negative margin can be reliably obtained. o The success of resection of hepatic colorectal metastases has resulted in looking for ways to extend resectability. One technique is through preoperative portal vein embolization. The procedure is based on the physiologic phenomenon of liver atrophy of the embolized lobe and liver hypertrophy of the contralateral lobe. This augments the volume of the remaining liver and allows safe large-volume resectional strategies. The portal vein is usually reached through a percutaneous transhepatic route under ultrasound and fluoroscopic guidance.  Ablative options: o Ablative therapies are also available for patients with unresectable disease who do not have apparent extrahepatic metastases. Patients with inadequate hepatic reserve despite technically resectable lesions are also candidates. Local ablative techniques such as radiofrequency ablation (RFA) and cryoablation treat tumors in situ, effecting tumor killing by thermal mechanisms.

 RFA can be employed through percutaneous, laparoscopic, thoracoscopic, and open procedures.  Size and location of the tumor dictate choice of treatment. Ultrasound guidance facilitates accurate and safe probe placement.  Small lesions(< 3 cm) located on the periphery are best suited for percutaneous approach.  The laparoscopic approach with intraoperative ultrasonography has higher accuracy for detecting hepatic lesions than transcutaneous ultrasonography and is safer for mobilizing and ablating tumors that are close to or in contact with surrounding organs.  A thoracoscopic approach is useful for tumors situated on the dome of the liver that are difficult to reach percutaneously or laparoscopically.  When a question of efficacy or safety arises, an open approach should be used. o Portal vein embolisation: Portal vein embolization (PVE) is another modality used preoperatively for patients where the extent of liver resection is expected to result in less than the optimal functional liver volume of 25% to 40%, necessary to prevent postoperative liver failure o Hepatic arterial infusion: . The use of hepatic arterial infusion pump (HAIP) placement and administration of chemotherapy is based on the observation that hepatic tumors derive their blood supply primarily from the hepatic artery, in contrast to normal hepatic parenchyma, which is principally supplied by the portal vein. Therefore, infusion of chemotherapeutic agents through the hepatic arterial circulation should lead to high concentrations of the agents within tumor cells while sparing the normal hepatic parenchyma. . In addition, several agents are efficiently extracted within the liver such that systemic concentrations and concomitant toxicities are further reduced. Fluorodeoxyuridine is the most common agent administered by the intra-arterial route, although FU-LV has also been delivered in this fashion.

2. What are the indications and preoperative evaluation for surgery for morbid obesity? Describe in short the different types of surgery in morbid obesity. 20 Answer. Indications:  Age greater 18 years  BMI above 40kg/m 2  BMI between 30kg/m 2 and 40 kg/m 2 with a major weight related medical condition (hypertension, diabetes, sleep apnea, fatty liver disease, heart disease)  Weight at least 100 pounds overweight. Criteria for obesity surgery, including:  Documented supervised weight loss attempt with failure  Medically cleared and approved by our multidisciplinary bariatric team members, including required evaluation by our bariatrician, psychologist, dietitian and surgeon  Mentally and emotionally prepared and motivated  No substance abuse or unhealthy lifestyle habits (drug abuse, alcohol abuse and tobacco use)  Have good understanding of procedure and lifestyle changes required for long-term success.  Have realistic expectations regarding outcome after surgery.  Have intact support structure to be successful following bariatric surgery (work, family, friends who are engaged in process with patient)  Weigh less than 400 pounds Pre-operative evaluation:  Initial screening:  History and physical examinations: o Medical and Surgical History o Family History/Social History o Medications/Allergies o Diet History o Physical Exam- HT, WT, BMI.  Laboratory investigations: o Chemistry, liver function, renal function o Lipid Profile o CBC o Iron Profile- TIBC, total iron, saturation o B-1, B-12 levels o HbA1c o H-Pylori o Drug Screen (optional)  Radiology: o U/S of liver/GB o CXR o UGI Swallow Study (optional)  Cardiac: o EKG o Venous Doppler Studies (optional)

The multidisciplinary approach includes: o Medical management of comorbidities o Dietary instruction, o Exercise training, Specialized nursing care and psychological assistance as needed. Preoperative Behavior Change: o Preoperative exercise program. o Patients sometimes asked to maintain body weight or lose weight prior to surgery. o Patients asked to quit smoking prior to surgery. Dietary Evaluation: Registered Dietitian o Address dietary concerns and begin making changes now o Avoid the Last Supper Syndrome o Diabetes Education Preoperative exercise program: o Assessment. o Mobility Issues. o Physical Conditioning. o Education. o Motivation.

Gastro-Intestinal Birth control Cardiac Pulmonary Endocrine Evaluation: counseling Risk assessment Evaluation Evaluation o Endoscopy. o Absence of o Stress Testing o Obstructive o Diabetes o Ulcers (Helicobacter pregnancy. o Echocardiogram Sleep Apnea Management. pylori). o Birth control. o Medication (testing and o Diabetes o Esophageal Disorders. o Risky adjustment treatment). Education. o Irritable Bowel pregnancy in o Cardiac o Asthma. o Thyroid Syndrome. the early post- Clearance o Smokers. disease. o Crohn’s Disease. op. period (1- 2 years). o Weight loss may improve fertility.

See the answer of question 1 of Paper – IV of 2010.

3. Write short notes of the following: 5x6 a) PAIR therapy for Hydatid cyst. b) Recent trends in management of varicose veins. c) Pancreatic necrosectomy. d) Perioperative fluid management. e) Composite mesh.

a) PAIR therapy for Hydatid cyst. Answer. PAIR – Percutaneous therapy by puncture, aspiration, injection and re-aspiration, combined with albendazole is a good method for the treatment of hydatid cyst. Scolicidal agents used in PAIR:

 95% sterile ethanol.  Absolute alcohol.  Mebendazole 2.4ug/ml.

Indications for PAIR: Contraindications to Complications of PAIR: PAIR:

.

 Refusal of surgery.  Inaccesible or hazardous  Urticaria.  Inoperable cases. location of cyst.  Anaphylaxis.  Type I, II and III cysts.  Multiple septal divisions.  Subcapsular haematoma.  Cystic lesion>= 5cm in  Dead or inactive cysts  Fever. diameter.  Communicating cysts.  Biliary fistula.  Multiple cysts >= 5cm in  Lung or bone cysts.  Secondary infection of diameter in different liver cyst cavity. segments.  Hypotension/hypotensive  Relapse post surgery. shock.  Lack of response to chemotherapy.

b) Recent trends in management of varicose veins. Answer. Physical examination:

The physical examination should include inspection of the limb in a standing position. Notation should be made of:

 Ankle venous flare  Telangiectasias (dilated interdermal venules <1 mm)  Reticular veins (non-palpable subdermal veins 1–3 mm)  Varicose veins (>3 mm)  Oedema  Skin pigmentation  Venous ulcers – chart on a diagram and note size, depth, character of the ulcer base.

Photo documentation is increasingly being used to:

 Evaluate the response to therapy  Track the natural progression of the disease  Decide when to refer  Keep a medicolegal record (remember to get consent from the patient to take and store a photograph as part of the medical record).

It is also important to document arterial pulses in all patients. In unusually sited or appearing varicosities, consider anteriovenous malformation. Traditional non-invasive office based testing (eg. Trendelenberg) has largely been replaced by duplex ultrasound.

Diagnostic testing:

Venous duplex ultrasound has become the standard of care for the investigation of varicose veins. Duplex ultrasound should document the:  Patency and competency of the deep system  Patency and competency of the superficial system, including site of incompetent segments, site of junctions and perforators  Location and relationships of the sapheno-popliteal junction  Diameter of the affected vein – this may be important in some endoluminal applications.

Magnetic resonance venography: o Magnetic resonance venography is safe, does not involve ionizing radiation as does phlebography and is not operator dependent like duplex ultrasound. o At present Magnetic resonance venography is in its infancy but its role will undoubtedly increase in future. o The efficacy of Magnetic resonance venography in deep vein thrombosis have been assessed against duplex ultrasound, contrast phlebography or both using a variety of MR techniques and it has been shown that in many cases MR can be as effective as or , on occasion, superior to these other imaging studies19. But, it is expensive and not available at all places.

Treatment:

 Conservative treatment with stockings and external compression is an acceptable alternative to surgery, but worsening cutaneous findings or symptoms despite these measure usually warrant intervention. Nonetheless, a patient's desire for surgical management over conservative treatment or for cosmetic purposes alone are both reasonable relative indications for surgery.  Patients with venous outflow obstruction should not have their varicosities ablated, because they are important bypass pathways that allow blood to flow around the obstruction.  Those patients who cannot remain active enough to reduce the risk of postoperative deep vein thrombosis (DVT) should not undergo surgery.  Surgery during pregnancy is contraindicated because many varicose veins of pregnancy spontaneously regress after delivery.

Choice of treatment depends on many factors, including local expertise. Taking into account age, general health and symptomatology, patients with varicose veins may be offered one or more of the following management options:

 Reassurance only  Conservative management with compression therapy  Endovenous therapy o Sclerotherapy – using liquid or foam (can be ultrasound guided, UGFS) o Endovenous laser therapy (EVLT) o Radiofrequency ablation (RFA) o Mechanical – using steam or rotating catheter  surgical treatment.

A recent review of the literature revealed the following:  No significant difference in short term outcomes between the modalities  Complications were few and mostly minor  New varices may develop in 10% of patients within the first year after any treatment  The technical failure rate was highest with ugfs  Both RFA and UGFS were associated with a faster recovery and less postoperative pain than evlt and stripping  The shortest time to return to work was seen in the ugfs and rfa groups  There were significantly more cases of superficial phlebitis in the ugfs and rfa groups  The mean cost per treatment was lowest in the UGFS group and highest in the RFA and EVLT groups.

Recent guidelines recommend against UGFS alone for the treatment of GSV incompetence.

 Overly large (>2 cm diameter) – thermal treatment may fail, and there is a high risk of deep vein thrombosis  Overly tortuous – endoluminal catheters may not pass up the vein  Very close to the skin – skin burns are more likely, and in cases of thrombophlebitis – endoluminal catheters may not pass up the vein.

Open surgery: Surgical treatment of varicose veins: Indications for Treatment: o When recognizable changes appear in the skin of the lower leg, i.e. presence of ankle flare, lipodermatosclerosis or venous ulcer. o If there are problems with hemorrhage or recurrent superficial thrombophlebitis o If the patient wishes to be treated for symptomatic or cosmetic reasons. o Those who are medically unfit because of presence of varicose veins.

Options available for surgical treatment of varicose veins are as follows:

nterruptions

Surgical interventions should be individualised according to the patient’s preoperative evaluation. An important principle is to ‘treat the patient and not the duplex scan’. A combination of ligation, axial stripping, and stab phlebectomy may be applied as needed to the GSV, SSV, tributary veins and perforating veins.

Alternatives to stripping: New venous surgical techniques have been developed in an effort to reduce the number and size of lower-extremity incisions and hematomas, to eliminate postoperative discoloration, and to reduce the recuperation time. o Radio frequency (RF) ablation: The intervention employs radiofrequency (RF) energy mediated heating of the vein wall to destroy the intima and denature collagen in the media with resulting fibrous occlusion of the vein. o Endovenous laser therapy: Endovenous laser therapy (EVLT) is similar to RF ablation, but laser energy is used for ablation of the saphenous vein.The procedure is faster and easier to perform than RF ablation and there is no size limitation of the saphenous vein that can be treated. o Foam Sclerotherapy: An increasing number of authors have recently reported successful injection of incompetent GSV with 3% polidocanol in the form of foam.

Saphenopopliteal Ligation: Some surgeons advocate routine stripping of the short saphenous vein should be disconnected and never stripped. The short saphenous vein operation should be carried out first, if a long saphenous vein operation is to be performed under the same anaesthetic. Ligation of the Lower Leg Perforating Veins: Surgery for these veins is usually required in patients with lipodermatosclerosis or ulceration. The presence of incompetent perforators in patients with advanced CVI (clinical classes 4 to 6) is an indication for surgical treatment in a fit patient. Whereas open perforator ligation is done only in those with healed ulceration, a clean, granulating open ulcer is not a contraindication for subfascial endoscopic perforator vein surgery (SEPS). Subfascial endoscopic perforator surgery (SEPS): o The major drawback of open procedure was a high incidence of wound complications. Edwards in 1976 designed a device called the phlebotome to ablate the incompetent perforators from sites remote from the diseased skin. o SEPS is a minimally invasive surgical technique used to treat chronic venous ulcers caused by perforating veins that may have been damaged due to deep vein thrombosis or chronic venous insufficiency. o The goal of treatment is to heal the ulcer to prevent serious complications or to prevent ulcer recurrence. o Patients with chronic venous ulcers may have developed chronic venous insufficiency as a result of damage to their perforating veins, which are veins located above the ankle that carry blood from the superficial veins into the deep veins. o If not corrected, patients may experience prolonged symptoms of severe chronic venous insufficiency and recurrent venous stasis ulcer formation. c) Pancreatic necrosectomy. Answer.  Pancreatic debridement is indicated for patients with pancreatic necrosis and progressive clinical sepsis as a complication of severe acute pancreatitis. Pancreatic and peripancreatic necrosis occur in approximately 20 percent of patients with pancreatitis, as a result of inflammation and vascular compromise.  Pancreatic necrosis can lead to secondary infection or symptomatic sterile necrosis, which is characterized by chronic low grade fever, nausea, lethargy, and inability to eat. Both infected pancreatic necrosis and symptomatic sterile necrosis are accepted indications for debridement.  The goal of pancreatic debridement is to excise all dead and devitalized pancreatic and peripancreatic tissue while preserving viable functioning pancreas, controlling resultant pancreatic fistulas, and limiting extraneous organ damage.  For patients with biliary pancreatitis, cholecystectomy with intraoperative cholangiography is an important secondary objective of the surgery because it will prevent recurrent disease.  Infected pancreatic necrosis — Secondary infection of the necrotic pancreatic or peripancreatic tissue with either bacteria or fungus occurs soon after the initial inflammatory reaction subsides and is heralded by tachycardia, hypotension, fevers, and/or deteriorating organ function.

Open and Minimally Invasive Approaches to the Treatment of Pancreatic Necrosis

Open surgery approaches Pancreatic resection Necrosectomy + wide tube drainage Necrosectomy + relaparotomy (staged reexploration) Necrosectomy + drainage + relaparotomy Necrosectomy + laparostomy ± open packing Necrosectomy + drainage + closed continuous lavage Minimally invasive approaches Laparoscopic necrosectomy Laparoscopic intracavity necrosectomy Laparoscopic assisted percutaneous drainage Laparoscopic transgastric necrosectomy Percutaneous necrosectomy and sinus tract endoscopy MRI–radiologically assisted necrosectomy Translumbar extraperitoneal retroperitoneoscopy Video-assisted retroperitoneal débridement

 Necrosectomy is a careful process, best accomplished by an educated finger. The extent of the cavity can be explored and the gentle separation of necrotic material accomplished. Necrotic extensions from the primary cavity need to be explored, often into the root of the small bowel mesentery and down the retrocolic gutters.  Care must be taken to remove only what easily separates and to avoid injury to major vessels. The removal of necrotic material may be assisted by irrigation, pulsatile irrigation, gauze, and sponge forceps. When contained by a mature wall, it is advisable to avoid opening up the area too widely. The next step is placement of large-bore, soft, dependent drains to cover all the regions of what is often a complex area.  Continuous lavage with peritoneal dialysis fluid, at flow rates of 300–1000 mL/h, may reduce the need to reoperate and often is continued for 2–3 weeks.  The most common postoperative complications are hemorrhage and fistulization (pancreas, small and large intestine). The use of packing is lifesaving for major hemorrhage that occurs at the time of necrosectomy, but when used routinely, it is associated with a higher incidence of enteric fistulas. d) Perioperative fluid management. Answer. Goals of Fluid Administration: • O2 delivery / blood flow - perfusion • Maintain electrolyte composition, • Glycemia, • Body temperature Requriment of fluid depends on: – Type of patient – Type of surgery – Amount of trauma – Acute injury vs. elective operation – Anesthetic, positioning Classic fluid management: • Deficits • Maintenance • 3rd Space • Blood loss

 Deficits: – Preop NPO (hourly maintenance x duration) – Preop bowel preparation (1-1.5L) – Preop blood loss (trauma) or fluid loss (burns) Typically replaced over first 2-4 hours.

 Maintenance: • 4 ml/kg/hr for first 10 kg of body weight • 2 ml/kg/hr for 2nd 10 kg of body weight • 1 ml/kg/hr for each kg of body weight above 20 kg

 Replace fluid losses: • Third space” 2-10 ml/kg/hr • Blood losses: • 3 to 1 ratio of crystalloid to EBL • 1 to 1 for colloid or blood (or hypertonic saline) e) Composite mesh. See the answer of question 4.a of Paper –IV of 2012

1/ 4. Answer briefly on the following. 4x7 2 a) Robotic Surgery. b) PET scan in Surgery. c) Modern surgical management of haemorrhoids. d) Surgery for chronic pancreatitis.

Answer. a) Robotic Surgery, computer-assisted surgery, and robotically-assisted surgery are terms for technological developments that use robotic systems to aid in surgical procedures. Robotically-assisted surgery was developed to overcome the limitations of pre-existing minimally-invasive surgical procedures and to enhance the capabilities of surgeons performing open surgery. Comparison to traditional methods: o Major advances aided by surgical robots have been remote surgery, minimally invasive surgery and unmanned surgery. Due to robotic use, the surgery is done with precision, miniaturization, smaller incisions; decreased blood loss, less pain, and quicker healing time. Articulation beyond normal manipulation and three-dimensional magnification helps resulting in improved ergonomics. Due to these techniques there is a reduced duration of hospital stays, blood loss, transfusions, and use of pain medication. o The robot normally costs $1,390,000 and while its disposable supply cost is normally $1,500 per procedure, the cost of the procedure is higher. o Additional surgical training is needed to operate the system.Surgeons report that, although the manufacturers of such systems provide training on this new technology, the learning phase is intensive and surgeons must operate on twelve to eighteen patients before they adapt. During the training phase, minimally invasive operations can take up to twice as long as traditional surgery, leading to operating room tie ups and surgical staffs keeping patients under anesthesia for longer periods. Patient surveys indicate they chose the procedure based on expectations of decreased morbidity, improved outcomes, reduced blood loss and less pain. o Compared with other minimally invasive surgery approaches, robot-assisted surgery gives the surgeon better control over the surgical instruments and a better view of the surgical site. In addition, surgeons no longer have to stand throughout the surgery and do not tire as quickly. Naturally occurring hand tremors are filtered out by the robot’s computer software. Finally, the surgical robot can continuously be used by rotating surgery teams.

Uses:

 General surgery: In early 2000 the field of general surgical interventions with the daVinci device was explored by surgeons at Ohio State University.  Cardiothoracic surgery: Robot-assisted MIDCAB and Endoscopic coronary artery bypass (TECAB) operations are being performed with the Da Vinci system. Mitral valve repairs and replacements have been performed. The Ohio State University, Columbus has performed CABG, mitral valve, esophagectomy, lung resection, tumor resections, among other robotic assisted procedures and serves as a training site for other surgeons. In 2002, surgeons at the Cleveland Clinic in Florida reported and published their preliminary experience with minimally invasive "hybrid" procedures. These procedures combined robotic revascularization and coronary stenting and further expanded the role of robots in coronary bypass to patients with disease in multiple vessels. Ongoing research on the outcomes of robotic assisted CABG and hybrid CABG is being done.  Cardiology and electrophysiology: The Stereotaxis Magnetic Navigation System (MNS) has been developed to increase precision and safety in ablation procedures for arrhythmias and atrial fibrillation while reducing radiation exposure for the patient and physician, and the system utilizes two magnets to remotely steerable catheters.

At present, three types of heart surgery are being performed on a routine basis using robotic surgery systems. These three surgery types are:

 Atrial septal defect repair – the repair of a hole between the two upper chambers of the heart.  Mitral valve repair – the repair of the valve that prevents blood from regurgitating back into the upper heart chambers during contractions of the heart.  Coronary artery bypass – rerouting of blood supply by bypassing blocked arteries that provide blood to the heart.

 Colon and rectal surgery:Results to date indicate that robotic-assisted colorectal procedures outcomes are "no worse" than the results in the now "traditional" laparoscopic colorectal operations. Robotic-assisted colorectal surgery appears to be safe as well.  Gastrointestinal surgery: Multiple types of procedures have been performed with either the 'Zeus' or da Vinci robot systems, including bariatric surgery and gastrectomy for cancer. Surgeons at various universities initially published case series demonstrating different techniques and the feasibility of GI surgery using the robotic devices. Specific procedures have been more fully evaluated, specifically esophageal fundoplication for the treatment of gastroesophageal reflux and Heller myotomy for the treatment of achalasia.

Other gastrointestinal procedures including colon resection, pancreatectomy, esophagectomy and robotic approaches to pelvic disease have also been reported.

 Gynecology: Robotic surgery in gynecology is of uncertain benefit with it being unclear if it affects rates of complications. Using the robotic system, gynecologists can perform hysterectomies, myomectomies, and lymph node biopsies.  Neurosurgery: Several systems for stereotactic intervention are currently on the market. The NeuroMate was the first neurosurgical robot, commercially available in 1997.  Orthopedics: The ROBODOC system was released in 1992 by Integrated Surgical Systems, Inc. which merged into CUREXO Technology Corporation.The Navio System is a navigated, robotics-assisted surgical system that uses a CT free approach to assist in partial knee replacement surgery.  Pediatrics: Surgical robotics has been used in many types of pediatric surgical procedures including: tracheoesophageal fistula repair, cholecystectomy, nissen fundoplication, morgagni's hernia repair, kasai portoenterostomy, congenital diaphragmatic hernia repair, and others.  Radiosurgery: The Cyber Knife Robotic Radiosurgery System uses image guidance and computer controlled robotics to treat tumors throughout the body by delivering multiple beams of high-energy radiation to the tumor from virtually any direction.  Transplant surgery: Transplant surgery (organ transplantation) has been considered as highly technically demanding and virtually unobtainable by means of conventional laparoscopy. The development of robotic technology and its associated high resolution capabilities, three dimensional visual system, wrist type motion and fine instruments, gave opportunity for highly complex procedures to be completed in a minimally invasive fashion. Subsequently, the first fully robotic kidney transplantations were performed in the late 2000s. The use of the robotic-assisted approach has allowed kidneys to be transplanted with minimal incisions, which has virtually alleviated wound complications and significantly shortened the recovery period. In this way, robotic kidney transplantation could be considered as the biggest advance in surgical technique for this procedure since its creation more than half a century ago.  Urology: Robotic surgery in the field of urology has become very popular. It has been most extensively applied for excision of prostate cancer because of difficult anatomical access. It is also utilized for kidney cancer surgeries and to lesser extent surgeries of the bladder.  Vascular surgery: In September 2010, the first robotic operations with Hansen Medical's Magellan Robotic System at the femoral vasculature were performed at the University Medical Centre Ljubljana (UMC Ljubljana), Slovenia. b) PET scan in Surgery. Answer. See the answer of question 3.b of Paper – IV of 2008. c) Modern surgical management of haemorrhoids.  Scalpel surgery is now rarely needed.  The treatment choices for internal haemorrhoids include infrared coagulation, radiofrequency coagulation, direct current coagulation, rubber band ligation, sclerotherapy, cryosurgery, scalpel surgery, and laser surgery.  Scalpel surgery is generally reserved for advanced fourth degree haemorrhoids and is most often done on inpatients. Laser surgery is said to be less painful, but this has proved difficult to verify.  Sclerotherapy is usually indicated only in first and second degree lesions.  Cryotherapy is also little used because of the profuse and prolonged discharge, the complications such as excessive sloughing and sphincter injury, and the poor results.  The least expensive and possibly the most widely used equipment is a rubber band ligator. This is suitable for first to third degree haemorrhoids. The bands are easy to apply, but the drawback is that two people are needed, one to hold the anoscope and the other to apply the bands. The treatment can cause severe pain if the bands are placed too low, and there is a small risk of perineal sepsis, which can, very rarely, be fatal.  The infrared coagulator is gaining rapid acceptance for outpatient treatment of internal first and second degree haemorrhoids and some third degree ones. o LigaSure and Harmonic Scalpel haemorrhoidectomy: Both LigaSure (Valleylab, Covidien) and Harmonic Scalpel (Ethicon Endo-Surgery) have been used more recently for haemorrhoidectomy due to their ability to coagulate and cut tissue. LigaSure is a bipolar electrothermal device, while Harmonic Scalpel relies on ultrasonic waves that have their effect locally with reduced lateral thermal effect; the theoretical advantage of using such devices being a reduction in tissue damage adjacent to the excised haemorrhoidal tissue. Results from trials comparing Harmonic Scalpel with more traditional methods of haemorrhoidectomy have generated differing conclusions. o Doppler-guided, transanal hemorrhoidal dearterialization is a minimally invasive treatment using an ultrasound doppler to accurately locate the arterial blood inflow. These arteries are then "tied off" and the prolapsed tissue is sutured back to its normal position. It has a slightly higher recurrence rate, but fewer complications compared to a hemorrhoidectomy. Stapled hemorrhoidectomy, also known as [procedure for prolapsing o haemorrhoids (PPH)], involves the removal of much of the abnormally enlarged hemorrhoidal tissue, followed by a repositioning of the remaining hemorrhoidal tissue back to its normal anatomical position. It is generally less painful and is associated with faster healing compared to complete removal of hemorrhoids. However, the chance of symptomatic hemorrhoids returning is greater than for conventional hemorroidectomy so it is typically only recommended for grade 2 or 3 disease.

d) Surgery for chronic pancreatitis.  Indications for Surgery in Chronic Pancreatitis: Intractable pain Pancreatic ductal stenosis Biliary obstruction (Wadsworth syndrome) Duodenal obstruction Left sided portal hypertension from splenic vein thrombosis Colonic obstruction Pseudocyst Pancreatic ascites Pancreatic fistula Pancreatic carcinoma

• Pain in chronic pancreatitis

Dialated pancreatic Longitudinal PJ duct HRCT No question of malignancy Distal disease ? Malignancy Distal pancreatectomy

No question of Normal/ narrow pancreatic duct ERCP/EUS malignancy Malignancy Pancreaticoduodenectomy / Local head resection +/- drainage

Pancreatico duodenectomy / distal pancreatectomy

The type of operation depends on the anatomy of the pancreatic ductal system, and whether or not the pancreas is diffusely involved with the disease or it involves one part of the gland more than the others. Operations to relieve pain in these patients are designed to either (1) drain a dilated pancreatic ductal system, or (2) resect diseased pancreatic tissue if the duct is not enlarged. The main pancreatic duct normally measures 4 to 5 mm in the head, 3 to 4 mm in the body, and 2 to 3 mm in the tail of the pancreas. The duct is considered dilated when it is at least 7 mm in diameter in the body of the gland. Thus, patients with a dilated pancreatic duct (>7 mm in the body of the gland) are candidates for a drainage operation that decompresses the duct (longitudinal pancreaticojejunostomy, ). Those with a duct that is of normal caliber will probably require resection of a part of the pancreas, usually the head of the gland (pancreaticoduodenectomy, Frey or Beger procedure).

Choice of Surgical Procedures for Treatment of Pain in Chronic Pancreatitis Duct drainage procedure Lateral pancreaticojejunostomy (Partington-Rochelle modification of Puestow procedure) Combined duct drainage-resection procedures Pancreaticoduodenectomy (Kausch-Whipple) Longitudinal pancreaticojejunostomy with local head resection (Frey) Duodenum-preserving resection of the head of the pancreas (Beger) Pure resection procedure Total pancreatectomy with islet autotransplantation Neuroablative procedure Thoracic splanchnicectomy

THE WEST BENGAL UNIVERSITY OF HEALTH SCIENCES MS (General Surgery) Examination, 2012 PAPER IV

Time Allowed: 3 Hours Full Marks: 100 Attempt all questions 1. Define GIST. Discuss the pathogenesis and characteristics of GIST. Discuss the treatment modalities of GIST. 5+5+10 2. Discuss recent trends regarding management of Axilla in breast cancer. 20

3. Write short notes of the following: 5 x 6 a) Gastric outlet obstruction in adults. b) Laparoscopic Total Mesorectal Excision( TME) for rectal cancer. c) Complication in endovascular stents. d) Portal vein embolisation. e) Preoperative Ultrasound.

4. Answer briefly on the following: 4 x 71/2 a) Prosthesis in Hernia Surgery. b) Oncogenes. c) Uroflowmetry. d) Surgery for Advanced Gall Bladder cancer.

THE WEST BENGAL UNIVERSITY OF HEALTH SCIENCES

MS (General Surgery) Examination, 2012

April 2012

PAPER IV

Time Allowed: 3 Hours Full Marks: 100 Attempt all questions 1. Define GIST. Discuss the pathogenesis and characteristics of GIST. Discuss the treatment modalities of GIST. 5+5+10

Answer. Gastrointestinal stromal tumors (GIST) are rare malignancies. Although they are the most common sarcoma of the gastrointestinal (GI) tract, they represent only 0.2% of all GI tumors. Pathologic Features:

 GIST exhibit heterogeneous histologic features.GIST are thought to arise from the interstitial cells of Cajal (ICC), which are components of the intestinal autonomic nervous system that serve as pacemakers regulating intestinal peristalsis.  Mutations of the KIT proto-oncogene in the vast majority of GIST. KIT is a receptor tyrosine kinase that is activated when bound to a ligand known as steel factor or stem cell factor.KIT is important in the development and maintenance of components of hematopoiesis, gametogenesis, and intestinal pacemaker cells. GIST are now identified by the near universal expression of the CD117 antigen (~95%), part of the KIT receptor, in the appropriate histopathologic context. CD117 expression is characteristic of most GIST, but not of other gastrointestinal smooth muscle tumors, which are more likely to express high levels of desmin and smooth muscle actin.

 Molecular Biology Of Gastrointestinal Stromal Tumors: These tumors are nearly universally characterized by the expression of KIT, a transmembrane receptor tyrosine kinase encoded by the c-kit protooncogene and recognized by the immunohistochemical stain for CD117, an antigen to an epitope on the extramembranous portion of the KIT molecule.  The c-kit gene is a cellular homologue of the v-kit oncogene found in the genome of the feline sarcoma virus. Stem cell factor is the natural ligand for KIT, and under normal physiologic conditions, two molecules of KIT form a dimer by binding to two molecules of the ligand, with the resulting dimerization leading to activation of the intracellular tyrosine kinase. Ligand independent activation of the KIT kinase leads to signal transduction abnormalities favoring proliferation and enhancement of cell survival mechanisms.  The gain-of-function kinase activity results in ligand-independent KIT autophosphorylation and constitutive activation and is probably a hyperproliferative transforming event. The mutations are often physically located within 11 amino acids (Lys-550 to Val-560) in the juxtamembrane domain (exon 11) and can be characterized as either deletions, point mutations, or insertions. In the unusual event that these mutations occur in a germ cell line, the family lineage is characterized by instances of multiple GISTs. Further compelling evidence for KIT mutation in the pathogenesis of GIST has been substantiated by transfection experiments of mutated KIT leading to cellular clonal transformation of the transfectant.

Characteistics of GIST:

 GIST is most commonly diagnosed in adults 50 to 80 years of age, with a mean age of 60.  The male-to-female ratio is approximately 1:1.  The majority of patients with GIST are symptomatic, with the most common symptoms being abdominal pain, early satiety, and bloating related to the presence of a space-occupying mass. GI bleeding and anemia are frequently noted and are due to erosion of mucosa by the tumor, even though GISTs originate within the muscular layer of the bowel wall and are often manifested by evidence of an extrinsic mass.  The most common location is the stomach (60% to 70%), followed by the small bowel (20% to 25%), colorectum (5%), and esophagus (5%). Rarely, cases of GIST originating within the retroperitoneum, omentum, appendix, gallbladder, pancreas, and mesentery have been described.  Approximately 20% of GISTs are asymptomatic and discovered incidentally by radiographic imaging or endoscopy, or they may be an unexpected surgical finding . These incidental tumors are often asymptomatic, noncystic submucosal masses most commonly found during endoscopy.  Recommendations for management of an asymptomatic incidental GIST with regard to biopsy or resection are not based on large cohort follow-up studies. However, in general, even a subcentimeter, submucosal suspected GIST should undergo biopsy or be removed, or both, if the clinical situation allows because of the necessity for long-term observation and the potential for malignant clinical progression based on the defined risk profile.  Although some GISTs are small submucosal solid tumors found incidentally, the majority are larger than 5 cm and are symptomatic because they are a space-occupying abdominal mass. GI bleeding and the insidious anemia of chronic blood loss are not uncommon with large necrotic GISTs.

Management:

 The initial work-up should consist of a history and physical examination, which may reveal the presence of a palpable abdominal mass, followed by a cross-sectional abdominal imaging study, usually a contrast-enhanced computed tomography (CT) scan.  Routine chest radiographs and blood work, including liver function tests, are indicated.  Endoscopy, endoscopic ultrasound, and possibly endoscopic biopsy can be recommended if the clinical situation warrants because the majority of GISTs are of gastric origin.  Early surgical involvement should address the potential for complete resection, and in patients with locally advanced or metastatic disease, consultation with medical oncology, evaluation for systemic therapy with imatinib mesylate, and close surgical follow-up should be considered. Close surgical follow-up is especially important for patients with large GISTs involving bowel mucosa, which may subsequently bleed either intra-abdominally or intraluminally as a manifestation of rapid tumor response after the initiation of systemic therapy. Percutaneous core or intraoperative biopsy of a suspected GIST that is localized and presumed to be resectable is not necessary. These tumors tend to be soft, well vascularized, and friable, and tumor spill or spontaneous rupture portends a poor prognosis and compounds the difficulty of treatment decisions. It is relatively safe to biopsy the tumor by endoscopic means. Image-guided percutaneous tumor biopsy is considered as part of the work-up, however, for a metastatic tumor or a primary tumor that is marginally resectable. Additionally, careful biopsy and tissue diagnosis can be considered when immediate tumor resection could lead to considerable morbidity or functional disability.

Classification of Primary Gastrointestinal Stromal Tumors by Risk of Metastasis

Risk Category Size Mitotic Count Very Low <2 cm <5 per 50 HPFs* Low 2–5 cm <5 per 50 HPFs Intermediate <5 cm 6–10 per 50 HPFs 5–10 cm <5 per 50 HPFs High >5 cm >5 per 50 HPFs >10 cm Any mitotic rate Any size >10 per 50 HPFs

HPF, high powered field

Targeted Therapy: Imatinib mesylate (Gleevec), a rationally designed small-molecule oral drug that is a selective inhibitor of type 3 tyrosine kinases, showed remarkable efficacy in preclinical studies against the KIT oncoprotein. This activity was manifested by antiproliferative effects and a measurable decrease in the tyrosine phosphorylation of KIT in KIT-expressing malignant cell lines

A partial explanation for the GIST malignant phenotype without KIT mutant protein may be found in the recent identification of PDGFR-α gain-of-function mutations in GIST samples. The uncommon patient with a PDGFR-α mutation can have a GIST that is morphologically and clinically identical to those with a KIT mutation.

GIST patients harboring a KIT mutation outside of exon 11 are less likely to respond to imatinib mesylate. They may therefore be candidates for alternative therapies as they become available. It is especially important, however, to emphasize that approximately 5% of GISTs will be KIT “negative” by CD117 immunostaining. These KIT-negative GISTs, when compared with KIT- positive GISTs, are more likely to have epithelioid morphology, arise in the omentum/peritoneal surface, and contain PDGFR mutations. These GISTs may contain imatinib mesylate–sensitive clones, and therefore patients with morphologically and clinically confirmed KIT-negative GIST should not be denied a trial of imatinib mesylate therapy. Imaging:

 Cross-sectional imaging, particularly in the initial work-up of a patient with suspected primary or recurrent GIST, is the diagnostic procedure of choice. A contrast-enhanced (oral and intravenous) CT scan is recommended and generally allows assessment of the extent of the primary, as well as the potential presence of metastatic disease.  The typical primary GIST will be manifested as an intestinal-based mass with the bulk of the tumor extrinsic to the bowel, thus often providing an opportunity for complete resection, even in patients with a large primary tumour.  In many instances the primary tumor, although large, may be pedunculated, particularly if originating from the stomach.  The frequency of metastasis of GIST to the lung is quite low, and therefore routine screening chest CT is not necessary.  The characteristic CT scan changes in GIST, consistent with a favorable response to a molecularly targeted agent, include a change in density of the measurable tumor to a more myxoid or hypodense appearance rather than a definitive decrease in size. This response can take place within a month of initiation of therapy and can be quantitated by measurement of Hounsfield units.  Within 2 months of therapy a decrease in tumor size can be considered predictive of a beneficial response, although a maximal decrease in tumor size may take 6 months or longer. Conversely, drug resistance may be manifested as the appearance of a small intratumoral nodule without a change in overall density or size of the tumor mass  Functional imaging with 18F-fluorodeoxyglucose (FDG)–positron emission tomography (PET) can be an important adjunct to standard CT by assessing early changes in metabolic activity in GIST. FDG-PET can provide an immediate and sensitive measure of response in GISTs, which are generally PET avid tumors. Imatinib mesylate may have a dramatic effect on abrogation of GIST cell glycolysis.  If FDG-PET is being considered as a diagnostic modality to monitor continued response, detect recurrence, or complement an ambiguous CT scan, a baseline PET scan should be obtained before initiation of therapy.

Surgical management: o Surgical management of GIST is based on sound oncologic principles, and before the introduction of imatinib mesylate as an effective systemic therapy, surgical resection was the only viable therapeutic option. o However, approximately 50% of all GIST patients will have evidence of overt malignant disease at initial evaluation, and survival after surgical management of these patients has historically been quite poor. o Optimal patients with primary tumors predominantly gastric in origin and treated by complete tumor resection still had a relatively poor reported 5-year survival rate of approximately 50%.

o Tumor size is a predominant factor in surgical resection series for primary GIST, and patients with tumors larger than 10 cm have reported survival rates in the 20% range at 5 years. o The most common sites of disease failure after complete resection are the peritoneal cavity and the liver. The finding of an extra-abdominal site is quite uncommon. o Approximately half to two thirds of GIST primaries will have disease failure within the liver, and nearly 40% of patients will have the liver as the only site of failure. o Generally, hepatic involvement is multifocal. Surgical resection for recurrent GIST has found limited use with rare long-term success, even after complete tumor removal or ablation. o Palliative surgery for bleeding or obstruction can be entertained as a viable alternative in a patient with limited disease and a good performance status.  The goal of surgery in the management of primary GIST is complete gross resection with an intact pseudocapsule. At laparotomy the abdomen should be carefully explored for any evidence of metastatic disease on the peritoneal surfaces or in the liver. GISTs should be handled gently and with care to avoid rupture.  Segmental resection of the stomach or intestine can be performed with the intent of negative margins. Partial or wedge gastrectomy is a viable option because GIST cells do not manifest submucosal spread, as is common with adenocarcinoma.  Lymphadenectomy is unnecessary because lymph node metastases are rare.  The value of microscopically negative resection margins, especially in a large tumor, is questionable since margin status does not appear to be a prognostic indicator for recurrence when it has been evaluated as a meaningful risk factor.  The value of reoperative surgery for microscopically positive margins is unproven and should depend on many factors, such as tumor size, potential morbidity, and the availability of adjuvant therapy.  Laparoscopic wedge resection may be used for small GISTs (<2 cm) when the risk for rupture is minimal, but data on this approach are lacking.  Surgical resection remains the standard form of treatment of primary GIST, the efficacy of the KIT-targeted oral agent imatinib mesylate has transformed therapeutic considerations into a multimodality paradigm.  Imatinib mesylate is a rationally based therapeutic agent that was developed as an antagonist to the KIT receptor on GIST cells and has shown remarkable effectiveness against metastatic and unresectable GIST.  Addition of this drug to surgical management decisions in GIST leads to the possibility of effective adjuvant therapy for improving long-term outcomes.  Advantages of this combined approach could also lead to organ preservation, an important consideration for GIST patients with primary tumor location in the proximal stomach, rectum, or duodenum.

Adjuvant therapy: Surgery is the primary initial treatment of resectable GIST, but it is seldom curative in patients with a high risk for recurrent disease. Surgery is even less effective in patients with locally recurrent or metastatic disease. Before the availability of imatinib mesylate the only treatment of GIST, other than surgical resection, was conventional chemotherapy and radiotherapy. However, lack of efficacy has been a constant in studies evaluating these modalities in the management of GIST patients. The recent availability of an effective systemic therapy raises the issue of its potential use as adjuvant or neoadjuvant treatment in conjunction with surgical tumor resection with the objective of either cytoreduction of disease before surgery or improvement in long-term survival after successful resection.

Patients typically at high risk for recurrence, such as those with tumors larger than 10 cm or tumors with greater than five mitoses per 50 high-powered fields, are obvious candidates for such a trial. Patients with other risk factors, such as tumor perforation or rupture or a known specific drug-sensitive genotype, might also be considered for clinical trial participation. The possibility of pharmacologic debulking with imatinib mesylate, followed by surgical resection, may be a rational strategy for organ preservation, in vivo drug sensitivity testing, optimization of therapy for focal metastatic disease, and abrogation of emerging drug-resistant clones. Short follow-up clinical experience has suggested the usefulness of limited surgery in some patients with multiple-site recurrent disease whose tumor demonstrated a mixed pattern of response to imatinib mesylate and in whom resection or ablation was successful in removing unresponsive sites while the bulk quiescent residual disease was managed successfully with continued imatinib mesylate therapy. At this time the optimal dose plus duration of imatinib in the surgical adjuvant or neoadjuvant strategy has not been defined, and this combination in treating GIST in standard actual practice awaits elucidation in clinical trials.

2. Discuss recent trends regarding management of Axilla in breast cancer. 20 Answer.

Introduction: The role of axillary surgery in breast cancer is to stage the axilla and in those with lymph node metastases to treat the axilla with axillary clearance. Adequate axillary dissection is important in node-positive patients both to ensure removal of all involved nodes to optimise local control and to obtain the maximum prognostic information. When staging the axilla, an additional goal, particularly in node-negative patients, is to minimise morbidity. Various strategies for doing so have been developed, the most recent being dual localisation sentinel lymph node biopsy (SLNB).

Sentinel lymph node mapping has become the standard axillary surgery used for patients who present with Stage I and Stage II breast cancer. This procedure is performed in conjunction with the breast surgery (lumpectomy or mastectomy) or as a stand-alone operation to help determine the patient’s need for chemotherapy prior to surgery or eligibility for breast reconstruction procedures.

Sentinel lymph node operation steps: 1. Injection of a technetium-sulfur colloid into the parenchyma around the breast cancer or dermal tissues above the cancer. This is commonly performed in nuclear medicine either the day before or 2 hours before the surgical procedure 2. Injection of a blue dye (Lymphazurin blue is the commonly selected dye) into the parenchyma around the cancer or the subareolar space 5 minutes prior to the surgery 3. The sentinel lymph node mapping surgery removes on average 2-3 Level 1 or 2 nodes 4. The sentinel lymph nodes are processed with standard Hemotoxylin and Eosin stains 5. A hand-held gamma counter is then employed transcutaneously to identify the location of the sentinel lymph node. A 3- to 4-cm incision is made in line with that used for an axillary dissection, which is a curved transverse incision in the lower axilla just below the hairline. After dissecting through the subcutaneous tissue and identifying the lateral border of the pectoralis muscles, the clavipectoral fascia is divided to gain exposure to the axillary contents. The gamma counter is employed to pinpoint the location of the sentinel lymph node. As the dissection continues, the signal from the probe increases in intensity as the sentinel lymph node is approached. The sentinel lymph node also is identified by visualization of isosulfan blue dye in the afferent lymph vessel and in the lymph node itself. Before removing the sentinel lymph node, a 10-second in vivo radioactivity count is obtained. After removal of the sentinel lymph node, a 10-second ex vivo radioactive count is obtained, and the lymph node is then sent to pathology for either permanent or frozen section analysis. The lowest false-negative rates for sentinel lymph node biopsy have been obtained when all blue lymph nodes and all lymph nodes with radiation counts greater than 10% of the 10-second ex vivo count of the sentinel lymph node are harvested (10% rule). Based on this, the gamma counter is employed before closing the axillary wound to measure residual radioactivity in the surgical bed. When necessary, a search is made for a second sentinel lymph node. This procedure is repeated until residual radioactivity in the surgical bed is less that 10% of the 10-second ex vivo count of the most radioactive sentinel lymph node.

For patients whose sentinel lymph nodes are negative by pathologic review, no additional axillary surgery is recommended.

Aims of axillary surgery

 30-40% of patients with early breast cancer  Surgical evaluation important and should have nodal involvement be considered for all patients with  The aims of axillary surgery is to: invasive cancer  To eradicate local disease  Levels of axillary clearance are assessed  To determine prognosis to guide adjuvant relative to pectoralis minor therapy  Level 1 - below pectoralis minor  Clinical evaluation of the axilla is  Level 2 - up to upper border of pectoralis unreliable (30% false positive, 30% false minor negative)  Level 3 - to the outer border of the 1st rib  No reliable imaging techniques available  Axillary samplings removes more than 4 nodes.

Arguments for axillary clearance Arguments for axillary sampling

 Axillary clearance both stages  Only stages the axilla and treats the axilla  Must be followed by axillary radiotherapy  Sampling potentially misses  The 60% of patients with node negative disease have nodes and understages the axilla unnecessary surgery  Surgical clearance possibly gains  Radical lymphadenectomy in other cancers (e.g. better local control melanoma) produces disappointing results  Avoids complications of axillary  Avoids morbidity of axillary surgery radiotherapy  The combination of axillary clearance and  Avoids morbidity of axillary radiotherapy is to be avoided recurrence  Produces unacceptable rate of lymphoedema

Surgery to the axilla:

 Invasive breast cancer: Minimal surgery, rather than lymph node clearance, should be performed to stage the axilla for patients with early invasive breast cancer and no evidence of lymph node involvement on ultrasound or a negative ultrasound-guided needle biopsy. o Sentinel lymph node biopsy (SLNB) is the preferred technique. o SLNB should only be performed by a team that is validated in the use of the technique. o Perform SLNB using the dual technique with isotope and blue dye. o Breast units should audit their axillary recurrence rates.

 Ductal carcinoma in situ: o Do not perform SLNB routinely in patients with a preoperative diagnosis of DCIS who are having breast conserving surgery, unless they are considered to be at a high risk of invasive disease. o Offer SLNB to all patients who are having a mastectomy for DCIS.

Evaluation and management of a positive sentinel lymph node: Offer further axillary treatment to patients with early invasive breast cancer who: • Have macrometastases or micrometastases shown in a sentinel lymph node • Have a preoperative ultrasound-guided needle biopsy with histologically proven metastatic cancer. The preferred technique is axillary lymph node dissection (ALND) because it gives additional staging information. Do not offer further axillary treatment to patients found to have only isolated tumour cells in their sentinel lymph nodes. These patients should be regarded as lymph node-negative.

3. Write short notes of the following: 5 x 6 a) Gastric outlet obstruction in adults. b) Laparoscopic Total Mesorectal Excision( TME) for rectal cancer. c) Complication in endovascular stents. d) Portal vein embolisation. e) Preoperative Ultrasound. Answer. (a) Gastric outlet obstruction in adults. Answer. See the answer of Q. No.2 of Paper – II of 2006. Gastric outlet obstruction – Definition:

It is a clinical condition of the pylorus or pyloric canal of the stomach that causes blocks the entry of food particles from the stomach to the duodenum (first section of the small intestine). Pyloric canal is a narrow tubular passage that forms an opening between the stomach and small intestine. It is generally not considered as a separate clinical entity and is usually a physiological consequence of other abdominal disorders. The condition is usually abbreviated as GOO. It is known by various names like:

 Pyloric obstruction  Congenital hypertrophic pyloric stenosis  Hypertrophic pyloric stenosis

Symptoms:  Projectile vomiting: Patients are found to forcibly eject the contents of the stomach through the mouth, owing to pyloric obstruction.  Constipation:While emptying the bowels, patients may face difficulties and acute discomfort that usually occur infrequently. In this condition hard, dry and small stools are passed. Eliminating these can be quite painful.  Weight loss: Loss of fluids and reduced appetite may result in sudden decrease in weight.  Epigastric pain: It is one of the most common symptoms of this stomach disorder. It is generally associated with acute pain arising in the upper middle part of the abdomen.  Dehydration:Persistent vomiting may lead to significant loss of body fluids, primarily causing excessive water deficiency.  Electrolyte imbalance: Hypochloremia, a deficiency of chloride ion in bloodstream due to prolonged vomiting, is a common symptom observed during this condition. Excessive loss of potassium (Hypokalemia) and Hydrochloric acid (Metabolic alkalosis), which occurs as a consequence of dehydration, highly contributes to this sudden reduction in body chlorides.  Wasting:Continuous degeneration of the body muscle and fat tissues may occur in the affected individuals, owing to low intake and high loss of nutrients (especially that of proteins). Malignant conditions of the stomach could be the primary factor, causing this gradual deterioration of the muscle tissues.

Causes of Gastric outlet obstruction: Benign Malignant

Peptic ulcer Tuberculosis Gastric adenocarcinoma Pyloric stenosis Amyloidosis Gastric Lymphoma Bouveret’s syndrome Gastrointestinal stromal tumor (GIST)

Gastric outlet obstruction in adults

Majority of cases of GOO are seen in the adult population, which is due to some benign conditions with peptic ulcer disease being the most common cause. Several infants are also known to suffer from this condition due to some congenital abnormalities in the abdomen, such as Pyloric stenosis.

Diagnosis

During diagnosis of GOO, physicians generally perform a physical examination of the stomach to reveal signs of dehydration. Affected patients may have a swollen stomach. This is generally followed by few diagnostic tests.

X-ray

An abdominal x-ray usually shows a stomach filled with gastric fluids and a distended or swollen small bowel, owing to the blockage at the level of the pylorus.

Barium meal

In this process, affected patients are made to ingest Barium sulfate. This is followed by x-ray imaging of the stomach and duodenum. X-ray images generally reveal distension of abdomen and narrowing of the pyloric canal due to Pyloric stenosis.

Endoscopy/ Gastroscopy

In this non-invasive technique, an endoscope is inserted through mouth into the upper gastrointestinal tract that includes esophagus, stomach and duodenum. This technique can help detect any tumor or ulcer in the stomach. Imaging studies

Ultrasound examination, computerized tomography (CT) and magnetic resonance imaging (MRI) may aid in detecting abnormal enlargement of the abdomen, or presence of any form of tumor.

Electrolytes:

It is a blood test that measures the levels of electrolytes and carbon dioxide. Abnormal levels of chloride, potassium and hydrochloric acid may indicate electrolyte imbalance in the body due to pyloric obstruction. Gastric outlet obstruction Treatment

On the basis of the cause of the disorder, physicians generally recommend an appropriate cure for affected patients. Treatment can be primarily classified into two groups.

Medical Treatment Nasogastric suction

It is a medical process where a plastic tube called nasogastric tube is inserted into the stomach through the nose and throat to drain the gastric contents. If continuous drainage of the contents is required then a collector bag is attached to the tube and placed below the level of the stomach. However, constant suction can be damaging to the lining of the stomach and must only be used in emergency situations. Electrolyte treatment

Sodium chloride solution is generally administered to patients for curing chloride deficiency in the body. Potassium supplementation is required and the mineral should be taken along with zinc and magnesium.

Proton pump inhibitors (PPIs)

This is a group of drugs that are most often used to reduce gastric acid production in the stomach that causes peptic ulcers. Omeprazole, Lansoprazole, Pantoprazole, Rabeprazole, Esomeprazole and Dexlansoprazole are some of the commonly used drugs.

Surgical Treatment

Surgical procedures are usually opted for when patients show least or no response towards any form of medical therapy.

Antrectomy

In this procedure, the lower section of the stomach called antrum is surgically excised to inhibit the effects of peptic ulcers which might lead to Pyloric obstruction.

Vagotomy

Acid secretion in the stomach can also be reduced by surgically removing the vagus nerve. This particular nerve plays a central role in the production and regulation of acid secretion. The removal of this nerve can cure ulcer complications.

Billroth I

In this surgical technique, the pylorus is entirely removed followed by direct reconnection or anastomosis of the upper portion of the stomach to the duodenum. Patients suffering from gastric tumors or peptic ulcer disorder are greatly benefitted from this method.

Gastrojejunostomy It is a surgical procedure that directly connects the stomach to the jejunum, second portion of the small intestine with the help of a small tube. In this way, the contents of the stomach bypass the duodenum and enter straight into the jejunum to enable normal absorption of the ingested food. When benign, Gastric outlet obstruction is not a life-threatening condition. Its symptoms can be treated with appropriate medication and surgery. Timely diagnosis is essential, especially if the cause is malignant. If cancerous developments are suspected, medical assistance should be availed instantly.

Endoscopic balloon dilation (EBD):

However, after the advent of through-the-scope balloon dilators, endoscopic balloon dilation (EBD) has emerged as an effective alternative to surgery in selected groups of patients. So far, this form of therapy has been shown to be effective in caustic-induced GOO with short segment cicatrization and ulcer related GOO. In the latter, EBD must be combined with eradication of H. pylori.

Gastroduodenal stent :

Gastroduodenal stent placement is a very safe and effective palliation method in patients with unresectable malignant tumors causing gastric outlet obstruction, with adequate palliation obtained in most cases. The procedure can be performed under fluoroscopic guidance or with a combination of fluoroscopic and endoscopic techniques. Advantages of gastroduodenal stent placement over surgical palliation include suitability as an outpatient procedure, more rapid gastric emptying, greater cost effectiveness, fewer complications, and improved quality of life. Covered duodenal stents are currently being evaluated and may play an increasingly important role in preventing recurrent obstruction secondary to tumor ingrowth. Moreover, simultaneous palliation of biliary and duodenal malignant strictures is possible with the use of metallic stents. Gastroduodenal stent placement is a promising new alternative for the palliation of malignant gastroduodenal obstruction.

(b) Laparoscopic Total Mesorectal Excision( TME) for rectal cancer. Answer. Total mesorectal excision is a common procedure used in the treatment of colo rectal cancer in which a significant length of the bowel around the tumor is removed. Total mesorectal excision addresses earlier treatment concerns regarding adequate local control of rectal cancer while performing an anterior resection. The term total mesorectal excision strictly applies when performing a low anterior resection for tumors of the middle and the lower rectum, wherein it is essential to remove the rectum along with the mesorectum up to the level of the levators. The principles of total mesorectal excision (sharp mesorectal excision) are also applied during an abdominoperineal excision of the rectum and for tumors of the upper rectum, although these are considered distinct from standard total mesorectal excision. In an abdominoperineal excision of the rectum where the tumor exists below the level of the levators, the lateral margins of the tumor are inferior to the mesorectum and the benefits of total mesorectal excision do not apply. Anterior resections involving the upper rectum may be completed with mobilization of the rectum to beyond 5 cm of the lower margin of the tumor, and which is often above the level of the levator and is sometimes referred to as partial mesorectal excision. The treatment of rectal cancers is multimodal with adjuvant radiotherapy and chemotherapy having benefits in some settings. In addition, accurate preoperative staging is dependent on good radiological support. It is therefore necessary to subject all rectal cancers to multidepartment conference and to design individualized treatment plans based on a well-defined protocol. This serves the dual purpose of maintaining a consistent evidence-based approach as well as creating a dataset for prospective analysis and feedback. Indications Total mesorectal excision is indicated as a part of low anterior resection for patients with adenocarcinoma of the middle and lower rectum. Total mesorectal excision is now considered the gold standard for tumors of the middle and the lower rectum. Outcomes Circumferential resection margin positivity rate is about 5% or less for low anterior resections with total mesorectal excision, whereas it is between 10% and 25% for abdominoperineal excision of the rectum. There is understandably a higher local recurrence rate following abdominoperineal excision of the rectum. Five-year survival and disease-free survival rates are significantly lower with total mesorectal excision. Laparoscopic TME is a feasible but technically demanding procedure (12% conversion rate). Excellence of surgical technique is of particular relevance in the treatment of rectal cancer. Routine excision of the intact mesorectum during resection of cancers of the middle and lower rectum has resulted in the lowest incidence of local recurrences ever reported. Patients with tumors located in the low and middle rectum can be selected for laparoscopic TME based on the following criteria: elective surgery, absence of occlusion, American Society of Anesthesiologists (ASA) status I to III. Neither morbid obesity nor prior pelvic surgery is considered a contraindication to laparoscopic TME. When the neoplasm at digital examination reached the anatomic anal canal or was fixed to the pelvic floor, the patient is not a good candidate for laoparoscopic TME. Preoperative work-up consists of clinical evaluation, total colonoscopy, computed tomographic scan, upper abdominal ultrasound, and endoscopic ultrasound. Patients preoperatively staged T3 or T4 without distant metastases are treated by preoperative radio-chemotherapy (45 Gy during 4 weeks together with systemic 5-fluouracil intravenous infusion) and are evaluated with clinical examination and computed tomographic scan 21 days after the completion of the treatment. Definitive inclusion in this study was decided at this point, excluding locally advanced tumor (i.e., T4 in the TNM classification). Patients are followed up with physical examination, digital examination, serum carcinoembryonic antigen (CEA) assay, ultrasound of the liver, computed tomographic scan, chest radiograph, and colonoscopy. Every case of suspected local recurrence is histologically confirmed. Laparoscopic TME is feasible and safe. The laparoscopic approach to the surgical treatment of operable rectal cancer does not seem to entail any oncologic disadvantages. Laparoscopic TME with preservation of anal sphincter is a safe and minimally invasive technique with less postoperative pain and rapid recovery, and importantly, it has preserved the function of the sphincter. Laparoscopic TME and Sphincter Preservation have the following advantages: (1) it helps surgeons identify accurately the interspace of loose connective tissue between visceral and parietal pelvic fascia, and select operative access by amplifying inner image on the monitors. (2) 25 or 30 laparoscope, the third eye of the surgeon, can reach the narrow lesser pelvis and magnify the local vision; (3) it is more definite to identify and protect the pelvic autonomic nerve fiber and plexus due to its magnifying function. (4) hemostatic benefit was owed to minimally invasive sharp dissection with minimal blood loss under laparoscopic view. However, the laparoscopic technique has its disadvantages, such as long operative time, short of direct hand feeling, technical constraints of the narrow pelvis, difficulty in assessing adequate surgical margins and in ultralow rectal cross clamping, and a long learning course for surgeons.

(c) Complication in endovascular stents. Answer. An endovascular stent graft is a tube composed of fabric supported by a metal mesh called a stent. It can be used for a variety of conditions involving the blood vessels, but most commonly is used to reinforce a weak spot in an artery called an aneurysm. Over time, blood pressure and other factors can cause this weak area to bulge like a balloon and it can eventually enlarge and rupture. The stent graft is designed to seal tightly with your artery above and below the aneurysm. The graft is stronger than the weakened artery and it allows your blood to pass through it without pushing on the bulge. Physicians typically use endovascular stent grafting to treat abdominal aortic aneurysms (AAAs), but they also use it to treat thoracic aortic aneurysms (TAAs) and less commonly, aneurysms in other locations. The potential complications of endovascular stent grafting include:

 Leaking of blood around the graft (“endoleaks”)  Infection  Movement of the graft away from the desired location (“migration”)  Graft fracturing  Blockage of the blood flow through the graft Sometimes fever and an increase in white blood cell count can happen shortly after endovascular stent grafting. These symptoms usually last 2 to 10 days and are treated with medications such as aspirin and ibuprofen. Other complications that are rare but serious include a burst artery, injury to your kidney, paralysis, blocked blood flow to your abdomen or lower body, and delayed rupture of AAA. Endovascular stent grafts can sometimes leak blood through the areas where the graft components join together, or they can allow blood to leak back into the aneurysm sac through small arteries feeding the aneurysm sac. These leaks are called "endoleaks," and while this name is somewhat confusing, it does not imply that your aneurysm has ruptured or is leaking. Some of the leaks stop by themselves and are not dangerous, but others need to be treated immediately. These leaks can even occur years after your procedure and can be dangerous if the aneurysm continues to enlarge. Thus, after endovascular aneurysm repair, physicians require their patients to undergo long term surveillance with periodic CT scans for the rest of their life to detect and treat problems before they become threatening. Since problems with the graft or endoleaks can occur even years after successful placement, it is important to comply with the follow-up regimen advised by the vascular surgeon.

(d) Portal vein embolisation. Answer. Portal vein embolisation (PVE) is a procedure carried out radiologically (using x-rays), in order to help towards treating cancer of the liver. It is not a treatment for cancer but it may enable potentially curative/beneficial surgery to take place. It is achieved by blocking off the portal venous flow to part of the affected liver, enabling the non-affected part to grow bigger.

PVE is performed in patients who have liver cancer that is suitable for surgical resection but where the potential remaining liver is of inadequate size in order to work properly. This could lead to hepatic (liver) failure after the operation. Therefore PVE is carried out before surgery. The aim is to shrink the part of liver affected with cancer and enlarge the healthy/normal part of liver. PVE therefore blocks blood flow into the cancerous part of the liver that will be removed later during surgery. Blocking off this part of liver causes shrinkage of this segment and in turn causes extra growth of the remaining healthy part of the liver. Therefore by the time surgery is done, the remaining liver is already enlarged, reducing the risk of liver failure.  The procedure itself involves a puncture of the liver to inject gluelike material into the relevant branch of the portal vein. This is done in the x-ray department by a radiologist, using ultrasound and angiography techniques to direct the needle into the correct position. It is usually done as a day case or involves an overnight stay in hospital. Alternatively the branch of the portal vein may be tied off during a surgical operation.  There are some risks involved, including bleeding or bile leak from the puncture into the liver. Spillage of the glue into parts of the circulation is another small risk. It is difficult to predict exactly how much liver growth this procedure will cause in a particular patient, and one has to wait and see.

(e) Preoperative Ultrasound. Answer. Introduction: Ultrasound is the second commonest method of imaging. It relies on high- frequency sound waves generated by a transducer containing piezoelectric material. The generated sound waves are reflected by tissue interfaces and, by ascertaining the time taken for a pulse to return and the direction of a pulse, it is possible to form an image.

uses frequencies in the Dedicated transducers have also been developed for range 3–20 MHz. endocavitary ultrasound such as transvaginal  Abdominal imaging uses transducers with a scanning and transrectal ultrasound of the prostate, frequency of 3–7 MHz allowing high frequency scanning of organs by  Higher-frequency transducers are used for reducing the distance between the probe and the superficial structures, such as musculoskeletal organ of interest .

A furtherand application breast. ultrasound. of dedicated probes has been in the field of endoscopic ultrasound, allowing exquisite imaging of the wall of a hollow viscus and the adjacent organs such as the gastric wall and the pancreas.

Doppler ultrasound : Reflection of an ultrasound wave from moving objects such as red blood cells causes a change in the frequency of the ultrasound wave. By measuring this frequency change, it is possible to calculate the speed and direction of the movement. Doppler imaging is widely used in the assessment of arterial and venous disease, in which stenotic lesions cause an alteration in the normal velocity. Furthermore, diffuse parenchymal diseases such as cirrhosis may cause an alteration in the normal Doppler signal of the affected organ.

Uses:  It is cheap and easily available. It is the first-line investigation of choice for assessment of the liver, the biliary tree and the renal tract. Ultrasound is also the imaging method of choice in obstetric assessment and gynaecological disease. High-frequency transducers have made ultrasound the best imaging technique for the evaluation of thyroid and testicular disorders. It is also an invaluable tool for guiding needle placement in interventional procedures such as biopsies and drainages, allowing direct real-time visualisation of the needle during the procedure.  The role of preoperative ultrasound in the evaluation of cervical lymph nodes in patients with known thyroid cancer has been widely explored.  For hepatobiliary system: Normal measurements on ultrasound — Measurements of components of the hepatobiliary tree depend upon the skill of the ultrasonographer obtaining the measurements, and there is variability in terms of what is considered "normal." However, some general estimates are available regarding the expected sizes of structures in the hepatobiliary tree:

o Gallbladder: The gallbladder wall should be less than or equal to 2 mm (in a distended or fasting gallbladder). Collapsed , seen when the subject has eaten, typically appear thickened. The maximum dimension of the gallbladder is 5 X 10 cm. o Common hepatic duct: Common hepatic duct (inner wall to inner wall) is usually measured at the level of hepatic artery. In the normal fasting state it should be <7 mm in patients <60 years, and <10 mm in patients older than 60. o Common bile duct (CBD): The normal CBD diameter increases with age and in patients who have had a cholecystectomy. In patients in their 40s, the normal mean diameter is 4 mm. The normal mean diameter then increases by 1 mm every decade, and authors have proposed that in elderly patients the normal upper limit of normal be set at 8.5 mm. The CBD is commonly up to 10 mm in patients who have undergone a cholecystectomy. o Liver: The span of the liver in the right mid-clavicular line should be less than 16 cm.

So, ultrasound is useful for the preoperative evaluation for the patient with diseases of liver and biliary system.

 Ligament, tendon and muscle injuries are also probably best imaged in the first instance by ultrasound. The ability to stress ligaments and to allow tendons to move during the investigation gives an extra dimension, which greatly improves its diagnostic value. The use of ‘panoramic’ or ‘extended field of view’ ultrasound provides images that are more easily interpreted by an observer not performing the examination, and are of particular assistance to surgeons planning a procedure.  Preoperative staging of colon cancer patients: ultrasound can be a valuable alternative to computed tomography.  Ultrasound will demonstrate most foreign bodies in soft tissues, including those that are not radio-opaque.

The disadvantages of ultrasound are that it is highly operator dependent, and most of the information is gained during the actual process of scanning as opposed to reviewing the static images. Another drawback is that the ultrasound wave is highly attenuated by air and bone and, thus, little information is gained with regard to tissues beyond bony or air-filled structures; alternative techniques may be required to image these areas.

Ultrasound

Strengths Weaknesses ■ No radiation ■ Interpretation only possible during the ■ Inexpensive examination ■ Allows interaction with patients ■ Long learning curve for some areas of ■ Superb soft-tissue resolution in the near field expertise ■ Dynamic studies can be performed ■ Resolution dependent on the machine ■ First-line investigation for hepatic, biliary available and renal disease ■ Images cannot be reliably reviewed away ■ Endocavitary ultrasound for gynaecological from the patient. and prostate disorders ■ Excellent resolution for breast, thyroid and testis imaging ■ Good for soft tissue, including tendons and ligaments ■ Excellent for cysts and foreign bodies ■ Doppler studies allow assessment of blood flow

1 4. Answer briefly on the following: 4 x 7 /2 a) Prosthesis in Hernia Surgery. b) Oncogenes. c) Uroflowmetry. d) Surgery for Advanced Gall Bladder cancer. Answer. (a) Prosthesis in Hernia Surgery. Properties of an ideal mesh are:  Strong but pliable and easy to handle.  Inert, nonallergic, nonbiodegradable ,noncarcinogenic.  Should have an optimum thickness.  Should stimulate adequate fibroblastic activity and get rapidly incorporated into the tissues.  Should provide mechanical support through permanent biocompatibility.  Should achieve early, rapid and optimum ingrowth of fibrocollagenous tissue to preventdislocation or migration.  Macroporous, monofilament, transparent.  Resistant to infection.  Should provide a barrier to adhesions in intra – abdominal placement.

Types of prostheses:  Poly propylene: preferable as it is less likely to give rise to infection. Poly propylene is also available in the form of a device (PHS) and a plug. Plug is implanted into the deep inguinal ring and may be used alone or combined with a patch. (PHS) - Prolene Hernia System consists of three parts – underlay, connector and onlay.it is indicated for open technique in all groin hernias and small umbilical and epigastric hernias.  Polyester : can be used in hernia repair by Stoppa technique.  PTFE: Polytetrafluroethylene and e-PTFE(expanded Polytetrafluroethylene) meshes are smooth, soft and strong. There are different varieties:  Soft tissue patch.  Micromesh.  Micromesh plus.  Dual mesh.  Dual mesh plus.  Dual mesh with holes.  PTFE impregnated with silver and chlorhexidine. These prostheses are suitable for repair of all groin and ventral hernias by open technique as well as by TAPP or TEP since they do not come in contact with abdominal viscera. Latest technical advances have now made available prosthetic materials, which prevent bowel adhesions.  Lightweight Composite Meshes without Adhesion Prevention “Barrier”  VYPRO II.  ULTRAPRO.  New Composite Meshes with Adhesion Prevention “Barrier” Repair of ventral and incisional hernia may be undertaken by minimal access surgery rather than the conventional open technique. To achieve this repair, intraperitoneal placement of mesh requires a material which has both high tissue ingrowth towards the abdominal wall and nonadhesiveness on the other side to prevent bowel adhesions.  Absorbable barrier composite mesh: . Sepramesh: Sepramesh Biosurgical composite (Genzyme Biosurgery, Cambridge, USA) is a dual-component prosthetic biomaterial composed of macroporous polypropylene on one side, with bioresorbable, nonimmunogenic membrane of sodium hyaluronate and carboxymethyl cellulose on the other side. Seprafilm was designed to provide protection against intra-abdominal adhesion formation throughout the critical period of remesothelialization during the first postoperative week. The absorbable barrier turns to a gel in 48 h, remains on the mesh for ~7 days and is cleared from the body in 28 days. This antiadhesive material forms a physical barrier on damaged surfaces to prevent adherence or reduce viscosity between opposing tissues. The physical barrier should allow injured tissues to heal separately from each other. In addition, the sodium hyaluronate and carboxymethyl cellulose are anionic polyaccharides that form a membrane that is negatively charged, a molecular property that promotes the separation of healing tissues. . Parietex Composite Mesh and Parientene Composite Mesh: Parietex composite (Sofradim, France) is composed of multifilament polyester mesh with a purified, oxidized bovine atelocollagen type I coating covered by an absorbable, antiadhesion film of polyethylene glycol and glycerol. Polyethylene glycol is a hydrogel that decreases tissue adherence and glycerol is a hydrophobic lipid. The collagen coating functions to promote collagen ingrowth by increasing the hydrophilicity of the polyester mesh and decreasing the fibrous tissue reaction to the 'foreign' material (mesh). The collagen, polyethylene glycol and glycerol film are resorbed in ~3 weeks. Parientene composite mesh consists of the same antiadhesive barrier but coated to polypropylene.

. PROCEED Surgical Mesh: PROCEED surgical mesh (Johnson and Johnson, India) is a sterile multilayered, thin, flexible, laminate mesh comprised of an oxidized regenerated cellulose (ORC) fabric; and PROLENE soft mesh, a nonabsorbable polypropylene mesh which is encapsulated by a polydioxanne polymer. The polypropylene mesh side of the product allows for tissue ingrowth, while the ORC side provides a bioresorbable layer that physically separates the polypropylene mesh from underlying tissue and organ surfaces during the wound-healing period to minimize tissue attachment to the mesh. The polydioxanone provides a bond to the ORC layer.

It has a lightweight macroporous mesh construction, leaves behind less residual foreign body, allows the fluid to flow through easily and does not harbor bacteria.  Non Absorbable barrier composite mesh: . Bard Composix Mesh: This is a combination of polypropylene that has a thin coat of ePTFE on one side to prevent bowel adhesions. Introduction through a laparoscopic port is difficult because it cannot compress the mesh and hence requires a larger port (12 mm) for its introduction. . GORE-TEX Dual Mesh: The Gore-Tex Dual mesh (W. L. Gore, USA) material has two surfaces; one is very smooth (micropores 3 mm) and the other is rough (micropores approximately about 22 mm). It is designed to be implanted with the smooth surface against the visceral organs - tissue to which no or minimal adhesion is desired - and the other surface against which tissue incorporation is desired. The Dual mesh comes in two choices: one is a solid sheet and the other is perforated to allow for greater tissue incorporation. A recent innovation is the incorporation of silver and chlorhexidine into the ePTFE. This results in a significant antimicrobial action.

The use of novel absorbable and nonabsorbable barriers on composite meshes to reduce the incidence of adhesion and adhesion-related complications has been evaluated in animal models and a few clinical studies have been reported. The incidence of adhesions and tenacity of adhesions were reduced for all of the barrier meshes compared with the macroporous polyester mesh.

The development of new prosthetic biomaterials with the addition of absorbable and nonabsorbable barriers for adhesion prevention after intra-abdominal placement of mesh during open and laparoscopic hernia repair is potentially a significant advancement in the management of ventral and incisional hernias. Long-term follow-up is desirable to determine if Sepramesh, Parietex Composite, Parientene Composite, PROCEED surgical, Bard Composix and Gore-Tex Dual meshes will decrease the incidence of mesh-related complications compared to nonbarrier, macroporous meshes.  Biological Mesh: Chart of biological meshes: Donor tissues Product name Human dermis Alloderm ,Allomax Porcine dermis Flex HD, Permacol, Collamend, XenMatrix Porcine small intestine submucosa Surgisis Bovine dermis Surgi Mend Bovine pericardium Veritas Selection of mesh: An appropriate mesh should be selected and implanted. For inguinal hernia repair by TAPP or TEP, any of the following meshes can be used:  Conventional polypropylene mesh  PROLENE soft mesh  VYPRO II  ULTRAPRO For intra-abdominal placements, any mesh that will prevent bowel adhesions should be used. It can be either ePTFE, PROCEED surgical mesh or any one of the newly engineered meshes with an absorbable or a nonabsorbable barrier.

Next important matter to consider is the size of the mesh. It must be at least 15 x 15 cm for an inguinal hernia. For repair of umbilical, ventral and incisional hernia, it should be at least 4 cm wider than the defect. It is better to initially measure the size of the defect with the scale and then select a mesh of appropriate size. It should be wide enough to cover the defect in all directions since a smaller size may lead to protrusion of the mesh into the defect and result in a recurrence.

(b) Oncogenes. Answer. See the Answer of Q. No. 3(c) of Paper – IV, of 2007.

(c) Uroflowmetry.

Answer. Uroflowmetry is a simple, diagnostic screening procedure used to calculate the flow rate of urine over time. The test is noninvasive (the skin is not pierced), and may be used to assess bladder and sphincter function.

Uroflowmetry is performed by having a person urinate into a special funnel that is connected to a measuring instrument. The measuring instrument calculates the amount of urine, rate of flow in seconds, and length of time until completion of the void. This information is converted into a graph and interpreted by a physician. The information helps evaluate function of the lower urinary tract or help determine if there is an obstruction of normal urine outflow. During normal urination, the initial urine stream starts slowly but almost immediately speeds up until the bladder is nearly empty. The urine flow then slows again until the bladder is empty. In persons with a urinary tract obstruction, this pattern of flow is altered, and increases and decreases more gradually. The uroflowmeter graphs this information, taking into account the person’s gender and age.

Indications: Uroflowmetry is a quick, simple diagnostic screening test that provides valuable feedback about the health of the lower urinary tract. It is commonly performed to determine if there is obstruction to normal urine outflow. Benign prostatic hypertrophy (BPH)--a benign enlargement of the prostate gland that usually occurs in men over age 50. Enlargement of the prostate interferes with normal passage of urine from the bladder. If left untreated, the enlarged prostate can obstruct the bladder completely.

 Cancer of the prostate, or bladder tumor  Urinary incontinence--involuntary release of urine from the bladder  Urinary blockage--obstruction of the urinary tract can occur for many reasons along any part of the urinary tract from kidneys to urethra. Urinary obstruction can lead to a backflow of urine causing infection, scarring, or kidney failure if untreated.  Neurogenic bladder dysfunction--improper function of the bladder due to an alteration in the nervous system such as a spinal cord lesion or injury  Frequent urinary tract infections (UTI’s)

Uroflowmetry may be performed in conjunction with other diagnostic procedures, such as cystometry and cystography.

Normal results: Normal values vary depending on age and sex. In men, urine flow declines with age. Women have less change with age:

 Ages 4 - 7 o The average flow rate for both males and females is 10 mL/sec.  Ages 8 - 13 o The average flow rate for males is 12 mL/sec. o The average flow rate for females is 15 mL/sec.  Ages 14 - 45 o The average flow rate for males is 21 mL/sec. o The average flow rate for females is 18 mL/sec.  Ages 46 - 65 o The average flow rate for males is 12 mL/sec. o The average flow rate for females is 18 mL/sec.  Ages 66 - 80 o The average flow rate for males is 9 mL/sec. o The average flow rate for females is 18 mL/sec.

 Results are compared with your symptoms and physical exam. A result that may need treatment in one patient may not need treatment in another patient.  Several circular muscles normally regulate urine flow. If any of these muscles becomes weak or stops working, you may have an increase in urine flow or urinary incontinence.

 If there is a bladder outlet obstruction or if the bladder muscle is weak, you may have a decrease in urine flow. The amount of urine that remains in your bladder after urinating can be measured with ultrasound.

 Risks of the Procedure: Because uroflowmetry is a noninvasive procedure, it is safe for most persons. The test is usually done in privacy to ensure that the person voids in a natural setting.There may be risks depending upon your specific medical condition. Be sure to discuss any concerns with your doctor prior to the procedure.Certain factors or conditions may interfere with the accuracy of uroflowmetry. These factors include, but are not limited to, the following:

 Straining with urination  Body movement during urination  Certain medications that affect bladder and sphincter muscle tone

(d) Surgery for Advanced Gall Bladder cancer. Answer. Gall bladder cancer is the most common cancer of the biliary tract worldwide. In majority ofcases, the outcome of patients with advanced gall bladder cancer is not good. It has been historically considered as an incurable malignancy. Often gall bladder cancer presents in advanced stage with invasion of the contagious adjacent organs, e.g. liver, duodenum, colon or bile duct. Advanced Gall bladder cancer – T3/N1 or any T and disease between II and III TNM. TNM Staging for Gallbladder Cancer T1 Tumor invades lamina propria (T1a) or muscular (T1b) layer T2 Tumor invades perimuscular connective tissue, no extension beyond the serosa or into the liver T3 Tumor perforates the serosa (visceral peritoneum) and/or directly invades into liver and/or one other adjacent organ or structure such as the stomach, duodenum, colon, pancreas, omentum, or extrahepatic bile ducts T4 Tumor invades main portal vein or hepatic artery or invades multiple extrahepatic organs and/or structures N0 No lymph node metastases N1 Regional lymph node metastases M0 No distant metastases M1 Distant metastases Stage Stage Grouping IA T1 N0 M0 IB T2 N0 M0 IIA T3 N0 M0 IIB T1 N1 M0 T2 N1 M0 T3 N1 M0 III T4 Any N M0 IV Any T Any N M1

N1 : cystic duct, pericholedochal, periportal, hilar , proper hepatic artery group of lymph node. N2 : posterosuperior and posteroinferior pancreaticoduodenal artery, common hepatic artery , celiac artery and superior mesenteric artery group of lymph node. o CECT is more helpful in demonstrating a mass in the GB or LN in the HDL and peripancreatic region. EUS is helpful for examination of the peripancreatic and periportal lymphadenopathy. Levels of tumour markers CEA, CA 19-9 AND CA 242 may be raised in GB cancer. o Curative resection of the tumour and its locoregional spread provides the only hope of long term survival. The extent of resection may range from an extended cholecystectomy with a 2 cm non anatomic wedge of liver in the gall bladder bed in segments IVB plus V and lymphadenectomy to an extended right hepatectomy and pancreaticoduodenectomy depending upon the location and spread of the tumour. o Lymph node evaluation is a critical component of radical surgery. Patients with N1 disease achieve long term survival with radical resection if the positivenodes are confined to the HDL. Positive nodes in more distant groups are associated with poor outcome. o The risk of (+) ve regional LN in patients with T3 or T4 disease can be estimated in individual cases by preoperative sampling of the key nodes by EUS and frozen section. o The probability of LN metastasis in patients with T3 or T4 disease is as high as 64%. However the risk of lymphatic spread is minimal if N12b ( pericholedochal), N12c ( cystic duct ) and N13a ( pripancreatic posterosuperior ). o Routine resection of bile duct is not advisable unless it is involved primarily by a Tor pericholedochal node. o If the tumour has invaded organs like duodenum, stomach or colon then limited resection(wedge/sleeve) should be considered instead of formal organ resection. o Once porta hepatis has been involved and patient presents with jaundice, curative resection is unlikely. o Planned cytoreduction or debulking surgery should not be done.

MS (General Surgery) Examination, 2011 PAPER IV

Time Allowed: 3 Hours Full Marks: 100 Attempt all questions

1. Liver resection: Indications, brief steps and results. 20 2. Describe Abdominal compartment syndrome. How does it affect the fuction of individual organs? How do you manage the complications? 20

3. Write short notes of the following: 5 x 6 a) Balloon . b) Intestinal lengthening for short bowel syndrome. c) Gene expression profiling of Breast cancer. d) C.A.B.G. e) V.A.T.S.

4. Answer briefly on the following: 4 x 71/2 a) Current status of staging Laparotomy in Lymphomas. b) Diabetic foot – recent trend in management. c) Recent trend in management of urinary lithiasis. d) Renal Transplant - steps.

MS (General Surgery) Examination, 2011

April 2011

PAPER IV

Time Allowed: 3 Hours Full Marks: 100 Attempt all questions

1. Liver resection: Indications, brief steps and results. 20

Answer. Indications: I. Most are performed for the treatment of hepatic neoplasms, both benign or malignant. Benign neoplasms include ,hepatic hemangioma and focal nodular hyperplasia. II. The most common malignant neoplasms (cancers) of the liver are metastases; those arising from colorectal cancer are among the most common, and the most amenable to surgical resection. The most common primary malignant tumour of the liver is the hepatocellular carcinoma. III. Hepatectomy may also be the procedure of choice to treat intrahepatic gallstones or parasitic cysts of the liver. IV. Partial Hepatectomies are also performed to remove a portion of a liver from a live donor for transplantation. Nomenclature for Most Common Major Anatomic Hepatic Resections GOLDSMITH AND SEGMENTS COUINAUD, 1957 WOODBURNE, 1957 BRISBANE, 2000 V-VIII Right hepatectomy Right hepatic lobectomy Right hemi-hepatectomy IV-VIII Right lobectomy Extended right hepatic Right trisectionectomy lobectomy II-IV Left hepatectomy Left hepatic lobectomy Left hemi-hepatectomy II, III Left lobectomy Left lateral segmentectomy Left lateral sectionectomy II, III, IV, V, Extended left Extended left lobectomy Left trisectionectomy VIII hepatectomy

Technique

 A Hepatectomy is considered a major surgery done under general anesthesia.  Access is accomplished by laparotomy, typically by a bilateral subcostal ("chevron") incision, possibly with midline extension (Calne or "Mercedes-Benz" incision).  Hepatectomies may be anatomic, i.e. the lines of resection match the limits of one or more functional segments of the liver as defined by the Couinaud classification; or they may be non-anatomic, irregular or "wedge" hepatectomies.  Anatomic resections are generally preferred because of the smaller risk of bleeding and biliary fistula; however, non-anatomic resections can be performed safely as well in selected cases. For details on the variety of anatomic hepatectomies and the specific nomenclature, cf. the International Hepato-Pancreatico-Biliary Association (IHPBA) Terminology for Liver Resections.  The maneuver is usually performed during a hepatectomy to minimize blood loss - however this can lead to reperfusion injury in the liver due to ischemia .  A liver resection must consider the disease to be treated and the goals of the operation, whether a margin-negative resection of a malignancy or the removal of benign tissue to alleviate symptoms.  The most basic steps can be distilled down to inflow (portal vein, hepatic artery, bile duct) control, outflow (hepatic veins) control, and parenchymal transection with preservation of a liver remnant of adequate size with intact inflow, biliary drainage, and venous outflow. The most common approach to an anatomic resection, in the most common order, is mobilization of the liver to be resected, dissection of inflow and outflow structures, division of the inflow, division of the outflow, and parenchymal transection.  Mobilization of the liver involves division of the right or left triangular ligaments, freeing up the liver from the diaphragm. Often, the liver must be mobilized completely off of the vena cava (which it straddles), and this requires careful dissection and division of multiple retrohepatic caval venous branches.  For major resections, the hepatic vein of the resected portion of liver is often encircled before the resection. There are a variety of techniques to dissect, control, and divide inflow vessels. Classic inflow control is obtained by a dissection of the liver hilum with control of the portal vein and hepatic artery to the hemi-liver to be resected. These can be suture-ligated or can be divided with vascular staplers.  Unless tumor proximity mandates, we advocate dividing the bile duct within the liver substance to absolutely minimize contralateral biliary injuries related to anatomic anomalies. Inflow control can also be obtained by dissection of the intrahepatic inflow pedicle to the anatomic section of liver to be resected. Recall that the inflow structures invaginate peritoneum at the hepatic hilus and run intrahepatically as an invested pedicle of the three inflow structures.  The inflow pedicle can be encircled by making flanking hepatotomies or by splitting parenchyma down to the pedicle of interest. The pedicle can usually be divided with a vascular stapler, but suture ligation is sometimes necessary.  Classically, the hepatic vein is divided in its extrahepatic position and can also usually be done with a vascular stapler. The hepatic vein can also be divided within the substance of the liver during parenchymal transection. There are numerous methods of parenchymal transection, ranging from complex ultrasonic irrigators to radiofrequency energy coagulators to a simple clamp crushing technique. All of these, in experienced hands, can be used effectively to minimize blood loss, and it is important to develop a specific technique that you are comfortable with.  Ultimately, parenchymal transection is about dissecting intrahepatic anatomy, controlling vascular and biliary structures, minimizing blood loss, and avoiding injury to the future liver remnant. Complications: Bleeding is the most feared technical complication and may be grounds for urgent reoperation. Biliary fistula is also a possible complication, albeit one more amenable to nonsurgical management. Pulmonary complications such as atelectasis and pleural effusion are commonplace, and dangerous in patients with underlying lung disease. Infection is relatively rare. Liver failure poses a significant hazard to patients with underlying hepatic disease; this is a major deterrent in the surgical resection of hepatocellular carcinoma in patients with cirrhosis. It is also a problem, to a lesser degree, in patients with previous hepatectomies (e.g. repeat resections for reincident colorectal cancer metastases). 2. Describe Abdominal compartment syndrome. How does it affect the fuction of individual organs? How do you manage the complications? 20

Answer. Abdominal compartment syndrome occurs when the abdomen becomes subject to increased pressure. Specific cause of abdominal compartment syndrome is not known, although some causes can be sepsis and severe abdominal trauma. Increasing pressure reduces blood flow to abdominal organs and impairs pulmonary, cardiovascular, renal, and gastro-intestinal (GI) function, causing multiple organ dysfunction syndrome and death. Abdominal compartment syndrome (ACS) is the end result of untreated Intra-abdominal hypertension (IAH). Abdominal compartment syndrome may result from large volume intravenous fluid resuscitation. Pathophysiology It occurs when tissue fluid within the peritoneal and retroperitoneal space (either oedema, retroperitoneal blood or free fluid in the abdomen) accumulates in such large volumes that the abdominal wall compliance threshold is crossed and the abdomen can no longer stretch. Once the abdominal wall can no longer expand, any further fluid leaking into the tissue results in fairly rapid rises in the pressure within the closed space. Initially this increase in pressure does not cause organ failure but does cause organs to not work properly - this is called intra-abdominal hypertension and is defined as a pressure over 12 mm Hg in adults. However, if the pressure continues to rise over 20 mm Hg and organs begin to fail, the syndrome has now progressed to the end stage highly fatal process termed abdominal compartment syndrome. These pressure measurements are relative. Small children get into trouble and develop compartment syndromes at much lower pressures while young previously healthy athletic individuals may tolerate an abdominal pressure of 20 mm Hg very well. The diagram provided here shows more details regarding the pathophysiolgy of this process. The underlying cause of the disease process is capillary permeability caused by the systemic inflammatory response syndrome (SIRS) that occurs in every critically ill patient. SIRS leads to leakage of fluid out of the capillary beds into the interstitial space in the entire body with a profound amount of this fluid leaking into the gut wall, mesentery and retroperitoneal tissue. Peritoneal tissue edema secondary to diffuse peritonitis, abdominal trauma, . Fluid therapy due to massive volume resuscitation . Retroperitoneal hematoma secondary to trauma and aortic rupture . Peritoneal trauma secondary to emergency abdominal operations . Reperfusion injury following bowel ischemia due to any cause . Retroperitoneal and mesenteric inflammatory edema secondary to acute pancreatitis . Ileus and bowel obstruction . Intraabdominal masses of any etiology . Abdominal packing for control of hemorrhage . Closure of the abdomen under undue tension . Ascites / intraabdominal fluid accumulation

Grading of abdominal compartment syndrome Grade Bladder Pressure (mm Hg) Recommendation I 10–15 Maintain normovolemia II 16–25 Hypervolemic resuscitation III 26–35 Decompression IV >35 Decompression and reexploration

ABDOMINAL COMPART MENT SYNDROME CLINICAL MANIFESTATIONS CENTRAL NERVOUS SYSTEM GASTROINTESTINAL  Intracranial pressure  Celiac blood flow  Cerebral perfusion pressure  SMA blood flow CARDIAC  Mucosal blood flow Hypovolemia  pHi  Cardiac output RENAL  Venous return  Urinary output  PCWP and CVP  Renal blood flow  SVR  GFR PULMONARY HEPATIC  Intrathoracic pressure  Portal blood flow  Airway pressures  Mitochondrial function  Compliance  Lactate clearance  PaO2  PaCO2 ABDOMINAL WALL  Shunt fraction  Compliance  Vd/Vt  Rectus sheath blood flow

Non-operative medical management There are now multiple papers noting that early intervention using medical management for intra-abdominal hypertension can prevent abdominal compartment syndrome from occurring and can result in reduced costs of care. All of these papers suggest early detection of elevated intraabdominal pressure via bladder pressure monitoring and introduction of a treatment protocol are required for optimal results Conditions Requiring Temporary Closures A. Primary closure of the abdominal cavity cannot be accomplished B. Primary closure of the abdominal cavity should not be done 1. Stage, or "damage control" laparotomy 2. "Second-look" operation required 3. Abdominal compartment syndrome prophylaxis C. Development of abdominal compartment

When the decision is made that the abdominal cavity should not be closed, all efforts should be made to preserve fascia for later definitive closure. Although some authors advocate suturing material to the fascial edge to form a "silo" over the abdominal wound, this practice should be avoided. Fascia should always be spared until definitive closure. There are several temporary closures of the abdominal cavity:

A. Endogenous—nonfascial closure 1. Towel clip closure (skin only) 2 cm from the edge and 2 cm apart, covered by an — adherent plastic drape 2. Suture closure—skin only ("the mortician closure")

B. Closure with prosthetics 1. Plastic silo—polyvinyl plastic silo, which was first performed by Dr. Oswaldo Borray of Bogotá, Colombia. Also, plastic wound drape (Steri-Drape, 3M, St. Paul, MN) can be used for this temporary closure. Some authors have advocated the 3-liter plastic bag, primarily used on the urologic service for irrigation. All silo material should be sewn to the skin. 2. Mesh a. Absorbable-polyglactin (Vicryl) or polyglycotic (Dexon) are often used when it is felt that primary closure cannot be achieved within a brief interval of days and that secondary closure by granulation tissue over the midgut will be necessary. Fabian et al. highlighted four stages in the use of absorbable mesh to cover the abdominal cavity:

1. Coverage of the midgut with absorbable mesh 2. Removal of mesh after formation of granulation (2 to 3 weeks)

3. Application of split thickness skin grafts 4. Definitive closure in 6 to 12 months

b. Nonabsorbable—there are more permanent meshes such as polypropylene mesh or Gore-Tex patch. However, as noted by Stone and others, such a rigid prosthesis has twice the incidence of postoperative wound sepsis, as

well as the subsequent development of 6 times as many enterocutaneous fistulae. This form of temporary closure should not be the coverage of choice for the open abdomen in the grossly contaminated operative field.

3. Zippers or Velcro devices—the overall concept of these devices is appealing, because they allow repeated evaluations of the abdominal cavity, along with the ability to perform necessary irrigations of the contaminated field.

C. Open—there are situations when the abdominal cavity is left open for a period of time because of either early or late abdominal sepsis because of failure of the

closed/semiclosed methods. Although the exposed bowel can be directly covered with moist gauze packs, the risk of intestinal fistula formation is still exceedingly high.

Vacuum-assisted temporary abdominal closure device: thin plastic sheet, a sterile towel, closed suction drains, and a large adherent operative drape. This dressing system permits increases in intra-abdominal volume, without a dramatic elevation in IAP.

3. Write short notes of the following: 5 x 6 a) Balloon Enteroscopy. b) Intestinal lengthening for short bowel syndrome. c) Gene expression profiling of Breast cancer. d) C.A.B.G. e) V.A.T.S. a) Balloon Enteroscopy. Answer. Introduction:The use of upper and lower gastrointestinal endoscopes has revolutionized the diagnosis and treatment of diseases of the esophagus, stomach, duodenum, and colon (large intestine). The last remaining frontier in the intestines has been the small intestine. Wireless allows physicians to visualize the inside of the intestines from the esophagus through to the colon, but capsule endoscopy has limitations, the most notable of which are the inability to control the capsule's passage and to perform therapy, for example, biopsy and electrocautery. Although capsule endoscopy is likely to remain an important diagnostic procedure because of its simplicity, the limitations of capsule endoscopy have been overcome by the development of balloon endoscopy, also known as enteroscopy. There are two types of balloon endoscopy: single balloon and double balloon.

Single balloon endoscopy: For single balloon endoscopy, a 200 cm long flexible, fiberoptic, endoscope (a hose-like tube one centimeter in diameter with a light and a camera on the tip) is fitted with an equally long overtube that slides the full length of the endoscope. On the tip of the overtube is a balloon that can be blown up and deflated. The balloon when blown up is used to anchor the overtube within the intestine. While the overtube is anchored, the endoscopy can be advance further into the small intestine. By withdrawing the overtube the small intestine can be shortened and straightened to make the passage of the inner endoscope easier. The balloon may then be deflated so that the overtube can be inserted further and the endoscope advanced again.The endoscope itself is a standard endoscope with working channels that allow the intestine to be inflated with air, rinsed with water, or to guide biopsy or electrocautery instruments to the tip of the endoscope. Double-balloon enteroscopy, also known as push-and-pull enteroscopy is an endoscopic technique for visualization of the small bowel. It was developed by Hironori Yamamoto in 2001. It is novel in the field of diagnostic gastroenterology as it is the first endoscopic technique that allows for the entire gastrointestinal tract to be visualized in real time. The technique involves the use of a balloon at the end of a special enteroscope camera and an overtube, which is a tube that fits over the endoscope, and which is also fitted with a balloon. The procedure is usually done under general anesthesia, but may be done with the use of conscious sedation. The enteroscope and overtube are inserted through the mouth and passed in conventional fashion (that is, as with gastroscopy) into the small bowel. Following this, the endoscope is advanced a small distance in front of the overtube and the balloon at the end is inflated. Using the assistance of friction at the interface of the enteroscope and intestinal wall, the small bowel is accordioned back to the overtube. The overtube balloon is then deployed, and the enteroscope balloon is deflated. The process is then continued until the entire small bowel is visualized.\ The double-balloon enteroscope can also be passed in retrograde fashion, through the colon and into the ileum to visualize the end of the small bowel. Indications Double-balloon enteroscopy has found a niche application in the following settings:

. Bleeding from the gastrointestinal tract of obscure cause . Iron deficiency anemia with normal colonoscopy and gastroscopy . Visualization and therapeutic intervention on abnormalities seen on traditional small bowel imaging . ERCP in post-surgical patients with long afferent limbs. Advantages and disadvantages Double-balloon enteroscopy offers a number of advantages to other small bowel image techniques, including barium imaging, wireless capsule endoscopy and push enteroscopy:

. It allows for visualization of the entire small bowel to the terminal ileum . It allows for the application of therapeutics . It allows for the sampling or biopsying of small bowel mucosa, for the resection of polyps of the small bowel, and in the placement of stents ordilatation of strictures of the small bowel. . It allows for access to the papilla in patients with long afferent limbs after Billroth II antrectomy. The key disadvantage of double-balloon enteroscopy is the time required to visualize the small bowel; this can exceed three hours, and may require that patients be admitted to hospital for the procedure. There have also been case reports of acute pancreatitis and intestinal necrosis associated with the technique. b) Intestinal lengthening for short bowel syndrome.

Answer. Introduction:Short-bowel syndrome is a disorder clinically defined by malabsorption, diarrhea, steatorrhea, fluid and electrolyte disturbances, and malnutrition. The final common etiologic factor in all causes of short-bowel syndrome is the functional or anatomic loss of extensive segments of small intestine so that absorptive capacity is severely compromised. Although resection of the colon alone typically does not result in short-bowel syndrome, the condition's presence can be a critical factor in the management of patients who lose significant amounts of small intestine. Massive small intestinal resection compromises digestive and absorptive processes. Adequate digestion and absorption cannot take place, and proper nutritional status cannot be maintained without supportive care. Today, the most common causes of short-bowel syndrome in adults include Crohn’s disease, radiation enteritis, mesenteric vascular accidents, trauma, and recurrent intestinal obstruction. In the pediatric population, necrotizing enterocolitis, intestinal atresias, and intestinal volvulus are the most common etiologic factors. Other conditions associated with short-bowel syndrome include congenital short small bowel, gastroschisis, and meconium peritonitis. Not all patients with loss of significant amounts of small intestine develop short-bowel syndrome. Important cofactors that help to determine whether the syndrome will develop or not include the premorbid length of small bowel, the segment of intestine that is lost, the age of the patient at the time of bowel loss, the remaining length of small bowel and colon, and the presence or absence of the ileocecal valve. Operative therapies for short-bowel syndrome can be divided into 2 broad categories: (1) intestinal or combined liver-intestinal transplantation, and (2) nontransplant operations. Nontransplant components of the surgical armamentarium for the treatment of short-bowel syndrome include intestinal lengthening (Bianchi) procedures, intestinal tapering for dilated dysfunctional bowel segments, , and creation of intestinal valves or reversed bowel segments for patients with rapid intestinal transit times. Surgical procedures to lengthen dilated bowel include the Bianchi procedure (where the bowel is cut in half and one end is sewn to the other) and a newer procedure called serial transverse enteroplasty (STEP—where the bowel is cut and stapled in a zigzag pattern).

c) Gene expression profiling of Breast cancer.

Answer. Cancer is a disease characterized by uncontrolled cell growth and proliferation. For cancer to develop, genes regulating cell growth and differentiation must be altered; these mutations are then maintained through subsequent cell divisions and are thus present in all cancerous cells. Gene expression profiling is a technique used in molecular biology to query the expression of thousands of genes simultaneously. In the context of cancer, gene expression profiling has been used to more accurately classify tumors. The information derived from gene expression profiling often has an impact on predicting the patient’s clinical outcome. Gene expression profiling is a technique used in molecular biology to query the expression of thousands of genes simultaneously. While almost all cells cell in an organism contain the entire genome of the organism, only a small subset of those genes is expressed as messenger RNA (mRNA) at any given time, and their relative expression can be evaluated. Techniques include DNA microarraytechnology or sequenced-based techniques such as serial analysis of gene expression (SAGE). Current cancer research makes use primarily of DNA microarrays in which an arrayed series of microscopic spots of pre-defined DNA oligonucleotides known as probes are covalently attached to a solid surface such as glass, forming what is known as a gene chip. DNA labeled with fluorophores (target) is prepared from a sample such as a tumor biopsy and is hybridized to the complementary DNA (cDNA) sequences on the gene chip. The chip is then scanned for the presence and strength of the fluorescent labels at each spot representing probe-target hybrids. The level of fluorescence at a particular spot provides quantitative information about the expression of the particular gene corresponding to the spotted cDNA sequence. DNA microarrays evolved from Southern blotting which allows for detection of a specific DNA sequence in a sample of DNA. There are a few gene expression profiling tests currently available. Here's a list of those available in the United States or in Europe for breast cancer:

 MammaPrint. This test is designed to help determine how likely early-stage lymph node negative breast cancers are to recur elsewhere in your body.  MapQuant DX. This test is designed to predict the risk of recurrence of certain types of estrogen receptor positive breast cancer.  Oncotype DX. This test is used to help determine whether to use chemotherapy after surgery to treat early-stage estrogen receptor positive, lymph node negative breast cancers with a low chance of recurrence.  Theros CancerType ID. This test is designed to help identify women with estrogen receptor positive breast cancer who are at high risk of recurrence and help predict their response to endocrine adjuvant therapy. Other tests are in development. The effectiveness of gene expression profiling is still being studied in a number of clinical trials. And while some doctors are using gene expression profiling to plan treatment, others are waiting for the results of these studies. Discuss with your doctor the pros and cons of gene expression profiling. These tests are not for people with more advanced breast cancer that has spread to distant parts of the body. d) C.A.B.G.

Answer. Coronary artery bypass grafting or CABG is an open-heart procedure to relieve the blockages of the arteries of the heart.

Indications for Coronary Artery Bypass Grafting (CABG) depend on various factors, mainly on the individual's symptoms and severity of disease. Some of these include -

 Left main artery disease or equivalent  Triple vessel disease  Abnormal Left Ventricular function.  Failed PTCA.  Immediately after Myocardial Infarction (to help perfusion of the viable myocardium).  Life threatening arrhythmias caused by a previous myocardial infarction.  Occlusion of grafts from previous CABGs.

Bypass grafting may be contraindicated in patients, for e.g. absent viable myocardium or the artery that needs grafting is too small.

Choice of Grafts for Coronary Artery Bypass Grafting (CABG)

If the decision has been made that your coronary artery disease is going to be treated with coronary artery bypass graft surgery, one important thing is to decide which grafts will be used to bypass your blocked arteries. There are 3 types of grafts, venous grafts, arterial grafts and synthetic grafts.

Venous grafts: During your operation, the surgical team will need to harvest and prepare a vein from elsewhere in the body to use as a graft. The most commonly used vein is the long saphenous vein. The long saphenous vein is a very long vein which runs on the inner surface of your leg starting from the ankle and moving up till it ends in the groin. The saphenous vein is long enough and if its whole length is taken, it can be used to bypass at least three coronary arteies. The good news is that when this vein is taken, nothing harmful happens to our leg except for some swelling, tingling and numbness.

If the long saphenous vein is diseased with varicosities, another vein on the back of your leg named the short saphenous vein can be used.

Although the saphenous vein is generally a good-quality blood vessel and can reach any coronary arteries, there is about a 5 percent chance per year that it will become narrow or totally blocked. Thus, its patency rate over time (or chance of the vein staying open with time) is not as good as that of some other grafts.

The saphenous vein graft is usally harvested by the physician assistant and can be taken through a long skin incision or recently by the endoscope (EVH). Benefits of endoscopic vein harvesting include more cosmetic incision and lower incidence of infection.

Arterial grafts: Certain arteries in the body can be harvested without doing much problems to the areas they were taken from. Here are examples of some of the arteries which can be harvested:

The left internal mammary artery: very commonly used vessel which is also called the left internal thoracic artery. It runs under the breastbone (sternum) on the left side. When the left internal mammary artery is harvested, it is usually freed in its lower part and its upper part is kept attached to its branch of origin from the aorta where it gets the blood from. After freeing its lower part , it is sutured to the left anterior descending coronary, which generally is the most important coronary artery for bypass.

The very good thing about the left internal mammary artery is its excellent patency rate in which there is about a 95 percent chance it will remain open for twenty or more years later. Sometimes its size is a disadvantage. It may only be a millimeter or less in diameter which is smaller than the coronary artery being bypassed, and sometimes the blood flow through it is inadequate. Occasionally, the internal mammary artery will not reach the point on the coronary that it needs to be grafted. That obstacle can frequently be overcome by disconnecting the upper end and sewing one end to the coronary and one to the aorta or another artery.

The right internal mammary: run under the breastbone on the right side, and is also frequently used to bypass blockages in the coronary arteries. This artery usually reaches the right coronary, the left anterior descending and some branches of the circumflex. If it does not, the approach is generally the same. The upstream end of the artery is disconnected, and one end is sewn on the coronary artery and the other is attached to the aorta or to another bypass graft.

One of the disadvantages of harvesting the internal mammary arteries is that they are the arteries which give blood supply to the breastbone. In some patients the healing of the breastbone, which is opened during surgery, is delayed and can be infected as well.

Radial Artery: which is located in the arm and runs from the elbow to the wrist towards the thumb.. Although some surgeons were using this artery for coronary bypass twenty-five years ago, recently it has become popular again. There is a single main artery in the upper arm called the brachial artery, which divides into two main branches near the elbow. One branch, the radial artery, runs along the inner forearm toward the thumb. The other branch, the ulnar artery, runs along the outer edge heading toward the little finger. Before harvesting the radial artery, surgeons will do special tests and sometimes Doppler ultrasound to make sure that the ulnar artery can take over and supply the hands when the radial artery is removed.

The patency rates with radial arteries so far indicate that it has a greater chance of staying open longer than saphenous vein grafts but not quite as long as the left internal mammary artery.

One of the disadvantages of the radial artery is that it can develop some spasm and narrowing after harvesting. This can be overcome by bathing the artery in some solutions after harvesting and by giving you special intravenous drugs in the ICU that will protect the artery from this spasm

Doctors sometimes use an abdominal vessel called the gastroepiploic arteryas the bypass graft. To use this artery, the abdomen must be opened. When using this artery, one end of it can be left attached to the stomach while the other end is threaded through a hole in the diaphragm, or breathing muscle, and joined to the appropriate coronary artery. The gastroepiploic artery can also be used as a free graft when both ends are disconnected. In this case, the other end is sewn to the aorta or another coronary bypass graft. The gastroepiploic artery graft seems to have a better patency rate than the saphenous vein graft but a somewhat poorer patency rate than the left internal mammary artery. The disadvantage of using this artery is that the surgeon has to make a second major incision to open the abdomen and take part of the blood supply of a portion of the stomach.

In certain other cases, veins from a human cadaver have been used, but, again, the patency rates are not very good. This may be because of a rejection process that occurs from using tissue from another human.

Synthetic grafts made of Dacron or other material have also been used. These grafts generally work quite well in other areas of the body, particularly in the larger arteries and the aorta, but the patency rates for coronary artery grafting have not been very good, and these synthetic arteries are not routinely used.

A new concept, called total arterial revascularization, is now emerging in which arterial grafts are only used because of their patency rates. However lot of surgeons still believe in using the left internal mammary artery for bypassing the left anterior descending artery and using saphenous vein grafts to bypass other coronary arteries.

On pump CABG:

The surgeon or an assistant surgeon removes the graft from legs or hands, ensuring the other ends are ligated properly. He then cleans the unwanted fat and other tissues attached to the graft. Injects saline to ensure that all branches are properly ligated or clipped. Nowadays saphenous vein can be harvested with minimal incision to avoid scarring and delay in healing.

Meanwhile, the chest layers are cut open right in the middle almost on top of the sternum. When the sternum is reached, the bone is sawed and the ribcage is retracted to expose the heart. The patient is connected to a heart lung machine by diverting blood from the systemic circulation through sterile plastic tubes.

The blood is oxygenized and filtered in the machine and sent back to the aorta thereby maintaining oxygen and nutrient supply to other vital organs. When the heart stops working, the surgeon identifies the block, makes incision (small cut) below the block. He then sutures the graft to that incision. Then the top end of the graft is sutured to the aorta. The left internal mammary artery that comes as a branch from aorta is left connected to its origin , however the distal or far end of the artery is divided for connection to the diseased coronary artery.

When all the grafts are in place and sutured, the heart is allowed to fill with blood and the heart lung machine is slowly weaned off. The surgeons make sure that there are no leaks in the connection between the graft and the aorta. Once the heart regains its function the pacing wires and drainage tubes are placed in the chest cavity to drain any fluid that normally collects after the procedure. The ribs are usually brought together and closed together with sternal wires and the rest of the muscles are closed in layers. The graft-harvested area is also sutured or closed in layers.

Following the surgery the chest wound is cleaned of the blood marks and dressed. The leg wound is dressed and bandaged with pressure to prevent swelling or limb edema.

Throughout the procedure an anesthesiologist will monitor the blood pressure, oxygen saturation and body temperature. Few blood tests may be repeated in intervals, especially when the patient is connected to the bypass machine.

Off-Pump Coronary Artery Bypass (OPCAB) Surgery - The same procedure can be performed without the use of the heart lung machine. It is often called as OPCAB or Off-Pump coronary artery bypass. This procedure can be performed when the heart is still beating.

Exposure of the heart is done by cutting the sternum open. Then a stabilizing device is used to hold and stabilize pertinent regions of the myocardium or heart muscle. Beating heart has to be handled very carefully, hence the lesions present on the front or anterior side is easier to graft as opposed to the ones on the lateral or posterior sides. Similar restrictions can be found in minimally invasive procedure, where the incision is made on the left side of the chest. A part of the ribcage is removed making the anterior part of the heart accessible. Patients with single vessel disease, who need one or two grafts, can be opted for OPCAB procedures.

Coronary Artery Bypass Surgery is a major surgical procedure and it carries some risks and complications. These complications are higher in patients who are heavy smokers or if they have a major problem with this other vital organs such as kidneys or lungs or have reduced blood supply to the brain.

Infection and bleeding are examples of risks and complications any surgery may present.

Some of these complications may be uncommon. Complications after surgery include-

 Ankle swelling  Heart Attack  Stroke  Emboli  Arrhythmias  Vein graft occlusion  Kidney failure or temporary shutdown  Death  Stress and Depression - could be long-term problem. Some patients may lose some of their cognitive ability,especially older women, however most will regain it within six to 12 months. Usually bypass surgery will not cause dementia however it may worsen this conditon if is is present prior to surgery.

The risk of death from CABG is approx 1%. The main reason of death is heart attacks occuring during or immediately after surgery. Heart attacks occurs in about 5% of the patients.

Neurological complications are minimal and occurs more in women than men, especially elder.

Incidence of stroke is about 1-2 %.

Long term studies show that exercise reduces mortality rate by 20%.

e) V.A.T.S.

Answer. Video-assisted thoracoscopic surgery (VATS) is a minimally invasive surgical technique used to diagnose and treat problems in your chest. During this surgery, a tiny camera (thoracoscope) and surgical instruments are inserted in the chest through small incisions. The thoracoscope transmits images of the inside of your chest onto a video monitor, guiding the surgeon in performing the procedure. Video-assisted thoracoscopic surgery (VATS) can be used for many purposes, ranging from a biopsy to removal of tumors or entire lobes from the lung. Its advantages include:

 Less pain and need for pain medication  Smaller scar  Shorter hospital stay  Faster recovery time Uses for VATS surgery Chest (thoracic) surgeons at Mayo Clinic use VATS to diagnose and treat a range of conditions, including:

 Pleural diseases. The pleura is a two-layer membrane that lines the chest cavity and surrounds both lungs. VATS is used to remove air or fluid which can collect in the pleural cavity following a chest injury or disorder and lung surgery. Surgeons may also use VATS to perform pleurodesis, a procedure where medicine is injected into your chest to fuse the space between the lung and chest wall and prevent fluid from returning to the chest.  . A rare cancer of the membranes lining the chest, abdominal cavity or heart, mesothelioma can be diagnosed with a VATS biopsy.  Lung cancer. VATS is used to remove pieces (wedge resection), lobes (lobectomy), fluid-filled lumps (nodules) and any suspicious spots on the lung along with a surrounding margin of healthy tissue. These tissue samples are then analyzed by an experienced pathologist to determine the stage of cancer.  Interstitial lung disease. VATS is used to gather pieces of lung to make the proper diagnosis and guide treatment decisions.  Empyemas. An empyema is a collection of pus and fluid that develops from a lung infection such as pneumonia. VATS is used to drain the excess fluid and allow the lung to re-expand.  Granulomas: VATS can be used to diagnose and remove swollen (inflamed) tissue called a granuloma which can develop in the lungs.  Lymphomas. Surgeons can remove tissue samples of abnormal lymph nodes from the chest cavity with a VATS biopsy to determine the type of lymphoma present.  Hyperhidrosis (excessive sweating). Chest surgeons use VATS to remove or cut the nerves that cause sweating (sympathectomy or sympathotomy).  Pericardial effusion. VATS can be used to drain fluid from the sac surrounding the heart (pericardium), caused by a range of diseases and conditions. Candidates The best candidates for VATS procedures are:

 Individuals who have never had chest surgery. Scar tissue from previous procedures can impede access to the chest cavity.  People who are at high risk for complications if they undergo traditional, open chest surgery.

1 4. Answer briefly on the following: 4 x 7 /2 a) Current status of staging Laparotomy in Lymphomas. b) Diabetic foot – recent trend in management. c) Recent trend in management of urinary lithiasis. d) Renal Transplant - steps.

a) Current status of staging Laparotomy in Lymphomas.

Answer. Advances in the treatment of Hodgkin’s disease have transformed a once fatal disease into a highly curable one. At present, virtually all patients given the diagnosis of Hodgkin’s disease have a chance for cure, with at least 75% of newly diagnosed patients expecting long- term survival . As the cure rate for Hodgkin’s disease improves, the implications of long-term complications of therapy become a paramount consideration in making therapeutic decisions. Disease stage remains the principle determinant of therapy , with early stage patients (I-II) treated primarily with radiotherapy and advanced stage patients (III-IV) receiving combination chemotherapy or combined-modality therapy.

Ann Arbor staging for Lymphoma:

Stage I

Either of the following means the disease is stage I:

 The lymphoma is in only 1 lymph node area or lymphoid organ such as the thymus (I).

 The cancer is found only in 1 area of a single organ outside of the lymph system (IE).

Stage II Either of the following means the disease is stage II:

 The lymphoma is in 2 or more groups of lymph nodes on the same side of (above or below) the diaphragm (the thin band of muscle that separates the chest and abdomen). For example, this might include nodes in the underarm and neck area but not the combination of underarm and groin nodes (II).

 The lymphoma extends from a single group of lymph node(s) into a nearby organ (IIE). It may also affect other groups of lymph nodes on the same side of the diaphragm. Stage III Either of the following means the disease is stage III:

 The lymphoma is found in lymph node areas on both sides of (above and below) the diaphragm.

 The cancer may also have spread into an area or organ next to the lymph nodes (IIIE), into the spleen (IIIS), or both (IIISE). Stage IV Either of the following means the disease is stage IV:

 The lymphoma has spread outside of the lymph system into an organ that is not right next to an involved node.

 The lymphoma has spread to the bone marrow, liver, brain or spinal cord, or the pleura (thin lining of the lungs). THE role of staging laparotomy in lymphoma  Staging laparotomy remains the gold standard for evaluating microscopic disease in the abdomen, including the spleen and liver .  From its introduction, staging laparotomy was intended only for patients in whom radiation therapy potentially could be the sole therapeutic modality.  Surgical staging allows treatment to be chosen with specific knowledge regarding the extent of disease.  Patients who prove to have pathologic early stage disease in the absence of bulky mediastinal adenopathy may be treated safely with radiation alone with an anticipated disease-free survival (DFS) of greater than 75% for pathologic stage IA and IIA (PS IA-IIA) disease.  Furthermore, pathologic staging may permit the use of smaller radiation fields. In a group of patients with pathologic early stage disease and other favorable prognostic factors (such as young age or nodular sclerosis or lymphocyte predominant histology), mantle field irradiation alone may be sufficient. In those patients who still require subdiaphragmatic irradiation after laparotomy, splenectomy allows delivery of a smaller dose to the abdomen without jeopardizing the left kidney and lung base .  Despite these attractive advantages, compelling reasons exist to avoid surgical staging. A laparotomy delays by at least two to three weeks the implementation of therapy, as the patient recuperates after a five- to seven-day hospitalization. While laparoscopic splenectomy may shorten the recuperative period, this procedure does not afford the full nodal examination that can be obtained with the open procedure .  Staging laparotomy has associated morbidity and mortality. Published mortality rates range from 0.3% to 1%, but should not represent a major concern at most centers .  Morbidity poses the greater threat. Major complications include cardiac arrest, wound infection or dehiscence, postoperative hemorrhage, subphrenic abscess, pulmonary embolism, pneumonia, sepsis and small bowel obstruction from adhesions. Minor complications include postoperative atelectasis, urinary tract infections, urinary retention and prolonged ileus. b) Diabetic foot – recent trend in management. Answer. A diabetic foot is a foot that exhibits any pathology that results directly from diabetes mellitus or any long-term (or "chronic") complication of diabetes mellitus. Presence of several characteristic diabetic foot is called diabetic foot syndrome. These are thus umbrella terms. One of the serious foot complications in diabetes is: Diabetic foot ulceration. It occurs in 15% of all patients with diabetes and precedes 84% of all lower leg .

 Diabetics are prone to foot ulcerations due to both neurologic and vascular complications.  Peripheral neuropathy can cause altered or complete loss of sensation in the foot and /or leg. Similar to the feeling of a "fat lip" after a dentist's anesthetic injection, the diabetic with advanced neuropathy looses all sharp-dull discrimination. Any cuts or trauma to the foot can go completely unnoticed for days or weeks in a patient with neuropathy. It's not uncommon to have a patient with neuropathy tell you that the ulcer "just appeared" when, in fact, the ulcer has been present for quite some time. There is no known cure for neuropathy, but strict glucose control has been shown to slow the progression of the neuropathy.  Charcot foot deformity occurs as a result of decreased sensation. People with "normal" feeling in their feet automatically determine when too much pressure is being placed on an area of the foot. Once identified, our bodies instinctively shift position to relieve this stress. A patient with advanced neuropathy looses this important mechanism. As a result, tissue ischemia and necrosis may occur leading to plantar ulcerations. Microfractures in the bones of the foot go unnoticed and untreated, resulting in disfigurement, chronic swelling and additional bony prominences.  Microvascular disease is a significant problem for diabetics and can lead to ulcerations. It is well known that diabetes is called a small vessel disease. Most of the problems caused by narrowing of the small arteries cannot be resolved surgically. It is critical that diabetics maintain close control on their glucose level, maintain a good body weight and avoid smoking in an attempt to reduce the onset of small vessel disease.

Treatment:

 First, determine the cause of this ulcer. Is it neuropathic, ischemic or a combination?  Treatment protocol should be based on the etiology of the ulcer.  Assuming that there is adequate perfusion to heal a plantar ulcer, one should have appropriate shoe modifications made to disperse weight away from the ulcerative area. Absorb any excess discharge and maintain a moist wound environment with appropriate product selection. Keep the wound edges dry. Make sure no sinus tracking occurs. Watch for infection. Debride necrotic debris and the hyperkeratotic rim as they are niduses of for infection.  Successful treatment of diabetic foot ulcers consists of addressing these three basic issues: debridement, offloading, and infection control.

Debridement:

 Debridement consists of removal of all necrotic tissue, peri-wound callus, and foreign bodies down to viable tissue. Proper debridement is necessary to decrease the risk of infection and reduce peri-wound pressure, which can impede normal wound contraction and healing. After debridement, the wound should be irrigated with saline or cleanser, and a dressing should be applied.  Dressings should prevent tissue dessication, absorb excess fluid, and protect the wound from contamination. There are hundreds of dressings on the market, including hydrogels, foams, calcium alginates, absorbent polymers, growth factors, and skin replacements. Becaplermin contains the β-chain platelet-derived growth factor and has been shown in double-blind placebo-controlled trials to significantly increase the incidence of complete wound healing. Its use should be considered for ulcers that are not healing with standard dressings.  In case of an abscess, incision and drainage are essential, with debridement of all abscessed tissue. Many limbs have been saved by timely incision and drainage procedures; conversely, many limbs have been lost by failure to perform these procedures. Treating a deep abscess with antibiotics alone leads to delayed appropriate therapy and further morbidity and mortality.

Offloading:

 Having patients use a wheelchair or crutches to completely halt weight bearing on the affected foot is the most effective method of offloading to heal a foot ulceration. Total contact casts (TCCs) are difficult and time consuming to apply but significantly reduce pressure on wounds and have been shown to heal between 73 and 100% of all wounds treated with them.. Inappropriate application of TCCs may result in new ulcers, and TCCs are contraindicated in deep or draining wounds or for use with noncompliant, blind, morbidly obese, or severely vascularly compromised patients.  Clinicians often prefer removable cast walkers because they do not have some of the disadvantages of TCCs. Removability is an advantage in that it allows for daily wound inspection, dressing changes, and early detection of infection. But removability is also the greatest disadvantage in that studies have shown that patients wear them only 30% of the time they are walking (usually to and from the doctor's office). ∼  Postoperative shoes or wedge shoes are also used and must be large enough to accommodate bulky dressings. Proper offloading remains the biggest challenge for clinicians dealing with diabetic foot ulcers.

Infection control:

 Limb-threatening diabetic foot infections are usually polymicrobial. Commonly encountered pathogens include methicillin-resistant staphylococcus aureus, β-hemolytic streptococci, enterobacteriaceae, pseudomonas aeruginosa, and enterococci. Anaerobes, such as bacteroides, peptococcus, andpeptostreptococcus, are rarely the sole pathogens but are seen in mixed infections with aerobes. Antibiotics selected to treat severe or limb- threatening infections should include coverage of gram-positive and gram-negative organisms and provide both aerobic and anaerobic coverage. Patients with such wounds should be hospitalized and treated with intravenous antibiotics.  Mild to moderate infections with localized cellulitis can be treated on an outpatient basis with oral antibiotics such as cephalexin, amoxicillin with clavulanate potassium, moxifloxacin, or clindamycin. The antibiotics should be started after initial cultures are taken and changed as necessary. c) Recent trend in management of urinary lithiasis.

Answer. Risk Factors for the Development of Urinary Calculi

Risk factor Mechanisms Bowel disease Promotes low urine volume; acidic urine depletes available citrate; hyperoxaluria Excess dietary meat Creates acidic urinary milieu, depletes available citrate; promotes (including poultry) hyperuricosuria Excess dietary Promotes hyperoxaluria oxalate Excess dietary Promotes hypercalciuria sodium Family history Genetic predisposition Insulin resistance Ammonia mishandling; alters pH of urine Gout Promotes hyperuricosuria Low urine volume Allows stone constituents to supersaturate Obesity May promote hypercalciuria; other results similar to excess dietary meat Primary Creates persistent hypercalciuria hyperparathyroidism Prolonged Bone turnover creates hypercalciuria immobilization Renal tubular acidosis Alkaline urine promotes calcium phosphate supersaturation; loss of citrate (type 1)

Risk factor Mechanisms Clinical Clues to the Diagnosis of Urinary Calculi

Evaluation Possible findings Laboratory evaluations Complete blood count Leukocytosis with struvite calculi Serum chemistry Elevation in creatinine levels with obstructing calculi; hypokalemia and hyperchloremia with renal tubular acidosis; elevated serum calcium levels with parathyroid disease Serum parathyroid Elevated in hyperparathyroidism hormone levels Urinalysis Microscopic or gross hematuria; acidic urine; alkaline urine (with struvite calculi); pyuria; crystals from involved calculi 24-hour urinalysis Elevated urinary calcium, oxalate, and sodium levels; decreased urinary volume and citrate levels Radiographic evaluations Abdominal, kidney, Urinary calculi larger than 2 mm may be visible. and upper bladder radiography CT (stone protocol) Nearly all calculi are visible on CT. Evaluates renal parenchyma, hydronephrotic changes, and surrounding organs for other etiologies of abdominal pain. Intravenous Calculi visible on scout film. Delay in contrast pyelography excretion if obstruction is present. Calculi may appear as filling defect. MRI Conventional MRI is not useful for imaging calculi. Ultrasonography Calculi appear as hyperechoic lesions that cast acoustic shadows. Not reliable for ureteral calculi. May demonstrate dilation of collecting system.

d) Renal Transplant - steps. Answer. Kidney transplantation or renal transplantation is the organ transplant of a kidney into a patient with end-stage renal disease. Kidney transplantation is typically classified as deceased- donor (formerly known as cadaveric) or living-donor transplantation depending on the source of the donor organ. Living-donor renal transplants are further characterized as genetically related (living-related) or non-related (living-unrelated) transplants, depending on whether a biological relationship exists between the donor and recipient.

The indication for kidney transplantation is end-stage renal disease (ESRD), regardless of the primary cause. This is defined as a glomerular filtration rate <15ml/min/1.73 sq.m. Common diseases leading to ESRD include malignant hypertension, infections, diabetes mellitus, and focal segmental glomerulosclerosis; genetic causes include polycystic kidney disease, a number of inborn errors of metabolism, and autoimmune conditions such as lupus and Goodpasture's syndrome. Diabetes is the most common cause of kidney transplantation, accounting for approximately 25% of those in theUS. The majority of renal transplant recipients are on some form of peritoneal dialysis, or the similar process of hemofiltration—at the time of transplantation. However, individuals with chronic renal failure who have a living donor available may undergo pre-emptive transplantation before dialysis is needed. Procedure for a living kidney donor

If you are donating a kidney, you will be placed under general anesthesia before surgery. This means you will be asleep and pain-free. Usually, surgeons today can use small surgical cuts with laparoscopic techniques.

Procedure for the person receiving the kidney (recipient)

People receiving a kidney transplant are given general anesthesia before surgery. The surgeon makes a cut in the lower belly area.

 Surgeon places the new kidney inside the lower belly. The artery and vein of the new kidney are connected to the artery and vein in your pelvis. The tube that carries urine (ureter) is then attached to the bladder.

 Patient’s kidneys are left in place, unless they are causing high blood pressure, infections, or are too large for your body. The wound is then closed.

Kidney transplant surgery takes about 3 hours. People with diabetes may also have a pancreas transplant done at the same time. This will usually add another 3 hours to the surgery.

Procedure In most cases the barely functioning existing kidneys are not removed, as this has been shown to increase the rates of surgical morbidities. Therefore, the kidney is usually placed in a location different from the original kidney, often in the iliac fossa, so it is often necessary to use a different blood supply:

. The renal artery of the kidney, previously branching from the abdominal aorta in the donor, is often connected to the external iliac artery in the recipient. . The renal vein of the new kidney, previously draining to the inferior vena cava in the donor, is often connected to the external iliac vein in the recipient.